Top Banner
Oxford Cambridge and RSA Examinations GCE History A Advanced GCE A2 H506 Advanced Subsidiary GCE AS H106 Mark Schemes for the Units January 2010 HX06/MS/R/10J
255

History A Mark Schemes for the Units January 2010 - The ...

Feb 26, 2023

Download

Documents

Khang Minh
Welcome message from author
This document is posted to help you gain knowledge. Please leave a comment to let me know what you think about it! Share it to your friends and learn new things together.
Transcript
Page 1: History A Mark Schemes for the Units January 2010 - The ...

Oxford Cambridge and RSA Examinations

GCE

History A Advanced GCE A2 H506

Advanced Subsidiary GCE AS H106

Mark Schemes for the Units January 2010

HX06/MS/R/10J

Page 2: History A Mark Schemes for the Units January 2010 - The ...

OCR (Oxford Cambridge and RSA) is a leading UK awarding body, providing a wide range of qualifications to meet the needs of pupils of all ages and abilities. OCR qualifications include AS/A Levels, Diplomas, GCSEs, OCR Nationals, Functional Skills, Key Skills, Entry Level qualifications, NVQs and vocational qualifications in areas such as IT, business, languages, teaching/training, administration and secretarial skills. It is also responsible for developing new specifications to meet national requirements and the needs of students and teachers. OCR is a not-for-profit organisation; any surplus made is invested back into the establishment to help towards the development of qualifications and support which keep pace with the changing needs of today’s society. This mark scheme is published as an aid to teachers and students, to indicate the requirements of the examination. It shows the basis on which marks were awarded by Examiners. It does not indicate the details of the discussions which took place at an Examiners’ meeting before marking commenced. All Examiners are instructed that alternative correct answers and unexpected approaches in candidates’ scripts must be given marks that fairly reflect the relevant knowledge and skills demonstrated. Mark schemes should be read in conjunction with the published question papers and the Report on the Examination. OCR will not enter into any discussion or correspondence in connection with this mark scheme. © OCR 2010 Any enquiries about publications should be addressed to: OCR Publications PO Box 5050 Annesley NOTTINGHAM NG15 0DL Telephone: 0870 770 6622 Facsimile: 01223 552610 E-mail: [email protected]

Page 3: History A Mark Schemes for the Units January 2010 - The ...

CONTENTS

Advanced GCE History (H506)

Advanced Subsidiary GCE History (H106)

MARK SCHEMES FOR THE UNITS

Unit/Content Page

AS/A2 HISTORY SYLLABUS-SPECIFIC MARKING INSTRUCTIONS 1

F961 British History Period studies 14

F962 European and World History Period Studies 29

F963 British History Enquiries 57

Option A: Medieval and Early Modern 1066-1660 57

F964 European and World History Enquiries 72

F966 Historical Themes 88

Grade Thresholds 110

Page 4: History A Mark Schemes for the Units January 2010 - The ...

Marking Instructions

AS/A2 HISTORY SYLLABUS-SPECIFIC MARKING INSTRUCTIONS

AS UNIT F961 & UNIT F962 – PERIOD STUDIES Distribution of marks for each level that reflects the Unit’s AOs and corresponds to the UMS 2 answers: each maximum mark 50.

A01a A01b

IA 21-24 24-26

IB 18-20 22-23

II 16-17 19-21

III 14-15 16-18

IV 12-13 13-15

V 9-11 11-12

VI 4-8 6-10

VII 0-3 0-5

Notes:

(i) Allocate marks to the most appropriate level for each AO.

(ii) If several marks are available in a box, work from the top mark down until the best fit has been found.

(iii) Many answers will not fall at the same level for each AO.

(iv) Analysis refers to developed explanations; evaluation refers to the argued weighing up/assessment of factors in relation to their significance in explaining an issue or in explaining linkages between different factors.

1

Page 5: History A Mark Schemes for the Units January 2010 - The ...

Marking Instructions

AOs AO1a AO1b Total mark for each question = 50

Recall, select and deploy historical knowledge appropriately, and communicate knowledge and understanding of history in a clear and effective manner.

Demonstrate understanding of the past through explanation, analysis and arriving at substantiated judgements of: - key concepts such as causation, consequence, continuity, change and significance within an historical context; - the relationships between key features and characteristics of the periods studied

Level IA

Uses a wide range of accurate, detailed and relevant evidence

Accurate and confident use of appropriate historical terminology

Answer is clearly structured and coherent; communicates accurately and legibly

21-24

Clear and accurate understanding of key concepts relevant to analysis and to the topic

Clear and accurate understanding of the significance of issues in their historical context

Answer is consistently and relevantly analytical with developed and substantiated explanations, some of which may be unexpected

The argument evaluates a range of relevant factors and reaches clearly substantiated judgements about relative importance and/or links.

24-26

Level IB

Uses accurate, detailed and relevant evidence Accurate use of a range of

appropriate historical terminology Answer is clearly structured and

mostly coherent; writes accurately and legibly

18-20

Clear and accurate understanding of most key concepts relevant to analysis and to the topic

Answer is mostly consistently and relevantly analytical with mostly developed and substantiated explanations

Clear understanding of the significance of issues in their historical context.

Substantiated judgements about relative importance of and/or links between factors will be made but quality of explanation in support may not be consistently high.

22-23

Level II

Uses mostly accurate, detailed and relevant evidence which demonstrates a competent command of the topic Generally accurate use of historical terminology Answer is structured and mostly

coherent; writing is legible and communication is generally clear

16-17

Mostly clear and accurate understanding of many key concepts relevant to analysis and to the topic

Clear understanding of the significance of most relevant issues in their historical context

Much of the answer is relevantly analytical and substantiated with detailed evidence but there may be some description

The analysis of factors and/ or issues provides some judgements about relative importance and/or linkages.

19-21

Level III

Uses accurate and relevant evidence which demonstrates some command of the topic but there may be some inaccuracy

Answer includes relevant historical terminology but this may not be extensive or always accurately used

Most of the answer is organised and structured; the answer is mostly legible and clearly communicated

14-15

Some/uneven understanding of key concepts relevant to analysis and of concepts relevant to their historical context

Answers may be a mixture of analysis and explanation but also simple description of relevant material and narrative of relevant events OR answers may provide more consistent analysis but the quality will be uneven and its support often general or thin.

Answer considers a number of factors but with very little evaluation of importance or linkages between factors/issues

Points made about importance or about developments in the context of the period will often be little more than assertions and descriptions

16-18

2

Page 6: History A Mark Schemes for the Units January 2010 - The ...

Marking Instructions

AOs AO1a AO1b Level IV

There is deployment of relevant knowledge but level/accuracy of detail will vary; there may be some evidence that is tangential or irrelevant.

Some unclear and/or under-developed and/or disorganised sections; mostly satisfactory level of communication.

12-13

Understanding of key concepts relevant to analysis and the topic is variable but in general is satisfactory.

Limited and patchy understanding of a few relevant issues in their historical context.

Answer may be largely descriptive/ narratives of events and links between this and analytical comments will typically be weak or unexplained OR answers will mix passages of descriptive material with occasional explained analysis.

Limited points made about importance/links or about developments in the context of the period will be little more than assertions and descriptions

13-15

Level V

There is some relevant accurate historical knowledge deployed: this may be generalised and patchy. There may be inaccuracies and irrelevant material also

Some accurate use of relevant historical terminology but often inaccurate/ inappropriate use

Often unclear and disorganised sections; writing will often be clear if basic but there may be some illegibility and weak prose where the sense is not clear or obvious

9-11

General and sometimes inaccurate understanding of key concepts relevant to analysis and of concepts relevant to the topic

General or weak understanding of the significance of most relevant issues in their historical context

Attempts at analysis will be weak or generalised, based on plausible but unsubstantiated points or points with very general or inappropriate substantiation OR there may be a relevant but patchy description of events/developments coupled with judgements that are no more than assertions

There will be some understanding of the question but answers may focus on the topic not address the focus of the question

11-12

Level VI Use of relevant evidence will be limited; there will be much irrelevance and inaccuracy Answer may have little

organisation or structure; weak use of English and poor organisation

4-8

Very little understanding of key concepts Very limited understanding of the topic or of the

question’s requirements Limited explanation will be very brief/ fragmentary The answer will be characterised by generalised

assertion and/or description/ narratives, often brief

6-10 Level VII No understanding of the topic or of

the question’s requirements; little relevant and accurate knowledge Very fragmentary and disorganised

response; very poor use of English and some incoherence

0-3

No understanding of key concepts or historical developments. No valid explanations Typically very brief and very descriptive answer

0-5

3

Page 7: History A Mark Schemes for the Units January 2010 - The ...

Marking Instructions

AS UNIT F963 & UNIT F964 – Historical Enquiries Maximum mark 100. 1 answer: 2 parts. Question (a) Maximum mark 30 A01a A01b AO2a

IA 6 8 16

IB 6 7 13-15

II 5 6 11-12

III 4 5 9-10

IV 3 4 7-8

V 2 3 5-6

VI 1 2 3-4

VII 0 0-1 0-2

Notes related to Question (a) (i) Allocate marks to the most appropriate level for each AO (ii) If several marks are available in a box, work from the top mark down until the best fit has

been found (iii) Many answers will not fall at the same level for each AO Question (b) Maximum mark 70 A01a A01b AO2a AO2b

IA 9-10 11-12 26-28 20

IB 8 9-10 23-25 17-19

II 7 8 20-22 14-16

III 6 6-7 17-19 11-13

IV 4-5 4-5 14-16 8-10

V 3 3 11-13 6-7

VI 2 2 5-10 3-5

VII 0-1 0-1 0-4 0-2

Notes related to Part B: (i) Allocate marks to the most appropriate level for each AO (ii) If several marks are available in a box, work from the top mark down until the best fit has

been found (iii) Many answers will not be at the same level for each AO

4

Page 8: History A Mark Schemes for the Units January 2010 - The ...

Marking Instructions

Marking Grid for Question (a)

AOs AO1a AO1b AO2a Total for each question = 30

Recall, select and deploy historical knowledge appropriately, and communicate knowledge and understanding of history in a clear and effective manner.

Demonstrate understanding of the past through explanation, analysis and arriving at substantiated judgements of: - key concepts such as causation, consequence, continuity, change and significance within an historical context; - the relationships between key features and characteristics of the periods studied.

As part of an historical enquiry, analyse and evaluate a range of appropriate source material with discrimination.

Level IA

Accurate use of a range of appropriate historical terminology

Answer is clearly structured and coherent; communicates accurately and legibly

6

Answer is consistently and relevantly analytical with developed comparison and judgement

Clear and accurate understanding of key concepts relevant to analysis and to the topic

Clear and accurate understanding of the significance of issues in their historical context

8

Response provides a focused comparison and/or contrast of both content and provenance

Evaluates qualities such as reliability, completeness, consistency, typicality, and especially utility, in relation to the question

16 Level IB

Accurate use of a range of appropriate historical terminology

Answer is clearly structured and coherent; communicates accurately and legibly

6

Judgements are supported by appropriate references to both content and provenance

Very good level of understanding of key concepts

Clear and accurate understanding of the significance of issues in their historical context

7

Response provides an effective comparison and/or contrast of both content and provenance

Evaluates a range of qualities of authenticity, completeness, consistency, typicality and usefulness in relation to the question

13-15 Level II

Generally accurate use of historical terminology Answer is structured and mostly coherent; writing is legible and communication is generally clear

5

Good attempt at explanation/ analysis but uneven overall judgements

Mostly clear and accurate understanding of key concepts

Clear understanding of the significance of most relevant issues in their historical context

6

Provides a relevant comparison and/ or contrast of both content and provenance

Answer lacks completeness in evaluating most of the range of available criteria (eg. limited use of the introductions and/ or attributions)

11-12

5

Page 9: History A Mark Schemes for the Units January 2010 - The ...

Marking Instructions

AOs AO1a AO1b AO2a

Level III

Answer includes relevant historical terminology but this may not be extensive or always accurately used

Most of the answer is organised and structured; the answer is mostly legible and clearly communicated

4

A mixture of internal analysis and discussion of similarities and/or differences. A judgement is unlikely Some/uneven understanding

of many key concepts relevant to analysis and of many concepts relevant to the topic

Uneven understanding of the significance of most relevant issues in their historical context

5

Provides a comparison and/ or contrast

Makes limited links with the sources by focusing too much on content or on provenance

The organisation is uneven, confining the comparison to the second half of the answer or simply to a concluding paragraph

9-10

Level IV

There may be some evidence that is tangential or irrelevant

Some unclear and/or under-developed and/or disorganised sections; mostly satisfactory level of communication

3

Mostly satisfactory understanding of key concepts

Mostly satisfactory explanation but some unlinked though relevant assertions, description / narrative

There is no judgement

4

Response attempts a comparison and/or contrast but the comment is largely sequential

Few points of comparative provenance or discussion of similarity/difference of content

7-8 Level V

There may be inaccuracies and irrelevant material.

Some accurate use of relevant historical terminology but often inaccurate/ inappropriate use

Often unclear and disorganised sections; writing will often be clear if basic but there may be some illegibility and weak prose where the sense is not clear or obvious

2

General and sometimes inaccurate understanding of key concepts relevant to analysis and of concepts relevant to the topic

General or weak understanding of the significance of most relevant issues in their historical context

3

Identifies some points of agreement and/or disagreement

The comparison and/or contrast is implicit

There is no judgement

5-6 Level VI

There will be much irrelevance and inaccuracy Answer may have little organisation or structure; weak use of English and poor organisation

1

Limited explanation but mainly description / narrative

Very little understanding of key concepts

2

Very weak commentary on one point of agreement/ disagreement

Sources may be paraphrased with no real attempt to compare and/or contrast

3-4

Level VII

No understanding of the topic or of the question’s requirements Totally irrelevant answer Very poor use of English

0

Weak explanation, and descriptive / narrative commentary on the sources

No understanding of key concepts

0-1

No attempt to provide a comparison and/or contrast

Sources are paraphrased or copied out

0-2

6

Page 10: History A Mark Schemes for the Units January 2010 - The ...

Marking Instructions

Marking Grid for Question (b)

AOs AO1a AO1b AO2a AO2b Total mark for the question = 70

Recall, select and deploy historical knowledge appropriately, and communicate knowledge and understanding of history in a clear and effective manner.

Demonstrate understanding of the past through explanation, analysis and arriving at substantiated judgements of: - key concepts such as causation, consequence, continuity, change and significance within an historical context; - the relationships between key features and characteristics of the periods studied.

As part of an historical enquiry, analyse and evaluate a range of appropriate source material with discrimination.

Analyse and evaluate, in relation to the historical context, how aspects of the past have been interpreted and represented in different ways.

Level IA

Uses a wide range of accurate, detailed and relevant evidence

Accurate and confident use of appropriate historical terminology

Answer is clearly structured and coherent; communicates accurately and legibly

9-10

Clear and accurate understanding of key concepts relevant to analysis and to the topic

Clear and accurate understanding of the significance of issues in their historical context

Answer is consistently and relevantly analytical with developed explanations leading to careful judgements

11-12

Excellent analysis and evaluation of all sources with high levels of discrimination

Analyses and evaluates the limitations of the sources and what is required to add to their completeness as a set

26-28

Excellent analysis and evaluation of the historical interpretation using all sources and own knowledge to reach a clear conclusion

Fully understands that the sources may either support or refute the interpretation

20 Level IB

Uses accurate, detailed and relevant evidence Accurate use of a

range of appropriate historical terminology

Answer is clearly structured and mostly coherent; writes accurately and legibly

8

Clear and accurate understanding of most key concepts relevant to analysis and to the topic

Clear understanding of the significance of issues in their historical context

Judgements are supported by appropriate references to both content and provenance

9-10

Focussed analysis and evaluation of all sources with high levels of discrimination

Analyses and evaluates the limitations of the sources and what is required to add to their completeness as a set

23-25

Focussed analysis and evaluation of the historical interpretation using all sources and own knowledge to reach a clear conclusion

Understands that the sources may either support or refute the interpretation

17-19

7

Page 11: History A Mark Schemes for the Units January 2010 - The ...

Marking Instructions

AOs AO1a AO1b AO2a AO2b

Level II

Uses mostly accurate, detailed and relevant evidence which demonstrates a competent command of the topic Generally accurate use of historical terminology Answer is structured and mostly coherent; writing is legible and communication is generally clear

7

Mostly clear and accurate understanding of key concepts

Clear understanding of the significance of most relevant issues in their historical context.

Good attempt at explanation/ analysis but uneven overall judgements

8

Focussed analysis and evaluation of most of the sources with good levels of discrimination

Analyses and evaluates some of the limitations of the sources and what is required to add to their completeness as a set

20-22

Focussed analysis and evaluation of the historical interpretation using most of the sources and appropriate own knowledge to reach a clear conclusion

There may be some imbalance between discussion of the sources and use of external knowledge in evaluating the interpretation

14-16 Level III

Uses accurate and relevant evidence which demonstrates some command of the topic but there may be some inaccuracy

Answer includes relevant historical terminology but this may not be extensive or always accurately used

Most of the answer is organised and structured; the answer is mostly legible and clearly communicated

6

Shows a sound understanding of key concepts. Sound awareness of

the significance of issues in their historical context

Attempts an explanation/ analysis but overall judgement may be incomplete

6-7

Refers to most of the sources to illustrate an argument rather than analysing and evaluating their evidence

Aware of some of the sources’ limitations either individually or as a set

17-19

Sound analysis and evaluation of the historical interpretation.

There may be some description and unevenness between use of own knowledge and use of sources

Answers which use the sources but no own knowledge in assessing the interpretation have a Level III ceiling

11-13 Level IV

There is deployment of relevant knowledge but level/ accuracy of detail will vary; there may be some evidence that is tangential or irrelevant

Some unclear and/or under-developed and/or disorganised sections; mostly satisfactory level of communication

4-5

Mostly satisfactory understanding of key concepts

Some explanation but not always linked to the question

Assertions, description / narrative will characterise part of the answer

4-5

Sources are discussed sequentially

Considers some of the limitations of the sources; but may not establish a sense of different views

14-16

Some analysis and evaluation of the historical interpretation with increasing amounts of description

Response is more imbalanced than Level III in using sources and own knowledge

Answers that use own knowledge but make no use of the sources in assessing the interpretation have a Level IV ceiling

8-10

8

Page 12: History A Mark Schemes for the Units January 2010 - The ...

Marking Instructions

AOs AO1a AO1b AO2a AO2b

Level V There is some relevant historical knowledge deployed: this may be generalised and patchy. There may be inaccuracies and irrelevant material Some accurate use of relevant historical terminology but often inaccurate/ inappropriate use Often unclear and disorganized sections; writing will often be basic and there may be some illegibility and weak prose where the sense is not clear or obvious

3

General and sometimes inaccurate understanding of key concepts relevant to analysis and of concepts relevant to the topic General or weak understanding of the significance of most relevant issues in their historical context

3

Limited attempt to use the sources or discriminate between them; they are discussed sequentially Sources will be used for reference and illustration of an argument

11-13

Mainly description with limited comment on the context of the question Little effective analysis of how far the sources support the interpretation

6-7 Level VI

Use of relevant evidence will be limited; there will be much irrelevance and inaccuracy Answer may have

little organisation or structure

Weak use of English and poor organisation

2

Very little understanding of key concepts.

No explanation. Assertion, description

/ narrative predominate

2

Weak application of the sources to the question

Weak attempt at analysis

5-10

Weak contextual knowledge

Mainly description with weak evaluation of the historical interpretation

3-5 Level VII

No understanding of the topic or of the question’s requirements; little relevant and accurate knowledge Very fragmentary and disorganised response; very poor use of English and some incoherence

0-1

No understanding of key concepts Weak explanation,

assertion, description / narrative

0-1

Very weak application of the sources to the question

No attempt at analysis

0-4

Very weak attempt at evaluating the historical interpretation

Heavily descriptive No contextual

knowledge

0-2

9

Page 13: History A Mark Schemes for the Units January 2010 - The ...

Marking Instructions

F966 Maximum mark 120 for this unit. 2 answers: Each maximum mark 60

A01a A01b

IA 18-20 36-40

IB 16-17 32-35

II 14-15 28-31

III 12-13 24-27

IV 10-11 20-23

V 8-9 16-19

VI 4-7 8-15

VII 0-3 0-7

Notes:

(i) Allocate marks to the most appropriate level for each AO.

(ii) If several marks are available in a box, work from the top mark down until the best fit has been found.

(iii) Many answers will not fall at the same level for each AO.

(iv) Candidates will demonstrate synoptic skills by drawing together appropriate techniques, knowledge and understanding to evaluate developments over the whole of the period

10

Page 14: History A Mark Schemes for the Units January 2010 - The ...

Marking Instructions

AOs AO1a AO1b

Total mark for each question = 60

Recall, select and deploy historical knowledge appropriately, and communicate knowledge and understanding of history in a clear and effective manner.

Demonstrate understanding of the past through explanation, analysis and arriving at substantiated judgements of: - key concepts such as causation, consequence, continuity, change and significance within an historical context; - the relationships between key features and characteristics of the periods studied

Level IA

• Uses a wide range of accurate and relevant evidence • Accurate and confident use of appropriate historical terminology • Answer is clearly structured and coherent; communicates accurately and legibly. 18-20

• Excellent understanding of key concepts (eg. continuity and change) relevant to analysis in their historical context • Excellent synthesis and synoptic assessment • Answer is consistently and relevantly analytical with developed explanations and supported judgements • May make unexpected but substantiated connections over the whole period 36-40

Level IB

• Uses accurate and relevant evidence • Accurate use of a range of appropriate historical terminology • Answer is clearly structured and mostly coherent; communicates accurately and legibly

16-17

• Very good level of understanding of key concepts (eg. continuity and change) in their historical context. • Answer is consistently focused on the question set • Very good level of explanation/ analysis, and provides supported judgements. • Very good synthesis and synoptic assessment of the whole period 32-35

Level II

• Uses mostly accurate and relevant evidence • Generally accurate use of historical terminology • Answer is structured and mostly coherent; writing is legible and communication is generally clear 14-15

• Good level of understanding of key concepts (eg. continuity and change) in their historical context • Good explanation/ analysis but overall judgements may be uneven • Answer is focused on the issues in the question set • Good synthesis and assessment of developments over most of the period 28-31

11

Page 15: History A Mark Schemes for the Units January 2010 - The ...

Marking Instructions

AOs

AO1a AO1b

Level III

• Uses relevant evidence but there may be some inaccuracy • Answer includes relevant historical terminology but this may not be extensive or always accurately used • Most of the answer is structured and coherent; writing is legible and communication is generally clear 12-13

• Shows a sound understanding of key concepts, especially continuity and change, in their historical context • Most of the answer is focused on the question set • Answers may be a mixture of analysis and explanation but also description and narrative, but there may also be some uneven overall judgements; OR answers may provide more consistent analysis but the quality will be uneven and its support often general or thin • Answer assesses relevant factors but provides only a limited synthesis of developments over most of the period 24-27

Level IV

• There is deployment of relevant knowledge but level/ accuracy will vary. • Some unclear and/or underdeveloped and/or disorganised sections • Mostly satisfactory level of communication 10-11

• Satisfactory understanding of key concepts (eg. continuity and change) in their historical context • Satisfactory focus on the question set • Answer may be largely descriptive/ narrative of events, and links between this and analytical comments will typically be weak or unexplained • Makes limited synoptic judgements about developments over only part of the period 20-23

Level V

• General and basic historical knowledge but also some irrelevant and inaccurate material • Often unclear and disorganised sections • Adequate level of communication but some weak prose passages 8-9

• General understanding of key concepts (eg. continuity and change) in their historical context • Some understanding of the question but answers may focus on the topic and not address the question set OR provides an answer based on generalisation • Attempts an explanation but often general coupled with assertion, description / narrative • Very little synthesis or analysis and only part(s) of the period will be covered 16-19

12

Page 16: History A Mark Schemes for the Units January 2010 - The ...

Marking Instructions

13

AOs

AO1a AO1b

Level VI • Use of relevant evidence will be limited; there will be much irrelevance and inaccuracy • Answers may have little organisation or structure • Weak use of English

4-7

• Very little understanding of key concepts (eg. continuity and change) in their historical context • Limited perhaps brief explanation • Mainly assertion, description / narrative • Some understanding of the topic but not the question’s requirements 8-15

Level VII • Little relevant or accurate Knowledge • Very fragmentary and disorganised response • Very poor use of English and some incoherence 0-3

• Weak understanding of key concepts (eg. continuity and change) in their historical context • No explanation • Assertion, description / narrative predominate • Weak understanding of the topic or of the question’s requirements 0-7

Page 17: History A Mark Schemes for the Units January 2010 - The ...

F961 Mark Scheme January 2010

F961 British History Period studies

Option A: Medieval and Early Modern England 1035-1642 From Anglo-Saxon England to Norman England 1035-1087

1 How far was Edward the Confessor’s personality the most important cause of the

problems he faced as king of England?

No set answer is looked for but candidates will need to address the question. Candidates should consider a range of reasons for the problems and at the top levels evaluate their relative importance in causing the problems. It might be said that Edward lacked the strong qualities to make an effective king. His piety and artistic interests were admired but for his ideals rather than their practical relevance to kingship. Better answers may start by identifying the problems that Edward faced and this may include issues such as his lack of knowledge of the country, his upbringing, the power of the Godwin family, the problems created by his marriage to Edith, his support base and the problem of the lack of an heir. It is likely that candidates may suggest that the power of the Godwin family played a large role in causing many of the problems as Edward was heavily dependent on them, some may use their exile to show the power they had. The power of the Godwins may be linked to many of the problems and this may be an approach taken by those reaching the higher levels-for example his upbringing meant that he was even more dependent upon the support of the Godwin’s and therefore it might be argued that his marriage to Edith was almost inevitable and that this created further problems and may even have led to the succession crisis at the end of his reign. There may be some consideration of the problems that followed from his continuing patronage of Normans. There might be mention of foreign dangers, especially from Scandinavia. A king’s powers were limited and he needed to be able to implement whatever authority he possessed.

2 ‘Military factors were the most important reason for William of Normandy’s success

at the Battle of Hastings.’ How far do you agree?

No set answer is looked for but candidates will need to address the question. Military factors is a wide ranging term and might include issues such as tactics, forces and weaponry available, military leadership and the previous invasion by Harald. If candidates use the term to encompass all of these they may find it difficult to consider other issues, however issues such as luck for William or misfortune for Harold may be considered as candidates might point to the timing of Harald’s invasion and the impact it had on Harold, particularly following the changing direction of the wind, which allowed William to invade. Some may consider the mistakes made by Harold as more important, suggesting that if he had not rushed back from the north and waited until he had rested and had a full force he might have won, given how close Hastings was, even with such a depleted force. Some answers might also consider religious factors and argue that it was only with papal blessing that William was able to gather a large enough force to be able to make the challenge.

14

Page 18: History A Mark Schemes for the Units January 2010 - The ...

F961 Mark Scheme January 2010

3 To what extent did William I change the government and administration of England?

No set answer is looked for but candidates will need to address the question. There is a wide range of material available for candidates to consider. Some may consider the issue of personnel and the fate of the Anglo-Saxon earls and their replacement by Normans. However, some answers may focus on the nature and methods of government and this may result in consideration of the use of the feudal system, but it must be linked to methods of government. There may be some consideration of the nature of the monarchy and candidates might consider the use made of crown wearing sessions. The personal rule of the monarchy became more important. Writs were used, a legacy of Anglo-Saxon government, although they were not usually in English and they were used more frequently to enforce William’s orders. Sheriffs and shire courts were continued but sheriffs were evidently more important as royal officials.

15

Page 19: History A Mark Schemes for the Units January 2010 - The ...

F961 Mark Scheme January 2010

Lancastrians, Yorkists and Tudors 1450-1509 4 How successful was Richard III’s government of England?

No set answer is looked for but candidates will need to address the question. Some may argue that Richard was not successful in his government of England because he lost at Bosworth, however this point needs to be fully explained if it is to be relevant. Candidates could point to his failure to win the support of the nobility and that his government was over-reliant on northern nobility at the expense of the southerners. Answers may also consider his relationship with parliament and the issue of finances, this may lead to a discussion of benevolences and candidates might discuss whether his approach was successful. Some may argue that his government was not successful as he did not possess a wide enough basis of power because of the nature of his accession and therefore lacked sufficient patronage. There may be some consideration of the nature of his accession, but this needs to be linked to the question.

5 How effectively did Henry VII deal with England’s domestic problems?

No set answer is looked for but candidates will need to address the question. It is likely that better answers will identify the domestic problems that Henry faced and then consider how well he was able to deal with them. Many answers are likely to focus on the problem of the Yorkist challenge, particularly Simnel and Warbeck. Candidates may argue that these were dealt with successfully as both were defeated and some may also argue that the threat was reduced by his marriage to Elizabeth of York. Candidates may also consider the problem of the nobility, some may conclude that Henry’s policy was successful as he prevented the emergence of over mighty subjects and through his policy of bonds and recognisances was able to reduce their power, but at the same time win loyalty through such methods as the Order of the Garter. However, others might argue that his last years were so oppressive that the country was close to civil war. The question of the succession might also be discussed and some may argue that initially this was successful with Arthur and Henry, but that Arthur’s death left the succession hanging by a thread. There might be some consideration of the financial problems and how well they were solved.

6 ‘Marriage agreements were the most important achievement of Henry VII’s foreign

policy.’ How far do you agree?

No set answer is looked for but candidates will need to address the question. Candidates should consider a range of achievements and evaluate their relative importance in order to access the higher levels. Some may argue that the marriages were important as they brought him European recognition, which was important because of his weak claim, and also gave him an ally with the most powerful nation-Spain. It might also be argued that the marriage of Margaret to James brought at least short term peace with Scotland and also removed the potential threat of Warbeck, which was important to Henry. However, others may consider that his most important achievement was achieving security from the Yorkist threat, although it might be argued that this was only achieved at the end of the period. There might be some consideration of how successful he was in dealing with the threat presented by Margaret of Burgundy. Some might argue that financial gain was the most important achievement given the nature of his finances and use the French pension to support this and also the development and protection of trade, although the latter issue can be debated.

16

Page 20: History A Mark Schemes for the Units January 2010 - The ...

F961 Mark Scheme January 2010

Henry VIII to Mary I 1509-1558 7 How successful were Wolsey’s domestic policies?

No set answer is looked for but candidates will need to address the question. There is a wide range of domestic policies that candidates might consider and it is not expected that candidates should cover all; it is the quality of analysis that matters. However, candidates should cover a range of areas and this might include legal, financial, social and economic aspects and the church. Some candidates might establish criteria against which to judge success and this could include pleasing Henry so as to remain in power, gaining personal wealth and prestige or improving the government of the country. It is possible that candidates will argue that his legal reforms were the most successful and point to the increase in cases and the availability of justice for all. However, it is possible to argue that his financial reforms were successful, particularly in the early years and candidates may use the example of the subsidy to support this and the funding of Henry’s foreign policy, which won Wolsey support. However, if this line is taken it can be balanced by consideration of the Amicable Grant. In discussing social and economic policies candidates might focus on the issue of enclosure and argue that in the short term it appeared to be successful, but had to be abandoned because of financial needs. The problem of the church may figure in some essays and although some might point to his success in dissolving some monasteries others might argue that, given the power he had, this was a missed opportunity and that he even brought the church into disrepute. There might be some consideration of the divorce and his failure and the consequences, but this should not dominate the answer.

8 How far did Tudor government and administration change in the 1530s?

No set answer is looked for but candidates will need to address the question. There is a wide range of issues that candidates might consider and it should not be expected that all will be addressed, what matters is the quality of analysis, although examiners should expect to see a range. There may be consideration of the changing role and regularity of parliament and its increased competence as it became involved in religious issues and some might raise the issue of the importance of statute law or point to Henry’s comment about power in the time of parliament. There might be some consideration of the financial courts that were established, although it should be noted that most were short-lived. Candidates might consider the issue of Wales and the Act of Union of 1536, with the establishment of the county system etc. Some answers might raise the Elton ‘Tudor Revolution’ debate, but this is not to be expected as historiography is not a requirement at AS and examiners should also be aware of answers that simply describe the Elton thesis and do not use it to answer the question.

9 How effectively did the governments of Edward VI and Mary I deal with unrest?

No set answer is looked for but candidates will need to address the question. Candidates understanding of the idea of unrest may be a determining factor in the quality of the answer. It is likely that many answers will focus solely on the rebellions of the period: Western, Kett and Wyatt, although the Lady Jane Grey affair may also receive mention. Candidates may argue that Mary was more successful as Wyatt was defeated without battle, whereas the unrest of 1549 was at least a contributory factor in the downfall of Somerset. However, they may also argue that ultimately both the Western and Kett were crushed. Some answers may take a broader approach and consider economic and social issues, such as vagrancy or the problems created by the collapse of the cloth trade and this is acceptable.

17

Page 21: History A Mark Schemes for the Units January 2010 - The ...

F961 Mark Scheme January 2010

Church and State 1529-1589 10 How important was Thomas Cromwell in influencing religious policy in the 1530s?

No set answer is looked for but candidates will need to address the question. There are a large number of factors that candidates may consider, but it is not expected that they will consider all issues, even at the highest level. There should be some consideration of Cromwell’s role and some may argue that his more radical beliefs were important in influencing the king to dissolve the monasteries and introduce some more protestant views. However, this might be balanced against Cromwell’s desire and need to please the king, arguing that he dissolved the monasteries to make Henry’ the richest man in Christendom’. It may also be argued that Cromwell’s views were not important as once he displeased Henry and became too radical he was removed. Candidates might suggest there were other more important factors and issues such as power, money, the foreign situation and threat of a Catholic crusade and Henry’s own religious beliefs might be considered. There might also be some who argue that the condition of the church and the need to reform in response to popular pressure was important.

11 How much support was there for Protestantism in England in 1558 and 1559? No set answer is looked for but candidates will need to address the question. The focus of 1558 and 1559 is important and candidates could therefore discuss the problems Elizabeth faced in the passing of the Elizabethan Settlement in Parliament. Candidates are also likely to look at the level of support for Catholicism and/or Protestantism at the end of Mary’s reign. Some may argue that the country was largely Catholic on Mary’s death, whilst others may argue that the burnings and Marian persecution had turned England protestant, depending on their argument so they will determine the strength of Protestantism in England on Elizabeth’s accession. Candidates whose answers range back into Edward’s reign and either suggest that it was difficult for Edward to turn England protestant or that he succeeded and therefore Elizabeth had a harder/easier task can receive credit, but the focus of the essay must be on the situation in 1559.

12 How successfully did Elizabeth I deal with the Catholic challenge from 1559 to 1589? No set answer is looked for but candidates will need to address the question. Candidates may identify the nature of the Catholic challenge and consider how it changes over the period. Answers may look at the threat from home and abroad and suggest that at the start of the period it was the threat from home that was the strongest, given the strength of Catholicism. However, they may argue that Elizabeth handled this well; there was no serious unrest, the moderate nature of the settlement and her avoidance of creating martyrs. Some answers may also consider how well she handled the Catholic challenge of the Northern Earls. There may also be consideration of her policy towards seminary priests and again the avoidance of creating martyrs, but executing for treason. The handling of the foreign threat might include her ambivalent policy in the early years, although some may argue that Philip needed her support just as much. There may be some who argue that her policy towards the end of the period was less successful as she provoked war with Spain and this could have led to Catholics at home rising.

18

Page 22: History A Mark Schemes for the Units January 2010 - The ...

F961 Mark Scheme January 2010

England under Elizabeth I 1558-1603 13 How important was the Privy Council in the government of England during the reign

of Elizabeth I?

No set answer is looked for but candidates will need to address the question. Candidates will need to consider the role of the Privy Council in the government of England, but this should be weighed up against other elements of government in order to reach a judgement about its relative importance. It is likely that many will write in greater depth about the role of parliament and some may be sidetracked in to the debate about parliament. Better answers might also consider local government and the role of JPs etc.

14 ‘Inflation was the most serious financial problem facing Elizabeth I and her

government’. How far do you agree?

No set answer is looked for but candidates will need to address the question. There is a range of financial problems that candidates can consider, but they must give due attention to the named factor, even if they argue that it was not the most important. Inflation had a major impact on crown revenue and impacted on the cost of warfare, which would be a major item of expenditure at the end of the period. Inflation also had an impact on taxation returns, although some may suggest that it was Elizabeth’s failure to update assessments that was the bigger problem. Some may consider the problem of crown expenditure and selling of crown lands, others may look at customs or monopolies as issues.

15 How successful was Elizabeth I in dealing with the issue of the succession during

her reign?

No set answer is looked for but candidates will need to address the question. The issue of the succession concerned many, but Elizabeth did not want the matter discussed by parliament and did not want to name a successor. Candidates might argue that parliament did try to discuss the issue, but were largely unsuccessful in getting any answer from Elizabeth. Some answers might argue that Elizabeth handled the situation very well, given the fact she was seen as illegitimate by some. They may point to her handling of the issue of Mary Queen of Scots, who was the potential heir, but by not naming her it discouraged attempts to hasten her accession. Elizabeth was also masterful in exploiting her position as the ‘Virgin Queen’ and candidates might consider the various marriage proposals and how well they were handled and exploited by her. There might be some consideration of the last years and the position of James VI.

19

Page 23: History A Mark Schemes for the Units January 2010 - The ...

F961 Mark Scheme January 2010

The Early Stuarts and the Origins of the Civil War 1603-1642 16 Assess the reasons why financial issues caused conflict between James I and his

parliaments.

No set answer is looked for but candidates will need to address the question. There is a range of issues that candidates can consider and it is not expected that all areas will be covered, what is important is the quality of analysis. Some answers may identify the financial problems that James faced, such as the inherited debt and the inadequacy of royal finances and suggest that it was the scale of the problem that was the major issue. However, others may suggest that it was James’ extravagance, particularly money spent at court or on royal favourites that caused conflict. There may be some consideration of foreign policy and the differing views of James and parliament and this can be linked to financial clashes. Some answers might argue that parliament used the issue of supply to try to obtain redress of grievances, whilst others may suggest that there was a lack of trust between the two, shown in the failure of the Great Contract. Issues such as monopolies and impositions may also receive consideration.

17 ‘Charles I desire for financial independence from parliament was the most important

reason for the establishment of personal rule in 1629’. How far do you agree?

No set answer is looked for but candidates will need to address the question. The focus of the question should be on the establishment of Personal rule and candidates who write about the nature of rule should not receive high credit. Candidates will need to focus on Charles’ aims and problems in the period from 1625 to 1629 to be able to address fully the demands of the question. Answers may consider the problematic relationship between Charles and his parliaments in this period and suggest that he wanted political independence and link this to his belief in Divine Right or even suggest that parliament was not a permanent part of the constitution. Some answers will look at other areas of conflict, such as foreign affairs or his relationship with Buckingham and again may argue that Charles wanted to avoid criticism and prevent parliament from linking supply to redress of grievance. Charles’ attitude towards parliament and his view of their role is also an area that might be considered.

18 To what extent was Charles I personal rule the most important cause of the outbreak

of Civil War in England in 1642?

No set answer is looked for but candidates will need to address the question. The question invites candidates to weigh up a range of factors causing the civil war. Those who focus on personal rule may consider the impact of the financial and religious policies of Charles and the growing opposition generated by polices such as Thorough and the fear it created. However, this may be balanced against the lack of a united opposition or the lack of a royalist party, suggesting that at this stage there could not be a war. Candidates who argue that the causes were short term will focus on developments during the period 1640-2. Some may argue that the war was unlikely in the summer of 1641 as Charles had compromised, others may suggest that the Grand Remonstrance was the turning point, others may suggest it was attempted arrest of the Five MP’s, whilst others may suggest it was either parliament taking control of the army or the Nineteen Propositions. There is a great deal that candidates could consider and it is not expected that all issues will be looked at, what matters is the quality of analysis.

20

Page 24: History A Mark Schemes for the Units January 2010 - The ...

F961 Mark Scheme January 2010

Option B: Modern 1783-1994

From Pitt to Peel 1783-1846

1 To what extent was the support of the crown the most important reason for Pitt’s domination of politics from 1783 to 1793?

No set answer is looked for but candidates will need to address the question. Candidates will need to write a good paragraph on the named factor if they are to access the higher levels, even if they conclude that it was not the most important reason. Candidates will need to assess the role of George III in ensuring the survival of Pitt’s ministry, particularly in the 1784 election and the Regency Crisis of 1788 and then attempt to evaluate this against other factors if they are to achieve the higher levels. The king played an important role through patronage, control of the frequency of elections and his own distaste for the Whigs, particularly Fox and North. The partnership between Pitt and George should not be understated. It might also be noted that once the king withdrew support Pitt soon fell. Other factors for survival could include Pitt’s successful domestic policy in the 1780s, which saw economic and financial recovery and his use of repression in the 1790s, the division and weakness of the Whig opposition, made worse by splits over the French Revolution and Pitt’s mastery of parliamentary business and debate.

2 How far would you agree that the Conservative party was more liberal from 1822 to 1830 than from 1812 to 1822?

No set answer is looked for but candidates will need to address the question. The question of how liberal the Tories were in the period after 1822 remains an open one, but most are likely to argue that they were more liberal after 1822 than before. To support the view candidates could use: the appointment of younger politicians such as Canning, Peel and Huskinsson after 1822-3, the economic legislation passed by both Robinson and Huskinsson and the reforms of Peel at the Home Office. However candidates might argue that there were also some illiberal measures such as the refusal to accept Roman Catholic Emancipation, the fact that the repeal of the Test and Corporation Acts were forced upon them, and the refusal to entertain the issue of parliamentary reform. These issues should be contrasted with the measure of the earlier period in order to reach a conclusion. Candidates are likely to argue that even if the later period was not that liberal, the earlier period was repressive and point to the Corn Laws, the Suspension of Habeas Corpus, the Six Acts and Seditious Meetings. However, this might be balanced against the need to tackle unrest and that the acts were no more repressive than Pitts. There might also be some consideration of the more liberal reforms of the earlier period.

3 How successful was Peel’s leadership of the Conservative party to 1846?

No set answer is looked for but candidates will need to address the question. Answers must focus on Peel as the party leader and not simply examine his reforms in the ministry of 1841-6, although some of these might be used to show he was not a great party leader. Some may argue that in the early years he was a good party leader as he reorganised the party after the disasters of the Great Reform Act and with his Tamworth Manifesto adapted the party to a changed set of electoral conditions. There might also be consideration of the reforms at the centre with reorganisation, the creation of the Carlton Club and Registration issues. Peel’s attempts to broaden the appeal of the party might also be discussed and candidates might be aware that the 1841 election results suggest he failed in this aspect and was returned to power on traditional Tory votes. There might be some discussion about the significance of the 100 days and also his support for some Whig measures to argue that he had shown the party was responsible and fit to govern. It is likely that many will consider his treatment of backbenchers once he was in power and his belief that it was his duty to serve the nation and monarch not the party. This might result in some discussion of his abandonment of key Tory ideas over protection and issues in Ireland. Many are likely to suggest that his action over the Corn Laws suggests he was a poor party leader as he split the party and the result was years in the political wilderness. Some might argue that it was not Peel’s successes that brought the Conservatives back into power but the mistakes and failings of the Whigs

21

Page 25: History A Mark Schemes for the Units January 2010 - The ...

F961 Mark Scheme January 2010

Liberals and Conservatives 1846-1895 4 How important was the influence of Gladstone in the emergence of the Liberal party

by 1868?

No set answer is looked for but candidates will need to address the question. Candidates are likely to focus on how Gladstone contributed to the key issues in Liberal development such as party development, winning elections, party cohesion and winning key electoral groups. Gladstone was able to identify the party with an attractive financial and economic package of low taxation and free trade in his budgets. Some may argue that it was Palmerston who founded the Liberal party in 1859 over the Italian principle and also note that Gladstone was absent from the Willis Rooms meeting. However, some might argue that Gladstone was important as he created a liberal press by his repeal of the Paper Duties; he made contacts with the Trade Unions and forged contacts with the radicals. These issues will be balanced against other factors, such as the split in the Conservative party on the role of the Peelites. There might be consideration of the roles of radicals such as Bright and Cobden, unity over Italy, common support for free trade, trust in the party over finance and the abandonment of the aristocratic Whiggish image associated with Grey and Melbourne.

5 ‘The loss of working class support was the most important reason for the defeat of

the Liberal party in the 1874 election.’ How far do you agree?

No set answer is looked for but candidates will need to address the question. Candidates will need to write a good paragraph on the named factor if they are to access the higher levels, even if they conclude that it was not the most important reason. Candidates will need to explain how, despite a generally good press for Gladstone’s legislative achievements, he lost the 1874 election. They must consider the loss of working class support, even if they conclude it was the not the most important reason. In looking at this factor they might include artisan and working class disapproval of Trade Union legislation that was repressive on peaceful picketing and acts in restraint of trade as well as resentment over the Licensing Act. They may assess the fall out from many of his reforms – The Whig upper class was unhappy over the Irish reform, particularly land, they also disliked the Abolition of the Purchase of Commissions in the Army and exams in the Civil Service. They might consider the importance of the non-conformists, unhappy with Forster’s Education Act which implicitly sided with the Anglicans. The administrative reforms and increased efficiency, which the government achieved, would hardly bring votes. Candidates might also consider the impact of an ‘apparently weak foreign policy’, Gladstone losing steam and a faltering leadership in 1873/4 and the failure to find a rallying cry beyond income tax.

22

Page 26: History A Mark Schemes for the Units January 2010 - The ...

F961 Mark Scheme January 2010

6 To what extent did Disraeli’s ministries of 1867 and 1874 - 1880 follow the ideas of Tory democracy?

No set answer is looked for but candidates will need to address the question. Candidates will need to show an understanding of the concept of Tory democracy – knitting up the social divisions between rich and poor by paternalistic social reform, cementing an upper and working class bond at the expense of the middle classes. In discussing this issue candidates might consider the Second Reform Act. There might be some mention of the origins of Tory democracy in his novels and Young England to suggest that he was likely to follow such a policy, although some may argue these were simply idealistic. In order to support the argument candidates might make reference to his speeches at Crystal Palace and Manchester, from the reforms themselves, Artisan Dwellings Act, Trade Union Act, Public Health Act, Food and Drink, Pollution and Merchant Shipping. In considering these acts candidates might discuss their intentions and practice to show whether they did uphold the ideals. Against the argument candidates might suggest that such reforms were already in the pipeline, that they built on existing practice, they simply followed a liberal framework, that developments in technology made such developments possible, that their impact was limited and that Tory Democracy was a later phrase which made little sense electorally.

23

Page 27: History A Mark Schemes for the Units January 2010 - The ...

F961 Mark Scheme January 2010

Foreign and Imperial Policies 1856-1914 7 To what extent was the maintenance of the balance of power the most important

factor influencing British foreign policy from 1856 to 1902?

No set answer is looked for but candidates will need to address the question. Candidates will need to write a good paragraph on the named factor if they are to access the higher levels, even if they conclude that it was not the most important reason. There are a number of factors that candidates might consider when addressing the question and examiners should not expect them all to be discussed, what is important is the quality of analysis. Some might argue that although the balance of power was an important issue the countries that threatened it changed from Russia to Germany. This issue might also be linked to trade and the need to preserve trade routes, particularly to India and how this impacted on relations with Russia. This might also be linked to imperial concerns over India and therefore the issue of the Mediterranean and the Suez Canal might feature. Issues of naval supremacy and ‘blue water’ might also be considered and the question of the two power principle. There might be some consideration of the importance of support for nationalist movements and Britain’s relationship with Italy, Germany and Poland in this period might be considered.

8 How far did support for imperialism decline from 1880 to 1902?

No set answer is looked for but candidates will need to address the question. Examiners need to ensure that candidates focus on ‘how far’ and do not simply assume that it did decline and assess ‘why’. Some answers might challenge the assumption that it did decline and point to the positive images of the ultimate victory or the result of the Khaki election. It is possible to argue that victory in the Boer War in 1902 only reinforced an image of invincibility and there was no decline and this can be seen in the popular jingoism of the music hall and added to the ideal that the ‘sun never sets ‘ on the British Empire. They might also point to the celebrations of Queen Victoria. However, this might be balanced against the negative reaction to the Boer War and some may suggest that it was a turning point as a small force had caused such difficulty for the might of the Empire; mention might also be made of the brutal use of concentration camps that damaged the reputation of imperialism. This can be supported by reference to the questioning of the wisdom of Chamberlain’s imperial vision and the social and medical problems highlighted by the Boer War caused some to argue that Britain should concentrate on domestic reform.

9 Assess the reasons why Britain’s attitude to major European powers changed from

1902.

No set answer is looked for but candidates will need to address the question. Candidates might argue that this was due to the resolution of areas of dispute with France, particularly in colonial issues following the Fashoda incident. They may argue that this led to the Entente Cordiale and some might develop this and suggest that relations with Germany changed following the Entente as Germany feared what might have been agreed. However, some candidates might suggest that it was the growing power and fear of Germany that caused Britain’s attitude to change; there might be reference to the development of the German navy, her economic growth or support for the Boers. Some candidates might explain how these developments, particularly after the Anglo Japanese alliance of 1902 encouraged an improvement in relations with Russia. There might be some candidates who argue that Britain’s attitude to European countries did not change and argue that the Entente did not commit Britain to war, that the agreements made followed on from the earlier Mediterranean agreements or that Britain still followed a policy of splendid isolation: this approach is valid and should be credited accordingly. Some might argue that attitudes did not change and that Britain was just concerned to avoid war and that it was the means that changed.

24

Page 28: History A Mark Schemes for the Units January 2010 - The ...

F961 Mark Scheme January 2010

Domestic Issues 1918-1951 10 How successful was the Conservative party from 1918 to 1929?

No set answer is looked for but candidates will need to address the question. Some candidates might argue that the party was very successful and point to the electoral recovery after 1906. It can be argued that they dominated the period electorally: the Coupon election and resulted in Conservative dominance, they won in 1922, were still the largest party in 1924 and won the subsequent election. However, some might balance this against the importance of Lloyd George in their victory of 1918, the party ‘split’ in 1924 and the subsequent Second Eleven Cabinet and the success of Labour in 1929. Some might debate how well the Conservatives handled the major issues such as the General Strike and social issues or they might consider whether Baldwin was successful in creating a new Conservatism, which resulted in electoral dominance throughout this period and up to 1945. It might be argued that Baldwin was able to heal the divides in society that had been created by the General Strike.

11 ‘Poor Trade Union leadership was the most important reason for the failure of the General Strike.’ How far do you agree? No set answer is looked for but candidates will need to address the question. There are a number of reasons that candidates might assess in order to decide the most important reason for failure. Candidates will need to write a good paragraph on the named factor if they are to access the higher levels, even if they conclude that it was not the most important reason. They might argue that the Unions were reluctant to embark on a General Strike and did so in light of their apparent weak actions on Black Friday. They might link this to the shortness of the General Strike and why the miners were abandoned so quickly. This might lead them to argue that they were pushed into it by the Coal Unions. On the other hand some might argue that the government was well prepared for the strike, having stockpiled coal and were aided by the timing. They might also point to the government’s ability to win the propaganda war and the role of Churchill and the British Gazette in this. Some might consider the reaction of a section of the public who were willing to help and enjoyed the opportunities the strike presented.

12 How successful were the Labour governments’ reforms of 1945-51 in improving social and economic conditions? No set answer is looked for but candidates will need to address the question. The approach to the question might depend on how success is judged; is it seen against the desires of many committed socialists or against the problems and difficulties they faced. It might be argued that Attlee’s government accomplished a modest redistribution of wealth through fiscal policy and therefore improved social conditions, there was also full employment and improved living standards for the working class this could be contrasted with the feeling of relative deprivation among the middle class. It might be argued that they suffered more austerity with food shortages and rationing than during the war. In considering social conditions it is likely that many will focus on issues such as the NHS and education opportunities. In considering social conditions candidates might refer to the National Insurance Act, the industrial Injuries Act and National Assistance Act and Family Allowance Act. Some historians have argued that these measures were so significant that they should be seen as achieving a social revolution, suggesting they were a success. There might also be a discussion as to whether the nationalisation programme was successful in improving economic conditions. However, this might be balanced against the economic problems they faced such as debts and argue that even though there was some recovery it was not enough to meet expectations and link this to the dollar gap and the defence spending associated with the Cold War. Some may argue that the government lacked any carefully thought-out plans of social reconstruction.

25

Page 29: History A Mark Schemes for the Units January 2010 - The ...

F961 Mark Scheme January 2010

Foreign and Imperial Policies 1945-1990 13 How far did British foreign policy change from 1945 to 1964?

No set answer is looked for but candidates will need to address the question. There are a large number of areas and issues that candidates might consider, but it is not expected that they will deal with all, what matters is the quality of the analysis. Candidates might choose to approach this by looking at either themes or their relationship with individual countries or the EEC. If they take the latter approach it is likely that they will focus on Britain’s relationship with the USA and USSR and this might be linked to the issues of the Cold War. Some might argue that there was a significant change after the war because of Britain’s financial position and point to the change seen in Greece. The development of a close relationship with the USA might be stressed, although some might argue that after Suez this did see a shift. The desire to remain a great power remained a constant and answers might mention Britain’s place on the Security Council and the desire for an independent nuclear deterrent.

14 How far was the decline of the British Empire the most important reason for the change in British attitudes towards Europe from 1945 to 1973? No set answer is looked for but candidates will need to address the question. Candidates are likely to consider a number of reasons, but in order to access the higher levels they must write at least a good paragraph on the given factor, even if they conclude it was not the most important. Candidates may consider the perception that Britain had to make a choice between the Empire and Europe and with ‘the wind of change’ shift to de-colonisation altered Conservative views. There might also be consideration of the perception that Britain had to make a choice between USA and Europe; this might also be linked to the new direction under Eden and the application to join the EEC in 1963. There might be some consideration of Heath’s attitudes. Some answers might consider the economic success of the EEC and compare this with the failure of EFTA. This might be compared with Britain’s go it alone attitude up to 1960. Many in Britain did not take European integration seriously until 1960, pointing to Britain’s world status and her desire to shape Europe rather than be shaped by it. This might be linked to distrust of the Schumann Plan and EEC.

15 To what extent did Thatcher achieve her aims in foreign policy? No set answer is looked for but candidates will need to address the question. It is likely that candidates will identify Thatcher’s aims before assessing how far she achieved them. There might be some consideration of her desire to reassert Britain’s position as a major power and this might be linked to the Falklands War, although some might balance this with the Grenada incident. There is a case that Thatcher wanted to improve relations with the US and that this was achieved through events such as the Libya bombings. Answers may discuss her aims regarding the EU, particularly the question of the budget and are likely to conclude that she achieved her aims, even if some have argued that her stance damaged Britain’s position in Europe. Thatcher wanted to see the defeat of communism and it is likely that some will argue that this was achieved and that her role was of some significance. There might be some consideration of the Rhodesian question where it might be argued that Thatcher achieved her aim of improving Britain’s position in the Commonwealth.

26

Page 30: History A Mark Schemes for the Units January 2010 - The ...

F961 Mark Scheme January 2010

Post-War Britain 1951-1994 16 ‘Labour weakness was the most important reason for Conservative dominance from

1951 to 1964’. How far do you agree?

No set answer is looked for but candidates will need to address the question. There are a large number of reasons that candidates might consider, but it is not expected that they will cover all areas, what matters is the quality of analysis. However, in order to achieve the higher levels they must consider the named factor even if they conclude it was not the most important. Candidates will need to identify the weakness of Labour opposition during the period. The weakness of Labour opposition in this period was focused on the left versus right wing debate over the future of the party. Modernisers or revisionists under Gaitskell wished to increase the private sector involvement and remove Clause 4. They were opposed by traditionalists under Bevan who wished to expand public sector involvement and oppose nuclear weapons. The divisions did not go down well with the electorate. However, it is likely that this will be contrasted with the positive elements of Conservative rule. It might be argued that prosperity made Labour disputes appear petty. Economic recovery was underway and the Conservatives were able to dismantle the apparatus of austerity and gain the credit. Politics were devised to manage this by Butler, Maudling, Powell and MacLeod. A property owning democracy had more appeal than Bevan’s expanded public sector or Gaitskell’s social democracy. The Conservatives were able to reduce taxes yet maintain and even increase social expenditure, completing the promised ‘homes’ programme. Full employment spread the gain more widely and affluence became more marked. The Conservatives timed elections well to coincide with ‘boom’ and avoided moments of disaster such as Suez or Profumo.

17 How successful was Heath as leader of the Conservative party? No set answer is looked for but candidates will need to address the question. Candidates may argue that Heath was a failure as leader and point to his defeat in 1974, when he called an election despite a large majority, and subsequent loss of the leadership of the party to Thatcher. He failed in nearly of all his aims: reducing the price rise, increasing productivity and reducing unemployment. In particular candidates might discuss his failure to deal with industrial relations, which led to his eventual downfall. It resulted in conflict with the miners and his misjudgement in calling an early election resulted in defeat. Some might argue that he failed to carry through his tough programme of economic and industrial reform on which the party had won the election and therefore should be judged to have failed and these were the grounds of criticism from the Thatcherite wing of the party, even their emergence might be used to argue he failed. There might be consideration of the number of U turns he made and the weak image that this created among the electorate. However, some might argue that he should not be judged as failing in his leadership, but these problems need to be seen in the context of the extraordinary and unstable domestic and international problems with which he was faced.

27

Page 31: History A Mark Schemes for the Units January 2010 - The ...

F961 Mark Scheme January 2010

28

18 Assess the reasons why governments were unable to solve the Irish problem in the period to 1994. No set answer is looked for but candidates will need to address the question. Answers may start by identifying the problems faced by the governments and this might include issues such as the divisions, the growth of terrorism, economic discrimination, the use of violence, the development of paramilitary groups and perceptions of the police and army. Although the topic starts in 1951 some answers might place the problems in context of earlier developments and this can be credited provided the main focus is on the period from 1951. Candidates might consider the problem of the division between Nationalists and Loyalists and the emergence of more extreme forms within the period which resorted to more violent approaches. The sizeable proportion of the population, about 1/3, who felt resentment against the government made the problems more difficult to resolve. Candidates might examine the importance of the events of 1968-9 in exacerbating the problems and they might also argue that the issues had largely been ignored until then and that this had matters worse. There might be an examination of the issues of discrimination in policing, social and economic areas and the anti-Catholic nature of the Unionist majority that added to the difficulties and this might be linked to the problem of a Unionist dominated government. Answers are likely to consider the role of the IRA and the emergence of the Provisional IRA following the split in 1969. Attitudes towards the British army among many Catholics did not make the situation any easier. Policies such as internment may also have exacerbated the problem and this was added to by events such as Bloody Sunday. There might be an exploration as to why the Sunningdale Agreement and power sharing failed and this might include the change in government and a lack of decisive leadership. There might also be some consideration of international support for terrorism and reference made to the hunger strikes of the 1980s. Some candidates might also examine the problem of relationship between Britain and the Irish government, particularly when Fianna Fail was in power, particularly during the Falklands crisis. Candidates might also make reference to the political success of Sinn Fein, the divisions within the Unionist movement and the limited support among both communities for a lasting peace.

Page 32: History A Mark Schemes for the Units January 2010 - The ...

F962 Mark Scheme January 2010

F962 European and World History Period Studies

Option A: Medieval and Early Modern 1095-1609 The Crusades and Crusader States 1095-1192 1 Assess the reasons why people joined the First Crusade.

No set answer is looked for but candidates will need to address the question. This question seeks to elicit responses which assess the relative significance of the different motives that people had for joining the First Crusade. There needs to be real assessment for the top bands. Candidates may discuss religious motivation in some detail and focus in on the crusade as an ‘armed pilgrimage’ and the plenary indulgence that Pope Urban II promised crusaders. Such discussion may be balanced against other motives such as those that can be inferred from the reports of Urban’s sermon at Clermont: revenge for the atrocities committed against eastern Christians by the Turks; aid to Christians in the East; the chance of ‘righteous’ warfare; the recovery of the Holy Land (and the focus on Jerusalem that emerged as a key factor as the crusade recruitment campaign got underway). Candidates may also suggest more worldly motives: the prospect of a land of milk and honey, an escape from the hardships of life in western Christendom, the chance to carve out reputations and lands as a result of victory and conquest. Candidates may discuss motivation in relation to general groups as well as particular individuals.

2 To what extent was shortage of manpower the main problem facing the Crusader

States in the twelfth century?

No set answer is looked for but candidates will need to address the question. Candidates must deal with the question of shortage of manpower even if they wish to argue other problems were more significant. Candidates may discuss the chronic nature of the manpower problem from which the erratic and sporadic arrival of ‘crusaders’ from Western Europe at best provided temporary alleviation. Even when there were major crusades, most crusaders saw their sojourn in the Holy Land as temporary. The appeals for western aid can be used as evidence of the manpower shortage. The problem of controlling the fragile crusader states with limited manpower was one which faced every ruler in every state. Candidates may discuss the role of the military orders, the strategy of establishing defensible strongpoints manned by relatively small numbers of knights and men, the relatively small size of the total forces available to the rulers of the states, even when they came together and so on. Such discussion needs to be balanced against other problems and candidates may discuss some of the following: the divided natures of the Christian states (even accepting the nominal sovereignty of the King of Jerusalem); the long and indefensible frontiers; the hostility (and growing unity) of neighbouring states; the rivalries between states; the problems associated with governance; the rivalries between different rulers and within states between different factions; the problems associated with succession crises and so on.

29

Page 33: History A Mark Schemes for the Units January 2010 - The ...

F962 Mark Scheme January 2010

3 To what extent was the rivalry between Richard I of England and Philip II of France the main reason for the limited success of the Third Crusade?

No set answer is looked for but candidates will need to address the question. Candidates must deal with the given factor even if they wish to argue other factors were more significant. In relation to the given factor, candidates may refer to the background to the tension between the two monarchs, the journey to Acre, the differences that emerged there, the departure of Philip after the siege and the difficulties with the French contingent thereafter. Certainly it would be hard to argue that the divisions between the two had no impact; indeed, candidates may also argue that fears about what Philip might do in Europe whilst Richard remained in the Holy Land affected the conduct and outcome of the Crusade. Against this candidates may argue that success at Acre would not have been possible without some cooperation between the two. In discussing other reasons for the limited success of the Crusade candidates may refer to the disaster that befell Frederick Barbarossa and the disintegration of the German contingent, the parlous position in the Holy Land, the rivalries within the Christian camp at Acre over who should be King of Jerusalem, and the strengths of the opposition and Saladin in particular.

30

Page 34: History A Mark Schemes for the Units January 2010 - The ...

F962 Mark Scheme January 2010

The Renaissance from c. 1400-c. 1550 4 How important was the fall of Constantinople (1453) in the development of the

Renaissance in Italy?

No set answer is looked for but candidates will need to address the question. This is a question about the Renaissance generally and not about art specifically. Candidates must deal with the given factor even if they wish to argue other factors were more significant. In relation to the given factor, candidates may discuss the argument that after the fall of Constantinople there was a flood of Greek scholars into Europe who brought with them manuscripts of the Greek classics that were to inform much of the development of Renaissance ideas. This may be countered by the suggestion that the Renaissance pre-dated 1453 and that western scholars had had access to classical learning before this date. They may argue therefore that at best the effects of the Fall of Constantinople accelerated a process that had already begun. Candidates need to address ‘How important?’ and are likely to do this not just by the above but also by setting the fall of Constantinople in the context of other factors that contributed to the development of the Renaissance, such as the cultural, economic and political conditions in 15th century Italy and in particular the significance of wealth, trade, city states (like Florence), patronage and other influences.

5 ‘Renaissance artists and architects did no more than copy classical art and

architecture.’ How far do you agree?

No set answer is looked for but candidates will need to address the question. Candidates are likely to argue against the contention in the question along the lines that, although Renaissance artists and architects did draw inspiration from the works of Rome and Greece, they developed something innovative in their various fields. They may draw on their knowledge of individual artists and architects to illustrate their argument. They may point, for example, to Brunelleschi’s marriage of classical features such as Corinthian columns and a concern with proportion with Romanesque arches and Byzantine inspired domes. In art they may point to the classical themes and the use of light and atmospheric colour that inspired much Renaissance art and the revival (by, for example, Donatello) of free-standing sculpture, but stress the development of perspective, realism and the close observation of nature that is apparent in the works of artists from Masaccio onwards. They may also discuss the differences of subject matter in Renaissance art. No specific answer is looked for but the quality of exemplar material is likely to be a key discriminator.

31

Page 35: History A Mark Schemes for the Units January 2010 - The ...

F962 Mark Scheme January 2010

6 To what extent did the Italian Renaissance influence cultural developments in northern Europe?

No set answer is looked for but candidates will need to address the question. Candidates may well argue that whilst the Italian Renaissance did influence developments in art, architecture and ideas north of the Alps, developments here were essentially distinct. In developing their ideas candidates may refer to the importance of Rome and Italy more generally as a place of pilgrimage, art and learning (with its universities) to which scholars and artists from across Europe came. They may also point to the spreading of Renaissance ideas and influences via merchants and diplomats. Candidates may also refer to the Italian influences apparent in the works of painters like Holbein and Dürer. They may also point to the influence of Renaissance humanism. To balance this, candidates may discuss the distinctiveness of developments north of the Alps such as the protestant prejudice against religious art and the development of Christian humanism that was less inspired by the examination of Greek and Roman classics and more by a concern to apply humanist ideas in a Christian context. In relation to this they may refer to the Devotio Moderna and the importance placed on the reading of scripture and the reality of religion.

32

Page 36: History A Mark Schemes for the Units January 2010 - The ...

F962 Mark Scheme January 2010

Exploration and Discovery c. 1445-c. 1545 7 How important was royal patronage in Portuguese overseas exploration in this

period?

No set answer is looked for but candidates will need to address the question. Candidates must deal with the role of royal patronage even if they wish to argue that other factors were more or as significant in explaining Portuguese overseas exploration. In relation to the given factor, candidates are likely to focus on Henry the Navigator and John II. In relation to the former, candidates may refer to: his sponsorship of voyages to explore the African coast, leading to the discovery of the Azores, the Cape Verde Islands and the coast of West Africa (a slave trading base at Lagos); and, his attracting some leading cartographers to help map the coast. In relation to the latter, John II sponsored and planned expeditions to find Prester John and a route to the Asian sources of spices. In this context, they may refer to the voyages of Bartholomew Diaz and the expedition of Covilha (overland) to India. Such discussion may be set in the context of other factors that promoted or helped develop Portuguese exploration, such as Portugal’s geographic position, its established sea-faring tradition, its relative political stability, the interest of nobles (not least in a desire to serve their rulers), the role of individuals, such as Diaz, Cabral and Da Gama, and the incentives to find gold, slaves, and spices and to find Prester John and spread Christianity.

8 ‘No more than a series of trading posts.’ How far do you agree with this view of the

Portuguese Empire?

No set answer is looked for but candidates will need to address the question. Candidates are likely to argue that to a degree the Portuguese Empire was a series of trading posts, and that trade was central to all of her empire. However, they are likely to argue also that to conclude it was ‘nothing more’ is wrong. In relation to the notion that the Empire was a series of trading posts, candidates are likely to point to the strategy of Henry the Navigator and those that followed of establishing secure bases for trade and security of trade routes along the coast of Africa and the estimated 50 forts and trading posts established between southern Africa and Japan during the 16th century. No attempt was made to colonise Africa but bases were used to exchange European goods for slaves and the raison d’être of posts from Goa to Macao was the lucrative spice trade. Against this, candidates may argue that this is not the whole picture, as in the islands of the coast of West Africa (Madeira and the Cape Verde Islands) active settlement took place and a thriving agriculture based on sugar was established. Similarly in Brazil, the east coast was settled for plantation agriculture supported by the export of slaves from Africa. What is more the capture of important trading posts like Goa, Malacca and Macao led in time to the development of colonies as from these the Portuguese could monopolise not only trade but could also plunder and tax.

33

Page 37: History A Mark Schemes for the Units January 2010 - The ...

F962 Mark Scheme January 2010

9 To what extent was the spread of Christianity Spain’s main aim in the development of its Empire?

No set answer is looked for but candidates will need to address the question. Candidates must deal with the given factor even if they wish to argue other factors were more significant. In relation to the given factor, candidates may discuss the religious motivation of royal patrons such as Isabella, the injunctions of the Pope (as in his bull of 1493 to Isabella), and the accompaniment of priests with Spanish expeditions to the New World. It would be hard to argue that religion was not an important motive as Christianity was spread with conquest. However, candidates are likely to argue that it was not the main motive and certainly not the only one. Candidates are likely to argue that the main motive was profit (gold, silver or spices) and discuss the expeditions of Cortes and Pizarro in Latin America as evidence of this. Other motives that candidates may discuss include the desire for settlement and farming, the desire for fame and reputation (not least for Cortes). Candidates should explore a range of aims and draw a reasoned conclusion as to the relative importance of the spread of Christianity.

34

Page 38: History A Mark Schemes for the Units January 2010 - The ...

F962 Mark Scheme January 2010

Spain 1469-1556 10 How successful were Ferdinand and Isabella’s religious policies?

No set answer is looked for but candidates will need to address the question. Success may be assessed in a number of ways: against aims; against results; and in the light of historical context. Candidates will need to identify and analyse Ferdinand and Isabella’s religious policies. The areas considered may include: the conquest of Granada, policies towards Muslims in Granada including the expulsion of 1502; policies towards the Catholic Church (to achieve royal domination of the Church, to access the Church’s wealth, and to reform the abuses within the Church); and the role of the Inquisition and policies towards the Jews. Candidates may argue that whilst the conquest of Granada was successful the policies towards the Muslims there had mixed success (the Morisco ‘problem’ was to remain until the 17th century, for example). They may argue also that Ferdinand and Isabella enjoyed considerable success in their policy towards the Church, effectively controlling appointments and winning considerable concessions from the Papacy in Granada and the New World. However, they may argue that Cisneros’ attempts to reform the clergy were less successful. Judgement on the work of the Inquisition, policies towards conversos and the expulsion of the Jews is likely to be one of success.

11 Assess the reasons why Charles I faced so many problems in Spain from 1516 to

1524.

No set answer is looked for but candidates will need to address the question. Candidates will need to analyse reasons and evaluate their relative significance and/or links between them. Candidates may well suggest that some problems were inherited from Ferdinand and Isabella and that others were of his own making or that his actions/inaction exacerbated the situation. Candidates are likely to discuss some or all of the following: the Communeros and Germania revolts (the latter not fully resolved until the pardon issued in 1524); the tensions between towns and grandees; the problem of raising money via the Cortes of Castile, Aragon and other provinces; the privileges of the same; the appointment of ministers; Charles’ delay in arriving and his subsequent absence; his other ambitions and commitments (and the use of Spanish resources to pursue them) and so on.

12 Assess the strengths and weaknesses of Spain in 1556.

No set answer is looked for but candidates will need to address the question. Candidates need to assess both strengths and weaknesses. In discussing strengths and weaknesses, candidates may address some of the following areas: the Church and religion; government and administration; Castile and the other provinces; relations with the nobility; finance; the impact of the New World; the impact of foreign policy; the economy. They may argue that the pattern of strengths and weaknesses is a mixed one. The failure of the Reformation to make any headway in Spain may be considered a strength – religious unity remained strong – although the problem of the moriscos remained; royal government at a local level was perhaps dependent on the nobility, but worked reasonably effectively at the centre through its councils and candidates may pay tribute to the work of Gattinara, los Cobos and the bureaucracy of letrados – the conciliar system, despite corruption worked well enough in normal circumstances, but proved less effective at times of crisis. Many may argue that finance was an area of weakness, partly because of noble exemption from taxation, but mainly because of the demands made on Castile especially by Charles’ expensive commitments outside Spain. New World bullion became more significant later in the reign, but debt was a constant feature. Many will argue that the economy was Charles’ greatest failure: heavy taxation and the failure to use New World revenues effectively distorted the economy.

35

Page 39: History A Mark Schemes for the Units January 2010 - The ...

F962 Mark Scheme January 2010

Charles V: International Relations and the Holy Roman Empire 1519-59 13 ‘Without the protection of princes, Lutheranism would not have survived.’ How far

do you agree?

No set answer is looked for but candidates will need to address the question. Candidates must deal adequately with the issue raised in the quotation even if they wish to argue that other factors were as or more significant. However, many are likely to argue a strong case in favour of the quotation, pointing to the role from early on of Frederick of Saxony, the difficulties facing Charles V in imposing his will without the support of the princes, the formation of the Schmalkaldic League and the eventual acceptance of Lutheranism in the Peace of Augsburg (cuius regio, eius religio). They may also point to the reluctance of Catholic princes to take up arms against Protestant princes. Such discussion needs to be balanced against other considerations such as: the power of Luther’s ideas; their spread (including the role of the printing press and the context of anti-papal feeling); the role of the towns, peasants, Imperial Knights; the intermittent attention Charles V was able to give to the issue given the distractions of Habsburg-Valois rivalry, his absences, his desire for religious division to be settled by a Church Council, the Ottoman threat and so on.

14 To what extent was personal rivalry the main reason for the Habsburg-Valois wars?

No set answer is looked for but candidates will need to address the question. Candidates must deal adequately with the issue of personal rivalry even if they wish to argue that other factors were as or more significant. In relation to the given factor, candidates may argue that in an age of personal monarchy, personal rivalry was bound to play a part. Certainly there was rivalry: Charles defeated Francis I in the bid to become Holy Roman Emperor and Francis would never accept that Charles was the pre-eminent ruler in Christendom. Their contrasting characters also helped to shape events. However, candidates may well argue that conflict between these rivals was more than personal. The election of Charles as Holy Roman Emperor and his interests in Italy and the Netherlands meant that strategically France felt surrounded, whilst Charles also wanted to recover his ancestral Burgundian lands and Francis Navarre. Candidates should support their arguments by reference to the developments in the struggle (including the reign of Henry II) and may refer to events in Italy, the Holy Roman Empire, relations with the Turks and England and so on to illustrate and develop their argument.

15 Assess the reasons why Charles V was unable to remove the Ottoman threat.

No set answer is looked for but candidates will need to address the question. Candidates must identify and analyse a range of reasons and evaluate their relative significance and/or linkages. Candidates are likely to discuss the Ottoman threat in relation both to the Holy Roman Empire and Habsburg lands in Austria, Bohemia and Hungary, and to the Mediterranean. They may point to the aggressive and expansionist nature of the Ottoman Empire (and refer to the actions of Barbarossa and Dragut in the Mediterranean as well as the opening up the Balkans by the Ottoman army’s victory at Belgrade in 1521). They may also discuss the impact of the different commitments that Charles V’s vast territories imposed upon him, that meant he could never focus on one issue for long. They may also argue in relation to that that France was willing to use the Ottoman threat as a weapon in its wars with Charles. They may also argue that the resources available to Charles were limited in a number of ways (costs; no effective navy to counter the naval forces available to the Turks; the refusal of the German army to cross the frontier into Hungary) and that (as with the Turks) distance also limited what could be achieved.

36

Page 40: History A Mark Schemes for the Units January 2010 - The ...

F962 Mark Scheme January 2010

37

Philip II, Spain and the Netherlands, 1556-1609 16 ‘The most serious problems Philip II faced in ruling Spain were economic and

financial.’ How far do you agree?

No set answer is looked for but candidates will need to address the question. Candidates must deal with the claim made in the quotation even if they wish to argue that other problems were as or more serious for Philip. However, many are likely to agree with the quotation arguing that insufficient finance (itself dependent on the economy) underpinned and limited his ability to deal with many other serious problems. Candidates may discuss the general inadequacy of funds and how New World bullion shipments provided at best short term relief. They may argue that there was a vicious circle of rising debt as future income was mortgaged and interest rates rose. Increases in taxation impacted on the Castilian economy and by the 1590s the strain told. Candidates may argue that financial problems were also a symptom as well as a cause of other problems. It was the strain of constant warfare that demanded increased taxation and inefficient administration meant corruption. Candidates may also discuss other problems, such as faction (Perez affair), relations with the nobility, the problem of the moriscos and conversos.

17 ‘Philip II enjoyed more success in his dealings with the Turks than with England or

France.’ How far do you agree?

No set answer is looked for but candidates will need to address the question. In assessing success, candidates may take note of aims, results and the historical context. Candidates may suggest that at best Philip had not dealt with the Turkish menace, but had contained it and secured a measure of peace by the 1580s. In reaching a judgement they may discuss the extent of the Ottoman/Corsair threat, the defeat at Djerba in 1560, the subsequent aggression of the Corsairs (even raiding Granada), the relief of Malta, the victory at Lepanto, and the armistice of 1580. Candidates may argue that Philip’s chances of success were limited by the diversion of resources to other problems (such as the Netherlands) and the different interests of the Papacy and Venice that made cooperation difficult. In relation to England, candidates may refer to marriage to Mary, attempts to woo Elizabeth, growing differences, conflict over the New World and the Netherlands and the failures of his armadas. Candidates may well judge his policy here as a failure (although England was excluded from the New World). In relation to France, candidates may discuss early fears of a Guise empire, but are likely to focus on his support for the Catholic League against Henry of Navarre. They, again, may judge his policy a failure, although in the end France remained Catholic.

18 How far was Maurice of Nassau’s leadership the main reason for the success of the

northern provinces by 1609?

No set answer is looked for but candidates will need to address the question. Candidates must deal with the role of Maurice of Nassau even if they wish to argue that other factors were as or more significant in the success of the northern provinces. Candidates may argue that Maurice’s key contribution was as a military leader who was able to deliver a series of victories to the northern provinces that made it clear that the Spanish, whilst they might retain a hold on the south could not re-take the north. Candidates may balance such discussion with consideration of other factors such as: the role of William of Orange and the failures of Spain prior to Maurice’s prominence; the role of England and France; the diversion of Spanish forces from the Netherlands against England and France in the 1580s and 90s; the financial difficulties facing Spain and the mutinies that affected their forces; the skills of Oldenbarnevelt; the divisions within the government of the southern provinces and so on.

Page 41: History A Mark Schemes for the Units January 2010 - The ...

F962 Mark Scheme January 2010

Option B: Modern 1795-2003 Napoleon, France and Europe 1795-1815 1 ‘The weaknesses of the Directory were the main reason for Napoleon’s rise to

power.’ How far do you agree? No set answer is looked for but candidates will need to address the question. Candidates must deal with the given factor even if they wish to argue other factors were more significant. In relation to the given factor, candidates may discuss: the nature of the constitution of the Directory which made it weak and increasingly dependent on the military; the internal politics of the Five Directors and their rivalries; unrest at home and the growing desire for change; and the impact of defeat against the Second Coalition. Candidates may well argue that the weakness of the Directory provided Napoleon with the opportunity to seize power, but that, on its own, it does not explain his rise. Candidates may refer to other factors, such as: Napoleon’s rise in the military and the reputation he gained from Toulon to Egypt; the significant role played by politicians like Barras; aspects of the Coup of Brumaire such as the role of Napoleon’s brother and the miscalculation of Sieyes and others who had hoped for a tame general (and here the reluctance of generals like Moreau to play the role is significant); and, of course, Napoleon’s own ambitions.

2 To what extent was Napoleon responsible for his own downfall? No set answer is looked for but candidates will need to address the question. This question focuses on the reasons for Napoleon’s downfall. Candidates must deal with the given factor even if they wish to argue other factors were more significant. The arguments for Napoleon’s own responsibility include: whilst he was a great warrior, he was not a great statesman and failed to seek a permanent settlement for Europe; the Spanish and Russian campaigns revealed the limitations of Napoleon as a grand strategist; the view that Napoleon’s abilities declined in his later years; Napoleon became predictable. Candidates may question some of these points and certainly they should discuss some of the alternative explanations, such as: the relative decline of the French army; the determined opposition of Britain, supreme at sea, critical in the Iberian peninsula and providing finance for those willing to take up arms against Napoleon; the reorganization of enemy armies in the light of French victories; the impact of Napoleon’s defeat in Russia in 1812; Napoleon’s loss of support in France; the impact of the Continental System; the drawing up of the Fourth Coalition; the significance of Leipzig and Waterloo, and so on.

38

Page 42: History A Mark Schemes for the Units January 2010 - The ...

F962 Mark Scheme January 2010

3 To what extent did Napoleon export the ideas of the French Revolution to the areas he conquered? No set answer is looked for but candidates will need to address the question. The question focuses on the impact of Napoleon’s hegemony over much of Europe for at least part of his reign. Candidates are likely to argue that the impact of French ideas varied from place to place and from time to time. They may argue that much of Napoleonic rule was pragmatic, and that although French ideas of government, principles of the Revolution and law were introduced in many areas it is difficult to discern a consistent pattern. Candidates may argue that much depended on nearness to France, the length of French influence and control, the attitudes of the local population, the differing status of conquered or occupied areas (absorbed within ‘France’ or satellite states or allies), and the exigencies of the particular time. There may be discussion of the impact of the Code Napoleon, relations with the Church, nobility and middle classes, the requirements of the French military machine and so forth.

39

Page 43: History A Mark Schemes for the Units January 2010 - The ...

F962 Mark Scheme January 2010

Monarchy, Republic and Empire: France 1814-1870

4 To what extent was Charles X responsible for his own downfall?

No set answer is looked for but candidates will need to address the question.

Candidates must deal with the issue raised in the question even if they wish to argue that other factors were as or more important in Charles X’ overthrow in 1830. In discussing Charles X’ degree of responsibility, candidates may argue that as he was king he was responsible for the ministers he appointed and the policies adopted, his association with the Ultras and the circumstances of his coronation suggested a reactionary policy from start. The policies pursued by Villele seemed to confirm this: compensation for émigrés, return of Jesuits and clerical control of education etc. Candidates may also argue that crucially Charles appointed the ultra Polignac in 1829 and then in 1830 issued the Ordinances of St Cloud at a time when the Crown’s best troops were in Algeria. On the other hand, candidates may point to Charles’ liberalization of the press on his accession, the existence of liberal opposition and press, the tradition of revolution and the return of economic crisis as key factors.

5 Assess the reasons why Louis Napoleon came to dominate the Second Republic.

No set answer is looked for but candidates will need to address the question.

Candidates will need to identify and analyse a range of reasons and evaluate their relative significance and/or linkages. Candidates may argue that there are longer term and shorter term reasons why Napoleon emerged as the dominant figure between 1848 and 1852. They may point to the longer term development of Bonapartism, Louis Napoleon’s failed expeditions of 1836 and 1840, the publication of the Extinction of Pauperism (1844), the association with Napoleon and the revival of the Napoleonic legend (return of body, and completion of the Arc de Triomphe) under Louis Philippe. In the shorter term they may point to the developments after the February Revolution in 1848: the growth of reaction, Louis Napoleon’s appeal to all classes (peasants, workers, businessmen, monarchists and ultramontane Catholics), Louis Napoleon’s alliance with conservative forces, problems in Paris, the election of November 1848, the Bonapartists’ efficient organization. Candidates may stress that Louis Napoleon’s greatest asset was his name. Some may argue that Louis Napoleon did not dominate the Second republic and that is why he abolished it.

6 Assess the reasons why Napoleon III’s foreign policies in the 1860s were unsuccessful.

No set answer is looked for but candidates will need to address the question.

Candidates will need to identify and analyse a range of reasons and evaluate their relative significance and/or linkages. In developing their answers, candidates are likely to discuss some of the following aspects of foreign policy: the attempts to secure territory along the Rhine (aired at Biarritz in 1865); attitude to the Polish Revolt in 1863; the Austro-Prussian war (1866); the Luxembourg crisis (1867); the Mexican adventure; colonial policy; the dispute over the Spanish succession and the background to the Franco-Prussian War. In discussing reasons for failure candidates may refer to: misjudgement, lack of clear aims, unrealistic ambitions and expectations, increasing isolation, pursuit of foreign aims as a means of appeasing domestic opinion (eg by keeping troops in Rome); policies which led to French isolation/loss of potential friends; the arousing of suspicions in Prussia, German states and Britain over attempts to expand French influence and territory on the Rhine; misconceived desire to offer support to Polish nationalism led to break-up of his understanding with the Tsar; the blow to French prestige when attempts to mediate in the Austro-Prussian war failed; unrealistic unilateral action against Mexico motivated by a mistaken desire for glory, economic opportunity and a desire to appease catholic opinion; being out-manoeuvred by Bismarck from 1865 onwards and so on.

40

Page 44: History A Mark Schemes for the Units January 2010 - The ...

F962 Mark Scheme January 2010

The USA in the 19th Century: Westward Expansion and Civil War 1803-c. 1890 7 How important were the policies of Federal governments in opening up the West to

settlement? No set answer is looked for but candidates will need to address the question. Candidates will need to deal with the given factor even if they wish to argue that other factors were as or more important. In assessing the importance of the policies of Federal governments, candidates may discuss some of the following areas: Federal sponsorship of exploration and surveying; Federal acquisition of territory; the organization of acquired lands into territories and states; the role of the Federal army in policing the frontier, the trails west and dealing with Native Americans; Federal sponsorship of communications (especially the trans-continental railway); Federal encouragement to settlement through legislation such as the Homestead Act. To balance such discussion candidates may argue that Federal policy often followed rather than preceded settlement and the real stimulus came from the needs of fur traders, cattlemen, farmers and miners as well as those seeking refuge, like the Mormons, from persecution, and the development of communications.

8 Assess the reasons why southern states decided to secede from the Union in 1860 and 1861. No set answer is looked for but candidates will need to address the question. Candidates need to identify and analyse reasons and evaluate their relative significance and/or linkages. Candidates may argue that the immediate cause of the first wave of secession was the election of a sectional president in Lincoln and that the stimulus to the secession of some Upper South states was the decision for war in the spring of 1861. Candidates may then go on to explore the short and longer term issues that led to secession. Central to their arguments is likely to be the issue of slavery and they may discuss some of the crises of the 1840s and 50s that made the issue of slavery and the issue of the possible westward expansion of slavery so contentious. In this context, candidates may refer to the Mexican War, Wilmot Proviso, Calhoun doctrine, the ‘Compromise’ of 1850, fugitive slaves, Dred Scott, ‘Bleeding Kansas’, the development of the Republican Party, Harper’s Ferry and so on. Candidates may also discuss the issue of States’ rights, the apparent social, cultural and economic divisions between North and South and the suspicions of ‘Slave Power conspiracy’ and ‘northern aggression’.

41

Page 45: History A Mark Schemes for the Units January 2010 - The ...

F962 Mark Scheme January 2010

9 ‘Grant was a better general than Lee.’ How far do you agree?

No set answer is looked for but candidates will need to address the question. This question requires candidates to compare the strengths and weaknesses of Grant and Lee as generals. Candidates may consider a range of issues in drawing their comparison: personal qualities, reputation, leadership, strategic thinking (eg at different levels – Grand Strategy, campaign strategy, battlefield command), relationship with political masters, use of resources, impact on morale, quality of opposition, quality of subordinates and so on. In making their analysis candidates may draw on their knowledge of specific campaigns and battles such as: in relation to Lee, Seven Days Campaign, Second Manassas, Antietam, Fredericksburg, Chancellorsville, Gettysburg, Wilderness Campaign, Petersburg; and in relation to Grant, Fort Donelson, Shiloh, Vicksburg, Lookout Mountain and Missionary Ridge, Wilderness Campaign, Petersburg, Atlanta and March through Georgia (Sherman in command). No specific answer is looked for and candidates can legitimately argue that both were great generals in their different ways.

42

Page 46: History A Mark Schemes for the Units January 2010 - The ...

F962 Mark Scheme January 2010

Peace and War: International Relations c. 1890-1941 10 To what extent was the alliance system the main cause of the First World War?

No set answer is looked for but candidates will need to address the question. Candidates will need to deal with the given factor even if they wish to argue that other factors were as or more important. In relation to the given factor, candidates may discuss the intentions and nature of the alliances that preceded the First World War. Certainly the division of Europe into two potentially hostile alliances (Ententes and Dual/Triple Alliances) could be said to make war a possibility and in the event the war between the Dual Alliance and the Entente. Candidates may, however, argue that other factors were more important and discuss the relative merits of other factors such as military and naval arms races, aggressive German foreign policy, Russia’s hopes and fears in relation to the Balkans, British and French policy, domestic problems and pressures and so on. They may discuss the significance of particular crises in contributing to making war more likely, such as the Bosnian Crisis, the Moroccan Crises and the July Crisis of 1914.

11 Assess the reasons why the First World War was not ‘over by Christmas’ 1914.

No set answer is looked for but candidates will need to address the question. Candidates will need to identify and analyse a range of reasons and evaluate their relative significance and/or links between them. Candidates may discuss the relative strengths and weaknesses of the Schlieffen Plan both conception and as it operated under Moltke. They may point to the role of the Belgians, the BEF, the ‘Miracle on the Marne’ and the subsequent ‘race to the sea’. They may also point to the relatively rapid mobilisation of the Russians and the impact of their offensives in the East. Candidates may also point to the issues of technology that affected the initial course of the war, pointing to the exposure of cavalry and infantry in attacking strong positions, the impact of disciplined rifle, machine gun and artillery fire, the difficulties of supply for a rapidly advancing army, the ‘digging in’ of defending forces and so forth. Candidates may also argue that no side had a decisive advantage in numbers, tactics or strategy.

43

Page 47: History A Mark Schemes for the Units January 2010 - The ...

F962 Mark Scheme January 2010

12 Assess the reasons why Japan followed an increasingly aggressive foreign policy in the period from 1931 to 1941.

No set answer is looked for but candidates will need to address the question. Candidates should discuss a number of reasons and assess their relative significance and/or the linkages between them. Candidates may well focus on the ambitions of Japan in China and South East Asia more generally. They may point to the aggressive foreign policy pursued as a consequence of the Depression, the growth of nationalism and historic claims on the Chinese mainland. There may be discussion of effects of the successful invasion of Manchuria, the outbreak of the Sino-Japanese war, the alliance with Germany and Italy, the Co-prosperity Sphere and the decision to attack Pearl Harbour. Discussion of Japanese motives needs to be set in the context of the attitudes of the USA, Britain and France, the weakness and failure of the League of Nations, the distractions of events in Europe, and the relative weakness of and internal divisions in China. Candidates may argue therefore that whilst there were compelling internal reasons why Japan wanted to pursue an expansive foreign policy, she was encouraged to do so by the weaknesses of her immediate opponents (notably China) and the failure of the major powers (Britain, France and the USA) to stop Japan because of their own internal problems and the developing events in Europe.

44

Page 48: History A Mark Schemes for the Units January 2010 - The ...

F962 Mark Scheme January 2010

From Autocracy to Communism: Russia 1894-1941 13 Assess the reasons for opposition and unrest in Russia from 1894 to 1905.

No set answer is looked for but candidates will need to address the question. Candidates will need to identify and analyse a range of reasons and evaluate their relative significance and/or linkages. Candidates may discuss the longer term context of opposition to the Tsars, but are likely to focus on the development of opposition and unrest after 1894. They may point to the economic causes of unrest and point to the economic downturn that provoked strikes and unrest in the period after 1900 alongside the appalling conditions for workers in Russia’s growing industries. They may also discuss the problem of land shortage for the peasants. Such discussion may be linked to the developments in political opposition through the Zemstva and in the development of Russian social Democracy, the Socialist Revolutionaries and the Union of Liberation Party. They may also point to the humiliation of defeat in the Russo-Japanese War, disillusion with Tsarism, the desire for liberal reforms, the problem of nationalities and the outbreak of the 1905 Revolution.

14 How far was Russia politically stable from 1905 to 1914?

No set answer is looked for but candidates will need to address the question. Candidates need to assess ‘How far?’. They may contrast the revolutionary crisis of 1905 with the relative quiet of the period of Stolypin’s influence and the development of renewed unrest in the few years before the First World War. Candidates may discuss the political repression that followed the October Manifesto and the Fundamental Laws and the nullifying of the potential of the Dumas as a check on Tsarism. They may point to the decline in agitation and the collapse in the membership of the RSDLP as evidence of increased stability. They may also point to Stolypin’s ‘wager on the strong’ as evidence of a different approach and the celebrations of the Romanov dynasty in 1913 of the Tsar’s popularity. They may also suggest that whilst there was comparative quiet in the period after 1906 stability was more apparent than real and that the pressures that brought about the revolutionary crisis of 1905 were still unresolved.

45

Page 49: History A Mark Schemes for the Units January 2010 - The ...

F962 Mark Scheme January 2010

15 Assess Lenin’s strengths and weaknesses as leader of Russia from 1917 to 1924. No set answer is looked for but candidates will need to address the question. Candidates must address both strengths and weaknesses. In discussing Lenin’s strengths and weaknesses candidates may refer to key decisions and developments in this period: the determination to ensure Bolshevik rule to the exclusion of other socialist parties (despite the brief flirtation with the left SRs); the ending of the war with Germany; the Civil War, reign of terror and war communism; the NEP and political repression. Candidates may discuss Lenin’s determination, his ruthlessness, his ability to force through unpopular decisions, his hard work, his ability to change policy when necessary, his commitment to the Bolshevik Party, his vision of a Communist Russia. What may be seen as strengths may also been seen as weaknesses – his unwillingness to compromise or work with other socialists, his pursuit of power (or Communist ideas depending on interpretation) whatever the cost, his decline after his strokes, and so on.

46

Page 50: History A Mark Schemes for the Units January 2010 - The ...

F962 Mark Scheme January 2010

Democracy and Dictatorship: Italy 1896-1943 16 Assess the reasons for unrest in Italy from 1896 to 1915.

No set answer is looked for but candidates will need to address the question. Candidates need to identify and analyse a number of reasons and evaluate their relative significance and/or linkages. Candidates may discuss some of the following areas: national humiliation after Adowa, famine and economic hardship; the desire for economic improvements (to working conditions); police violence; the North:South divide; the limited franchise; the growth of socialism; nationalist agitation and so on. Candidates may argue that real hardship was certainly an issue in the late 1890s when famine led to food riots. They may suggest that after 1900 much agitation was still economically based and strikes and protests aimed at better working conditions. They may suggest that whilst Giolitti’s economic policies led to economic expansion this only accentuated the divide between the industrial north and the poverty-stricken agricultural south. Candidates are likely to spend some time discussing the growth of socialism and the rise in political and economic unrest in the pre-war years, such as ‘red week’. They may also point to nationalist agitation that led to the expensive conquest of Libya in 1911-12.

17 To what extent was fear of socialism the main reason for Mussolini’s rise to power in 1922? No set answer is looked for but candidates will need to address the question. Candidates must deal with the given factor adequately even if they wish to argue that other factors were more important. In relation to the significance of fear of socialism, candidates may discuss the growth of socialism, its electoral profile and the red week and the fears aroused amongst the middle and upper classes, the Church and the establishment by the ‘red menace’. They may link such discussion to the impact of unemployment, inflation, post-war economic restructuring, problems in the countryside and the north-south divide to illustrate the potential scale of the threat. Mussolini and the Fascists were able to play on these fears and pose as the men of action - the direct action Mussolini was willing to take against strikers and communists (albeit after the main crisis had passed). Such a pose contrasted with the apparent failure of the liberal governments of Nitti and Giolitti to deal with the problems effectively. Candidates may also discuss the other weaknesses of the liberal governments (for example, the failure to gain a creditable peace settlement, the failure of transformismo), the legacy of nationalism, the ability and opportunism of Mussolini and the fascists, the attitude of the King and the establishment and the fateful decisions of 1922.

47

Page 51: History A Mark Schemes for the Units January 2010 - The ...

F962 Mark Scheme January 2010

18 ‘A dictator in name only.’ How far do you agree with this view of the extent of Mussolini’s power in Italy after 1922?

No set answer is looked for but candidates will need to address the question. Candidates should discuss the extent and nature of Mussolini’s dictatorship. Candidates may refer to some of the following in developing their argument: Mussolini’s consolidation of power after 1922 (Acerbo Law, Aventine Secession, abolition of the party system, the restrictions on the power of the monarchy, rule by decree, the fusion of state and party under the Duce). They may also refer to censorship, propaganda and other aspects of a police state (such as OVRA). To balance this they may also discuss the extent of Mussolini’s control of the party, the continued existence of the monarchy, the need to come to agreement with the Church (Concordat), the inefficiency of the Fascist state. Candidates could point to the circumstances of his overthrow in 1943 as evidence of his limitations to his power.

48

Page 52: History A Mark Schemes for the Units January 2010 - The ...

F962 Mark Scheme January 2010

The Rise of China 1911-1990 19 Assess the reasons why it took so long to establish effective government after the

1911 revolution.

No set answer is looked for but candidates will need to address the question. Candidates will need to discuss and evaluate a range of reasons. In assessing reasons, candidates may discuss some of the following: the state of China in 1911; ‘sudden’ nature of the revolution in 1911 and resulting power vacuum; ambitions of Yuan Shikai; the limited authority of government and local power/rivalries of warlords (the significance of the warlords may be stressed); the extent and nature of support for Sun Zhongshan (Sun Yat-sen) and the Nationalists (party formed only in 1912); the significance of the 4 May Movement; Sun Zhongshan (Sun Yat-sen) and the reorganization of the Guomindang; Foundation of CCP. Candidates may argue that whilst the overthrow of the Manchu dynasty met little resistance, there was no consensus about what next and that there was no one source of power able to assert its authority in the short term. Not until the 1920s were the nationalists in a position to establish their authority and this remained patchy. Some may argue that no really effective government was established until after 1949. No specific answer is being looked for.

20 Assess the reasons why the Nationalists were unable to crush the Communists in the period to 1945.

No set answer is looked for but candidates will need to address the question. Candidates need to identify and analyse a number of reasons and evaluate their relative significance and/or linkages. In discussing reasons for Nationalist failure, candidates may refer to the united front with Sun Yat-sen’s nationalists after the 4 May Movement during which time the party began to grow, the White Terror of 1927, the retreat to the countryside and the Chungkang Mountains, the development of the Red Army, the Nationalist campaigns and the resulting Long March of 1934-5, Yenan, the united front against the Japanese, and the Civil War. Candidates may discuss reasons to do with the communists: leaders like Mao, the active aid to and support from peasants, the skills of the Red army and its development of guerrilla tactics, the significance of the Long March and the honing of ideas in Yenan, the distinguished role in the war against Japan and so on. They may also discuss the failings of the Nationalists under Jiang: distraction of the warlords, corruption, failure to win over the workers and peasants (lack of support in the countryside), the loss of middle class support, poor performance in the war against the Japanese and so on. They may also discuss the fact that the Nationalists were also forced to accept communist help in the fight against the Japanese.

49

Page 53: History A Mark Schemes for the Units January 2010 - The ...

F962 Mark Scheme January 2010

21 To what extent were Mao’s domestic policies successful in the 1950s?

No set answer is looked for but candidates will need to address the question. Candidates need to identify and analyse a range of policies and assess their success; this can be done against aims, results and/or historical context. Candidates may consider some of the following policy areas: the establishment of communist rule including military rule and reunification campaigns, the use of terror, propaganda and the imposition of one party rule; the ‘three’ and ‘five’ ‘anti-movements’; attacks on the middle classes and landlords; the first Five Year Plan; the Hundred Flowers Campaign; collectivisation; the Great Leap Forward. The last may not be considered in terms of success as it lasted into the 1960s. Candidates may well argue that the CCP managed to establish its authority effectively and achieved considerable successes but at a cost. The results of the first five year plan, for example, were impressive, but heavily dependent on Soviet aid and support; the middle classes were attacked and denounced and maybe a million landlords in the countryside were killed. They may argue that the Hundred Flowers Campaign backfired and had to be abandoned.

50

Page 54: History A Mark Schemes for the Units January 2010 - The ...

F962 Mark Scheme January 2010

Democracy and Dictatorship in Germany 1919-1963 22 To what extent did the Weimar Republic in the 1920s overcome the problems it

faced? No set answer is looked for but candidates will need to address the question. Candidates need to identify a number of problems facing the Weimar Republic and assess how far these were overcome during the 1920s. Candidates may discuss some of the following problems: the economic and social problems arising out of the war; the Treaty of Versailles and its impact; the problems of political extremism; the problems arising from the nature of the new Weimar constitution; the difficulties of Germany’s international position and so on. In relation to economic and social problems candidates may refer to the unemployment, inflation and economic dislocation after the war and the impact of reparations, hyperinflation and the invasion of the Ruhr. They may discuss the work of Stresemann, the Dawes and Young Plans and the rise in foreign investment in relation to this. They may discuss the threats posed by left and right to the regime and the difficulties associated with a democratic constitution based on proportional representation and how (far) these were dealt with. They may discuss German attempts to revise the Treaty of Versailles and the impact of Stresemann’s foreign policy (Locarno, League of Nations). No specific answer is looked for.

23 ‘Their use of terror was the main reason that the Nazis retained control in Germany after 1933.’ How far do you agree? No set answer is looked for but candidates will need to address the question. Candidates must give adequate treatment to the use of terror even if they wish to argue that other factors were as or more significant. In relation to terror, candidates are likely to discuss various aspects of the police state such as the roles of censorship and propaganda, the nazification of the judicial system, the use of arbitrary imprisonment and the roles of the SS, Gestapo and concentration camps. They may also point to the stifling of political opposition with the arrest of communists and social democrats and the ‘law’ banning other political parties. However, candidates may balance their discussion of these areas with the role of indoctrination, the attempts to control all aspects of people’s lives and the impact of war (with reference to the German Labour Front, Strength through Joy, the Hitler Youth etc.). Candidates may also refer to the apparent benefits of Nazi rule: the end of the communist threat, the restoration of ‘order’, employment and economic recovery, and foreign policy successes.

51

Page 55: History A Mark Schemes for the Units January 2010 - The ...

F962 Mark Scheme January 2010

24 Assess the reasons why Adenauer kept power for so long after 1949.

No set answer is looked for but candidates will need to address the question. Candidates need to identify and analyse a number of reasons and evaluate their relative significance and/or linkages. Answers may discuss the personal role of Adenauer in maintaining power and weigh this up against other factors This may include factors such as the political situation in West Germany, economic issues and issues of foreign relations in developing their argument. In relation to politics, candidates may refer to the strength and stability of the CDU/CSU coalition and the reliable support this received from the Liberals until the early 60s; they may also point to the weaknesses of the main opposition party – the SPD – (internal divisions, unable to adapt to the new prosperous West Germany). They may also suggest that the strategy of emphasising reconstruction (rather than recrimination) was a powerful political argument. In relation to foreign policy, candidates may point to: acceptance of West Germany in Europe; the recognition given to the FRG after 1955 and the end of the ‘occupation’; Britain’s support for Germany’s entry to NATO and hence the creation of her own army; West Germany membership of the OEEC, the ECSC and then the EEC. In relation to the economy candidates are likely to point to the work of Erhard, the social market economy, the Marshall Plan, cheap labour, good industrial relations, the survival of much of Germany’s industrial base after the war. They may point to the fall in unemployment and the average growth rate of 8%, and improving living standards. Candidates may well point to economic factors as being most important in explaining Adenauer’s success but no specific answer is looked for.

52

Page 56: History A Mark Schemes for the Units January 2010 - The ...

F962 Mark Scheme January 2010

The Cold War in Europe from 1945 to the 1990s 25 Assess the reasons for the growth of tension between the Allies at the Yalta and

Potsdam Conferences. No set answer is looked for but candidates will need to address the question. Candidates need to identify and analyse a range of reasons and to evaluate their relative significance and/or linkages. Candidates may discuss the longer term context of ideological differences and tensions that had arisen in the wartime alliance that provided grounds for mutual suspicion. In relation to Yalta, candidates are likely to focus on discussion of the Polish issue and the differences this aroused. There may be discussion of Stalin’s desire for security. In relation to Potsdam, candidates may refer to the change in personnel and it significance (particularly in relation to Truman), the context of the successful testing of the atom bomb, the continuing issue of Poland, the question of governance in liberated states and the issue of reparations in relation to Germany. Candidates may suggest that underpinning apparent agreements lay real difficulties as mutual fear and suspicion grew.

26 How serious were the problems facing the Soviet Union in controlling Eastern

Europe in the 1950s and 1960s?

No set answer is looked for but candidates will need to address the question. Candidates will need to identify and analyse a range of problems and evaluate their relative seriousness. They may discuss particular developments such as the problems in East Germany, Poland, Yugoslavia, Albania and Hungary in the 1950s and the Prague Spring of 1968. However, they may place such discussion in the broader context of the issue for Soviet leaders in balancing control against local situations and reform, the impact of the Cold War and the impact of de-Stalinisation. Candidates may discuss individual crises and the threat they posed to the stability of Soviet controlled Eastern Europe more generally and here there may be developed treatment of Hungary in 1956, the problem of refugees and the building of the Berlin Wall and the Dubcek regime in Czechoslovakia. The seriousness of the problems may be assessed in relation to the reaction of the Soviet government and the actions it took.

53

Page 57: History A Mark Schemes for the Units January 2010 - The ...

F962 Mark Scheme January 2010

27 Assess the consequences of the end of Communist rule in the Soviet Union.

No set answer is looked for but candidates will need to address the question. Candidates need to identify and analyses a range of consequences and evaluate their relative significance and/or linkages. Candidates may place their discussion of the context of the loosening of Communist Party control within the Soviet Union under Gorbachev’s policies of glasnost and perestroika and date their treatment of the question from the formal surrender of Communist power in February 1990. Candidates may point to the disintegration of the Soviet Union as Baltic States, Georgia and others bid for independence; the attempted coup against Gorbachev and the rise of Yeltsin (elected president of the Russian republic); the continued collapse of the economy and food rationing; the independence of the Ukraine; the creation of the Commonwealth of Independent States and Gorbachev’s resignation which signalled the formal end of the Soviet Union; the freedom of the media and political life; Yelstin’s economic ‘shock programme’ and the economic and social problems of the 1990s. Candidates may distinguish between political, social and economic consequences and between the immediate and longer term consequences.

54

Page 58: History A Mark Schemes for the Units January 2010 - The ...

F962 Mark Scheme January 2010

Crisis in the Middle East 1948-2003 28 Assess the consequences of the first Arab-Israeli War (1948-49).

No set answer is looked for but candidates will need to address the question. Candidates need to identify and analyse a range of consequences and evaluate their relative significance and/or linkages. Candidates may set their discussion in the context of the fighting that had already begun between Zionists and the Arab League even before the British left and the state of Israel was declared. They may refer to immediate consequences of the fighting such as the casualties, the razing of Arab towns and villages within Israel, the defeats inflicted on the Arab League forces, the agreement of cease-fires with Egypt and Syria (but not Iraq). Candidates may point out that there was no formal peace and that therefore the wars of 1956 and 1967 can be seen as continuations of the conflict. Candidates may point out that Israel at the end of the war was larger than that envisaged by the UN partition proposal and that the remnants of the Palestinian state were effectively taken over by Egypt and Transjordan. There was also the plight of the approximately 1 million Palestinian refugees that remained a destabilizing element in the Middle East. Candidates may also point to the consequences for the Arab states that faced internal instability. Candidates may make links between the war and political developments in these states.

29 To what extent have the violent actions of some Palestinians been the main obstacle

to the establishment of a Palestinian state?

No set answer is looked for but candidates will need to address the question. Candidates must deal adequately with the given factor even if they wish to argue that other factors were as or more significant. In relation to Palestinian violence candidates may discuss some of the following: the sporadic attacks of fedayeen in the 1950s, the actions of Yasser Arafat and Fatah, raids in the 1960s, hijacks, Black September, the Munich Olympics, the PLO in Lebanon, the actions of Hezbollah, the First and Second Intifadas, the attitude and actions of Hamas, and so on. Candidates may argue that violence was the Palestinians only effective weapon, that it brought the issue of the Palestinians to world attention and made possible an eventual two-state solution (proposed by Arafat). However, candidates may also argue that the impact of violence made settlement with the Palestinians impossible. Others may argue that there were other obstacles to the establishment of a Palestinian state, not least the attitude and actions of Israel and the unwillingness of the USA to force a solution. Candidates may also discuss the policies and attitude of the PLO, the significance of the Six Day War and Resolution 242. The issue of the Palestinians was, of course, also bound up with the other causes of distrust and friction within the Middle East.

55

Page 59: History A Mark Schemes for the Units January 2010 - The ...

F962 Mark Scheme January 2010

56

30 Assess the reasons for United Nations military action against Iraq in 1991.

No set answer is looked for but candidates will need to address the question. Candidates need to identify and analyse a range of reasons and evaluate their relative significance and/or linkages. Candidates may point to growing concerns, especially in Israel and the US, over the development of Scud missiles with the potential to carry nuclear warheads, Saddam’s announcement of a new rocket with a range of over 1200 miles and the misspending of agricultural credits. Meanwhile the Soviet Union was reducing its involvement in Iraq under Gorbachev and relations with Britain deteriorated (eg over the so-called super-gun and the execution of a British journalist by the ‘butcher of Baghdad’). However, the main issue was Iraq’s actions over Kuwait. Despite diplomatic pressure from both the US and the Arab League Saddam did not back off from his threats to Kuwait and in August 1990 he invaded, annexed the state (and its oil) and held foreigners as hostages. Candidates may explain how the coalition against Iraq was built up with the West, most Arab states and Gorbachev (unwilling and unable to intervene). Candidates may suggest that it was the end of the Cold War that enabled the UN Security Council to vote for an ultimatum. Candidates may argue that underpinning all the minor and more obvious reasons to justify UN action was the question of oil supply security.

Page 60: History A Mark Schemes for the Units January 2010 - The ...

F963 Mark Scheme January 2010

F963 British History Enquiries

Option A: Medieval and Early Modern 1066-1660

1 The Normans in England 1066-1100

(a) Study Sources A and C Compare these Sources as evidence for military campaigns fought by the Norman Kings of England. [30]

Both sources show that the kings relied on taking important castles. In both sources the king of France is appealed to by the enemies of the English and in both peace is made. In both members of the royal family are pitted against each other. The sources differ in that in A the Bretons fought hard and defeated William I, a rare event for the victor of Hastings, whereas in C William Rufus was successful, albeit through bribery. In A William was even worsted by his son Robert. Similarly in A the intervention of the French king was successful but in C he was bought off by Rufus. C has more to say about the attitudes of the nobles some of whom wanted a peace made. Source A mentions the corollary of war in France, an invasion by the Scots and the results of this. Both authors are chroniclers who list and describe. They have similar attitudes to the fighting, but A is more focused on the impact the events in France had on England, whereas C shows the bad effects of the war in Normandy and the impact of land ownership on both sides of the Channel. In terms of provenance A is Anglo-Saxon and C is Anglo-Norman but both approach the campaigns from an English perspective. As an Englishman, Henry of Huntingdon stresses the danger to England of the threat from the Scots while the king is Normandy. William of Malmesbury is only referring to the Norman perspective. Source A might be seen to provide a broader perspective.

(b) Study all the Sources

Use your own knowledge to assess how far the Sources support the interpretation that the problems arising from the linking of Normandy and England were caused mainly by members of William I’s own family. [70]

Sources A, C and D make it clear that Robert of Normandy was a troublemaker, leading a rebellion and being cursed by his father in A, appealing to the French king for help against his brother in C and being castigated as a rebel in D. In D he is also condemned for his failure to keep order. Source B outlines the effects in England of William’s absence when his regents, one of whom was his half brother, exploited their position. This evidence agrees with the statement. But in C Robert lacked courage to resist so his threat may not have been that serious. Source C suggests there were other reasons, describing a civil war that lasted a long time and showing that the nobles took advantage of the situation. But Source C also shows that the nobles saw their interests were served by peace, In Source A there is intervention from the French king, taking advantage of the situation and Source C supports this point. Also in C the Scots were causing problems. Source D shows the problems inherent in the linking of England and Normandy and the dual loyalties of the barons. There is a hint that Rufus himself was less effective than his father. Candidates could argue that despite the assertions in B, William I was a strong ruler in both England and Normandy as D makes clear. Odo was later overthrown and Lanfranc was a better regent. Candidates could refer to the role of Ranulf Flambard under Rufus. The fear of the barons that they might have different overlords in England and Normandy is backed up in the sources and candidates might argue that the greatest problems arose in the period 1087-95 when the kingdom and the duchy had different rulers.

57

Page 61: History A Mark Schemes for the Units January 2010 - The ...

F963 Mark Scheme January 2010

2 Mid-Tudor Crises 1536-1569 Royal Advisers 1540-1569

(a) Study Sources A and E

Compare these Sources as evidence for relations between monarchs and their advisers. [30]

Focus: Comparison of two Sources. No set answer is expected, but candidates need to compare the contents, evaluating such matters as authorship, dating, utility and reliability, so using the Sources ‘as evidence for …’. The headings and attributions should aid evaluation and reference to both is expected in a good answer.

The Sources agree that the advisers of both monarchs were ambitious, ‘seeking their own profit’ and with ‘projects’ the king wished to stop succeeding in Source A, and ‘if you think to rule’ in Source E. Both Sources agree that monarchs had to deal firmly with advisers – ‘reproaching’ them in Source A and humbling Dudley in Source E. They also agree that advisers did serve the monarch well: in Source A Cromwell is referred to as ‘the most faithful servant’ Henry had had and in Source E Elizabeth has ‘many servants whom she favours’. In Source A Henry is ‘gloomy’ and mistrusts his advisers, as in Source E Elizabeth mistrusts Dudley. The Sources also agree that monarchs had the power to make advisers afraid – Dudley was ‘so alarmed’ in E, perhaps because of examples like the execution of Cromwell in Source A.

But the Sources disagree concerning the extent of control monarchs exercised over their advisers. Source A suggests that Henry had been persuaded to execute Cromwell by the lies of Cromwell’s enemies, whereas in Source E Elizabeth is said to have deliberately created parties and factions to ‘divide and rule’. In Source E the public reprimand is to Elizabeth’s favourite, while in Source A, Henry’s reprimand is to the advisers who have made him remove his minister. Contextual knowledge may be used to show understanding of the comparison. Perhaps Henry is wishing to salve his conscience. His annoyance at the Cleves marriage undermined Cromwell’s favour with the king, and the Bill of Attainder against him was introduced under the king’s prerogative. Dudley was Elizabeth’s favourite, recently made Earl of Leicester and a rival of Cecil on the Privy Council, but she had no intention of marrying him, as he hoped, because he was a ‘mere’ subject. Both Sources therefore suggest that factional rivalry played a part in the relationship between monarchs and their advisers.

The provenance of the Sources may be used to determine which is more useful or reliable for explaining relations between monarchs and their advisers. Neither Source can be fully trusted. In Source A this is because of the nationality of the author and the context of the Habsburg-Valois wars. As ambassador at a time when the Anglo-German alliance had collapsed, he might be reassuring leading advisers, such as Montmorency, that Henry was weak. Late in Henry VIII’s reign the conservative and reform factions were influential due to his poor health and increasing age. Source E is written by a Stuart politician who was a child at the time of the incident he describes. Source E might seem to be less useful evidence as the assertion of Elizabeth’s strong control of her advisers has little factual support except this one incident. In Source A there is more balance between an infirm king who knows the good servants from the flatterers, and can act forcefully, as with Cromwell, but can also be manipulated. No set conclusion is expected, but substantiated judgement should be reached for the top levels of the Mark Scheme.

58

Page 62: History A Mark Schemes for the Units January 2010 - The ...

F963 Mark Scheme January 2010

(b) Study all the Sources. Use your own knowledge to assess how far the Sources support the interpretation that advisers could be trusted to serve royal interests between 1540 and 1569. [70]

Focus: Judgement in context, based on the set of Sources and own knowledge. Successful answers will need to make use of all five Sources, testing them against contextual knowledge and evaluating their strengths and weaknesses, any limitations as evidence. A range of issues may be addressed in focusing upon the terms of the question but no set conclusion is expected.

The Sources contain references to both sides of the argument, so they may be grouped by interpretation. The supporting view is shown predominantly in Source D and to varying extents in Sources A, B and E, whereas the opposing view predominates in Sources A, B, and E.

The supporting view is that Tudor royal servants helped establish strong system and served the monarchy well. Source D suggests that at the start of her reign, as a young female ruler, Elizabeth I trusts Cecil as hardworking, honest and discreet. Own knowledge might add that he was an experienced politician and a committed Protestant to help her implement a Church Settlement. However, at the time of the Source England was religiously divided, and he had not yet been tested, so these are only Elizabeth’s hopes. Henry VIII, in Source A, is reflecting on the loyalty of Cromwell, but his execution suggests that Henry had previously felt he had failed him. In Source C, Renard himself is acting as a loyal adviser of Mary, keeping Charles V informed should problems require his support for her and his son Philip. Here the context is less typical, as the Spanish marriage has caused factions at court to develop a xenophobic flavour unlike those in Sources A, B and E. Some candidates might point out that factions were part of the normal running of all governments. Dudley, in Source E, is humbled to be a loyal servant because of the Queen’s forcefulness. But this may be Naunton’s attempt to make a contrast with the weaker Stuart kings. In Source B, Somerset claims to be serving Edward VI by encouraging the common people to rise in support of the Lord Protector, who is taking the side of the people oppressed by the greedy gentry class, though the authors of the Source see his action as undermining the monarchy and nation.

So the Sources also support the opposing view. Somerset’s claims are unconvincing as described in the charges against him in 1549, made by his enemies on the Privy Council, the authors of Source B. They blame him for political and social instability, though knowledge of rebellions might be added to suggest other causes. In Source B he is said to have acted against the King’s interests and for his personal ambition during the royal minority. Knowledge of provenance might reveal the Privy Council’s own ambition, to oust Somerset in favour of Northumberland, who might be added, from knowledge, to best exemplify an adviser seeking power against the interests of the Tudor dynasty by placing Lady Jane Grey on the throne in 1553. In Source C, Paget and Arundel are revealed to be scheming against Gardiner, the chancellor. Paget is also out of favour with Mary and cultivating Philip’s support, leading to factions and disunity in the Council, hence political instability. The author, Renard, however, is Mary’s close adviser, so may not be objective. Source A suggests factions had caused instability due to their schemes and personal ambition, whereas in Source E, individual favourites are suggested as failing to prosper under Elizabeth, who is fully in control of her advisers using the patronage system. A supported overall judgement should be reached on the extent to which the Sources accept the interpretation that advisers could be trusted to serve royal interests. No specific judgement is expected.

59

Page 63: History A Mark Schemes for the Units January 2010 - The ...

F963 Mark Scheme January 2010

Candidates are likely to consider a range of themes within the Sources: factional rivalry, disunity, personal ambition, manipulation and loyal service. They are likely to set the Sources within the context of strong or weak monarchies, perhaps due to age or gender. It is up to candidates to assess and decide upon relative importance here, there being no set conclusion.

3 The English Civil War and Interregnum 1637-1660

The Second Civil War and the Trial of King Charles I

(a) Study Sources A and C Compare these Sources as evidence for issues involved in the King’s negotiations during 1647-8. [30]

Focus: Comparison of two Sources. No set answer is expected, but candidates need to compare the contents, evaluating such matters as authorship, dating, utility and reliability, so using the Sources ‘as evidence for …’. The headings and attributions should aid evaluation and reference to both is expected in a good answer.

Both Sources are examples of the terms negotiated by the King. The provenance of the Sources should be integrated into the comparison. There are both similarities and differences in the terms discussed. In Source A, the King negotiates with the Scots to gain their military support which he had already had earlier with the Solemn League and Covenant, whereas in Source C he negotiates with commissioners sent by Parliament, who are hostile and in a position of strength after his defeat in the Second Civil War. Source A is signed by Charles I himself so is more reliable than Source C, which is a later record of hearsay information perhaps justifying a husband’s involvement in the later execution of the King. The purpose of the King in both sets of negotiations is to restore his power. In Source A the Scots ‘will restore him to his government, royal rights and full revenues’ and Source C refers to ‘terms they agreed for his restoration’.

Source A includes terms concerning religion: the King wishes ‘to preserve and establish religion’ and Source C agrees ‘He would not give up the bishops, but only lease out their revenues’. However, in Source A ‘Separatists and Independents will be suppressed’ and the King will set up ‘a Presbyterian system for three years’, whereas Source C takes an opposite view on bishops. Hutchinson is giving her husband’s view, that such terms ‘betrayed their whole cause’. Context might be used to explain the divisions which Hutchinson shows, between those Presbyterians who wished to continue negotiations and the Army officers who now took a hard line attitude towards the King as ‘that man of blood’, complicating the process of negotiation in 1648. Source A records the treaty with the Scots which began the Second Civil War, the basis of the charge that the King had caused bloodshed by waging war against his people, and this charge seems to have been put to him in Source C, which states that the King ‘acknowledged himself guilty of the blood spilt in the late war’. His untrustworthiness is shown in Source C, by his request that this concession ‘should not be used against him’. In Source A, he accepts Presbyterianism, though himself a crypto-Catholic, and concedes valuable fortresses to the Scots, showing his untrustworthiness.

Both Sources suggest that the King tried to use bribery to buy his restoration to power. In Source A, he offers the Scots fortresses, ‘arrears of £200,000 and all the expenses of their army in this future war’. In Source C he offers ‘great honours and offices’. But whereas in Source C, Colonel Hutchinson’s view is that acceptance of the terms is ‘inconsistent with the liberty of the people’, in Source A, Charles is claiming that his Engagement with the Scots is to ‘defend the privileges of Parliament and the liberties of the subject’. Context might be used to discuss this claim in the light of national and religious hostility between

60

Page 64: History A Mark Schemes for the Units January 2010 - The ...

F963 Mark Scheme January 2010

the Scottish Presbyterians and the Independents in the Army and Parliament, who would lose their liberty by the terms of the Engagement.

So the Sources are very different in provenance: authorship, date and context, but many of the issues involved remain the same. In relation to the attempt to gain a settlement with the King, Source C is better evidence, despite its unreliability. No set conclusion is expected, but substantiated judgement should be reached for the top levels of the Mark Scheme.

(b) Study all the Sources

Use your own knowledge to assess how far the Sources support the interpretation that the King was brought to trial because after 1647 he could no longer be trusted. [70]

Focus: Judgement in context, based on the set of Sources and own knowledge. Successful answers will need to make use of all five Sources, testing them against contextual knowledge and evaluating their strengths and weaknesses, any limitations as evidence. A range of issues may be addressed in focusing upon the terms of the question but no set conclusion is expected.

The untrustworthiness of the King was also a reason for his trial. Negotiations failed partly because Parliament and the Army did not trust him to keep his promises. He also is said, in Source C, to have tried to bribe the commissioners to get the treaty signed by Parliament. This certainly supports his view in Source E that he was leaving behind a ‘corruptible crown’. His claim, in Source E, that he did not rule by the power of the sword might be evaluated by cross-reference with Source A. By laying the foundations of the ‘cult of King Charles the Martyr’ with his propaganda in Source E, he is trying to overcome the view in the title.

There are two main charges against the King in his trial in Source D: firstly that ‘He traitorously and maliciously levied war against Parliament’; and was ‘the author and continuer of the unnatural, cruel and bloody wars’, ‘guilty of the treasons, murders, burnings and damages to this nation caused by these wars’. Secondly, that he tried ‘to erect an unlimited and tyrannical power’ and ‘overthrow the people’s liberties’. In Source E the King protests his innocence saying that he did not ‘rule in an arbitrary way, to have all laws changed according to the power of the sword’. Thus he refutes the second charge of trying to create an unlimited and tyrannous power. He claims to have tried to protect ‘the people’s liberty’ and ‘their life and goods’.

He denies that he acted in an arbitrary way. Cross reference might be made with Source A, where he invites a Scottish army into England to restore him, but here too he claims to be protecting the ‘liberties of the subject’. This claim might be evaluated in the light of English views of the Scots and his promise to establish Presbyterianism for three years. There is evidence in Source A that the King caused the Second Civil War by signing the Engagement with the Scots, so the charge, which is repeated in Sources B and C, may have some foundation.

But, on the other hand, the introduction to Source D shows that the High Court set up to try Charles was created by a Rump of the Commons after Pride’s Purge in December 1648. John Bradshaw represents this, and the King did not recognise the court. Thus the power of the Army lay behind the trial of the King.

The king had escaped from Army control when he called in the Scots with the Engagement, so Army power might be seen to lie behind this desperate move. However, there were still those parliamentarians who wished to continue to negotiate with the King, as revealed in Source C, and there was considerable debate in Parliament about the

61

Page 65: History A Mark Schemes for the Units January 2010 - The ...

F963 Mark Scheme January 2010

62

acceptance of the terms negotiated by the Parliamentary commissioners in the autumn of 1648. The provenance of this Source is unreliable, as there may be a purpose of justifying Lucy Hutchinson’s husband, and showing that the parliamentary cause would be lost if a treaty with the King were signed. However, if it is taken at face value, pressure from the Army played an important part in the decision to end negotiations.

As for the decision to bring the king to trial, Source B suggests that some parts of the Army had prejudged his guilt. Source B reveals that some elements of the Army viewed the King as ‘that man of blood’ as early as the Windsor prayer meeting in April 1648. The ‘reopened’ negotiations mentioned in Source C was unpopular with them, as it came after the defeat of the King in the Second Civil War ‘after his defeat and capture’, and the repeal of the ‘Vote of No Addresses’ which had originally been passed under pressure of the Army in January 1648. Therefore the power of the Army had played a part in the ending of negotiations, trying the king and finding him guilty. However, the author of Source B is an agitator, writing long after the event, emphasising the danger of the Second Civil War. The context of Source B is widespread revolts against the Army throughout 1648, so that may suggest the Army was not so powerful at that time. Certainly this context hardened their views of the king’s role in events.

A supported overall judgement should be reached on the extent to which the Sources accept the interpretation that the King was brought to trial because after 1647 he could no longer be trusted. No specific judgement is expected.

Candidates are likely to consider a range of themes within the Sources, such as the King’s part in the outbreak of the Second Civil War, his untrustworthiness, the extent of his defence of the law, the nature of his rule and the part played by the Army and the Scots. They are likely to set the Sources within the context of events, such as the Vote of No Addresses and Pride’s Purge. It is up to candidates to assess and decide upon relative importance here, there being no set conclusion.

Page 66: History A Mark Schemes for the Units January 2010 - The ...

F963 Mark Scheme January 2010

Option B: Modern 1915-1945 1 The Condition of England 1815-1853

(a) Study Sources A and E Compare these sources as evidence for attitudes towards improving the condition of factory workers. [30]

No set answer is expected, but candidates need to compare the contents, evaluating such matters as authorship, dating, utility and reliability, so using the sources ‘as evidence for…’. The headings and attributions should aid evaluation and reference to both is expected in a good answer. Both sources agree on the need for improvement, emphasising the need for some sort of education, although E is vaguer (the 3 ‘R’s in A, an ‘increase of intelligence’ in E). They also agree that the way forward is for factory workers to become ‘industrious’ (A) or to work in E. However they differ on how best to do it. There is disagreement on what holds workers back. The stress in A is on poor living and working conditions whilst in E it is government restrictions on both trade and labour. For Bright trade barriers, monopolies and factory reforms all deny workers the opportunity to find work. Similarly they disagree on labour. Owen in A has ended child labour under 8 and although Bright in E doesn’t refer to this he is adamant that all be given the opportunity or ‘liberty’ to work. The way forward for Owen (A) is for employers to take responsibility, providing education from 5 to 10, better streets and housing, restricting alcohol and ending the Truck System by effective bulk buying and then selling at low rates. For Bright in E it is the free market which allows both employers and workers to make individual choices (workers voluntarily deciding to limit their work). Owen stresses the employer as the key, Bright both the individual worker and employer operating in a free market. These differences are explained by the provenance. Both are employers and mill owners and perhaps significantly neither mentions better wages as the way forward, although Owen does refer to the need to lower the price of necessities. Both are radicals and not necessarily typical of their class, although Bright will speak for most of his type. There the similarities end. Owen was a radical paternalist and early socialist who believed in cooperation rather than competition as the basis for the new industrial society. In contrast Bright was an MP and spokesman of the northern millowners who campaigned for Free Trade and a society based on the principles of laissez faire, hence his stress on achieving a free market as the means of improvement (‘markets of the world’, ‘liberty to work’). Owen’s comments are based on social experimentation at New Lanark and he is keen to use the experience to ‘prove’ his case – that cooperation will lead to workers who are ‘industrious, faithful and kind’. This may have led to some exaggeration of the beneficence of his changes given that he is arguing a case in his book ‘New View of Society’. In contrast Bright is talking of the economy in general. He assumes that improvement will come not from employer paternalism but from individual effort, if only government would ‘give them the power’. Both are optimistic. There is also a difference in the dates. Owen is talking of an earlier experiment, before the first proper Factory Act in 1833, whilst Bright is opposing Graham’s Factory Act, the 3rd main instalment of workplace change, in a parliamentary speech that is strong on rhetoric. Both are useful pieces of evidence on how best to improve the industrial working class. Although there were other examples of paternal millowners Owen may be less typical than Bright, although many employers may not have taken Bright’s more theoretical view, preferring to stress profit.

63

Page 67: History A Mark Schemes for the Units January 2010 - The ...

F963 Mark Scheme January 2010

(b) Study all the Sources Use your own knowledge to assess how far the Sources support the interpretation that factories in the period 1831 to 1844 were places of exploitation for all workers. [70]

Successful answers will need to make use of all five Sources, testing them against contextual knowledge and evaluating their strengths and weaknesses, including any limitations as evidence. A range of issues may be addressed in focusing upon the terms of the question but no set conclusion is expected.

The sources may be used in a variety of ways to assess the interpretation that factories were places of exploitation for all. It is likely that candidates will see Sources B and D as more supportive of this view whilst A, C and E are critical of it, pointing to Factories as more positive places, the ‘best schools’ in the words of McCulloch in C. Evaluatively C and E may be considered less credible given their abstract and theoretical nature (both are referring to the economy). McCulloch in C is an economist who may well have less experience than Bright and Owen (A and E) in the running of factories. However they too have interests that lie elsewhere (Bright’s middle class radical campaign against aristocratic government and the Corn Laws and his position as an MP; Owen’s trade union and written work). B and D may be considered more useful given Horner’s hands-on evidence as the key Factory Inspector a decade into his work, and the report of the Leeds newspaper with a slightly better off readership in Yorkshire, yet whose tone appears sympathetic to the cause of reform (‘aptly named little victims’, an acceptance of the case that exploitation was prevalent). The case against factories is mainly to be found in B and D. The Manchester demonstration in B, organised by the Short Time Committees, was designed to stress the cruelty of the system for children and thus implicitly for men as well. It deftly demonstrates the experience of radicalism, the phrasing linking to an Anti Slavery movement that was about to triumph in 1833 (candidates may refer to Oastler’s Yorkshire Slavery article) – ‘Am I not a brother and a man’. Children sing of the 12 hours they work and emotively carry whips and straps. However this is obviously propaganda, milking a Manchester audience for support and clearly succeeding. It is the sort of ‘exaggeration’ that McCulloch in C complains of. Nonetheless the response and scale might suggest deeply felt grievances with workers yet to learn to love the factory. Horner in D may be considered better evidence given the provenance. It is based on much visiting and inspection and although the true picture may have been hidden on occasions his comments command authority and he was listened to by governments. However Horner does not comment on children, confining his comments to adult labour. He does not necessarily accept the argument that conditions and hours for men needed interference, but he is concerned that the rise of female labour is based on abuse (their alleged physical incapacity and the ‘deterioration of their health’) and that this has an impact on adult men (neglected domestic duties). He believes that women, as unfree agents, have been exploited as cheap labour. Whilst not a supporter of 10 hours, he does consider 12 exploitative. Candidates could also use Owen in A. Given that he refers only to his own experiment at New Lanark, one can infer exploitation was common elsewhere (‘the practice of employing children…’; a truck system that needed challenging, via his Cooperative movement, to prevent employers controlling their workforce as consumers). Own knowledge could extend the discussion here, pointing to findings in the reports of 1831 and 1832 and to the legislative struggle before and beyond 1833. Candidates could also note that McCulloch admits there is some foundation to these shocking reports and that ‘abuse’ is ‘certain’ in some factories (and own knowledge may point to the older, smaller and more water powered ones by the 1830s and 1840s). Bright in E could be interpreted as a theoretical justification of worker exploitation, given that no mention is made of wages and that labour shortages were increasingly a thing of the past.

64

Page 68: History A Mark Schemes for the Units January 2010 - The ...

F963 Mark Scheme January 2010

The case that hours and conditions were not necessarily exploitative for all can mainly be found in A, C and E. Yet Owen in A is not referring to a typical factory and candidates might discuss whether there were other model factories. He is certainly keen to include children and adults in his improving measures. C and E carry the main weight of the pro factory argument, as to be expected from wealthy men and employers who invested their time and careers in advancing the cause of a factory based society governed by the free market and self made men, offering the blessings and opportunities of hard work. Clearly they are concerned to resist any attempt by government to interfere via factory acts. However McCulloch was right to challenge some of the stories of exploitation emanating from the factory reformers – professional cripples and the coaching of witnesses by reformers was a tactic used in the early 1830s. Non factory labour (workshops and agriculture) may well have been just as exploitative, and with worse conditions for all, than the factories, the owners of which often felt unduly focused on. McCulloch refers to ‘other classes’ and comments on the alternatives – beggary in the streets and crime. He is convinced that the Factories especially have disciplined, ordered and protected children. Bright in E shares this view although candidates could question the reality behind his rhetoric. He is short on evidence, long on claims (‘speedily make them independent’?). Could workers look forward to the sort of means which would enable them to choose more ‘recreation and enjoyment’? Horner in D will not accept less hours for adult men whilst B could be dismissed as unreliable anti factory propaganda. The Sources provide mixed messages and much will depend on an evaluation of their respective worth and the relative conditions prevailing in the variety of workplaces at the time. There is much special pleading in all the sources.

65

Page 69: History A Mark Schemes for the Units January 2010 - The ...

F963 Mark Scheme January 2010

2 The Age of Gladstone and Disraeli 1865 -1886

(a) Study Sources B and D Compare these Sources as evidence for the attitudes towards Forster’s 1870 Education Act. [30]

No set answer is expected, but candidates need to compare the contents, evaluating such matters as authorship, dating, utility and reliability, so using the sources ‘as evidence for…’. The headings and attributions should aid evaluation and reference to both is expected in a good answer. The sources agree on the importance both of education and the 1870 Act, as to be expected from two such advocates, but both have reservations. These are religious. Bright in B is convinced that the Act was designed to enhance the role of the Church of England and to continue a system of religious voluntary schools that favoured Anglicans. As such the Nonconformists had long given up a demand for equity and instead put their trust in a nondenominational system that would strip the Church of its pre-eminence in elementary education, hence Bright’s hope that Board schools would be established everywhere and voluntary schools induced to come under their control. This explains Bright’s frustration that in many areas, especially rural ones, nondenominational Board Schools would not be established and thus a choice not forthcoming for nonconformists. Barry in D partly agrees but on different grounds, stressing those more respectable workers will want to send their children to largely Anglican voluntary schools, possibly on grounds of snobbery and class. He thus agrees with Bright that the Act in part advantages denominational Anglican education but he considers this to be the fault of Nonconformists like Bright. His argument is that by strongly objecting to fee payments for the poor to attend Voluntary schools they confine them in effect to what is perceived to be a second class Board School education. Where Barry in D disagrees is his view that the Act has dealt a huge blow to the religious voluntary system, financially (no building grants and a failure to plug into the local rates as an assured means of securing income) and religiously (the Cowper Temple conscience clause allowing withdrawal and an enforced nondenominational religion to be taught in Board schools). These differences are explained by provenance. Bright, as a key nonconformist MP, is denouncing the Act in a speech, possibly to a nonconformist audience. He represents militant nonconformity, anxious to achieve religious equity with established Anglicanism and his tone is evidence of this. In contrast Barry is more the educational expert and Headmaster and, although he approaches it from an Anglican viewpoint, he is concerned to make more thoughtful points about its impact. In part a little distance in date helps him. In 1873 Bright, who resigned as President of the Board of Trade, is still the outraged leader of thwarted Nonconformity whilst in 1874 Barry is more impressed with its secular, nondenominational drift. He can see, as Bright cannot, that the Anglican Voluntary schools will benefit from class distinctions (whether he approves is another matter) as better off workers seek to segregate their children from the urban poor, destined for the Board schools. On these grounds it may be the better evidence for more informed and balanced attitudes 4 years into its operation and on the eve of an election that saw Bright’s Nonconformists contribute to Gladstone’s downfall.

(b) Study all the Sources

Use your own knowledge to assess how far the Sources support the interpretation that the reforms of 1868-74 were designed to satisfy interest groups which supported the Liberal Party. [70]

Successful answers will need to make use of all five Sources, testing them against contextual knowledge and evaluating their strengths and weaknesses, including any limitations as evidence. A range of issues may be addressed in focusing upon the terms of the question but no set conclusion is expected.

66

Page 70: History A Mark Schemes for the Units January 2010 - The ...

F963 Mark Scheme January 2010

Sources A and in part B and E suggest that the reforms were intended to satisfy liberal interest groups and that those groups played an important part both in their occurrence and their drafting. These sources may be considered effective in their contribution, coming as they do from a variety of liberal or moderate and reformist views (Arnold from the vantage point of reason and justice, Bright from militant middle class nonconformity and Matthews from a modern historian’s view with a close understanding of Gladstone’s thinking). In contrast Sources C, D and in part B and E would suggest otherwise – that the reforms were designed to attack vested interest wherever it might be. They too come from a variety of liberal angles and provide a balanced view, Matthews in E being a case in point. He provides a balanced account of the Trade Union reforms. The satisfying of Liberal interests is seen in Arnold (A) who strongly suggests that the key interest group to be appeased were the nonconformists, citing their influence on Irish Disestablishment in 1869. Arnold would clearly have preferred a redistribution of property rather than disendowment and cites liberal concepts of equity and justice. Knowledge might support this. Gladstone, it has been argued, used Irish Church reform to unite a divided Liberal party in the wake of the debacle over the 2nd Reform Act and to win the 1868 election. Not just nonconformists were pleased by Disestablishment. Radicals and Whigs also united around it. However whether Arnold’s ideas were realistic is another matter. Clearly Forster in 1870 tried to satisfy the nonconformists and the Birmingham based National Education League over education but Bright in B and the League would settle for nothing less than an ending of grants to the voluntary schools and a compulsory, free, state, non denominational elementary educational system. They looked to his Board Schools to do it and there is evidence that he thought he had done enough to satisfy them. In E Bruce is clearly acting to address Lib/Lab concerns over Union legal status and funds, thinking it achieved by the two acts of 1871. Matthews makes it clear that the Liberal reforms intended to allow moderate peaceful picketing and negotiation, only to be frustrated by conservative and restrictive rulings in the Courts. Knowledge might also be used to demonstrate Whig interests (government posts and a more moderate Irish Land Act) being satisfied. Other Liberal and nonconformist concerns were addressed over alcohol, the aristocratic and patronage ridden army and over the principle of merit in Universities, Civil Service and the Army. The alternative view is provided in C, D and in parts of B and E – that far from pandering to interest groups, Liberal or otherwise, the reforms took on vested interest and indeed alienated key Liberal groups, notably the nonconformists, the Drink Trade and the Lib/Labs of the respectable artisan upper working class with their newly formed TUC. Bright in B is outraged by the apparent Anglican victory and candidates could point to clause 25 and other loopholes that allowed rate subsidy for Voluntary schools via poor pupils and became the later focus for nonconformist hostility. Far from satisfying those the reforms seemed to ride roughshod over this particular ‘interest’. The Economist in C is staggered at the Licensing Act’s treatment of both the brewers and the working class. It stresses how bold it was to take on such a vital electoral interest. Gladstone blamed his own second place in the Greenwich election in 1874 on this act – the torrent of gin and beer! Barry in D can be interpreted either way – that the Liberals achieved a balance between Anglican and nonconformist interests or that it was too concerned at economising by not building Board schools throughout and therefore did advantage Anglicans whilst simultaneously undermining their catechism. Matthews in E points to the lack of interest Gladstone and the Cabinet (or was it lack of agreement) had over addressing worker concerns over the Criminal Law Amendment Act. It would appear that the reforms were not that interested in pandering to specific Liberal interest groups. They were prepared to live with Union anger, something Disraeli was easily able to capitalise upon in 1875. Sources A to D are all varieties of Liberal thinking and provide a mixed message as to the focus of the reforms. Only A suggests out and out pandering to sectional interest and Arnold stood somewhat aloof from mainstream Liberalism. The rest, including the historian of Gladstone, Matthews, suggest a more mixed set of motives and, indeed, the antagonism of some liberal groups, whether Nonconformist, TUC, Brewer or Whig Irish landowner, to achieve retrenchment, reform, civic equality and justice.

67

Page 71: History A Mark Schemes for the Units January 2010 - The ...

F963 Mark Scheme January 2010

3 The Fortunes of the Conservative Party 1900-1914

(a) Study Sources C and D Compare these Sources as evidence for attitudes towards Tariff Reform. [30]

Focus: Comparison of two Sources No set answer is expected, but candidates need to compare the contents, evaluating such matters as authorship, dating, utility and reliability, so using the Sources ‘as evidence for...’. The headings and attributions should aid evaluation, and reference to both is expected in a good answer.

Content: Both Sources refer to Conservative opinion. But while Source C supports Tariff Reform and the necessity of Food Taxes, Source D is opposed to both. Source C is confident that Tariff Reform is the right policy, and claims that it is most successful when discussed openly and vigorously. The emphasis is on Tariff Reform protecting British jobs. But this argument does not play well with the Lancashire county association (Source D). The Source claims that ‘Lancashire hates Tariff Reform’, and candidates might pick up on the point that generally the cotton industry supported both free trade and low food prices. According to Lord Derby, Tariff Reform has been a divisive issue in the party, especially since the crushing electoral defeat of 1906, and now further failure in the two elections of 1910.

Provenance: Source C. The background is the General Election of January 1910, when the Conservatives narrowly failed to defeat the Liberal Government. Austen Chamberlain had taken over leadership of the Tariff Reform movement from his father Joseph Chamberlain. Here, he encourages Balfour to continue with the controversial policy. As this election was fought against the background of the crisis over the People’s Budget and the House of Lords, one might wonder if Chamberlain is exaggerating the importance of Tariff Reform in the minds of the electorate. However, at this stage, it remains true that the supporters of Tariff Reform (‘whole-hoggers’) are still an influential group in the Conservative Party. Source D. Nearly three years later, Balfour has gone, having initially promised a referendum on Tariff Reform. Bonar Law has now withdrawn this offer. Hence the anger in Lancashire. The source reference is local, but the dangers of a split in the party are wider. Walter Long was a moderate supporter of Tariff Reform. But Derby hopes he will respond to the danger. Soon after this, Bonar Law (always a ‘whole-hogger’) is forced to abandon Tariff Reform. A good answer may not require all of this information. The essential point is the potential split in Conservative leadership (and support) arising from these differing attitudes to Tariff Reform. The dates are important in bringing this out.

(b) Study all the Sources

Use your own knowledge to assess how far the Sources support the interpretation that the issue of cheap food was the main reason for fluctuating working class support for the Conservative Party between 1900 and 1914. [70]

Successful answers will need to make use of all five Sources, testing them against contextual knowledge, and evaluating their strengths and weaknesses and any limitations as evidence. A range of issues may be addressed in focusing upon the terms of the question, but no set conclusion is expected.

Fluctuating working class support for the Conservative Party is much in evidence during this period. After victory in the Khaki Election of 1900, the Conservatives faced many difficulties in an era of increasing working class political influence. These problems included: The unpopular policies of the Balfour Government 1900-1905. Electoral defeat in 1906. The domination of the Liberal Party 1906-1914. The rise of the Labour Party. The House of Lords Crisis. Continuing splits over Tariff Reform (the issue of cheap food). The

68

Page 72: History A Mark Schemes for the Units January 2010 - The ...

F963 Mark Scheme January 2010

problems of Ulster and Home Rule. And failure to oust the Liberals in the two elections of 1910. And yet, as the modern historian suggests in Source E, the Conservatives continued to enjoy an underlying electoral strength as the natural party of government. Despite the Liberal reforms, and Labour’s rise, there was still strong support for the Conservatives, even among the working classes. Certainly, the issue of cheap food weakened the Conservatives because their policy of Tariff Reform was associated with dearer food, and because not all Conservatives supported the policy. Nevertheless, for sound reasons, it remained a Conservative policy for almost a decade. Besides, there were many other factors for fluctuations in working class support. Most of the Sources can be used on either side of the debate, although a likely grouping is A, B and E questioning the centrality of ‘cheap food’ as the key factor in working class political allegiance, C and D asserting its primacy. Source A clearly explains electoral defeat in 1906 by reference to unpopular policies which alienated the working class (and others). It points to the Conservatives being seen generally as the party of the rich and selfish. However, ‘taxing the food of the poor’ is only one of the reasons given for loss of working class support. Chinese Slavery (selfish imperialism) and Taff Vale (attacks on trade unionism) are seen as equally important. In addition, candidates might suggest that the Quarterly Review (a Liberal magazine) is presenting a one-sided case. It could be argued that Joseph Chamberlain’s Tariff Reform campaign was a serious effort to deal with the problems of Empire, trade and national efficiency. Furthermore, own knowledge could provide examples of useful Conservative legislation appealing to working class support. For example: the Education Act; Wyndham’s Land Act; and the Unemployed Workmen Act. Balfour’s Government also introduced a commission to investigate the poor law. However, there is no doubt that the Liberals won a landslide in 1906. In Source C, Austen Chamberlain, who has taken over his father’s campaign, urges the uncertain Balfour to stick with Tariff Reform, despite recent defeat in the January Election of 1910. Its main thrust is to praise Tariff Reform, and to suggest that electoral defeats have occurred when the party members were insufficiently bold and open about the policy. Chamberlain’s optimistic view is that the working class could be converted to Tariff Reform by an emphasis on the threat of unemployment rather than on the price of food. However, he admits that Food Taxes may well have contributed to the party’s unpopularity with workers: ‘dear food, black bread and horse flesh’. Candidates might point out that the main issue in January 1910 was the House of Lords rejecting the People’s Budget rather than the food issue. In Source D, it is clear that ‘Lancashire hates Tariff Reform’ because of ‘Food Taxes’. Derby, as leader of the county association, would be well aware of working class discontent in Lancashire. However, as a member of the ‘shadow cabinet’, he has wider concerns for his party. The Source concentrates on divisions in the party resulting from the policy of Food Taxes, which are the main problem of continuing to support Tariff Reform. The Source also implies problems of Conservative leadership following Bonar Law replacing Balfour. Bonar Law stuck with Tariff Reform as long as possible. Balfour had been more diffident, and had offered a referendum on the question. Source B provides the clearest evidence against the assertion in the question. In the Source, Sir Edward Stanley plays down the impact of Tariff Reform as an explanation for Conservative electoral defeat in 1906. In particular, Stanley sees the rise of the Labour Party (Lib-Lab Pact implied?), the strengthening trade unions, and the growth of working class independence as the main factors in his own defeat in Lancashire. However candidates might point out that Stanley (the Lord Derby of Source D) will change his opinion by 1912, accepting the damage done to the party by the issue of cheap food. Credit answers that make an effective provenance point here – B, as a private conversation, may more accurately reflect Derby’s position than the letter to Long in D. In Source E, the modern historian explains the broadening popular appeal of the party despite all the difficulties of the period. The Source ignores the damaging issue of cheap food. Instead, it explains the revival of working class support by references to Irish Home Rule, Empire and Employment. The Conservatives were seen as patriotic; and this created

69

Page 73: History A Mark Schemes for the Units January 2010 - The ...

F963 Mark Scheme January 2010

jobs, an important contrast in some areas to Lancashire’s cotton industry. They supported the Empire against Little Englanders and Socialists. They backed Ulster. Candidates might also point out that, in the second election of 1910, the Conservatives received more votes than the Liberal Party, which now had to rely on Labour and Irish support to stay in power.

4 (a) Study Sources A and B.

Compare these Sources as evidence for views about self-government for India as expressed in 1931. [30]

Focus: Comparison of two Sources No set answer is expected, but candidates need to compare the contents, evaluating such matters as authorship, dating, utility and reliability, so using the Sources ‘as evidence’ for….The headings and attributions should aid evaluation and reference to both is expected in a good answer.

Content: A offers a view of a moderate and popular Indian policy in contrast to the view in B. Churchill in B speaks of surrender, while A speaks of all parties cooperating to bring a new constitution. There is no mention in A of the loss of trade which Churchill fears. In B India is being abandoned to upper caste Hindus, but in A Muslims and Hindus are in agreement and there is no reference to dominant caste politics. A sees the government acting to meet the political hopes of India, but B sees ignominious surrender to Gandhi and the ‘Brahmins’. Political liberties for all Indians are guaranteed in A, but the Untouchables are being consigned to tyranny in B. A sees peaceful conditions, but by implication that is far from the case in B’s view. B talks in terms of high emotion – ‘shame’ and ‘guilt’ but A is the language of agreement and consensus. The tone is very different. Provenance: A is a statement made in the calm of the House of Commons, pondered and governmental. B is a rousing rhetorical speech in a large hall before committed Imperialist opponents of self government for India. It appeals to emotion and self-interest in a way that B deliberated avoids. A is from a government minister wanting to show progress for moderation; B is from a political maverick, whose stance has alienated him from the Conservative establishment and who rejects moderation and cross-party agreement. The aim of A is to build agreement; the aim of B is to use extra parliamentary pressure to wreck agreement. A is typical of moderate opinion on India, hoping to end the Congress agitation and build on previous acts to share power. B is not very typical of Conservative opinion, but entirely typical of Churchill’s willingness to take on establishments and pursue anti-appeasement policies. A is useful for showing the calm tone of Macdonald’s approach; B is useful for seeing how India led Churchill to the rhetorical excesses which kept him in the political wilderness. Some may judge A to be more useful for establishing the cross-party view of India because B is so unrepresentative of all but a minority. Others may see B as more useful for understanding the disagreements and for seeing that the tone of the opposition to reform in India was not likely to be generally acceptable.

(b) Study all the Sources

Use your own knowledge to assess how far the Sources support the interpretation that Churchill’s policies towards India showed serious misjudgement on his part. [70]

Focus: Judgement in context, based on the set of Sources and own knowledge. Successful answers will need to make use of all five Sources, testing them against contextual knowledge and evaluating their strengths and weaknesses, any limitations as evidence. A range of issues may be addressed in focusing upon the terms of the question but no set conclusion is expected. The debate here is whether Churchill was blinkered by his early experiences in India and his determination to hold India at all costs and whether this led him to serious

70

Page 74: History A Mark Schemes for the Units January 2010 - The ...

F963 Mark Scheme January 2010

71

miscalculation. There have also been suggestions that Churchill was blinded by racial prejudice about India. So he has been seen as an out-of-touch imperialist who ruined his career in campaigning against India reform and turned moderate Conservative opinion against him, a misjudgement which made his criticisms of appeasement less effective when applied to Germany. There is a view that Churchill was wiser in practice than in making speeches, but he came in for a great deal of criticism for the governing of India during the war – by Amery, the Colonial secretary and Wavell, the Viceroy and Source D condemns him for the policy adopted over the Bengal famine. Source E attempts some justification and Source C shows that as Prime Minister he softened the approach he took to India. The most critical Sources are A, C and D. Source A by implication shows that the press and public opinion, and both Hindus and Muslims within India were prepared to come together for moderate reform. ‘Some quarters’ – presumably Churchill and his die hard allies – the extremists referred to in C cannot accept the consensus. However, a minister in Benn’s position would want to stress the degree of unity about this issue. The tone of B rather tells against Churchill here and the Imperial expert, Wrench, even at a time when Churchill’s reputation stood very high in the early 50s finds the 1931 position that Churchill took hard to understand. However this is a speech in a large hall to an audience likely to support the reactionary views of the organizers and Churchill’s public and private views were not always expressed in the same terms. Churchill after all had wide ministerial experience and had been sympathetic to national causes in South Africa, Ireland and Belgium, but he aligned himself with imperial extremists. The most damning criticism comes from an Indian source (D) looking back on the Bengal Famine. However, this is based on one family and the evidence is not given for British neglect. 1942 was a very difficult year for Britain and there were some hard choices. This is a journalistic rather than a historical account. It is also bound to be limited as it is essentially based on family experiences. However, other sources do confirm widespread resentment about British inaction. C has eye witness accounts of meetings with Gandhi and the Cripps mission of 1942; but whether Gandhi would actually in the end have been satisfied with dominion status may be questioned – certainly on the basis of the limited evidence given here. India and the Empire were causes of dissent between Churchill and the Americans and there is some doubt about whether a post-war Conservative government would have given up India. Candidates may be aware of Churchill’s continuing commitment to an imperial role right up to the end of the war. However Wrench (C) does offer some modification to the harsher view. Churchill himself defends his policy in 1931 on humanitarian grounds, though the sincerity and factual basis of this may be questioned. He was convinced that Congress would oppress lower castes and, indeed, Muslims. This was actually the case in Congress administrations after 1935 – at least in terms of Muslims. However, Churchill’s concerns were probably more about preserving the Victorian empire and a belief in the importance of trade links. Source E offers a justification in terms of eventual outcome, but this may be questioned. By the 1930s there was a consensus for change as Source A shows, and a gradual constitutional reform was seen as inevitable and more likely to prevent extremism and communalism. In 1931 Churchill was something of a failed politician and may well have played the Indian card with some irresponsibility. If subsequent events proved some of his concerns to have some justification, then that does not entirely justify his position in 1931. In terms of both assessing the needs of India and of his reputation within his own party and with the influential political leaders of his day, it could well be argued that Churchill had misjudged and candidates are free to assess how serious this was.

Page 75: History A Mark Schemes for the Units January 2010 - The ...

F964 Mark Scheme January 2010

F964 European and World History Enquiries

Option A: Medieval and Early Modern 1073-1555 The First Crusade and the Crusader States 1073-1130 1 The People’s Crusade

(a) Study Sources A and B Compare these Sources as evidence for the popular response raised by the preaching of the Crusade. [30]

Focus: Comparison of two Sources No set answer is expected, but candidates need to compare the contents, evaluating such matters as authorship, dating, utility and reliability, so using the Sources ‘as evidence for...’. The headings and attributions should aid evaluation and reference to both is expected in a good answer. A makes much of the divine nature of Peter’s appeal (‘little short of divine’), in the context of his audience and of the enthusiasm generated; B echoes elements of this but also points up the sense of a lack of control, an over-confidence, a belief in success, all features, arguably, of excessive popular enthusiasm. Both focus on the role of Peter; both imply receptive audiences, audiences waiting for a message. B refers to soldiers as well as unarmed men, women and children, adding to the sense of the range of appeal. The reference to palms and wearing crosses is of note. The tone and language of both Sources can be engaged here. So, too, the provenances, and, given the apparent hostility of the Byzantine rulers towards the People’s Crusade, the authorship of B may be significant. Source B mentions Peter ‘inspired people’, the wide range of those affected. The Source makes much of ‘palms and wearing crosses’ (links here to Palm Sunday) and of the mass response. There is a strong sense of popular involvement. Source A mentions ‘gifts’, ‘holiness’, ‘godlike’. Both point to a sense of the charismatic popular preacher as well as to a mass longing for such a leadership with purpose. A mentions ‘the common people’, ‘crowds of people’, while B mentions ‘unruly, difficult, restless’ supporters. Comments on the provenances will aid evaluation. Authorship is important. Both assess from hindsight, Anna from a highly privileged Byzantine perspective which expresses horror at the disorderly aspects of the Peoples’ Crusade and may well exaggerate this. The gap in dates may be viewed as important. A comes from a reliable source and offers insight while B, though later, comes from a source usually seen as critical of the crusaders yet here having some positive comments to make. Sympathy and empathy in A, a more cautious, possibly semi-critical viewpoint in B, can be assessed as well and the provenances linked to the contents and their nature.

(b) Study all the Sources

Use your knowledge to assess how far the Sources support the interpretation that Peter the Hermit was an ineffective leader. [70]

Focus: Judgement in context, based on the set of Sources and own knowledge Successful answers will need to make use of all five Sources, testing them against contextual knowledge and evaluating their strengths and weaknesses, any limitations as evidence. A range of issues may be addressed in focusing upon the terms of the question but no set conclusion is expected. The interpretation finds support in Sources C, some of D and some of E, while Sources A, B, some of D and E point to other possible factors; D says that the Emperor did send help in the end having neglected the crusade beforehand. Source E presents an overview and sees some merit in Peter’s leadership and mentions

72

Page 76: History A Mark Schemes for the Units January 2010 - The ...

F964 Mark Scheme January 2010

the nature, size and scale of the enterprise (linked to A and B) as well as sense of the loss of control by Peter (as in C). Peter is seen as having an impact as a preacher and a leader (A, B, E) but also as lacking the necessary qualities as a military and political leader (especially in C and E). His role was important but there were other factors involved (E is useful here), some related to his leadership, others less so (for example, the sheer military power and ferocity of the Turks his force encountered). Apart from the issues of the attitude of Alexius I, the Byzantine Emperor, and the nature of Peter’s leadership, military factors are raised in C and D, and suggested in E. A and B point to the intensity of responses to the call for a crusade. Source B does include the warning from the Byzantine Emperor. Source C mentions Peter’s visit to Constantinople, and the Turks ‘full of glee’, ready to attack. Source D also mentions Constantinople and conveys a sense of followers abandoned to the Turks. The popular element of this Crusade was large but the force was ill-equipped and ill-trained. Contextual knowledge can supply some details of the Crusade and its fate as well as of the role of the Byzantine Emperor; this Crusade had flaws in leadership, organisation, weaponry (Source A points to the advantages that princes and nobles had in preparations, resources); there is a wider issue of the Byzantine response to the presence of this Crusade, indeed of the Crusade as a whole. Source E gives a succinct survey of some of the key ‘requirements for success’ and these can be exemplified by reference to the problems encountered by the People’s Crusade. Peter was a charismatic preacher, but he was no military leader. The peasant-based force he gathered was transported into Asia Minor by Alexius I and, it could be argued, abandoned there. The lack of control by Peter, evident early on, may be viewed as significant; so, too, the very nature of this Crusade, as evidenced by references to its make-up in B and E. Both B and D point to the attitude of the Emperor; the reference to ‘permission’ in D is of note. The provenances of the Sources can add to analysis here; so, too, the tone, not least of C (the Emperor rejoiced, survivors were disarmed). Candidates who make valid comments on provenance should be rewarded. References to the Emperor in C and D could be used to qualify the reliability of Anna in B who writes from a generally pro-Byzantine stance. The author of C, as an ex-Crusader, might be deemed to have greater knowledge of conditions those of A and B, whose eye-witness accounts may not have been fully reliable. The time lag in B and D is also significant in assessing reliability. Topic knowledge will add in points about (for example) the general attitude of the Emperor to the Crusaders and about the military problems facing all crusaders as well as the Turkish advantages in battle against unprepared, undisciplined opponents.

73

Page 77: History A Mark Schemes for the Units January 2010 - The ...

F964 Mark Scheme January 2010

The German Reformation 1517-1555 2 Reactions to Luther and his Ideas 1519-21

(a) Study Sources A and C Compare these Sources as evidence for Catholic reactions to Luther’s teachings. [30]

Focus: Comparison of two Sources. No set answer is expected, but candidates need to compare the contents, evaluating such matters as authorship, dating, utility and reliability, so using the Sources ‘as evidence for…’. The headings and attributions should aid evaluation and reference to both is expected in a good answer.

The provenance of the Sources is a good starting point for a comparative analysis here. The author of Source A is an internationally respected and influential humanist, thought by some to have ‘helped him write his books’, though he seems to be trying to distance himself from the association. On the other hand, the author of Source C is Pope Leo X, the target of much of Luther’s criticism. Their authority is very different. Erasmus has intellectual and theological authority but no official religious authority within Germany. The audience of Source A is Albrecht of Mainz, who had aided the Pope in the sale of indulgences, and a corrupt churchman himself whereas the audience of Source C is Frederick the Wise of Saxony, Luther’s prince, whom the Pope hopes may be able to suppress Luther’s heresy even though he lies in saying that Frederick has never favoured Luther, opening the way for Frederick to distance himself from supporting Luther. This would avoid further confrontation before the Bull ‘Exsurge Domine’ becomes final and perhaps limit the damage to the Church: hence his purpose in writing to Frederick. Erasmus, on the other hand, has the purpose of trying to save his own reputation from being dragged down by Luther’s heresy from which he wishes to distance himself.

The Sources are similar, in that Source A says ‘All they can say is ‘heresies’, and Source C shows the Pope, having declared Luther a heretic, complaining that he ‘ignores the punishment of heretics, papal decrees and church councils’. Source A states that ‘churchmen publicly ridicule him with their crazed howling’, and the Pope, in Source C, calls Luther ‘that son of iniquity’, ‘mad’ and ‘a scabby sheep who infects the flock’. Source A suggests that his enemies wish only to catch and crush Luther, and similarly, in Source C, the Pope suggests Frederick takes him captive to suppress his views. Erasmus, in Source A mentions ‘the vicious venomous lies’ with which his enemies tear Luther apart, and in Source C the Pope is claiming Luther believes only his own opinion, whereas in Source A Erasmus states that Luther’s ideas are based on St Augustine, so the Pope would seem to be unreliable.

The Sources therefore also differ. In Source A, Erasmus is suggesting Luther should not be crushed, whereas in Source C the Pope is trying to suppress his views. Whereas Erasmus sees Luther’s teachings as showing ‘brilliant sparks of Gospel learning’, the Pope, in Source C, says Luther ‘perverts the faith’, a claim which Source A refutes as ‘they are considered orthodox, even godly’ in St Augustine. Source A suggests Luther’s critics are unlearned, and ‘have never read a word Luther has written’ whereas Source C is written by the Pope, the fount of canon law and Roman Catholic doctrine, who has selected particular errors by which Luther has ‘seduced the simple’. But whereas Erasmus, in Source A, says that Luther’s enemies would prefer him to be a ‘dead man rather than a good man’, Pope Leo, in Source C, is offering Luther clemency if he ‘returns to his sanity’.

74

Page 78: History A Mark Schemes for the Units January 2010 - The ...

F964 Mark Scheme January 2010

Brief comments on context must be credited only in so far as they aid the comparison. The recent publication of the Bull ‘Exsurge Domine’ urging Luther to recant within sixty days has made it more urgent to make one final attempt to put pressure on Luther by targeting his princely patron.

A supported judgement should be reached on their relative value as evidence. Source C is a last attempt at persuasion by the highest authority in the Roman Catholic Church, whereas Source A is a request for support from a leading German churchman in holding back the tide of the attack on Luther and justifying him, to prevent Erasmus getting tarnished by his previous association with Luther. They therefore represent the divided reactions within Germany at this time. Both are useful as evidence for ‘behind the scenes’ views, but Source C shows more of the official face of the Roman Catholic Church, asserting papal authority but also trying one last attempt at persuasion. No set conclusion is expected, but substantiated judgement should be reached for the top levels of the Mark Scheme.

(b) Study all the Sources

Use your own knowledge to assess how far the Sources support the interpretation that the main reason the authorities failed to suppress Luther’s heresy was because he had influential supporters. [70]

Focus: Judgement in context, based on the set of Sources and own knowledge. Successful answers will need to make use of all five Sources, testing them against contextual knowledge and evaluating their strengths and weaknesses, any limitations as evidence. A range of issues may be addressed in focusing upon the terms of the question but no set conclusion is expected.

The Sources contain references to different interpretations, so they may be grouped according to their view. The supporting view is shown predominantly in Sources A, C, D and implicitly E, whereas the opposing view features in Source B and explicitly in C, D and E, though the reference in C is false and misleading in E.

The supporting view of influential religious patronage is in Source A, while political patronage features in Sources C, D and implicitly E. Source C is an appeal to Elector Frederick of Saxony who had not pressured Luther to recant and supported him. Knowledge might be used to explain that the Pope’s attempt to bring Luther to Rome to face trial had failed due to Frederick’s support for Luther’s trial within Germany, where he had popular support. Frederick’s military influence is hinted at In Source D, by the reference to defending Luther with a ‘military bodyguard’. ‘Support with their resources’ also implies Frederick, but in Source C, the Pope denies that Frederick has supported him rather than accuse Frederick openly, due to his power in the Empire. The hint lies in the ‘lofty and dazzling dignity’ mentioned. The Pope had sent Frederick the coveted ‘Golden Rose’ to try and gain his support for the papal candidate in the Imperial election in 1519. Other princes are also said to have left Worms, in the introduction to Source E, before Charles V feels able to issue the Edict of Worms, suggesting Luther had influential princely support.

The Sources also support the opposing view. Source E shows that the most influential patron in Germany, Charles V, did not support Luther and was attempting to suppress his heresy. Knowledge might be used to explain the Emperor’s role as the political arm of the papacy, but also to assess the extent of his influence over the princes of the Holy Roman Empire. But though the Emperor is asserting that Luther is hated by all God-fearing persons, he contradicts this by implying that Luther has been protected and supported, not only by influential patrons but by the wider German public, who have published, bought, read and sold his books. Knowledge of the part played by the printing press might aid evaluation. However, Charles’ audience is the more compliant group of princes remaining

75

Page 79: History A Mark Schemes for the Units January 2010 - The ...

F964 Mark Scheme January 2010

at Worms after Frederick and Luther’s other princely supporters have left. Charles’ hope to ban or capture Luther remains in vain as long as his patrons and the German public continue to support him, as perhaps shown by using knowledge of Frederick’s secret ‘kidnap’ of Luther for his own protection, his lodging in the Wartburg and safe return to Wittenberg.

Further limitations of his support are suggested in Sources B and D. The author of Source B is not a patron, but a Luther supporter, with pupils he may influence. He shows this by his comment about ’wise, learned men’, suggesting that Luther may have some theologians on his side, such as Erasmus in Source A, but he has not gained majority support among the audience of his disputation with Eck. Erasmus at face value is attempting to gain Luther the patronage of Albrecht of Mainz, but knowledge of his part in the sale of Indulgences and his purchase of his office would suggest this is unlikely to happen. Rather Erasmus may be trying to gain himself an influential patron to protect him from association with Luther. Source C reveals that the simple folk, who are among Luther’s main supporters, may be influential, though not patrons, and are a large group who have been ‘seduced by him’. These are perhaps among the ‘god-fearing persons’, the ‘faithful subjects’ to whom Charles is appealing in Source E. Fear of civil war is always in the background.

The provenance of the Sources should be integrated into the discussion. The authorship, tone, audience and purpose of the Sources are particularly revealing, as shown above.

Supported overall judgement should be reached on the extent to which the Sources accept the interpretation that influential patrons were the main reason why the authorities failed to suppress Luther’s heresy. No specific judgement is expected.

Candidates are likely to consider a range of reasons within the Sources: the influence of patrons, support from theologians, hesitation by the authorities to condemn Luther, outbursts and lies which attracted support for Luther, the printing press, the weakness of the Emperor and public adulation. They are likely to set the Sources within the context of Luther’s condemnation for heresy. It is up to candidates to assess and decide upon relative importance here, there being no set conclusion.

76

Page 80: History A Mark Schemes for the Units January 2010 - The ...

F964 Mark Scheme January 2010

Option B: Modern 1774-1975 1 (a) Study Sources A and D.

Compare these Sources as evidence for opinions about Robespierre. [30]

Focus: Comparison of two Sources No set answer is expected, but candidates need to compare the contents, evaluating such matters as authorship, dating, utility and reliability, so using the Source ‘ as evidence for…..’ The Headings and attributions should aid evaluation and reference to both is expected in a good answer.

Content: Source A sees Robespierre as eloquent and a famous man whose speeches will be read in the future. Source D sees him as puffed up with pride and not revered but the butt of sarcastic comments by his fellows and of contempt by at least one member of ‘the people’. Robespierre is not being ‘elevated to the sky’ by mercy in D, but rather by eccentric delight in a pseudo-religious cult. The writer of A has affection for Robespierre as an old friend, D writes in a hostile and ironic way (With what joyful pride! Which contrasts with A’s ‘My dear Robespierre’). There is no suggestion in A that Robespierre has theocratic ambition and D does not suggest that he has been excessively bloodthirsty and severe in the way that A does. Robespierre’s popularity is directly mentioned in D and rather more indirectly in A in which his writings are thought to be likely to be read by posterity. A is a warning; D is a judgement. They both reflect criticisms of Robespierre among the political elite – Source A by its nature of a warning and Source D by the reporting of Robespierre’s mocking colleagues.

Provenance: Source A is a public article intended to be a clear warning against violence and immoderation, a warning that cost Desmoulins his life, despite his friendship with Robespierre. Source D is a public document, but not one that was so bravely written, as Robespierre had fallen and was dead by the time it was disseminated. It reflects on his fall while, in contrast, A is trying to prevent it. Both men were deeply involved in the politics of their day– therefore these are political ‘insiders’. Note the dates – A is written before the massive increase in terror which it failed to prevent; D after the terror which helped (see C) to bring about Robespierre’s fall. A is still full of the revolutionary idealism typical of this author, whereas D seems much more cynical in tone – with Robespierre’s idealism being mocked and his popularity creating envy – however, note the rapturous crowds. In terms of making a judgement about their utility, A could be seen as more useful from someone who knew Robespierre or B could be seen as more useful as revealing more about Robespierre’s actual rule. Neither is an objective source, but it could be argued that A is more balanced. No set answer is expected.

(b) Study all the Sources.

Use your own knowledge to assess how far the Sources support the interpretation that the main reason for the fall of Robespierre in 1794 was because his rule was dominated by the policy of Terror. [70]

Focus: Judgement in context, based on a set of Sources and own knowledge. Successful answers will need to make use of all five Sources, testing them against contextual evidence and evaluating their strengths and weaknesses, any limitations as evidence. A range of issues may be addressed in focusing upon the terms of the question but no set conclusion is expected. The debate here is whether the Terror brought the end of Robespierre, or whether it was Robespierre’s potential to be a dictator based on his popularity with the populace together with his increasing eccentricities and religious views. Did the easing of the external threat make his extremism less necessary or was the key the split in the convention and the

77

Page 81: History A Mark Schemes for the Units January 2010 - The ...

F964 Mark Scheme January 2010

committee about the irrational desire for Reason and the alienation of the propertied classes?

A and C stress concerns about terror – one before it reached its height by a personal friend and associate; C a private letter which does recognize the positive achievements – so neither by a necessarily unsympathetic or counter-revolutionary source, but both expressing concerns about violence. Neither may be typical – both are Paris-based and both supporters of revolution. Provincial opinion may have been more strongly against Robespierre and many may have found the Terror obliterated any good opinions about the Committee of Public Safety. Candidates may use own knowledge of the terror to good effect here. D and, by implication, B do not refer to violence but are evidence of unease about the eccentricity and pseudo religious views of Robespierre. B seems quite ridiculous with all manner of elements being celebrated on holy days. As a Decree its authenticity cannot be questioned, but the extent to which it reflected opinion in both government and country might be considered and how far it reflected aspiration rather than reality. This might be linked to other manifestations of extreme change and perhaps too to the association of Revolution with impiety and anticlericalism that fuelled provincial unrest. The scepticism, therefore, shown in D may be typical and candidates might know the relief in the Thermidor period when the fatuous processions, emblems and rhetoric stopped – and of course the violence. What D stresses is the splits in the revolutionary elite by 1794 and candidates may know the background of the receding threat from invasion and the reaction against the political murder of Robespierre’s previous opponents – Roland, Danton etc. ‘Liberty what crimes are committed in thy name’ etc. E tries to balance but the negativity of Robespierre’s political vision comes out strongly and this can be assessed in the light of knowledge of the terror and the political in-fighting. The fanatical self-belief is supported by D and the repression by C. The reliance on Robespierre on sans culotte support in C may be picked up and linked to own knowledge of his rise and appeal and also cross referenced to the envy shown in D.

2 (a) Study Sources B and D Compare these Sources as evidence for Cavour’s influence in Piedmontese politics in the period 1851-57. [30]

Focus: Comparison of two Sources No set answer is expected, but candidates need to compare the contents, evaluating such matters as authorship, dating, utility and reliability, so using the Sources ‘as evidence for …’. The headings and attributions should aid evaluation and reference to both is expected in a good answer. Candidates are likely to highlight some of the following factors which may be described as similarities or differences according to the emphases adopted by candidates. Both sources refer to the limited popularity of Cavour and the impact this had on his political actions. In Source B the suggestion is that Cavour adopted ‘fine liberal sentiments’ as a way of reinforcing ‘public faith in his liberalism’ and in Source D his reluctance to reform the press laws is explained by his ‘fear of lowering his own popularity’. The challenge Cavour faced from the Right is stressed in both sources with Source B implying it was increasingly confident to act independently of the government and Source D refers to a swing to the Right in the elections of November 1856. The connubio is identified as a key factor in Piedmontese politics with Source B explaining how Cavour created it and Source D confirming Cavour’s continued reliance on it. The way French affairs impacted on Cavour is referred to in Source B explaining how Napoleon III’s coup weakened the Left in Piedmont, to Cavour’s advantage, and Source D explicitly demonstrates the influence the French tried to exert and the support they enjoyed from the King which, by implication, weakened Cavour’s position. Most are likely to agree that an obvious difference is the position of Cavour for in Source B he is described as ‘in complete control’ but in Source D his position is ‘seriously weakened’.

78

Page 82: History A Mark Schemes for the Units January 2010 - The ...

F964 Mark Scheme January 2010

Candidates may claim the sources are reliable as they are both reports intended to be a matter of record. On the other hand each comment contains an element of subjective opinion. Candidates will consider the author of Source B to be either supportive or critical of Cavour depending on their interpretation of the references to Cavour’s speeches and the views expressed about the conduct of cabinet business. Either way the author was a member of the Cabinet with direct knowledge of the politics of the time. The accuracy of the comments of the author of Source D may be considered questionable as the views of one observer only and those of an outsider. The utility of the sources may be considered. Both provide an insight into the political manoeuvrings of the time and some may be able to substantiate with reference to the division between d’Azeglio and Cavour in 1852 and the rise of the Right in 1856 based on the anti-clerical policies of the previous years. Some may identify the comments in Source D about attacks on the Emperor with disappointment in Piedmont that France appeared reluctant to promote the Italian cause following the expectations of the peace of Paris in 1856.

(b) Study all the Sources

Use your own knowledge to assess how far the Sources support the interpretation that Piedmont developed into a liberal state in the 1850s. [70]

Focus: Judgement in context, based on the set of Sources and own knowledge. Successful answers will need to make use of all four Sources, testing them against contextual knowledge and evaluating their strengths and weaknesses, any limitations as evidence. A range of issues may be addressed in focusing upon the terms of the question but no set conclusion is expected.

How candidates define ‘a liberal state’ will determine the shape and direction of their response. Many are likely to group the sources into those that suggest Piedmont developed into a liberal state (Sources A, B and D) and others that suggest the opposite (Sources C and E) although there is room for debate within most of these sources. Some might adopt a thematic approach and consider the economic, political and religious constituents of the liberal state. If so they may treat Source A as indicative of a liberal trade policy. Sources B, C, D and E reflect the nature of political issues and Sources C and E religious matters. There is scope for differences of emphasis in the interpretation of these sources and the evaluation of their provenance Tariff reform in pursuit of free trade, championed in Source A, was a basic element of the liberal state of the period. Candidates should add details about the trade treaties agreed with foreign states and the increased competition opened up in Piedmont. They could also expand on the reference to ‘economic progress’ and explain how industry, agriculture and transport were modernised in the 1850s. However, the fawning tone of Cavour’s remarks, intended to win Cobden’s support for the engineer despatched to England, exaggerates the strength of the free trade lobby in Piedmont especially as the argument for free trade was not as secure as Cavour implies. Candidates might argue that Sources B, C and D confirm parliamentary government – a key feature of a liberal state – functioned in Piedmont. Further, the dominant politician of the period, Cavour, is portrayed as committed to liberalism (Sources B and D) and Source D makes it clear that a free press existed. Knowledge of the powers of Parliament, the political manoeuvrings described in Sources B and D and the type of criticism directed against the French Emperor referred to in Source D could be considered in support of these points. On the other hand, some will detect traces of authoritarianism in all three sources. Source B implies that cabinet government was undermined by Cavour’s actions and dominance. In Source D the freedom of the press is considered of little worth in terms of its influence. In Sources C and D the King appears to be conservative in his attitude to Church reform and press freedom. Candidates may comment on the opportunism of Cavour as a politician or the limitations of the Statuto such as the considerable powers of

79

Page 83: History A Mark Schemes for the Units January 2010 - The ...

F964 Mark Scheme January 2010

the King which was why the proposed marriage laws referred to in Source B were dropped. In December 1851 d’Azeglio was prepared to restrict the powers of the press to appease the political right. In Source E Acton argues that the pursuit of ‘the greatness of the State’ was more important to Cavour than ‘the liberty of the people’: candidates could refer to the events of 1859-61. The charge that freedom was alien to Piedmont could be explored by comparisons with England, even America. The specific interests of the authors of Sources B, C and D might be considered. Sources C and E illustrate the move to a more liberal secular State. ‘The law on religious orders’ mentioned in Source C refers to the closure of some monasteries. The King’s opposition to the law is implicit in the source which was consistent with his religious convictions, demonstrated in his hostility to the earlier marriage bill referred to in Source C. Some may stress the Pope’s threat to excommunicate anyone supporting the reform to explain the fawning tone of the King’s letter. Candidates might highlight the public anger against the Pope’s intervention as an indication of the strength of liberal opinion in Piedmont although loyalty to the Church, emphasised in the final lines, highlights the depth of conservatism. At face value Source E views Piedmont as an illiberal state and there is evidence to support certain accusations. D’Azeglio did introduce ecclesiastical reforms without consulting the Church, most notably the Siccardi Laws of 1850. That ‘her governments were profoundly hostile to the Church’ was true in so far as both d’Azeglio and Cavour were resolutely anti-clerical. However, the author was a Catholic whose loyalty to the Church could explain his opposition to the religious reforms of the period. Further, Acton ignores Cavour’s pledge to allow a ‘free Church in a free State’. Despite this Lord Acton was committed to liberty in the broadest sense so his views are not entirely prejudiced.

80

Page 84: History A Mark Schemes for the Units January 2010 - The ...

F964 Mark Scheme January 2010

3 (a) Study Sources A and B Compare these Sources as evidence for the right of South Carolina to nullify the Tariff Law. [30]

Focus: Comparison of two Sources. No set answer is expected, but candidates need to compare the contents, evaluating such matters as authorship, dating, utility and reliability, so using the Sources ‘as evidence for …’. The headings and attributions should aid evaluation and reference to both is expected in a good answer.

The sources disagree in every respect. Calhoun (A) considers nullification to be legal, describing the exercise of the veto as ‘legitimate’ whereas Webster (B) states clearly that nullification is ‘unlawful’. The justification offered for these positions differ too. Calhoun argues that the Constitution was formed by the will of the States which is denied by Webster who claims ‘the people erected it’. In similar vein Calhoun asserts ‘that no authority is higher than theirs’ (the States) in contrast to Webster who states ‘the laws of the United States are supreme’. To emphasise this further it is stated that intervention by the General Government would mean ‘violating the Constitution’. In contrast, Webster argues that resistance to ‘a law is treason which the US could not ignore’ implying that interference in the internal affairs of a State is justified. Calhoun implies nullification is vindicated to ensure liberty (first line) whereas Webster argues liberty is dependent on the Union, hence his reference to ‘Liberty and Union’.

The evaluation of the differences should consider the debate about States Rights and the impact of the Tariff. Calhoun was invoking some of the notions raised earlier by Jefferson and Madison in the Kentucky and Virginia Resolutions respectively which were interpreted by some as justifying nullification. The State of Massachusetts had consistently argued against any concession to States Rights so Webster’s views are unsurprising. The Tariff was resented in South Carolina because it was perceived to damage agricultural interests and, by implication, was a threat to the peculiar institution of slavery. Candidates should pick up on the vehemence of Calhoun’s hostility to the Tariff as ‘obnoxious’ and something that ‘impoverishes us’: it was called ‘The Tariff of Abominations’ in South Carolina. By contrast Webster considers the Tariff as ‘beneficent’ which can be explained because of the protection it offered northern manufacturers. Calhoun appears to favour accommodation with the General Government whereas Webster does not betray any doubt about his position. This was in part because Calhoun was the newly elected Vice-President with an obligation to uphold the integrity of the Union (which explains his desire for anonymity). Furthermore, Source A was written two years earlier when positions were less entrenched than when Webster was debating. Calhoun was trying to explain a theory whilst Webster was defending the status quo to an audience that was largely conservative on matters of the Constitution.

(b) Study all the Sources

Use your own knowledge to assess how far the Sources support the interpretation that the Nullification Crisis threatened the existence of the United States. [70]

Focus: Judgement in context, based on the set of Sources and own knowledge. Successful answers will need to make use of all four Sources, testing them against contextual knowledge and evaluating their strengths and weaknesses, any limitations as evidence. A range of issues may be addressed in focusing upon the terms of the question but no set conclusion is expected.

81

Page 85: History A Mark Schemes for the Units January 2010 - The ...

F964 Mark Scheme January 2010

Most candidates are likely to group Sources B, C and D as supportive of the interpretation with Sources A and E providing the counter-argument. Sources C and D consider nullification will result in the separation of South Carolina from the Union. Source C is effectively an ultimatum and provides a deadline after which the State will form its own government. Source D claims South Carolina is intent on secession. The reliability of both sources might be queried as they are public statements intended to stake positions unequivocally and to indicate the commitment of both parties to fulfil their duty; in the case of the Convention to articulate popular sentiment in the State and in the case of the President to show his determination to discharge his responsibility to uphold the Union. To some extent both may be seen as bluffs as both challenge the other side to make the first move: Jackson also appeals to the people of South Carolina by questioning the wisdom of those leading them. The fact that Jackson was a slave-owning Southerner, with sympathies for the South, might be assessed. However, candidates might argue that the threat to use force, made in both sources, is convincing. Following the revision of the Tariff in 1832 anger in South Carolina was widespread and radicals like McDuffie and Rhett in the Convention were sincere in the position expressed in Source C. Similarly, Jackson had just secured re-election for a second term so he was confident of a mandate to uphold the Union and, as an army general with a reputation for firmness, there is little doubt that he was sincere in his views. South Carolina was raising an army, as Jackson claims, if only in anticipation of having to defend itself. Candidates may cross refer to Source B which expresses similar concerns about the future of the Union even if they are less explicit. This may be because in 1830 there was still room for manoeuvre on nullification. At least Webster’s anticipation of a ‘broken union’ leading to ‘civil feuds’ suggests that he regarded the nullification issue to be a threat to the existence of the United States. Some may know that Webster retained an uncompromising stance on the issue till the very end arguing strongly against Clay in the debates of February 1833 so allowing some cross reference to Source E.

Sources A and E appear to refute the interpretation that nullification threatened the United States if for very different reasons. It is clear that Source A does not see any contradiction between nullification and the continuance of the Union. Calhoun clearly hopes for a peaceful solution to the problem of the Tariff and only as a last resort does he envisage applying a veto. Indeed, he is optimistic that the new President will address the concerns of South Carolina if only, perhaps, because as the new Vice-President, he believed he could represent the interests of the State directly in the White House. He does not refer to the separation of the State from the Union. At this stage the notion of nullification was only being defined as the introductory comments make clear so positions were more fluid and open-ended than was to be the case later. Source E explicitly rejects the idea that South Carolina ‘ever desired to become a separate State’ and as such refutes the interpretation that the nullification crisis threatened the existence of the United States. Clay bases his view on the impracticality of the State surviving out of the Union. Candidates might consider the financial, economic, political and military difficulties independence would create. In addition, Clay identifies a key weakness of South Carolina in that she lacked support ‘across the continent’. Candidates should know that no other State, even those in the South, supported her: indeed, many like Louisiana and Alabama publicly disowned her. Furthermore, the hostility of public opinion, alluded to by Clay, might be substantiated by the formation of so-called ‘Washington Societies’ of volunteers prepared to take action against South Carolina. Clay’s pitch may be seen for what it was: an attempt to win support for a compromise Tariff from both South Carolinians and nationalists, of which he was one, by his appeal to history. As such, some may argue he was glossing over the seriousness of the situation at a time when the Force Bill was being debated too.

82

Page 86: History A Mark Schemes for the Units January 2010 - The ...

F964 Mark Scheme January 2010

4 (a) Study Sources D and E Compare these Sources as evidence for the effectiveness of the Nazi regime’s attack on the Churches. [30]

Focus: Comparison of two Sources. No set answer is expected, but candidates need to compare the contents, evaluating such matters as authorship, dating, utility and reliability, so using the Sources ‘as evidence for …’. The headings and attributions should aid evaluation and reference to both is expected in a good answer.

Content: Similarities: Both sources are complaining of lack of success. Both refer to church attendance and both refer to restrictions put in the way of religious education. Both refer to the general absences from church services among young people. Both refer, albeit obliquely in D, to the influence of the Church on opinion. Differences: The obvious difference is that D is lamenting the continuing decline in church attendances among the population as a whole, especially among the young whereas E is lamenting the fact that, during war time, attendances are rising, again, apart from among the young. D implies that the Protestant church is effectively stifled or paralysed by rigorous Nazi activity whereas E is stating effectively the opposite, that the Church has more manpower at its disposal and is able to exert its influence over opinion, which it is unable to do in D. However, perceptive candidates might point out that in E the report suggests that the Catholic rather than the Protestant Church is exerting more influence. D refers exclusively to the Protestant Church whereas E refers to both Catholic and Protestant churches.

Provenance: There are obvious differences in the provenance and dates. D comes from a confidential report from Protestants in peacetime after five years of the consolidation of Nazi power. It comes from Bavaria where the Protestant Church was historically weaker, the Catholic stronger. Bavaria was also more supportive of the Nazis. E is a summary of reports from Nazi gauleiters in wartime where the effectiveness is questioned, especially amongst adults. Both are confidential summaries of reports, not aimed at the public and therefore inclined to be reliable, especially as both report negatively upon themselves. Both are of equal value.

In terms of ‘evidence for’, candidates should highlight the differences between peacetime and war time. The date of D is 1938, a time when the regime’s attacks on the Protestant Church have had mixed success; The ‘German Christian’ movement has largely failed, but, as pointed out in D, the regime has largely managed to woo youth away from Church influence and hinder religious teaching. The whole tone and content of D implies effectiveness.

In E, clearly the war time situation is fundamental. The date, mid 1943 is important. By this time, the tide has turned and the war is going badly. As more and more Germans read between the lines and realise this, naturally they turn more to the churches for spiritual comfort in the wake of huge losses at Stalingrad. Clearly then, by this time, the regime’s attacks on the Churches has largely ceased to be effective, particularly given the strain on manpower that the Churches would not be subject to.

83

Page 87: History A Mark Schemes for the Units January 2010 - The ...

F964 Mark Scheme January 2010

(b) Study all the Sources. Use your own knowledge to assess how far the Sources support the interpretation that the Churches were willing collaborators with the Nazi regime. [70]

Focus: Judgement in context, based on the set of Sources and own knowledge. Successful answers will need to make use of all four Sources, testing them against contextual knowledge and evaluating their strengths and weaknesses, any limitations as evidence. A range of issues may be addressed in focusing upon the terms of the question but no set conclusion is expected. In terms of grouping: A clearly suggests willing collaboration B suggests collaboration but for different reasons; necessity rather than willingness. The content of C is rather ambiguous, but collaboration of a kind particularly over the need to stamp out Communists is implied. In opposition to this view D strongly suggests weakness in the face of the regime, but not collaboration. E suggests clearly that not only have the churches survived, but are not collaborating. The supporting argument therefore is supplied by A and C, and the counter argument supplied by D and E Source B can be used to demonstrate that there was collaboration, but not ‘willing’ in all areas.

There is much in the five sources to construct an argument around. Candidates should use their contextual knowledge to evaluate the sources. The key element is to establish the existence of ‘collaboration’ and the extent to which this was ‘willing’. A shows clear collaboration, not in order for the Catholic Church to survive but because the Nazis were welcomed as a violently anti-bolshevik force. However, this was right at the beginning of the regime when the horrors were not yet apparent. It also comes from Catholic schoolteachers rather then the Church per se. Moreover, the churches were not alone in welcoming Hitler and fearing communists. Candidates can evaluate using contextual knowledge. There was much common ground between the Catholic Church and Nazi ideology over the issues of communism, anti-modernism in culture and the arts, anti-feminism, and, to an extent, anti-semitism. Much can be made of B. The context of course is the Catholic centre Party’s collaboration with Hitler in voting for the Enabling Law, thereby effectively destroying democracy. Candidates may pick up on the word ‘collaboration’ in the source which the author suggests (ironically) would not be possible in the future without the Party siding with Hitler in the vote. A clear sense of helplessness is evident here. However, good candidates will question whether this ‘collaboration’ was ‘willing’, or whether it was done out of necessity. C Implies a form of collaboration. There is continuing support for Hitler with protestations of loyalty, a misguided belief that he means to keep his promise made in 1933 not to interfere with the Church and is being ignored by the ‘little Hitlers’ There is the reference to ‘communists’ and ‘Marxists’ which can be cross-referenced with A. The source can be taken as a counter-argument against ‘willing collaboration’. Contextual knowledge can be used to point out that the Catholic Church did stand up and protest [Archbishop Galen, removal of crucifixes from schools etc] but only did so when its own interests were threatened. D Strongly implies helplessness, and can be cross-referenced with B, but not collaboration. Candidates should use own knowledge to evaluate here .The traditional mainstream of the Lutheran Church succeeded in breaking away from the German Christian Movement, and therefore did not collaborate.. However, this was at great cost as the content and tone of the source indicates. Though there is no implication of collaboration here, let alone ‘willing’, there is evidence of passive acceptance. E Clearly states neither ‘willingness’ nor ‘collaboration’. In fact, quite the opposite. However this is after nearly four years of war when discerning Germans are seeing through the propaganda and beginning to realise that the war might soon be lost. Death and destruction post-Stalingrad would account for religious revival. Candidates should point out the obvious differences between the relations between the regime and the churches during peacetime and wartime.

84

Page 88: History A Mark Schemes for the Units January 2010 - The ...

F964 Mark Scheme January 2010

5 The USA and the Cold War in Asia 1945-75 American Policies in Asia 1945-1950

(a) Study Sources C and E

Compare these Sources as evidence for US strategies for the military security of Asia between 1945 and 1950. [30]

Focus: a comparison of Sources No set answer is expected, but candidates need to compare the contents, evaluating such matters as authorship, dating, utility and reliability, so using the Sources ‘as evidence for …’. The headings and attributions should aid evaluation and reference to both is expected in a good answer.

The Sources agree that the main strategy is ‘to rely on the people attacked to resist’ in Source C, supported by the arming of South Korea to prevent border raids in Source E. They agree that a defensive perimeter is in place to hold back enemy advance - the defensive perimeter in Source C, and the thirty-eighth parallel in Source E. In both, the US strategy is to work with the United Nations in a collective security system to prevent or stop war. Both Sources agree that there are global considerations: ‘other pressing problems’ in Source C and threats to independent nations in Source E. Source C says that the USA cannot guarantee ‘other Asian Areas against military attack’, and this has been shown to be the case in Source E, where mainland Korea has been invaded. Even here the US makes no mention of using ground troops, only air and sea cover for the South Koreans.

But the Sources also disagree. Source C prioritises Japan and the Philippines as part of the chain of offshore islands of the ‘defensive perimeter strategy’, whereas Source E refers to the invasion of Korea, on mainland Asia, an area which had received no US military protection and was excluded from the defensive perimeter. This is shown by the South Korean forces being armed for ‘internal security’ only and proving unable to resist attack without US support now being supplied after the event. Source E suggests that the Communists have moved from subversion to open war, whereas in Source C the enemy is not identified, but a general global threat is identified.

Contextual knowledge may be used to show understanding of the comparison. Source C is set in the context of America’s confidence being shaken by the ‘fall’ of China to Mao’s CCP, creating a powerful communist bloc in Asia and public disquiet at home. McCarthyist criticism leads Truman into a more active policy towards Asia and he may wish to prove he has no communists within his administration. Nonetheless Acheson is concerned to put limits on what the US can do in mainland Asia. By the time of Source E, the Cold War has intensified and America has lost its nuclear monopoly after the successful Soviet nuclear test in August 1949. NSC 68 has been secretly proposed in April, suggesting a tripling of military expenditure and Truman is wishing to gain support in Congress in order to adopt a more forceful policy.

The provenance of the Sources may be used to determine which is more useful or reliable for explaining US military strategies. The author of Source C is the Secretary of State, Dean Acheson, who had established the half-hearted ‘defensive perimeter strategy' and failed to contain communism in Asia, a strategy he sticks with in C, whereas that of Source E is the President himself, so Source E carries extra weight. Truman’s purpose is to pave the way for a massive increase in funding from Congress for a more forward policy to rescue his and America's reputation.

A supported judgement should be reached on which Source provides better evidence. Source E is more useful and reliable in that it shows the strategy the President wishes to adopt rather than the ill-advised exposure of US weakness by his Secretary of State in the

85

Page 89: History A Mark Schemes for the Units January 2010 - The ...

F964 Mark Scheme January 2010

wake of the loss of China. However, Source E explains why the strategy in source C had to be changed. (b) Study all the Sources

Use your own knowledge to assess how far the Sources support the interpretation that US policy in Asia was weak in the period between 1945 and 1950. [70]

Focus: Judgement in context, based on the set of Sources and own knowledge. Successful answers will need to make use of all five Sources, testing them against contextual knowledge and evaluating their strengths and weaknesses, any limitations as evidence. A range of issues may be addressed in focusing upon the terms of the question but no set conclusion is expected.

The Sources contain references to both sides of the argument, so they may be grouped by interpretation. The view that US policy was weak, in achieving its aim to contain communism, is based on the key argument that the USA became too reliant on its nuclear monopoly, prioritised Europe and failed to realise the strength of Asian national movements. This view is predominantly in Source B and to an extent in Sources C, D and E, whereas the view that US policy was strong is in Sources A, D and to an extent E.

The view that US policy was weak is stressed by Source B, that though US policy was outwardly strong and aggressively imperialist, it was weak because it depended on the atomic bomb to back the spread of western ideologies. It calls the US atom bomb a 'paper tiger', the nuclear weapons underpinning defensive perimeter strategy boasted about in Source A merely alienating local peoples. The view is that the USA would not dare use atomic weapons again after the bombings of Hiroshima and Nagasaki in August 1945, which might be supplied from own knowledge. Source C may be linked to this view, suggesting ‘it is wrong to become obsessed with military considerations in Asia . . there are other pressing problems’. Own knowledge might be used in evaluation: the USA had made military cutbacks, as it had no tradition of taxation for the armed forces in peace-time. It prioritised defence of democracy in Europe. Source A talks of bases from ‘Alaska to the Azores in the South Atlantic’ and other global commitments, and Source B, ‘to dominate the world’. This point might be evaluated as 'US overstretch'. Source C suggests that the USA has prioritised Japan and the Philippines within the Asian defensive perimeter, omitting Korea. Own knowledge might be used of the policy of ‘model’ democratic states, a weak containment policy, as it depended on corrupt puppet rulers. Acheson in Source C expects local peoples in Asia to defend themselves, 'we rely on the people attacked to resist'. Sources B and D agree that 'Asian affairs will be settled by the Asian peoples themselves'. Knowledge of the fall of China to communism by the time of Sources C and D reveals the weakness of Truman's administration, especially Acheson's policy. Source E supports the view in C that the USA felt they could not defend other areas of Asia, and Source D is an example of American supplies to the Nationalists failing to be sufficient to prevent the fall of China. Own knowledge might be used to develop this idea, such as the misuse of aid to the Chinese Nationalists. This links to Source E where US 'air and sea forces' are to cover and support South Korean troops but there is no mention of US ground troops being sent except as part of UN forces, showing a weak policy. Knowledge might be used of the involvement of the UN to shield the US from the threat of a global nuclear war. Discussion of subjective provenance might mention the anti-American standpoint of B and D, both members of the Chinese Communist leadership, while the authors of C and E are Truman and his Secretary of State, much criticised by Republicans and McCarthyists. Own knowledge of this and of strong Soviet support for Communist China might be used to develop this view.

86

Page 90: History A Mark Schemes for the Units January 2010 - The ...

F964 Mark Scheme January 2010

87

US weakness can be contrasted with the confidence and strength of the view in Source A, which supports the view that US policy was strong. Source A takes a confident line based on the US nuclear monopoly a few months after they had defeated Japan with the dropping of atomic bombs on Hiroshima and Nagasaki. It suggests that off-shore defensive bases are all the strength needed for defence of Asia and the wider world, although it assumes a willingness to use them (‘guts’). Own knowledge and provenance might be used to evaluate this view - the USA had recently emerged as the stronger of the two superpowers, but the subjective, outspoken tone of Source A reveals this as unreliable, as the introduction mentions. This was a misplaced confidence based on the delusion it could be had on the cheap. Source D might also be used for this argument, that US policy was outwardly strong and aggressively imperialist, killing millions and removing freedoms by supplying arms and aid to Nationalists in the Chinese civil war, although Chou En-lai is clearly exaggerating US intent (annexation of China). Knowledge might be used to develop this point - at the time of Sources C, D and E the USSR had its own nuclear capability, ending the US monopoly. By 1950 also the USSR had formed an alliance with Communist China, explaining the more aggressive policy forced on the USA in Korea in Source E, though under a UN umbrella.

Candidates are likely to consider a range of themes within the Sources: strengths and weaknesses of defensive perimeter strategy, reliance on the atom bomb and weakness in cutting back US military forces, prioritising Europe over mainland Asia. They are likely to set the Sources within the shifting context of the Cold War. It is up to candidates to assess and decide upon relative importance here, there being no set conclusion.

Page 91: History A Mark Schemes for the Units January 2010 - The ...

F966 Mark Scheme January 2010

F966 Historical Themes

Option A: Medieval and Early Modern 1066-1715 English Government and the Church 1066 – 1216 1 Assess the view that reasons for rebellion remained the same in the period from

1066 to 1216. [60]

Rebellions happened at times across the period. They include those from 1066 to 1070, 1075-6, 1088, 1095, the rebellion against Stephen which ended in civil war, the Great Rebellion of 1173-4 and rebellion against John from 1215-6. Examiners should not expect equal treatment of all of these. Some reasons for rebellion are seen throughout the period and, although they are not all present in all rebellions, rebellions were multi-causal. Reasons include support for an alternative ruler. This is seen in 1088 when Odo of Bayeux supported Robert of Normandy instead of William II, in 1095 when the rebels planned to replace William with his nephew Stephen, in the civil war in Stephen’s reign when some of the barons supported Matilda’s claim, and in the 1173-4 rebellion where rebels supported Young Henry. The strength of royal government was also a cause of rebellion: in 1075-6 Roger of Hereford balked at William’s sending sheriffs to hear pleas on his marcher lands; in 1095 Robert of Mowbray resented royal interference in his fief; one of the reasons for the 1173 rebellion was the tension caused by 20 years of strong government and 40 years later, reaction to the strength of Angevin rule was an important factor in the rebellion against John. Some barons rebelled because of the problems caused by the continental possessions, especially the difficulty of serving two different lords. This is seen in 1088 when barons were faced with the prospect of serving Robert of Normandy for their lands in Normandy and William II for their English lands, and from 1144 in Stephen’s reign. Continental possessions caused other problems too, especially the heavy taxation needed to retain them, a possible factor in the rebellion against Henry II, or to try to win them back, certainly a factor in the rebellion against John. Behind much of this, throughout the period, lay baronial self interest, the belief that barons’ rights were being undermined in some way, as can be seen from some of the clauses of Magna Carta, and/or that they could win better personal rewards from another ruler. However, there are also reasons which apply only to specific rebellions. Those from 1066 to 1070 were reactions to the invader. The 1088 rebellion and that against Stephen were partly the result of disputed succession. The rebellion against John was prompted partly by military failure abroad. The rebels of 1215 also arguably took up arms not only against Angevin government but because they wished to enforce some regulation of the king’s rights. The best responses are likely to be analytical examinations of a range of reasons, looking at both continuity and change, and evaluating how consistent they were across the period. They will probably point to the multi-causal nature of all rebellions but might also examine how far there is a change in emphasis on particular reasons in different rebellions. Most candidates will deal with a number of rebellions, analysing reasons and reaching a conclusion. Weaker responses will probably do this sequentially, typically describing rebellions and then deducing reasons. Least effective responses are likely to deal with only continuity or change and a very limited range of rebellions. Examiners must be open to alternative approaches. If in doubt, they should consult their Team Leader.

88

Page 92: History A Mark Schemes for the Units January 2010 - The ...

F966 Mark Scheme January 2010

2 'They were never fully in control of the English Church.' How far do you agree with this view of the archbishops of Canterbury in the period from 1066 to 1216? [60]

Most candidates will probably limit their answers to Lanfranc, Anselm , Becket and Langton and this is acceptable as these are the only archbishops mentioned in the specification. However, credit should be given for relevant reference to others such as Theobald or Hubert Walter. Candidates are likely to argue that there is plenty of evidence for lack of control over the Church by archbishops of Canterbury. Absence through exile as in the case of Anselm or Becket, or inability to enter England in the case of Langton, reduced the amount of control they could exercise. There was also no unequivocal resolution of the primacy issue. There was recognition of Lanfranc's personal primacy but as Thurstan of York's refusal to profess obedience to Canterbury in 1115 demonstrated, this remained a problem. This was usually resolved by the pope granting legatine authority to Canterbury but this was not automatic, as when Henry of Blois was made papal legate instead of the archbishop. By the later twelfth century, Canterbury's primacy did not confer much real power over the Church, and at times both Canterbury and York were made papal legates in their own provinces. Archbishops' authority was deliberately undermined by popes who wished to eradicate primatial control in order to maximise their own authority over the Church and who took advantage of the prevailing English situation to do so as occurred under Becket and Langton. Popes also encouraged appeals to Rome (eg in Stephen's reign) which further reduced archbishops' control. Archbishops sometimes suffered lack of support from their own bishops. For example, Becket's quarrel with Henry II lost him the support of some of his bishops such as Foliot. In Stephen's reign the divided government of the English Church caused problems between Archbishop Theobald and Henry, Bishop of Winchester. Henry II later took the opportunity to weaken Becket's authority by having York instead of Canterbury crown Young Henry. Monasteries also created problems for archbishops as they tried to free themselves from archiepiscopal control eg the Canterbury monks. Good responses should examine a range of evidence, recognising that Canterbury's lack of control was not consistent. Most candidates are likely to discuss the degree of control exercised by Lanfranc, Anselm, Becket and Langton in turn before reaching a judgement. Weaker answers may be characterised by partial treatment of just a few archbishops. Examiners must be open to alternative approaches. If in doubt, they should consult their Team Leader.

89

Page 93: History A Mark Schemes for the Units January 2010 - The ...

F966 Mark Scheme January 2010

3 How far was the impact of the papal reform movement in England from 1066 to 1216 characterised more by change than by continuity? [60]

The papal reform movement led to increased papal power and it is this which had an impact in England. There was considerable change in the extent of the impact of the papal reform movement. It had very little impact under William I and Lanfranc when William refused to become a papal vassal and Lanfranc refused Gregory’s summons to Rome. By the thirteenth century, the power of the pope had increased so much as a result of the papal reform movement that Innocent III was able to impose an archbishop of Canterbury unacceptable to the king, excommunicate John, place England under an interdict and go on to suspend Langton, so demonstrating papal power over both the English monarch and the church. There were also changes in the nature of the impact. In the reigns of William II and Henry I its main manifestation was through the Investiture Contest which resulted in a compromise in 1107 in which the king lost investiture with ring and staff. Under Stephen the growing power of the papacy led to more appeals to Rome. At times the main impact could be seen as attempts by popes to further their power by undermining primatial authority, eg by making Henry of Blois papal legate or by supporting York against Becket. At other times worsening relations between kings and their archbishops could be regarded as the main impact: for example, relations between Anselm and Henry I deteriorated because of the Investiture Contest and the quarrel between Becket and Henry II was prompted partly by the papal reform movement’s encouragement of ideas of separate ecclesiastical justice. It is likely that most candidates will examine these changes and possibly conclude that change was more noticeable than continuity. Weaker answers might tend to deal with the changes chronologically.

However, candidates might also argue that the impact of the papal reform remained essentially the same throughout most of the period. It was characterised by increased freedom of the church from royal control starting with Investiture Contest, increased papal authority over the king, the archbishop and the English church as a whole, and, although not consistently, at times throughout the period after William I and Lanfranc when it led to poor relations between kings and their archbishops. Stronger responses will be aware of both change and continuity and will evaluate the relative strength of each, reaching a substantiated conclusion. Some might well point out that although there was much consistency in the nature of the impact the emphasis was on different aspects at different times. Examiners must be open to alternative approaches. If in doubt, they should consult their Team Leader.

4 ‘Tudor rebellions were essentially the responses of local communities to local

grievances.’ How far do you agree with this view on the causes of Tudor rebellions? [60]

It might be useful if candidates define ‘local communities’ and ‘local grievances’ to distinguish between causes that had their origin in a village or town before spreading more widely, and causes that were not particularly linked to local factors but owed more to high politics and personal attitudes. An argument in support of the latter might refer to dynastically motivated rebellions such as Simnel, Warbeck and Northumberland, or to rebellions whose leaders had personal grievances, eg Northern earls, Essex, Kildare and Shane O’Neil. Candidates are likely to suggest that rebellions that were mainly social and economic in origin usually reflected local issues. Reactions to unfair or excessive taxation in Yorkshire (1489), Cornwall (1497), Suffolk among several counties (1525), Lincolnshire (1536) and Devon (1549), complaints about enclosures triggered the Kett and Oxfordshire disturbances, and responses to biased local JPs and the conduct of county gentry led to uprisings in Devon and Norfolk in 1549. It may also be argued that religious grievances manifested themselves locally but were really a response to government policies that

90

Page 94: History A Mark Schemes for the Units January 2010 - The ...

F966 Mark Scheme January 2010

affected the whole country. Disturbances in 1536 in Lincolnshire and Yorkshire, for example, resulted from national policies and local circumstances; and similar cases can be made for the Catholic reaction to reforms in Yorkshire and Durham by a reform-minded bishop in 1569. Some rebellions, such as Wyatt and Tyrone, were actuated by a mixture of personal and political motives and had their origin in local responses to national issues. Candidates should demonstrate the interplay between local and national developments and focus on similarities and differences in the causes of rebellion. Examiners must be open to alternative approaches. If in doubt, they should consult their Team Leader.

5 ‘English rebellions were far more successful than those in Ireland.’ How far do you

agree with this view of the period from 1485 to 1603? [60]

Candidates should seek to compare the outcome of rebellions in England and Ireland before reaching a judgement on the extent of success. Good essays will be aware that while most rebellions failed, there were some successes in both countries – Tyrone O’Neil in Ireland and the Yorkshire, Amicable Grant and Mary’s defeat of Northumberland in England. Some answers might include Henry Tudor’s defeat of Richard III as it occurred in 1485. Several English rebellions achieved some of their subsidiary objectives eg taxation was reduced in 1497, illegal enclosures reversed in 1537 and 1597, the Statute of Uses repealed in 1540, religious reforms suspended after 1539 and the Council of the North reconstituted in 1572. Irish rebellions had fewer successes and most failed to achieve their main objectives. Tyrone recovered his dispossessed lands and received a pardon but lost his title of ‘The O’Neil’. The other revolts resulted in the leaders’ deaths, military defeat, the imposition of martial law and confiscated lands. Attempts to oust English settlers in some of the plantations and preserve the Catholic faith were successful but Kildare, Shane, Fitzgerald and Tyrone all failed to expel the English administration and garrisons. It could be argued that the Irish forced English governments to spend a disproportionate amount of time, money and resources in combating rebellions, which enabled them to frustrate English aims of colonisation and conquest. Candidates should be rewarded for discussing areas of similarity and difference and for evaluating successes and failures than for explaining the reasons for successes and failures, which may be a characteristic of weaker essays. Examiners must be open to alternative approaches. If in doubt, they should consult their Team Leader.

6 Assess the role of the nobility in maintaining political stability in Tudor England.

[60]

The nobility was vital to the crown in maintaining political stability in Tudor England. Candidates are likely to argue that many nobles served as royal councillors both in London and at a regional capacity, as sheriffs, JPs, lords lieutenants, and as special commissioners in the counties, and suggest how these officers upheld political stability in the country. As leading landowners, nobles were also expected to arm their tenants and servants to suppress disturbances and, when necessary, fight in royal armies. Nobles such as Surrey, Oxford and Pembroke assisted Henry VII; Norfolk, Suffolk and Shrewsbury suppressed rebellions in Henry VIII’s reign; Russell, Warwick and Grey led armies against the Western, Kett and Oxford rebels in 1549; and Pembroke, Clinton and Norfolk dealt with Wyatt and his rebels. Elizabeth similarly called upon Sussex, Clinton and Hunsdon to combat the Northern Earls, and the Earl of Nottingham was called upon to arrest Essex in 1601. Better candidates should point out that not all nobles supported the crown and in 1485 several families presented a serious challenge to internal stability. However, the Percy, Neville and Howard families declined between 1536 and 1572, especially in the north of England, and only a small minority of nobles remained a problem in Elizabeth’s reign. It may also be pointed out that the crown increasingly relied not just on the nobility but the gentry and lesser landowners for keeping law and order in the provinces and counties. These groups of people dealt with most local grievances and as town officials, JPs and MPs, took a keen interest in maintaining stability. The clergy, judiciary and

91

Page 95: History A Mark Schemes for the Units January 2010 - The ...

F966 Mark Scheme January 2010

monarchy might also be examined but it is important that candidates focus their answers principally on the nobility. Examiners must be open to alternative approaches. If in doubt, they should consult their Team Leader.

7 ‘The aims and methods of Tudor foreign policy were characterised more by change

than by continuity.’ How far do you agree with this judgement? [60]

Some candidates may well focus on either aims or methods but better essays should examine both elements in terms of continuity and change. Some candidates may assess Tudor relations with particular countries such as Spain, France, Scotland and Burgundy/ the Netherlands. Some might adopt a chronological approach though this may make sustaining a synoptic analysis more difficult. The main aims behind Tudor foreign policy were national security, trade agreements, continental expansion and dynastic alliances. National security and trade agreements were consistently pursued although, if there was a conflict of interest, trade yielded to political security. Keeping the Netherlands neutral or out of enemy hands was also a consistent objective. To this end, France and Burgundy were seen as England’s main enemies at the start of the period but had been replaced by Spain at the end. Dynastic and marriage alliances were pursued by Henry VII, Henry VIII and Mary, but less so by Edward VI and inconclusively by Elizabeth – although the latter used her unmarried status as a pawn in foreign diplomacy. There were inconsistencies, however, in the Tudors’ attitude towards war. Henry VII and Elizabeth avoided it if possible but engaged in continental alliances; Henry VIII and Edward devoted much time and money to waging war against Scotland and France; Mary was opposed to war but was drawn into it by Philip. Religious reforms also brought changes in allies and foreign commitments eg Anglo-Scottish relations. The best candidates are likely to explain the more prevalent trends and account for changes in aims and methods. Examiners must be open to alternative approaches. If in doubt, they should consult their Team Leader.

8 Which presented the greater threat to England’s security in the period from 1485 to

1603: Scotland or France? Explain your answer. [60]

Candidates should focus on and assess both Scotland and France before reaching a conclusion. Some answers are likely to compare the two countries thematically – perhaps in respect of military and naval threats in peacetime and at war, the support given to pretenders, claimants and English rebels, or the impact of making alliances against England and often together (Auld Alliance). Some essays might assess Scotland and France separately before reaching a judgement though this approach could restrict candidates’ ability to demonstrate synoptic skills. The strong military, naval and financial power of France, its desire to recover land held by England, the long-standing rivalry and its commitment to the Catholic faith, might suggest that it posed a serious threat, and every Tudor ruler went to war against France at least once. Better candidates, however, should point out that there were long periods of peace and stability between the two countries eg 1492-1512, 1527-42, 1564-1603. Scotland though much smaller and less powerful than France also presented a threat to England: it supported Warbeck, invaded northern counties at will and had a long-standing arrangement with France of embarrassing English governments. The deaths of James IV and James V after military conflicts weakened Scotland but left it open to a French presence and capacity to intrigue against England, which was not removed until 1560. Thereafter, Scotland did not present a serious threat. Candidates may well conclude that both countries presented rather different threats, which changed over time, before deciding in favour of one of them. Examiners must be open to alternative approaches. If in doubt, they should consult their Team Leader.

92

Page 96: History A Mark Schemes for the Units January 2010 - The ...

F966 Mark Scheme January 2010

9 Assess the ways in which Spain affected English domestic affairs during the period from 1485 to 1603. [60]

Candidates are likely to suggest that Spain had a considerable impact on English domestic affairs but should also be aware that the nature of its influence changed over the period. One approach that candidates might take is to look at each of the Tudors in turn and comment on developments that led to continuity and change. Another would be to assess the impact thematically. The main areas of discussion are likely to be political issues – Henry VII’s alliance of 1489 against pretenders and the betrothal of Arthur (and later Henry) to Catherine of Aragon; the influence of the Aragonese faction at court in the 1520s and 1530s, and Charles V’s pressure not to proceed with a divorce; hostility towards Philip in England in the 1550s at the court, council and parliament surfaced in Wyatt’s rebellion; putative Spanish support for Mary Stuart and Catholic plotters. Religious issues – pressure to remain a Catholic country after the break from Rome. This may have curtailed Henry VIII’s reform programme, compromised Somerset, supported and aided Mary’s restoration of the Church, influenced Elizabeth in determining the Settlement of 1559 and her subsequent treatment of Catholics. Economic issues – trade links since 1489 with Aragon and Castile, and from 1515 with the Low Countries. The collapse of the Spanish Netherlands’ woollen trade had a severe impact on England’s economy in the 1550s. English merchants traded extensively with Iberia until the 1580s and privateers became a source of friction from the 1560s. The outbreak of the Dutch Revolt had a serious effect and Elizabeth faced political pressure to intervene. Military issues – war with Spain against France in 1542 and 1557 and against Spain from 1585 had serious financial, economic and political consequences. Spain also intervened in Ireland in the 1580 and 1601 rebellions. Examiners must be open to alternative approaches. If in doubt, they should consult their Team Leader.

10 ‘The success of the Catholic Reformation in the period from 1492 to 1610 depended

mainly on secular rulers (emperors, kings and princes).’ How far do you agree with this view? [60]

Candidates should evaluate the contribution of a number of secular rulers to the Catholic Reformation and decide how far any success was due to their efforts and how much to other factors. Isabella and Ferdinand, Mary Tudor, Philip II, Ferdinand of Styria, Sigismund of Poland, Rudolph II of Austria, and Albert, William and Maximilian of Bavaria, could be usefully examined as secular rulers who advanced the Catholic Reformation in their own lands. Reasons for their success could be attributed to the use of patronage, military influence and political authority. Candidates might argue that some secular rulers who were in a position to implement improvements did little to advance the Church’s revival. Most German princes, Henry VIII, Francis I and the later Valois kings would fall into this category, and criticism can be made of Charles V and Philip II, who constantly quarrelled with the papacy. Candidates should compare secular rulers with other influences and might well reject the title’s premise. For instance, they could argue that the papacy was the key to the Catholic Reformation’s success, and illustrate the point by the negative and positive contributions of popes during this period. They might point to the role of individuals as members of the new orders or to reform-minded clerics, who were not secular rulers but who had a significant influence upon them eg Canisius in Bavaria, Borromeo in Milan, Pole in England. The work of institutions like the Council of Trent, the Inquisition and Index, might be evaluated but better essays will focus on the term ‘mainly’ and might well conclude that the successful impact of individuals and institutions rested on lay support. Where secular rulers were weak or opposed to reform, little headway was made. Examiners must be open to alternative approaches. If in doubt, they should consult their Team Leader.

93

Page 97: History A Mark Schemes for the Units January 2010 - The ...

F966 Mark Scheme January 2010

11 To what extent was the Catholic Reformation more concerned with continuity than change in the period from 1492 to 1610? [60]

Candidates can support or refute the proposition by examining a range of features that contributed to the Catholic Reformation. It may be helpful to define ‘continuity’ eg retaining key features, beliefs and institutions of the Catholic Church and faith. The Inquisition and Index might be assessed to show how they repressed liberal ideas and censored unacceptable views, such as those of Illuminists, Erasmians, Protestants, conversos and moriscos. Attempts to reconcile Catholics and Protestants at Regensburg in 1541 resulted in victory for the reactionaries led by Carafa who was intent on suppressing change. Even the new orders struggled to gain respect and recognition from traditional monastic orders. In education, biblical humanism, and its implicit changes to the way the Church interpreted the Scriptures, was rejected in favour of scholasticism. The management of Trent’s three sessions by Jesuits and Dominicans demonstrated the desire to defeat Augustinianism, Lutheranism and Calvinism in order to preserve orthodox ideas, which was underlined by the Tridentine Decrees. No provision was made for lay administrators and female reformers and reforms to the Curia were slow to take effect and very conservative. There are examples, however, of the Catholic Church showing a willingness to change. The Church learned from Protestantism the value of preaching and the sermon in developing the spiritual condition of ordinary people, and the benefit of advocating social welfare, the use of the consistory in keeping discipline, and the role of seminaries in educating the clergy. Candidates could refer to individual popes to suggest that the papacy was capable of embracing enlightened ideas. On balance, candidates are likely to argue that the Church was more conservative than progressive. Examiners must be open to alternative approaches. If in doubt, they should consult their Team Leader.

12 ‘The Inquisition and Index did little to advance the Catholic Reformation in the

period from 1492 to 1610.’ How far do you agree with this view? [60]

Some candidates will agree and some disagree with the proposition. The focus of the answer should be on what the Inquisition and Index set out to achieve, how far they succeeded and assess their contribution to the Catholic Reformation during these years. Most candidates are likely to assess the Spanish and Roman Inquisitions but the Portuguese, Dutch and Italian city state inquisitions are all valid lines of argument. Candidates should be aware that the role of the Index stayed constant from its inception in the 1540s, when titles of forbidden works were proscribed by universities, to the Roman and Spanish Indices which listed titles and authors. Revisions occurred periodically and candidates could usefully discuss the impact of the Index upon Catholic ideas, education and society in states such as Spain and Italy. Most candidates will probably spend more time assessing the Inquisition. Under Isabella and Ferdinand, the Spanish Inquisition tackled moriscos and conversos; under Charles V, Erasmians, Anabaptists and Lutherans were targeted; under Philip II and Philip III, it focused on immorality, paganism and moriscos in Spain, conversos in Portugal and Calvinists in the Netherlands. An argument may be made that the suppression of heresy and strengthening of the Church owed a great deal to the Inquisition. The Roman Inquisition similarly silenced Protestant movements, dealt with cases of immorality, and maintained papal authority in several Italian city states. A counter-view is that the Inquisition and Index were negative influences and real advances were principally due to other developments, such as the Jesuits, reform-minded popes, bishops and secular rulers who implemented the Tridentine Decrees. For Levels I and II, however, there should be a good evaluation of the Inquisition and Index, especially if the balance of the argument is on other factors. Examiners must be open to alternative approaches. If in doubt, they should consult their Team Leader.

94

Page 98: History A Mark Schemes for the Units January 2010 - The ...

F966 Mark Scheme January 2010

13 ‘The nobility strengthened rather than weakened royal power in France in the period from 1498 to 1610.’ How far do you agree with this view? [60]

Candidates are likely to assess the ways in which the nobility contributed to or hindered the development of the nation state, and the best answers will not stray from this path. The nobility held key offices in both church and state and not only served the king but also served themselves in the provinces. They led and raised armies during the Italian wars, which strengthened the state, but the same troops were turned against the well-being of the state and monarchy in the wars of religion. Nobles were royal governors, law enforcers and administrators, and most under a strong ruler such as Francis I served the state very loyally. A minority flouted the law, lined their pockets and disobeyed the king, at times demonstrating how much harm they could inflict at all levels of society. Some candidates may use particular noble families, such as the Montmorencys, Bourbons and Guises, to illustrate their argument throughout the period or to distinguish between noble groups eg nobles who were politically ambitious, princes of the blood, and those with strong religious convictions. Candidates should be aware of the changing relationship between the nobility and crown, most notably during the reigns of Francis I and Henry IV, when the major families were subdued, and in the reigns of Francis II, Charles IX and Henry III, when they exercised considerable political influence over the crown to the detriment of royal authority. Some answers might include the role of noble women eg as a mistress (Diane de Poitiers), a regent (Catherine de Medici) or as participants in the wars of religion (Marguerite d’Angouleme, Louise de Montmorency, Jeanne d’Albret). Examiners must be open to alternative approaches. If in doubt, they should consult their Team Leader.

14 To what extent did the French government overcome its economic problems in the

period from 1498 to 1610? [60]

Most candidates are likely to discuss the state of royal finances during this period but higher responses (Levels I and II) should explain some of the difficulties in industry, trade, commerce and agriculture as well. In finance the main problems were the inefficient and unequal tax system, tax farming and insufficient revenue to meet the state’s requirements; long periods of foreign war and civil wars disrupted administration and increased crown debts; and inflation was exacerbated by court affluence and patronage. Francis I implemented reforms that centralised the system but did not tackle issues of corruption and exemption. Henry IV could only begin to solve the difficulties caused by civil wars by cancelling debts and gradually initiating reforms after 1598. The nobility and officiers had the wealth and potential to invest in trade and industry but throughout the period showed little interest as long as rentes, crown pensions and from 1604 the Paulette were more profitable. Trade was in the hands of merchants who were heavily taxed and disadvantaged when competing with foreigners. There were few improvements in agriculture due to the depressed condition of the peasantry and disinterest among landowning nobility who preferred to hunt over the crops. There was little investment in industry and agriculture until Henry IV and Sully began to encourage state subsidies. Population levels rose to 17 million by 1610 (largest in Europe), which put pressure on urban employment and food supplies, and increased the likelihood of plague, poverty and revolts. Local and regional opposition to a more unitary transport system and an excessive number of tolls impeded the movement of goods. Overall the economy remained a weakness in the nation state though there were some improvements, notably under Francis I and Henry IV. Examiners must be open to alternative approaches. If in doubt, they should consult their Team Leader.

95

Page 99: History A Mark Schemes for the Units January 2010 - The ...

F966 Mark Scheme January 2010

15 ‘The Catholic Church held back the development of the French nation state in the years from 1498 to 1610.’ How far do you agree with this view? [60]

Candidates need to assess how far the Catholic Church impeded the development of the nation state. Weaker essays may focus heavily on the Wars of Religion as a prime source of national disunity. Some candidates may discuss the impact of religious groups upon the nation state from Francis I’s reign onwards, particularly in dividing the country socially, politically and religiously, and the response of the Catholic Church to them. Some candidates may see the Day of the Placards (1534) as a turning point in that thereafter humanists, Lutherans and Calvinists were under attack and the goal of national unity (one king, one law, one faith) disappeared. Some candidates will see this question in terms of the Wars of Religion and may compare the attitudes of Catholics towards Huguenots both during and after the civil wars. However, better essays will be aware that the Catholic Church was a source of strength to the crown and to most French people for much of the period. Of course, some candidates may point out factors other than the Church that weakened the nation state. Reference to geo-political borders, language, customary and Roman law, social and political disruption caused by civil wars, developments in administrative centralisation and fluctuations in the authority of the monarchy, would be valid areas of discussion in examining the development of the nation state. Examiners must be open to alternative approaches. If in doubt, they should consult their Team Leader.

16 ‘The development of absolute monarchy in France owed more to Louis XIII than

Louis XIV.’ How far do you agree with this judgement? [60]

The most successful responses are likely to compare the two rulers’ contribution to the development of absolute monarchy in France. Candidates might assess them in respect of their power and authority, their skill as politicians and statesmen, their qualities of leadership in domestic and foreign affairs, their appointment of ministers and the success of their reigns. An argument could be made that the extent to which they were absolute rulers rested heavily upon the competence of their ministers but the question requires more than a comparative evaluation of royal councillors. Some essays might compare the absolutism of the two kings, however implicitly, by examining how well they dealt with France’s problems. Among these, we can expect an assessment of their dealings with the princes of the blood and nobility, their management of royal finances, councils and policy making, how far they enhanced royal authority in the face of parlements and provincialism, their relationship with the Catholic Church and Huguenots, and how well they handled foreign affairs. Better candidates should be aware that the issues that faced French kings changed over time (eg rising population and inflation, the decline of Spain, the problem of the princes of the blood), though some continuity remained (eg financial system, particularism, strong Catholic Church), and, of course, the personality and ambitions of Louis XIII and Louis XIV were in sharp contrast. Moreover, Louis XIV was able to build upon the achievements of Louis XIII and his ministers but just how absolute was the monarchy in 1715 compared with 1610? Examiners must be open to alternative approaches. If in doubt, they should consult their Team Leader.

17 Assess which minister contributed most to France becoming a major European

power in the seventeenth century: Richelieu, Mazarin or Colbert. [60]

A case can be made for each of these ministers and much may depend upon how candidates define ‘a major European power’. Some candidates may compare Richelieu and Mazarin in terms of how they dealt with over-powerful nobles, the Estates and parlements, raised money for war, administered the state and church, strengthened the monarchy and waged war. They may suggest that Richelieu contributed more because he solved the Huguenot problem and worked closely with the Catholic Church. He expanded

96

Page 100: History A Mark Schemes for the Units January 2010 - The ...

F966 Mark Scheme January 2010

the army and navy, set up trading companies, reduced the power of the estates, humbled the aristocracy, expanded the intendants and laid the foundations for victory in war against Spain. Mazarin’s main contribution lay in negotiating beneficial terms at Westphalia and the Pyrenees, which gained France lands in Savoy, Alsace, the Netherlands and the Rhineland (1648) and lands in Luxembourg, the Netherlands, the Pyrenees and acquired a claim to the Spanish throne (1659). A negative feature was the Fronde, which was partly a result of his own unpopularity and financial mismanagement. Colbert’s claim lay in his management of the economy, which provided the basis for France’s military achievements under Louis XIV and the creation of Versailles. Revenue increased 400%, taxes rose 40%, corruption was reduced in administration and by 1672, the budget was balanced. Unlike Richelieu, he built up an effective navy of 300 ships and 4 new dockyards; improved road and canal transport and revitalised textile industries. He pursued mercantilist policies aimed at acquiring gold and silver bullion at the expense of the Dutch and English. He regulated industries, founded trading companies, established colonies in Canada and the West Indies, expanded the royal navy, maritime fleet and arsenals and naval stores. However, he failed to reform the fiscal system, his law codes could not be enforced and attempts to establish trading companies failed. A comparative synthesis is likely to characterise the better essays. Examiners must be open to alternative approaches. If in doubt, they should consult their Team Leader.

18 Which social groups in France gained most and which lost most during the period

from 1610 to 1715? Explain your answer. [60]

Candidates should consider a range of social groups, assess in what ways and to what extent they were affected by developments across the whole period, and explain why some benefited more than others. The best responses are likely to organise their arguments thematically according to either different social groups or to particular events, and demonstrate a good awareness of continuity and change. The principal groups that are likely to be examined are the nobility (and better candidates should distinguish between different categories ie princes of the blood, noblesse d’épée, noblesse de robe, noblesse de province), the clergy (Catholic and Protestant), merchants (perhaps those engaged in traditional trades, as well as those in newer companies and industries), town and rural workers (especially the urban poor, artisans and agricultural peasants). Explanations for change and continuity in the condition of these groups may be found in the growth of centralisation and administrative developments, long periods of warfare after 1635, an increase in taxation and size of armies, the growth of Paris and other large cities, the creation of Versailles, economic developments and religious issues, such as the Revocation of the Edict of Nantes. Candidates are not required to assess all groups or every development in the period but they are expected to produce a balanced and sustained comparison of different social groups before arriving at an overall judgement. Examiners must be open to alternative approaches. If in doubt, they should consult their Team Leader.

97

Page 101: History A Mark Schemes for the Units January 2010 - The ...

F966 Mark Scheme January 2010

Option B: Modern 1789-1997 The Challenge of German Nationalism 1789 – 1919

1 Assess the view that the German people were more divided than united during the period from 1789 to 1919. [60]

Candidates should focus on the extent to which the German people were more divided than united in this period. Candidates may argue both for and against this proposition. Candidates may view the period from 1789 to 1815 as a time when the German people became more united, geographically and in terms of aims. French domination helped to modernize and consolidate Germany and sparked the first upsurge of German nationalism. A popular uprising helped to drive Napoleon out of Germany in 1813. This common fight of people from different German states against the French enemy gave strong impulses to nationalism. The number of independent and semi-independent German states had been around one thousand in 1790 (with between three and four hundred fully independent units). Twenty-five years later only a little over thirty remained. Candidates may argue that the German nation was very divided from 1815 as a consequence of decisions taken at the Congress of Vienna, but could also argue that the German Confederation from 1815 did loosely bind most Germans into a Confederation with a Diet. Candidates may argue that the growing emergence of the nationalist movement after 1815 to the development of more radical nationalism in the late Nineteenth and early Twentieth Centuries demonstrates an emergence of national unity amongst the German people over this period. Candidates may understand that there were significant divisions even within those who were nationalist. Candidates may understand that the Prussian Empire in 1871 represented Kleindeutschland and an enlarged Prussia. They may argue that it was a Prussian Empire rather than a German Empire; it certainly did not unite all the German people even geographically. The exclusion of Austria from the process of German unification may be dealt with. Divisions within the German nation after 1871 might be illustrated through the Kulturkampf and the rise of socialism, or the domination of the Reich by the elites. Candidates may, however, argue that territorial boundaries rarely exactly match where the people of that nationality live and that divisions within a nation based on class or culture do not necessarily define the unity or otherwise of that nation. All modern nations have exhibited such divisions. They could certainly argue that the German Empire from 1871 physically united the majority of Germans. Candidates may argue that the First World War both united the German nation, at first, but that divisions soon arose and were entrenched by 1918. Similarly, whilst Versailles divided the nation geographically it united the nation in condemnation and bitterness.

Examiners must be open to alternative approaches. If in doubt, they should consult their Team Leader.

2 How far do you agree that the development of German nationalism was mainly dependent upon economic factors from 1789 to 1919? [60]

Candidates might focus on the relative importance of economic factors within the development of German nationalism in this period. Candidates should explain the importance of economic factors on developments, for example the impact of the Zollverein after 1834 in developing Prussia’s economic strength and Prussian leadership of Germany. Candidates should understand how developments in the economy in the 1850s paved the way for the military victories of 1864, 1866 and 1870 / 71. Military strength depended upon economic strength however. ‘Coal & Iron’ rather than ‘Blood & Iron’ could be usefully debated. The development of the railways may be seen as significant. The impact of the extraordinary developments in the German economy after 1871 should be discussed. Candidates must however show that they understand that economic factors were not the sole factors determining the fortunes of German nationalism in this period. For example, the Great War left Germany broken and half-starved despite the German

98

Page 102: History A Mark Schemes for the Units January 2010 - The ...

F966 Mark Scheme January 2010

economic domination of continental Europe in 1914. Economic factors undeniably contributed to Prussia’s domination of Germany from 1866, but opportunistic and skilful leadership, especially by Bismarck, should not be overlooked. Candidates may wish to place considerable importance on the diplomatic abilities of Bismarck both in terms of the unification of Germany and his management of German nationalism. The development and impact of ideas on the emergence and development of intellectual nationalism may also be usefully explored. Candidates may argue that initially this provided the impetus or springboard for later developments and that, in the Napoleonic period, it was the common fight of people from different German states against their French enemy that gave strong impulses to nationalism

Examiners must be open to alternative approaches. If in doubt, they should consult their Team Leader.

3 ‘Bismarck was much more effective in managing German nationalism than either Metternich or Kaiser Wilhelm II.’ How far do you agree with this view? [60]

Candidates should focus on how effectively Metternich, Bismarck and Wilhelm II managed German nationalism. Candidates will undoubtedly be more successful if they define ‘effective management’ in their answer. Candidates might define the ways in which the three were (or were not) effective: for example in controlling, harnessing or using nationalism. Clearly all three had different aims and different circumstances, which could enable candidates to make convincing cases for all of them. By 1848/49 no leader of the nationalist movement with mass appeal emerged. From 1815 to 1848 the nationalist movement was too weak to effectively challenge the Metternich System: arguably this demonstrates Metternich’s effective control over German nationalists. Equally Metternich fled Vienna in 1848, though his downfall was hardly dominated by German nationalism. Many candidates may argue in favour of Bismarck because of his critical role in the 1860s in the creation of the Second Reich; candidates may argue that he managed German nationalism by hijacking the nationalist cause for Prussia’s ends. This too could be considered effective management of German nationalism. Wilhelm II’s search for world power was undoubtedly populist, mirroring the development of radical nationalism, but it placed Germany in a vulnerable, dangerous position. The ultimate outcome of his policies was defeat in the Great War and humiliation at Versailles.

Examiners must be open to alternative approaches. If in doubt, they should consult their Team Leader.

99

Page 103: History A Mark Schemes for the Units January 2010 - The ...

F966 Mark Scheme January 2010

Theme 2: The Changing Nature of Warfare 1792 – 1945 4 Assess the extent to which conscription enabled armies to succeed in war in the

period from 1792 to 1945. The impact of conscription on warfare is not limited to the massing of larger numbers of soldiers but this will probably form the bulk of the answers met. Conscription is applied to warfare right at the start of the period by the development of ‘a nation in arms’ by France in the early part of the Revolutionary Wars developing into more regulated conscription in the later Revolutionary & Napoleonic period. This might be contrasted with the use of long service professionals and mercenaries by the dynastic armies of France’s enemies. The reaction of France’s enemies to conscription might include the tentative use of Frei Korps and Landwehr by Austria or the traditional use of conscripted serfs by Russia’s long service army. A good topic for discussion would be the development of the Krumper system in Prussia after 1808. Candidates might note that Britain never embraced conscription in this period and yet her army was successful. For the period of unification the different systems used by the combatant powers might be examined. Generally candidates will point to the superior organisation of manpower by Prussia and the resulting large size of her army in proportion to her population. This was illustrated by the defeat of France’s long service army by Prussia’s reservists despite superior French weapons technology in the ‘Imperial’ phase of the Franco-Prussian War. The expansion of the use of reservists in the last part of the 19th century is a profitable area for discussion. The First World War is an obvious example where conscription played a key role in warfare. Note that Britain used a long service professional army supported by Territorials at the start of the conflict, replacing this with Kitchener’s army of volunteers and finally conscription. WWII also saw the use of mass armies of conscripts but with more sophisticated technologies in the hands of these soldiers with resulting problems in training and use on the battlefield. The American Civil War falls into the mainstream of the debate, the north having a preponderance in manpower. Alternative explanations are possible and examiners must be open to alternative approaches. If in doubt, consult your Team Leader.

5 How far did developments in command and control of armies determine the

outcome of battles in the period from 1792 to 1945? No set conclusions are to be expected, but candidates must answer the question and address the theme over the full period. The question prompts a discussion of the organisation of war by the combatant powers and the control of armies during the period. Napoleon was the first to develop a true general staff and this was taken to new levels of effectiveness by the Prussian general staff of the middle and later part of the period. The planning of WWI and WWII is an obvious case for discussion. Candidates might point to ineffective command and control as a basis for argument, a good example of deficiencies in this era was the armies of Napoleon III and Austrians in 1866 versus Prussia. The failure of command and control might be a useful way to argue against the premise of the question. The American Civil War fits easily into the debate. The impact of command and control on the outcome of warfare can be discussed on many levels from grand strategy to the tactics. Better candidates may balance these two factors against others, but a discussion of the command and control of armies must form the core of the essay. Examples of the outcome of battles being determined by command and control could be drawn from the individual skills of a given commander in chief or from the use of command and control systems or both. In the Revolutionary and Napoleonic wars virtually any battle with Napoleon and the French staff system directing one side will apply, but of especial interest might be Austerlitz. The allies eventually developed similar methods but examples will have to come from later in the wars, the 1813 period would be useful but even then

100

Page 104: History A Mark Schemes for the Units January 2010 - The ...

F966 Mark Scheme January 2010

Napoleon tended to win all of his battles. A good example of success using more antiquated methods of command would be any of the Duke of Wellington’s battles. Of course the Napoleonic system sometimes was found wanting, for example at Borodino or Waterloo. For the battles of the mid century candidates should be aware that the size of actions increased and had an impact on command and control despite developments in this area, Magenta or Solferino in 1859 were both confused affairs. In 1866 and 1870-1 the Prussian staff system brought their army to the battlefield with some efficiency but once again many of the battles themselves demonstrated the difficulty of controlling armies fighting in long linear formations, Konigsgratz is an example as are many of the battles of the Franco-Prussian War although Gravelotte-St. Privat is of especial interest. The First World War has many obvious examples on the Western Front, so too the Second World War. Alternative explanations are possible and examiners must be open to alternative approaches. If in doubt, consult your Team Leader.

6 ‘Industrialisation was most successfully applied to warfare in the First World War.’

How far would you agree with this view of the period from 1792 to 1945?

No set conclusions are to be expected, but candidates must answer the question and address the theme over the whole period. The First World War is an obvious example of the first conflict where the application of industrialisation to conflict was the key to victory. This was partially due to the sheer scale of the conflict. Candidates might argue that the long duration of the war, the size of the armies involved and the geographic spread of the conflict meant that victory went to the side with the greatest industrial might. Alternatives might be the Napoleonic Wars where one might argue that Britain as the first industrial power played a key role in the conflict or that France was successful for much of the period due to an emerging proto-industrial economy. Another is the Crimean War where the industrial might of the Allies caused them to emerge victorious over an industrially backward Russia – a power that played a key role in the final defeat of Napoleonic France. Another alternative would be the impact of a newly industrialised Prussia on the conflicts of the mid-nineteenth century. For later conflicts the Second World War might fit the ‘successful application of industrialisation’ in the title better than the First, i.e. that industrialisation had a much wider impact on this conflict. The American Civil War can be discussed by candidates, the industrialised North defeating the non-industrialised South but the successful application of command in the question must be addressed in this context given that it took time for the Union’s industrial might to overcome the Confederacy. Candidates need to be aware of how industrialisation had an impact on the successful waging of war in the period. Alternative explanations are possible and examiners must be open to alternative approaches. If in doubt, consult your Team Leader.

101

Page 105: History A Mark Schemes for the Units January 2010 - The ...

F966 Mark Scheme January 2010

Theme 3: Britain and Ireland 1798-1921. 7 ‘Revolutionary nationalism in Ireland consistently failed to develop mass support.’

How far would you agree with this view of the period from 1798 to 1921?

No set conclusions are to be expected, but candidates must answer the question and address the theme over the whole period. Candidates may well agree with the question’s proposition, citing the low numbers involved in Emmet’s rising (1803), Young Ireland in 1848, the Fenian Outrages in 1867, the Phoenix Park murders of 1882 and in the Easter Rising of 1916. They could refer to the proscribed nature of Republicanism, their preference for rebellion, assassination and risings, the secretive nature of their methods and their violence, which was met with the same. The revolutionary tradition was one of martyrdom (Tone, Emmet, 1916 etc.). They did not hold mass meetings; their oratory was that of the scaffold, finance was a problem until they tapped into the Irish American diaspora, whilst their policy of an independent republic lacked appeal in peasant Ireland, until the Land League in the late 1870s made the connection with Alien landlords. They also failed to gain the support of the Catholic Church pending the development of a conservative and Catholic social policy in the 1910s. However candidates could also challenge the view of ‘consistent failure’ with reference to the post 1870 period when land issues were exploited by Davitt, especially the rural support gained during the Land war of 1879-82 (where boycott was preferred to violence). There was also more success after the 1890s in widening the basis of support through the harnessing of cultural nationalism (the Gaelic Athletics Association was hijacked by the IRB for propaganda and recruitment purposes). Connolly’s socialism made some headway amongst the Dublin working class but it lacked a rural base, as did Griffith’s Sinn Fein. It could be argued that considerable success came in and after 1917 following government mistakes (Conscription, Black and Tans) and with a popular mandate for Sinn Fein in the 1918 election. Collins’ strategies (the Anglo Irish War) and a comparison with Wolfe Tone may prove useful here, with the careful targeting of the police and local authorities. Alternative explanations are possible and examiners must be open to alternative approaches. If in doubt consult your Team Leader. [60]

8 Assess the view that Gladstone’s first Liberal government (1868-1874) was the most

important turning point in Britain’s relationship with Ireland in the period from 1798 to 1921.

No set conclusions are to be expected, but candidates must answer the question and address the theme over the whole period. Candidates could argue that it did, citing Gladstone’s unusual statement on hearing the news – ‘My mission is to pacify Ireland’, followed by a PM focus on solving the issue of an ‘Alien Church’ (Disestablishment of the Church of Ireland in 1869and its ending of the Protestant Constitution and the religious ascendancy of the Anglo Irish establishment); the attempt in the 1st Land Act in 1870 to address the legal issues (the three ‘Fs’) of Land and the encouragement of a more inclusive ruling class via University reform in 1873. In support of this it could be argued that Gladstone was under little pressure to introduce an Irish programme after 20 years of relative indifference and that it inaugurated a focus on the issue that was to last until 1921, by Gladstone and especially, but not exclusively, by Liberal governments in general. However, candidates might take a different view, arguing that Gladstone’s 1st government was entirely consistent with both previous and successive governments who remained wedded to the Union. Gladstone could be seen as following the tradition of Pitt (who wanted to stabilise the relationship and extend the economic benefits of Union) and Peel. The latter’s government had also sought to bring the Catholic Church on board (Maynooth). Peel’s Land Bill, following the Devon Commission, and the University Bill, prefigured almost exactly Gladstone’s 1870 Land Act and University Bill. Gladstone in 1868 was therefore not a turning point, unless one stresses Disestablishment, which

102

Page 106: History A Mark Schemes for the Units January 2010 - The ...

F966 Mark Scheme January 2010

neither Pitt nor Peel had wanted to adopt, although both had introduced Catholic Emancipation, arguably a much greater turning point. It could be argued that Gladstone’s 2nd Land Act in 1881 and the Conservative ones of the 1880s and 1890s were of much greater significance, whilst much could be made of the First Home Rule Bill in 1886 and its successors in 1893 and 1912 as marking a much more important turning point in that it represented a return to the pluralism of Grattan’s parliament in 1798-1800 and had become the policy of all governments after 1912, to be applied both North and South post 1918. Another route for candidates would be to argue that reform within the Union remained British policy throughout and as such the turning points lay at the beginning and end of the periods (1800 Act of Union and the Anglo-Irish Treaty 1921). Alternative explanations are possible and examiners must be open to alternative approaches. If in doubt, consult your Team Leader. [60]

9 How successfully did British governments deal with Irish land issues in the period

from 1798 to 1921?

No set conclusions are to be expected, but candidates must answer the question and address the theme over the whole period. Candidates will need to grasp the significance of Irish land issues in the period. Ireland was overwhelmingly peasant and agrarian outside parts of Ulster and Dublin. One key issue was the legal basis of the land settlement in Ireland. The British had imposed this in the 17th century, basing it on religious confiscation and the concept of landlord supremacy in relation to his tenants. This in turn was opposed by Irish tradition, Ulster tenant right, which effectively gave a form of dual ownership to landlord and tenant. Such grievances fuelled much of the discontent in Ireland in the late 18th and 19th centuries and were increasingly harnessed by both constitutional and revolutionary nationalism from the 1870s (the Land League). No British government sought to deal with this before Peel in the 1840s (the Devon Commission and subsequent Bill), Pitt seeing economic union and an efficient landlord class as the key to agrarian prosperity. The Whigs in the 1830s had de-linked the Tithe from tenant payment to landlord but had increased resentment of the latter that raised rents to compensate. Peel had failed in the face of parliamentary landlordism, which remained entrenched well into the 1870s. Gladstone sought to deal with tenant right in the 1870 Land Act but failed given the loopholes. He had more success when confronted with a Land War in 1881 (the Second Land Act which effectively granted dual ownership and a final recognition that Arrears had to be dealt with in an 1882 Act).Until the 1880s it was Tenant Right and evictions that were the focus. The other key issue was the backwardness of Land and it could be argued that this was not dealt with until post 1880. Pitt and Peel were frustrated by the failure of capitalist agriculture to emerge in anything other than moderate and regional form (East and the Midlands). The West remained backward, overpopulated and potato dependent, as the famine was to show. British governments dealt more successfully with this following the agricultural depression of the late 1870s which rendered Landlordism less powerful and more willing to embrace land sales and purchase schemes offered by Gladstone, and the Constructive Unionism of Ashbourne, Balfour and Wyndham. The latter, by providing loans for purchase at rates that were less than rentals, effectively solved the Land problem in relationship to issues of ownership. Nonetheless backwardness remained a problem. Both Conservative and Liberal governments in the 1890s and early 1900s poured money into marketing schemes and fisheries, with some effect. The pattern would seem to be some success from the 1880s onwards. Alternative explanations are possible and examiners must be open to alternative approaches. If in doubt, consult your Team Leader. [60]

103

Page 107: History A Mark Schemes for the Units January 2010 - The ...

F966 Mark Scheme January 2010

Theme 4: Russia and its Rulers 1855 – 1964

10 ‘The nature of Russian government was changed more by Stalin than by any other ruler.’ How far do you agree with this view of the period from 1855 to 1964? [60]

No set conclusions are to be expected, but candidates must answer the question and address the theme over the full period. Candidates should focus on the phrase ‘the nature of Russian government’ in their answers. Candidates may argue either for or against Stalin as having changed the nature of Russian government most, but must do so comparatively in the context of other rulers and leaders. Candidates may argue that the highly dictatorial nature of the Stalinist regime justifies this view and are likely to support this by reference to events such as the terror and the purges. Others may argue that this represented continuity with the nature of much previous Russian government, even if the scale was much greater. Many candidates may show awareness that some historians see great continuity between Lenin and Stalin whereas others view Stalin as significantly different from Lenin. This could be very usefully debated. Candidates may argue in favour of Alexander II because of the emancipation of the serfs and his other reforms such as the zemstva. Candidates may argue in favour of Alexander III because of ‘the Reaction’ though many will see this as a reversion to traditional autocracy. Candidates may argue that the end of over 300 years of Romanov rule in February 1917 was the most significant turning point in the nature of Russian government as it ended the 304 year old Romanov dynasty, but may argue that ultimately this led to the replacement of ‘Romanov Tsars’ by ‘red Tsars’. Many candidates will undoubtedly argue that October 1917 and the triumph of Bolshevism significantly changed the nature of Russian government as it crushed all possibility that a liberal democracy might emerge in Russia. Candidates may argue that Khrushchev’s secret speech of 1956 and his subsequent de-stalinisation marked a significant change in the nature of Russian government.

Examiners must be open to alternative approaches. If in doubt, they should consult their Team Leader.

11 Assess the view that all the rulers of Russia had similar aims in domestic policy in the period from 1855 to 1964. [60]

No set conclusions are to be expected, but candidates must answer the question and address the theme over the full period. Candidates should focus on the phrase ‘similar aims in domestic policy’ in their answers. Candidates are likely to want to argue both for and against this assertion. Candidates may well argue that retention of power, whether autocratic or dictatorial, and the crushing of opposition were priorities for all the rulers even if some were singularly unsuccessful in achieving those goals. Candidates may well argue that the modernisation of Russia was an aim for all the rulers, though candidates are likely to differentiate between rulers such as Alexander III and Stalin in terms of motives and extent. Candidates may however wish to argue that the communist rulers had very different core priorities to the Tsars in terms of political ideology and social priorities; others may contend that this should have been the case but that rulers, especially Stalin (though some will also indict Lenin). Candidates may argue that the Tsars were not uniform in their core aims; they are likely to see Alexander II as having different priorities to his successors, citing emancipation and the other reforms of the 1860s in support. Candidates may also argue that the communist rulers were not uniform in their core aims either; they are likely to argue that Khrushchev had very different priorities to Stalin, citing de-stalinisation as support. Candidates may well understand that whether Lenin and Stalin had similar aims is subject to historical debate.

Examiners must be open to alternative approaches. If in doubt, they should consult their Team Leader.

104

Page 108: History A Mark Schemes for the Units January 2010 - The ...

F966 Mark Scheme January 2010

12 Assess the view that the lives of the peasants in Russia did not improve in the period from 1855 to 1964. [60]

No set conclusions are to be expected, but candidates must answer the question and address the theme over the full period. Candidates should focus on the phrases ‘lives of the peasants’ and ’did not improve’ in their answers. Candidates may well consider how valid the phrase ’did not improve’ is. Candidates may argue that some rulers, for example Alexander II and Khrushchev made a sustained attempt to improve the lives of the people. Some candidates may argue that despite the brutality, Lenin and even Stalin did do some things that improved the lives of the people. Arguably the communists did much more to introduce social reform, for example in the sphere of education, than the Tsars. Candidates may also argue that there was little real improvement in the lives of the people. For example, peasants were serfs under the Romanovs until 1861, but candidates may argue that there was little real improvement and / or that collectivization was a ‘second serfdom’. Before and after 1917 there was harsh treatment of the peasantry by both regimes; ‘squeezed dry’ to finance industrialization. Famine hit, e.g. 1891, 1921 & 1932, regardless of regime, although arguably Stalin’s denial of the famine of the 1930s made its impact worse. Control over their lives, whether exercised through the Mir, the Land Captains & the Kolkhoz was a common feature, although distinctions may clearly be made. Candidates may use the systematic Russification of the non-Russian peasants both before and after 1917 as another clear example of there being no significant change. Candidates may also wish to argue that there were times when rulers did improve the lives of the peasants, but that these improvements were most typically temporary rather than embedded. For example, the peasants were given glimpses of reform, e.g. the Peasants Land Bank from the 1880s, the Decree on Land in 1917 and the NEP from 1921. All of these changes led to improvements, albeit temporary, in their living and working conditions. Both regimes had a temporary Kulak policy under Stolypin from 1906 & under the NEP from 1921-28 as peasants were encouraged to ‘enrich themselves’.

Examiners must be open to alternative approaches. If in doubt, they should consult their Team Leader.

105

Page 109: History A Mark Schemes for the Units January 2010 - The ...

F966 Mark Scheme January 2010

Theme 5: Civil Rights in the USA 1865 – 1992 13 To what extent did the aims of the campaigners for African American civil rights

remain the same in the period from 1865 to 1992?

No set conclusions are to be expected, but candidates must answer the question and address the theme over the whole period. The weaker candidates will concentrate almost exclusively on the various African American leaders and will probably not go beyond contrasting the non-violent methods of Booker T Washington and Martin Luther King with the more radical goals and methods of leaders such as Du Bois, Garvey, Malcolm X, the Black Power movement and the Black Panthers. Weaker candidates are unlikely to have much to say about civil rights campaigns after 1965 beyond observing that King was less successful in his final years. The majority of candidates should be able to contrast the limited aims of Booker T Washington – offering black co-operation with a white-dominated society in return for economic gains – with the legal campaigns of the NAACP to secure their rights under the 14th and 15th amendments and Martin Luther King’s high profile campaign to abolish the whole Jim Crow system. They should also be able to contrast these assimilationist goals with the separatism advocated by leaders such as Garvey and Malcolm X. The best candidates will explain how changed circumstances, especially in the post-war period enabled campaigners to pursue a more ambitious strategy than previous leaders. They will also refer to the importance of grass-roots activism when writing, for example, about the Sit-In campaigns and the Freedom Rides. The better candidates should also be able to analyse the lack of coherence in the aims of the Black Power movement and the Black Panthers as well as analysing the problems the civil rights movement experienced in the 1970s and 1980s with controversy over bussing and affirmative action and perceptions of continued police discrimination (reference could be made to the Rodney King incident of 1992). High level answers will make effective comparisons over the whole period, perhaps referring to the persistence of de facto discrimination and the existence of a large underclass trapped in a cycle of poverty, unemployment, poor housing in contrast to a prosperous black middle class which had taken advantage of the changed attitude to racism and the abolition of de jure discrimination. Examiners must be open to alternative approaches. If in doubt, they should consult their Team Leader.

14 ‘The internal divisions within the trade union and labour movement in the USA the

most important obstacle to the progress of labour rights in the period from 1865 to 1992’. How far do you agree? No set conclusions are to be expected, but candidates must answer the question and address the theme over the whole period. Weaker candidates will probably refer only in general terms to the divisions over membership between the Knights of Labor and the AFL in the late 19th century, and the AFL and the CIO in the 1930s. They might also refer to ethnic conflict between whites and blacks and between native-born workers and the new immigrants of the late 19th and early 20th centuries. Some weaker candidates may also discount (or even ignore) the internal divisions within the labour movement and provide a list of alternative factors without adequately comparing their importance. Many (perhaps most) candidates, while acknowledging the divisions within the labour movement, will concentrate their analysis on the other obstacles to the progress of labour rights. These include the use force of by government authorities to end strikes (for example, President Cleveland during the Pullman strike in 1894 or Massachusetts Governor Coolidge in Boston in 1919), the willingness of the courts to issue injunctions against strikers under the Sherman Anti-Trust Act, specifically anti-labour legislation such as the Taft-Hartley Act of 1947 and the Landrum-Griffin act of 1959, and the employers’ use of both yellow dog

106

Page 110: History A Mark Schemes for the Units January 2010 - The ...

F966 Mark Scheme January 2010

contracts and violent strike-breakers (such as Pinkerton guards in the Homestead strike). Most candidates should attempt some evaluation of the relative importance of these factors in relation to divisions among workers. Better candidates will be able to analyse how far labour divisions were exacerbated by a climate in the USA hostile to the progress of labour movements and point out that this climate was, in part, the result of extremist labour movements (such as the Molly Maguires or the Wobblies) but that it also made it easy for employers and governments to associate trade unionism with socialism and communism, thereby undermining middle class sympathy for workers. High level candidates will be aware that the arguments within the labour movement extended beyond union membership and tactics and were bedevilled by divisions over race, class, religion and gender. They will provide intelligently selected examples to illustrate these divisions. High calibre candidates might, as well as analysing these divisions and the obstacles, point out that the progress of labour rights depended crucially on the support of the federal government (as in the New Deal and the JFK-LBJ years). They will also be able to analyse the importance of wider economic conditions, especially in explaining the decline of trade union power since the 1950s. Examiners must be open to alternative approaches. If in doubt, they should consult their Team Leader.

15 ‘The Indian Reorganization Act in the New Deal was the most important turning

point in the development of Native American civil rights in the USA in the period from 1865 to 1992.’ How far do you agree with this view?

No set conclusions are to be expected, but candidates must answer the question and address the theme over the whole period. Most candidates will agree with this statement and point out that the Indian New Deal replaced forced assimilation with respect for Indian culture and customs which, arguably, has persisted to the present. Furthermore, the Indian Reorganization Act prevented the alienation of more tribal land and granted Indian communities a measure of governmental and judicial autonomy. Weaker candidates will probably see this as a stage of “good” policy towards Indians as opposed to the “bad” periods of forced assimilation from 1887 until 1933 and the termination policy of the 1950s. As such, they will regard the Indian Reorganization Act as a significant turning point, although such candidates are unlikely to be able to compare it with other turning points. Most candidates will offer some alternatives (such as the 1887 Dawes Act at the end of the Indian Wars, the end of the Indian New Deal in 1945, the end of termination in the 1960s or the first militant action of Red Power in 1969) but their attempts to compare them with the Indian Reorganization Act may be unconvincing. The best candidates will be aware of the limitations of Indian Reorganization Act and the degree of opposition to it (for different reasons) in Congress and among Native Americans. They might point out that Collier’s policies were as paternalistic in their way as all other federal Indian policies, and that the majority of white Americans (and a significant number of Indians themselves) continued to believe that Indians should assimilate into mainstream US culture. As such, the Indian Reorganization Act could be seen as an aberration rather than a turning point. The best candidates might also contrast the importance of federal policy shifts with the impact of economic and social change in the USA on Native American lives and attitudes (especially the two world wars, the Cold War and the post-war expansion of the US economy). Alternatively, the best candidates will make effective comparisons between the different turning points they analyse and explain fully why one is more important than the others.

Examiners must be open to alternative approaches. If in doubt, they should consult their Team Leader.

107

Page 111: History A Mark Schemes for the Units January 2010 - The ...

F966 Mark Scheme January 2010

Theme 6: The Development of Democracy in Britain 1868 – 1997 16 How far did economic factors influence the results of general elections from 1868 to

1997?

No set conclusions are to be expected, but candidates must answer the question and address the theme over the whole period. Economic change undoubtedly influenced the outcome of a significant number of elections during the period particularly where it was linked with a slow down in economic growth and associated rises in unemployment. For example, the parties that lost elections in 1868, 1880, 1922, 1929 and 1979 were all linked, and to an extent blamed for, high levels of unemployment, falling real wages and a lowering of living standards. Some candidates are likely to show that there were also other economic issues that dominated some election campaigns such as tariff reform (1906, 1923), fiscal policy (1910), changes in the industrial infrastructure (1966), a faltering balance of payments (1970), and rising fuel prices (1974). It would, of course, be highly simplistic to argue that elections were always fought over the issue of economic change. Most candidates should be able to discuss a range of other factors. These might include: external influences ( e.g. the effects of wars on the 1906, 1918, 1945 and 1983 elections); social changes ( e.g. the impact and debate over the NHS which influenced the result of the 1951 election; consumerism which prompted Macmillan’s election winning slogan ‘You’ve never had it so good’ in 1959); internal party divisions ( e.g. the Liberals in the 1880s and1920s, Labour in the mid-1950s) and the coming of class based politics ( e.g. the rise of the Labour party to push the Liberals into third place, the rise of general unionism and industrial unrest which affected the outcome of elections in the 1880s, 1906, 1951 and, most significantly, 1979). A strong case could be made for economic factors mainly (but not solely) influencing the results of elections although better responses will probably offer a more balanced analysis and evaluation. Also, a decent, but not exhaustive, range of elections should be referenced to illustrate change and/or continuity over the period. Examiners must be open to alternative approaches. If in doubt, they should consult their Team Leader.

17 ‘There was more continuity than change in the role that Prime Ministers played in

the development of the democratic system from 1868 to 1997.’ How far do you agree?

No set conclusions are to be expected, but candidates must answer the question and address the theme over the whole period. Over the period, parliaments were managed differently, monarchs varied, control of cabinets increased and there was more responsibility for international affairs (especially wars) and disposal of finance (e.g. the Secret Fund). It might be argued that prime ministers handled such changes more skilfully as time went by, whilst having their power ‘checked’ by political institutions such as political parties, pressure groups, the judiciary and the media. Thus, prime ministers were not allowed to abuse their power. They were expected to play a responsible role in the maintenance and enhancement of democracy. Another line of argument might be that the power of prime ministers was never kept in check enough, especially with respect to the latter part of the period. Developments worth discussing might include ‘Prime Ministers Questions’, the appointment system, collective decision making in the Cabinet, control over Cabinet meetings, the exploitation of new technology (e.g. Baldwin’s use of the radio), influence on the Budget, the power to call elections, the right to remove ministers (1905 onwards) and decisions to go to war (e.g Chamberlain, Thatcher). There should be a focus on assessing the extent of continuity and change in the role of prime ministers; there is little need for candidates to discuss the relative importance of other factors on democracy.

108

Page 112: History A Mark Schemes for the Units January 2010 - The ...

F966 Mark Scheme January 2010

109

Examiners must be open to alternative approaches. If in doubt, they should consult their Team Leader.

18 Assess the view that educational reform was the most important influence on the

development of democracy from 1868 to 1997.

It is plausible to argue that there was considerable continuity in the way in which educational reforms influenced the development of democracy. The education acts of 1870 and 1880 seemed to influence a raft of political reforms that were implemented before the end of the nineteenth century. Forster’s Act filled the gaps in elementary school provision especially in the countryside and it was no coincidence that the franchise was extended in 1884 to include £10 householders and lodgers residing in rural areas. The other major political reform of the time, the Redistribution of Seats Act, 1885, was closely linked to the Third Reform Act. This pattern of educational reform influencing changes to the democratic system continued in to the twentieth century. The reforms included the education policies of the Liberals from 1906 to 1910 which had a bearing on the 1910 constitutional crisis; Fisher’s Act of 1918 which occurred at the same time as the Representation of the People’s Act; and the policy initiatives of the inter-war years (Hadow, Spens, Tawney) which tied in with a further extension to the franchise (1928). Some educational reforms, such as that of 1870 and 1988, actually allowed direct participation in politics through the educational system (e.g. election to school boards, governing bodies). Although there were other influences on changes to the democratic system, the expansion in the number of people who were educated undoubtedly caused politicians to implement policies that engendered greater participation in the political process. Also, more generally, educational reforms resulted in a demand for a change to the way in which people were represented, hence the rise of the Labour party and Trades Unions. However, better candidates are likely to point out that some changes to education did not improve the democratic system. The 1902 act appeared to be an attempt to maintain the ruling status quo and the 1944 act created a tripartite system that, in the long run, seemed to reflect a divided society in which many experienced a limited educational and, subsequently, vocational choice. More contentiously, governments throughout the whole of the period continued to allow the existence and growth of the private education sector which could also be viewed as divisive and not characteristic of a truly democratic society. Finally, some candidates might point out that educational change was more likely to follow democratic change. This was partly true of the 1870 act (‘we must educate our masters’) and the creation of the Board of Education in 1899. Examiners must be open to alternative approaches. If in doubt, they should consult their Team Leader.

Page 113: History A Mark Schemes for the Units January 2010 - The ...

Grade Thresholds

Advanced GCE History (H506) Advanced Subsidiary GCE History (H106) January 2010 Examination Series Unit Threshold Marks

Unit Maximum Mark

A B C D E U

Raw 100 73 64 56 48 40 0 F961/01 UMS 100 80 70 60 50 40 0 Raw 100 73 64 56 48 40 0 F961/02 UMS 100 80 70 60 50 40 0 Raw 100 73 65 57 50 43 0 F962/01 UMS 100 80 70 60 50 40 0 Raw 100 73 65 57 50 43 0 F962/02 UMS 100 80 70 60 50 40 0 Raw 100 68 60 53 46 39 0 F963/01 UMS 100 80 70 60 50 40 0 Raw 100 68 60 53 46 39 0 F963/02 UMS 100 80 70 60 50 40 0 Raw 100 67 60 53 46 40 0 F964/01 UMS 100 80 70 60 50 40 0 Raw 100 67 60 53 46 40 0 F964/02 UMS 100 80 70 60 50 40 0 Raw 80 64 56 48 40 32 0 F965 UMS 80 64 56 48 40 32 0 Raw 120 85 76 68 60 52 0 F966/01 UMS 120 96 84 72 60 48 0 Raw 120 85 76 68 60 52 0 F966/02 UMS 120 96 84 72 60 48 0

Specification Aggregation Results Overall threshold marks in UMS (ie after conversion of raw marks to uniform marks) Maximum

Mark A B C D E U

H106 200 160 140 120 100 80 0

The cumulative percentage of candidates awarded each grade was as follows:

A B C D E U Total Number of Candidates

H106 18.56 46.74 75.61 92.42 99.02 100.00 1371

For a description of how UMS marks are calculated see: http://www.ocr.org.uk/learners/ums_results.html Statistics are correct at the time of publication.

110

Page 114: History A Mark Schemes for the Units January 2010 - The ...

OCR (Oxford Cambridge and RSA Examinations) 1 Hills Road Cambridge CB1 2EU OCR Customer Contact Centre 14 – 19 Qualifications (General) Telephone: 01223 553998 Facsimile: 01223 552627 Email: [email protected] www.ocr.org.uk For staff training purposes and as part of our quality assurance programme your call may be recorded or monitored

Oxford Cambridge and RSA Examinations is a Company Limited by Guarantee Registered in England Registered Office; 1 Hills Road, Cambridge, CB1 2EU Registered Company Number: 3484466 OCR is an exempt Charity OCR (Oxford Cambridge and RSA Examinations) Head office Telephone: 01223 552552 Facsimile: 01223 552553 © OCR 2010

Page 115: History A Mark Schemes for the Units January 2010 - The ...

Oxford Cambridge and RSA Examinations

GCE

History A Advanced Subsidiary GCE

Unit F963/02: Option B Modern 1815-1945

Mark Scheme for January 2011

Page 116: History A Mark Schemes for the Units January 2010 - The ...

OCR (Oxford Cambridge and RSA) is a leading UK awarding body, providing a wide range of qualifications to meet the needs of pupils of all ages and abilities. OCR qualifications include AS/A Levels, Diplomas, GCSEs, OCR Nationals, Functional Skills, Key Skills, Entry Level qualifications, NVQs and vocational qualifications in areas such as IT, business, languages, teaching/training, administration and secretarial skills. It is also responsible for developing new specifications to meet national requirements and the needs of students and teachers. OCR is a not-for-profit organisation; any surplus made is invested back into the establishment to help towards the development of qualifications and support which keep pace with the changing needs of today’s society. This mark scheme is published as an aid to teachers and students, to indicate the requirements of the examination. It shows the basis on which marks were awarded by Examiners. It does not indicate the details of the discussions which took place at an Examiners’ meeting before marking commenced. All Examiners are instructed that alternative correct answers and unexpected approaches in candidates’ scripts must be given marks that fairly reflect the relevant knowledge and skills demonstrated. Mark schemes should be read in conjunction with the published question papers and the Report on the Examination. OCR will not enter into any discussion or correspondence in connection with this mark scheme. © OCR 2011 Any enquiries about publications should be addressed to: OCR Publications PO Box 5050 Annesley NOTTINGHAM NG15 0DL Telephone: 0870 770 6622 Facsimile: 01223 552610 E-mail: [email protected]

Page 117: History A Mark Schemes for the Units January 2010 - The ...

F963/02 Mark Scheme January 2011

1

Question (a) Maximum mark 30

A01a and b AO2a

1 13-14 15-16

2 11-12 13-14

3 9-10 10-12

4 7-8 8-9

5 5-6 6-7

6 3-4 3-5

7 0-2 0-2

Notes related to Part A: (i) Allocate marks to the most appropriate level for each AO (ii) If several marks are available in a box, work from the top mark down until the best fit has

been found (iii) Many answers will not be at the same level for each AO

Page 118: History A Mark Schemes for the Units January 2010 - The ...

F963/02 Mark Scheme January 2011

2

Marking Grid for Question (a) A0s A01a and b A02a

Total for each question =30

Recall, select and deploy historical knowledge appropriately, and communicate knowledge and understanding of history in a clear and effective manner. Demonstrate understanding of the past through explanation, analysis and arriving at substantiated judgements of: - key concepts such as causation, consequence, continuity, change and significance within an historical context; - the relationships between key features and characteristics of the periods studied.

As part of an historical enquiry, analyse and evaluate a range of appropriate source material with discrimination.

Level 1 Consistent and developed comparison of the key issue with a balanced and well-supported judgement. There will be little or no unevenness.

Focused use of a range of relevant historical concepts and context to address the key issue.

The answer is clearly structured and organised. Communicates coherently, accurately and effectively.

13-14

Focused comparative analysis. Controlled and discriminating evaluation of content and provenance, whether integrated or treated separately.

Evaluates using a range of relevant provenance points in relation to the sources and question. There is a thorough but not necessarily exhaustive exploration of these.

15-16 Level 2 Largely comparative evaluation of

the key issue with a balanced and supported judgement. There may be a little unevenness in parts.

Focused use of some relevant historical context with a good conceptual understanding to address the key issue.

The answer is well structured and organised. Communicates clearly.

11-12

Relevant comparative analysis of content and evaluation of provenance but there may be some unevenness in coverage or control.

Source evaluation is reasonably full and appropriate but lacks completeness on the issues raised by the sources in the light of the question.

13-14

Level 3 Some comparison linked to the key issue. Is aware of some similarity and/or difference. Judgements may be limited and/or inconsistent with the analysis made.

Some use of relevant historical concepts and contexts but uneven understanding. Inconsistent focus on the key issue.

The answer has some structure and organisation but there is also some description. Communication may be clear but may not be consistent.

9-10

Provides a comparison but there is unevenness, confining the comparison to the second half of the answer or simply to a concluding paragraph. Either the focus is on content or provenance, rarely both.

Source evaluation is partial and it is likely that the provenance itself is not compared, may be undeveloped or merely commented on discretely.

10-12

Page 119: History A Mark Schemes for the Units January 2010 - The ...

F963/02 Mark Scheme January 2011

3

A0s A01a and b A02a Level 4 Some general comparison but

undeveloped with some assertion, description and/or narrative. Judgement is unlikely, unconvincing or asserted.

A general sense of historical concepts and context but understanding is partial or limited, with some tangential and/or irrelevant evidence.

Structure may be rather disorganised with some unclear sections. Communication is satisfactory but with some inaccuracy of expression.

7-8

Attempts a comparison but most of the comment is sequential. Imparts content or provenance rather than using it.

Comparative comments are few or only partially developed, often asserted and/or ‘stock’ in approach.

8-9 Level 5 Limited comparison with few links to

the key issue. Imparts generalised comment and /or a weak understanding of the key points. The answer lacks judgement or makes a basic assertion.

Basic, often inaccurate or irrelevant historical context and conceptual understanding.

Structure lacks organisation with weak or basic communication.

5-6

Identifies some comparative points but is very sequential and perhaps implicit

Comment on the sources is basic, general, undeveloped or juxtaposed, often through poorly understood quotation.

6-7 Level 6 Comparison is minimal and basic

with very limited links to the key issue. Mainly paraphrase and description with very limited understanding. There is no judgement.

Irrelevant and inaccurate concepts and context.

Has little organisation or structure with very weak communication.

3-4

Little attempt to compare. Weak commentary on one or two undeveloped points, with basic paraphrase. Sequencing is characteristic.

Comments on individual sources are generalised and confused.

3-5 Level 7 Fragmentary, descriptive,

incomplete and with few or no links to the key issue. There is little or no understanding. Much irrelevance.

Weak or non existent context with no conceptual understanding.

No structure with extremely weak communication.

0-2

No attempt to compare either content or provenance with fragmentary, brief or inaccurate comment.

Makes no attempt to use any aspects of the sources.

0-2

Page 120: History A Mark Schemes for the Units January 2010 - The ...

F963/02 Mark Scheme January 2011

4

Question (b) Maximum mark 70

A01a and b AO2a and b

1 20-22 42-48

2 17-19 35-41

3 13-16 28-34

4 9-12 21-27

5 6-8 14-20

6 3-5 7-13

7 0-2 0-6

Notes related to Part B: (i) Allocate marks to the most appropriate level for each AO (ii) If several marks are available in a box, work from the top mark down until the best fit has

been found (iii) Many answers will not be at the same level for each AO

Page 121: History A Mark Schemes for the Units January 2010 - The ...

F963/02 Mark Scheme January 2011

5

AOs A0Ia and b Ao2a and b

Total mark for the question = 70

Recall, select and deploy historical knowledge appropriately, and communicate knowledge and understanding of history in a clear and effective manner. Demonstrate understanding of the past through explanation, analysis and arriving at substantiated judgements of: - key concepts such as causation, consequence, continuity, change and significance within an historical context; - the relationships between key features and characteristics of the periods studied.

As part of an historical enquiry, analyse and evaluate a range of appropriate source material with discrimination. Analyse and evaluate, in relation to the historical context, how aspects of the past have been interpreted and represented in different ways.

Level 1 Convincing analysis and argument with developed explanation leading to careful, supported and persuasive judgement arising from a consideration of both content and provenance. There may be a little unevenness at the bottom of the level.

Sharply focused use and control of a range of reliable evidence to confirm, qualify, extend or question the sources.

Coherent organised structure. Accurate and effective communication.

20-22

A carefully grouped and comparative evaluation of all the sources with effective levels of discrimination sharply focused on the interpretation.

Analyses and evaluates the strengths, limitations and utility of the sources in relation to the interpretation. Uses and cross references points in individual or grouped sources to support or refute an interpretation.

Integrates sources with contextual knowledge in analysis and evaluation and is convincing in most respects. Has synthesis within the argument through most of the answer.

42-48

Level 2 Good attempt at focused analysis, argument and explanation leading to a supported judgement that is based on the use of most of the content and provenance.

A focused use of relevant evidence to put the sources into context.

Mostly coherent structure and organisation if uneven in parts. Good communication.

17-19

Grouped analysis and use of most of the sources with good levels of discrimination and a reasonable focus on the interpretation.

Analyses and evaluates some of the strengths and limitations of the sources in relation to the interpretation. May focus more on individual sources within a grouping, so cross referencing may be less frequent.

Some, perhaps less balanced, integration of sources and contextual knowledge to analyse and evaluate the interpretation. Synthesis of the skills may be less developed. The analysis and evaluation is reasonably convincing.

35-41

Page 122: History A Mark Schemes for the Units January 2010 - The ...

F963/02 Mark Scheme January 2011

6

AOs A0Ia and b Ao2a and b Level 3 Mainly sound analysis, argument

and explanation, but there may be some description and unevenness. Judgement may be incomplete or inconsistent with the analysis of content and provenance.

Some relevant evidence but less effectively used and may not be extensive.

Reasonably coherent structure and organisation but uneven. Reasonable communication.

13-16

Some grouping although not sustained or developed. Sources are mainly approached discretely with limited cross reference. Their use is less developed and may, in parts, lose focus on the interpretation. There may be some description of content and provenance.

Is aware of some of the limitations of the sources, individually or as a group, but mostly uses them for reference and to illustrate an argument rather than analysing and evaluating them as evidence. There is little cross referencing.

There may be unevenness in using knowledge in relation to the sources. Synthesis may be patchy or bolted on. Analysis and evaluation are only partially convincing.

28-34

Level 4 Attempts some analysis, argument and explanation but underdeveloped and not always linked to the question. There will be more assertion, description and narrative. Judgements are less substantiated and much less convincing.

Some relevant evidence is deployed, but evidence will vary in accuracy, relevance and extent. It may be generalised or tangential.

Structure is less organised, communication less clear and some inaccuracies of expression.

9-12

Sources are discussed discretely and largely sequentially, perhaps within very basic groups. Loses focus on the interpretation. The sources are frequently described.

May mention some limitations of individual sources but largely uses them for reference and illustration. Cross referencing is unlikely.

An imbalance and lack of integration between sources and knowledge often with discrete sections. There is little synthesis. Analysis and explanation may be muddled and unconvincing in part.

21-27

Level 5 Little argument or explanation, inaccurate understanding of the issues and concepts. The answer lacks judgement.

Limited use of relevant evidence or context which is largely inaccurate or irrelevant.

Structure is disorganised, communication basic and the sense not always clear.

5-8

A limited attempt to use the sources or discriminate between them. The approach is very sequential and referential, with much description. Points are undeveloped.

There is little attempt to analyse, explain or use the sources in relation to the question. Comment may be general.

There is a marked imbalance with no synthesis. Analysis and explanation are rare and comments are unconvincing.

14-20

Page 123: History A Mark Schemes for the Units January 2010 - The ...

F963/02 Mark Scheme January 2011

7

AOs A0Ia and b Ao2a and b Level 6 There is very little explanation or

understanding. Largely assertion, description and narrative with no judgement. Extremely limited relevance to the question.

Evidence is basic, generalised, patchy, inaccurate or irrelevant.

Little organisation or structure with poor communication.

3-4

Very weak and partial use of the sources for the question. No focus on interpretation.

A very weak, general and paraphrased use of source content.

No synthesis or balance. Comments are entirely unconvincing.

7-13 Level 7 No argument or explanation.

Fragmentary and descriptive with no relevance to the question.

No understanding underpins what little use is made of evidence or context.

Disorganised and partial with weak communication and expression.

0-2

Little application of the sources to the question with inaccuracies and irrelevant comment. Fragmentary and heavily descriptive.

No attempt to use any aspect of the sources appropriately.

No contextual knowledge, synthesis or balance. There is no attempt to convince.

0-6

Page 124: History A Mark Schemes for the Units January 2010 - The ...

F963/02 Mark Scheme January 2011

8

The Condition of England 1815-1853 1 (a) Study Sources C and D.

Compare these sources as evidence for views on the Tolpuddle case of 1834. [30] No set answer is expected, but candidates need to compare the contents, evaluating such matters as authorship, dating, utility and reliability, so using the sources ‘as evidence for…’. The headings and attributions should aid evaluation and reference to both is expected in a good answer. The context was the growing power of larger and more general Unions post 1829 and in particular the emergence of the GNCTU in early 1834. Both sources, from very different viewpoints, agree that the real issues involved are control of the workforce and the right to join a Union, the wage issue being secondary. Both focus on the use of the law to resolve the issue. Frampton in C is convinced rural labourers are ‘waiting to join the Union as soon’ as the case signals their right to do so confirmed by Loveless in D who has been instrumental in forming a Friendly Society and receiving direction from Trade Societies, a clear reference to the growing labour movement that produced organisations like the GNCTU. The testimonies of both show them to be the main protagonists in the case, Loveless labelling Frampton as one of the ‘unjust and cruel’ magistrates. Both agree that the impact of the case will be to deter trade unionism. They disagree over the case itself. Frampton (C) comments in a matter of fact tone that the judge ruled that labourers could not join a Union without punishment and that the 6 labourers had broken the law, a clear warning to the workforce. In contrast Loveless (D) considers the verdict to be not just a simple statement of existing law but a travesty of justice, a trial marked by unjust methods and intimidation – that the authorities had found an old Act of 1797 (mutiny in the navy) and twisted its meaning to apply to oaths to join a civil Friendly Society, applied after the event (‘placards were then displayed’). Frampton implies that those on trial were troublemakers, whilst Loveless demonstrates how this impression was unfairly implied (idle drunkards). They also partially disagree. Frampton (C) is convinced that the judgement will discourage labourers joining a Union and that it was welcomed by the ‘higher classes’. On the other hand Loveless (D), by his catalogue of injustice and the title of his pamphlet (‘Victims’), is trying to limit this by exposing the methods of the authorities (candidates may refer to the pardon issued in 1836 following the campaign against the harsh sentences). In terms of provenance both are highly slanted sources. Frampton (C) is a local squire, magistrate and landowner concerned that a stand be taken against rural unionism and with clear contacts with government (he writes with familiarity to the Home Secretary, as though reporting on a successful and coordinated campaign with government involvement –‘looked forward to’). He reveals his role in keeping the Home Secretary informed as to how the verdict has been received in Dorset, confirmed by Loveless (D) in his comment on Frampton – ‘a name I shall not forget’. Loveless writes from the labour and radical perspective. As one of the ‘victims’, his purpose is to campaign (after his pardon and return from Australia in 1837) to reverse the verdicts and use the case to advance the radical cause. He wants to link local and national authorities as jointly responsible for a miscarriage of justice – ‘Victims of Whiggery’. He clearly blames Frampton – the wage reduction seems to occur after an alleged negotiation was communicated to him. In terms of judgement candidates could regard both as equally useful. Together they provide a balanced view. The evidence of Frampton is valuable in providing an informed assessment of the impact and result of the case (deterring Unionism). It is better evidence for the attitudes of government and local landowners, whilst Loveless provides the

Page 125: History A Mark Schemes for the Units January 2010 - The ...

F963/02 Mark Scheme January 2011

9

perspective of the radicals and labourers with more detail as to the methods used to obtain conviction. Candidates may be aware of much harsher penalties than previously exacted (7 years’ transportation instead of the maximum 6 months hard labour under the old Combination Laws).

1 (b) Study all the Sources.

Use your own knowledge to assess how far the Sources support the interpretation that the workers and their leaders were the main reason for trade Union weakness in the period from 1824-1844 [70] Successful answers will need to make use of all five Sources, testing them against contextual knowledge and evaluating their strengths and weaknesses, including any limitations as evidence. A range of issues may be addressed in focusing upon the terms of the question but no set conclusion is expected. A variety of interpretations on weakness is possible here and all but Place (A) support a variety of views – alongside the views of workers and their leaders there is the hostility of employers, the opposition of the authorities, both central and local, and economic conditions in general. Three of the sources (Place in A, Loveless in D and Burt in E) are from radical and Union leaders and may be the more revealing as to admitted weakness (odds stacked against workers) whilst two are from an establishment view, (Frampton in C and the Cartoon in B) and might be expected to demonstrate hostility, stressing either worker weakness or government determination to make a stand. The case for workers and their leaders themselves undermining unions is clear in A, B, C and E, although candidates will need to be aware of the exaggerated nature of the cartoon evidence in B. These sources stress that both industrial and rural workers were reluctant to act by combining into Unions and lacked an appreciation of how to proceed, despite the efforts of leaders like Place and Loveless. Place in A is revealing given his role in working to repeal the Combination Laws in 1824. In an unpublished account, part of his working notes for the MP Hume, he is quite scathing on the workers he is trying to rehearse to give appropriate evidence to the Select Committee on Combinations in 1824. He comments on the parochial nature of their grievances, their stress on isolated issues like machinery and their lack of politicisation and polish, especially their lack of understanding that outside a union they were isolated and unequal units in the population/wage situation. This is confirmed by the stress on wages in D (Loveless). Bread and butter issues and local conflicts would lead workers to act individually or in local groups, (and here candidates may use knowledge to confirm this - by the late 1820s and 1830s Doherty’s National Association on the Protection of Labour of 1829 failed to show solidarity in the Lancashire Spinners strike in 1831; the GNCTU failed to mobilise other trades to support the Derbyshire silk weaver’s strike). The anti Trade Union cartoon (B) demonstrates feeble and corrupt leadership, greedy and drunken (a charge also evident in D at Tolpuddle). Policy in B would seem to amount to posturing – a procession will suffice or ‘sommat o’ that sort’. However this evidence is clearly exaggerated, although there were cases of Treasurers running off with funds (Doherty’s NAPL). It does highlight the problems of national organisation for large and radical unions (too large?), such as recruiting able officials and the problem of funding through subscriptions (few below the craft unions could afford them – the GNCTU charged 3d and the box in the cartoon is empty). The point about funding is reinforced by Burt (E) who confirms, from a labour perspective, that unions had no resources or reserve funds for financing strikes. He suggests, reliably, that there was no eagerness to strike or even to unionise amongst workers. The GNCTU only

Page 126: History A Mark Schemes for the Units January 2010 - The ...

F963/02 Mark Scheme January 2011

10

managed a membership, briefly, of 16,000 with the craft unions refusing to join (potters, builders, spinners and clothiers). Poor coordination, worsened by communication difficulties is referred to in the cartoon (B) when ignorance is shown about what was occurring in the west. However the existence of leaders like Place, Owen, Loveless and Doherty would suggest that not all were as the cartoon suggests, although Owen in particular might be seen as too idealistic. Loveless in D is clearly aware that rural workers would be put off joining a union given his fate. The cartoon reflects propertied fears at the burst of Union activity between 1829 and 1834. The sources thus provide a mixed view on the assertion in the question. The hostility of employers was also an important factor in union weakness (A, C, D and E). Place in A refers to the ‘will of the masters’ as a barrier to worker participation whilst Burt in E particularly emphasises this factor and is reliable given his personal experience in union and reform politics and the vantage of very informed hindsight. He speaks from personal experience, albeit as a child. He is referring to one of the most cohesive working communities in the country, a later Union vanguard, the N.E. miners. Yet in this period they are completely under the control of employers who could close employer owned shops, evict (mine owned pit cottages) and sack them, bringing in Welsh miners instead. It is not the workers, their leaders or the government which defeats them but employers who ‘treat them as though they do not exist’. This is corroborated in the rural areas by C and D (Frampton and Loveless), although there may be some disagreement as Loveless argues that some farmers and landowners were prepared to settle until Frampton galvanised them into making a stand. Candidates might refer to the ‘Document’, which many employers post 1825 forced workers to sign, saying they would never join or pay a subscription to a union. The hostility of government and local authorities were also important factors in weakening the unions. Place in A refers to the conduct of magistrates who were also employers and the context of this source is the illegality of Unionism before 1824 and the difficulty in obtaining repeal of the Combination Laws. Legal barriers are evident in the other sources. The cartoon in B has one of the leaders commenting on the need to be moderate enough ‘to escape being put down by the government’, the context of the Tolpuddle martyrs case in 1834 which helped kill off the GNCTU. The law was used in an increasingly restrictive manner, rendering union activity almost impossible (an illiterate workforce could not read and sign declarations, instead undergoing entry rituals and oaths, hence Loveless’ tactics, then declared illegal under an old naval mutiny law). Candidates may know that Melbourne and government law officers advised Frampton on how to use the 1797 law and Loveless in D catalogues the pressures applied. Candidates might confirm this with reference to the Glasgow Spinners Case in 1837 (where the spinners’ leader was deported after being charged with conspiracy over the murder of a blackleg). A case could also be made that weakness was due to the nature of the economy and conditions in general. Loveless in D refers to the very low wages in rural areas, and the cartoon mentions the non payment of subscriptions. Place in A mentions the link between population and wages and to technical changes in the economy (machinery) that deskilled workers. Later (1840s) this is corroborated by Burt in E when he refers to the abundance of unskilled labour desperate for work, hence low wages. In the rural areas there was mechanisation and an overstocked labour supply that disadvantaged Union activity whilst in the urban areas neither artisans nor factory workers found it easy to control new economic forms in which they had no ownership and in which the educated consensus was for a freer market and laisser faire. As Burt in E comments, from the vantage of hindsight, they were ‘not destined to prevail at that time’.

Page 127: History A Mark Schemes for the Units January 2010 - The ...

F963/02 Mark Scheme January 2011

11

The Age of Gladstone and Disraeli 1865-1886 2 (a) Study Sources A and B

Compare these Sources as evidence for the views of Disraeli on extending the right to vote. [30] No set answer is expected, but candidates need to compare the contents, evaluating such matters as authorship, dating, utility and reliability, so using the sources ‘as evidence for…’. The headings and attributions should aid evaluation and reference to both is expected in a good answer. Both sources are speeches to Parliament by Disraeli and both comment on extending the suffrage. In both there is caution. In 1865 it is only to happen if the opportunity is favourable and as a matter of necessity. Disraeli implies that he is merely considering a remote possibility. In 1867 he is again at pains to suggest that his proposal will extend the vote no more than the Gladstone/Russell Bill of 1866.He relies on the personal payment of rates to exclude the many that would, as Householders, otherwise get the vote. He is not pursuing mere numbers, although he goes on in the same speech to admit that change has created numbers which it is ‘desirable’ to enfranchise. In both proposals he is concerned to set limits. The differences are that Disraeli is less convinced of suffrage extension in 1865 than in 1867. In 1865 he is cautious and anti democratic, warning against extending downwards to the working man and skilled artisan. He prefers more votes for the middle class and stresses the traditional view that voting was a privilege to be earned rather than a right to be exercised. In 1867, in contrast, he has opted for household suffrage, restricted only by an insistence on the personal payment of rates and, if own knowledge is used, by other fancy franchises. This proposal would reach the skilled working man in a way that his 1865 ideas would not. In 1867 he goes on to talk grandly about change, population growth, knowledge and progress in an almost Gladstonian manner. As regards provenance the dates and changing contexts are crucial. Both are public speeches to Parliament by Derby’s deputy and are thus likely to be less than candid, moulded by the rhetoric of the constitution, progress, virtue and the common good in a world that had been Liberal for over two decades. In both he seeks to avoid over commitment but has to come up with a concrete proposal in 1867. In 1865 Disraeli is concerned to scotch a Liberal Bill, appealing in an anti democratic and privileged way to conservative Liberals worried by the death of Palmerston and the advent of more reform minded Liberals like Russell and Gladstone. He is also concerned to reassure members of his own party that any Conservative extension to the franchise would be in keeping with the traditional constitution. Household suffrage was not a political possibility in 1865. However, by 1867 it was. The ‘opportunity’ of power has arrived if not the ‘urgent necessity’ – the Hyde Park riots in 1866 had followed the rejection of the Liberal Bill. The context was Disraeli considering extending the vote to appeal to Liberal reformers whilst securing his own right wing and thus Tory unity, hence the reference to no greater numbers than in 1866. He is at pains, at the end of the speech, to appeal for the cooperation of all MPs. Candidates are likely to judge Disraeli’s views as changeable according to circumstance. He is attempting to leave his options open, approving an extension but qualifying via generalities and specifics according to the political situation.

Page 128: History A Mark Schemes for the Units January 2010 - The ...

F963/02 Mark Scheme January 2011

12

2 (b) Study all the Sources Use your own knowledge to assess how far the Sources support the interpretation that Disraeli’s approach to parliamentary reform was purely opportunistic during the period from 1865 to 1867. [70] Successful answers will need to make use of all five Sources, testing them against contextual knowledge and evaluating their strengths and weaknesses, including any limitations as evidence. A range of issues may be addressed in focusing upon the terms of the question but no set conclusion is expected. The sources support three possible interpretations – that he was entirely opportunistic and pursued party and personal power at the expense of consistency and principle; that he stuck to the general principle of a moderate extension of the vote based on household suffrage and had done so since 1859; that he had general principles but was prepared to be flexible as the only way of securing a Conservative measure of Parliamentary reform. The question covers the period from the death of Palmerston, a key opponent of reform, through various proposals, to the successful Conservative Act of 1867. The pure opportunist argument is to be found in Sources D and E and, according to interpretation, Disraeli himself in A, B and C as well. The latter three may be the more convincing sources as two are public speeches by Disraeli (A and B) and one a private letter to an influential colleague, Gathorne Hardy, sounding him out by explaining the reasoning behind the volte face of accepting the abolition of compounding. There is plenty in the public speeches of A and B to suggest opportunism. In 1865 (A) Disraeli himself stresses the importance of ‘opportunity’ and the let out clause of ‘urgent necessity’. His only concern is to play on post Palmerstonian Liberal divisions by opening the door to both liberal opponents of reform (Robert Lowe and the Adullamites) and moderate radicals whilst reassuring the aristocratic Conservative right (General Peel and Cranborne), hence the reference to more middle class voters (fancy franchises). In 1867 Liberal divisions have led to resignation and Derby heads a minority Conservative government. He and Disraeli had to consolidate their reform reputation if they were to break Liberal dominance. The expectations raised in 1866 could not be put back in the bottle (Hyde Park riots), hence the announcement in B of household suffrage, mitigated by the personal payment of rates. Disraeli was still hoping to keep the Tory right but failed to do so when Cranborne resigned. Source C refers to the most obvious moment of opportunism when Disraeli, exploiting splits in the liberals and putting an end to Gladstone’s attempts to force him to reintroduce some limit to borough household suffrage (£5, £6 or £7) by letting the hare of compound abolition run, accepted compound abolition. He thus admitted the numbers he had been so concerned previously to prevent (400,000 extra urban voters). The letter to Hardy admits the motive was to put an end to Gladstone’s dominance of the issue and silence radical agitation but also tries to gloss the volte face by asserting no retreat from principle. Candidates might also refer to other measures electorally to advantage the Conservatives. Sources D and E are very critical of Disraeli’s tactics and opportunism, one from a conservative Liberal, Lowe, angered that Disraeli failed to make common cause to resist reform and the other from Disraeli’s rival from the aristocratic right, Cranborne. Their view will be coloured by Disraeli’s defeat of their ‘conservative’ views. Lowe uses the telling metaphors of a shy horse and an overloaded ship to describe Disraeli’s tactical opportunism but there is a sense of admiration for how he conned his party into supporting a radical measure. A comparison of C, the letter to Hardy, with D is telling. In Source C Disraeli argues there is no retreat from a rating franchise based on residence, yet Lowe is right to suggest duplicity here by pointing

Page 129: History A Mark Schemes for the Units January 2010 - The ...

F963/02 Mark Scheme January 2011

13

out that he must have realised personal rating and compounding were different and without the former the electorate would double. Lowe’s points, albeit from an opponent, are convincing. Those of Cranborne in E are more personal and less convincing. His view is of one dishonest opportunism and intrigue, and he points to one of Disraeli’s possible motives, personal ambition. That he writes this to a local party organiser suggests that his anger and sense of betrayal is deep. The counter interpretation that Disraeli held to general principles is, not surprisingly, found in his own speeches and correspondence, Sources A, B and C. From 1859 he had been in favour of some extension and politicians on both sides, from the 1850s, had moved in favour of change, not least Gladstone. Disraeli’s progress is marked by careful reference to principle and he never became a democrat (‘the right to vote’ in A) or shared Bright’s view of universal household suffrage. He believed that the concession on compounding in C would be ineffective as they would fail to register. Thus there was no retreat from the rating and residential principle. In C he refers only to the ‘spirit’ of Hodgkinson’s amendment, not its actual implementation. This was why so many were entranced by his rhetoric and parliamentary performances in 1866/7. He still felt, as in A, that the vote would remain a privilege allowed if ‘virtue, intelligence, industry and integrity’ were in evidence. Candidates may be aware of the Edinburgh speech in late 1867 where he was to expound, in grander terms, on a consistent and noble strategy to include the working man. A third interpretation is a general belief in extending the vote when circumstances allowed (Source A) but like any politician in a minority he would have to be flexible to attract votes. He had to woo liberal, radical and conservative. To maintain absolute consistency was impossible. Lowe in D recognises Disraeli as an ‘able tactician’ and the rhetoric in Disraeli’s own speeches could be used to exemplify this. Cranborne too recognises his ‘mastery over the movements in his own party’. His ambition was to restore it and to secure the succession to Derby, as Cranborne bitterly recognised. The three public speeches (A, B and D) all suggest a belief in suffrage extension and all three provide evidence of Disraeli’s tactical skill and flexibility. The more private sources (C and E) are divided; Cranborne convinced of his naked opportunism, Disraeli himself juxtaposing principle and good timing, ‘the critical moment’ in C.

Page 130: History A Mark Schemes for the Units January 2010 - The ...

F963/02 Mark Scheme January 2011

14

3 (a) Study Sources A and D Compare these Sources as evidence for attitudes to State intervention in dealing with poverty. [30] No set answer is expected, but candidates need to compare the contents, evaluating such matters as authorship, dating, utility and reliability, so using the Sources ‘as evidence for …’. The headings and attributions should aid evaluation and reference to both is expected in a good answer. The sources agree on the ideology or principle of State intervention. Source A argues that ‘laissez-faire’ is ‘not the cure’ and ‘old Liberalism’ is castigated. Source D emphasises the duty of the State to the working man. Both stress that the State should only help the poor rather than control their lives. In Source A the key word is ‘assistance’ and Source D says the State should support the ‘able-bodied man’ to look after himself. Indeed, the inability of individuals to fend for themselves is highlighted in both. ‘Self-reliance (was) not powerful enough’, according to Source A, and in Source D ‘haggling in the market’ is considered inadequate. The means and power of the State is accepted by both: Source A claims the State to be ‘not incompetent for the work of social reform’ and Source D implies the same in references to ‘the State to ensure economic conditions’. Further, State intervention is considered a positive force for the freedom of the individual. Source A argues intervention ‘may extend the bounds of liberty’ and Source D presses the ‘right to work’ and ‘the right to a ‘living wage’’. If there is a difference it is that Source A is more general in terms of conditions and remedies in contrast to Source D which identifies specific problems of food, housing, clothing and wages. D identifies the right to work and to a living wage. Both also nod to traditional liberal ideas of self-reliance and hard work. In evaluating the sources candidates might suggest the similarities are unsurprising given that both were written by new Liberals. Further commentary on the authorship is possible. Written by a politician, Source A seems to place emphasis on the wellbeing of the individual whereas Source D, written by an academic, focuses on the broader concerns of society as well as the individual. Candidates may consider this consistent with their personal stance and interests. The dates of publication are important. Written before the Liberals came to power Source A is concerned to articulate and clarify the principles of new Liberalism, hence the denunciation of old Liberalism. Its purpose is more obviously political to convince people of the soundness of State intervention by reassuring the reader that freedom will not be lost. Source D has to be seen against the context of five years of Liberal government and many reforms. It is less concerned with presenting the case for new Liberalism as highlighting the priorities of the time, notably wages (candidates could link to the debate on a minimum wage) and the notion of the reciprocal duties of State and worker (the Insurance Act of 1911 might be mentioned). The reference to ‘the rights of person and property’ might be an allusion to the struggle with the House of Lords since 1909 culminating in the Parliament Act of 1911.

Page 131: History A Mark Schemes for the Units January 2010 - The ...

F963/02 Mark Scheme January 2011

15

3 (b) Study all the Sources Use your own knowledge to assess how far the Sources support the interpretation that the main reason for tackling poverty was a moral obligation to help the poor. [70]

Successful answers will need to make use of all four Sources, testing them against contextual knowledge and evaluating their strengths and weaknesses, any limitations as evidence. A range of issues may be addressed in focusing upon the terms of the question but no set conclusion is expected. Arguably only Source E is unequivocal in supporting the interpretation although all sources attest, in varying degrees, to a widely held view that society had a moral obligation to help the poor. In addition, political, economic, social and ideological motives explain why it was thought necessary to tackle poverty. Archbishop Lang articulates the feelings of those who were uncomfortable in a society racked by gross inequality: ‘poverty in all our cities is appalling’ and he is clear that inequality ‘ought not to be’. In part this is because of the ‘increasing prosperity’ of the period, denied the poor, but more likely, because he is stung by the denial of ‘every sign of the beauty of God’s earth’ to the poor. Candidates will rightly argue this is unsurprising in a man of the Church with strong religious convictions. Some may suggest he verges on the sentimental in his comments on the ‘great multitude’ whom he has ‘learnt to revere’. In both respects any evaluation of the source might conclude that the evidence of Source E is not entirely reliable. On the other hand the author had first-hand experience of the slums of three cities and he is described as ‘a strong advocate of ‘enlightened capitalism’’ in the introduction so it could be argued his remarks carry weight. Each of the other sources can be used to support Source E in so far as they denounce the scale and depth of poverty. Perhaps Source C is as forceful as any scorning Elgar’s notion of a ‘’Land of Hope and Glory’’ and suggesting that the poor were a ‘discredit’. This may be regarded by candidates as expected from a Commission that had spent 4 years investigating poverty and compiled 50 volumes of information cataloguing the horrors of poverty. Source A admits that it is impossible to tolerate poverty any longer. Like Source C the view expressed in Source A is based on evidence stating that ‘the facts of poverty are now known’ probably referring to the research of Booth and Rowntree. Source D seems to accept that poverty should be tackled as a moral obligation in so far as ‘the rights’ of the workman should be acknowledged and that ‘society owes him the means of maintaining a civilised standard of life’. This may be regarded as consistent with the whole tenor of the passage which places stress on the contract between State and individual. Many may dismiss Source B as not offering anything in support of the interpretation although some may refer to the prospect of ‘new social systems’ making ‘England a better place for the poor’. Churchill’s strong support for the reforms of the period might be mentioned to confirm the sincerity of his moral obligation to the poor. However, many may set Churchill’s remark against the comment that follows to argue that his motive in tackling poverty was political hoping that improvements in the lot of the poor would lead to ‘the country’ giving ‘solid support to the government’. Indeed, elsewhere in Source B the timing and type of reform is regarded in political terms calculating that ‘the miseries of this winter’ would secure support from the poor and expenditure on ‘social systems’ would find favour with the Lords. Churchill’s concerns seem to be as much to do with matching Germany – for economic and imperial reasons? – as any sense of moral obligation to the poor. It might be argued that Churchill is alert to the conservative instincts of Asquith to whom he is writing, at a time when he has just become Prime Minister, and that he is trying to win the latter’s support by presenting reform of poverty in political terms.

Page 132: History A Mark Schemes for the Units January 2010 - The ...

F963/02 Mark Scheme January 2011

16

Certainly the second sentence can be interpreted as a fawning attempt to flatter Asquith. Some candidates might link comments on Source B with Source A which, it could be argued, hints at a political agenda. Comments about the limitations of ‘laissez-faire’ and the attack on ‘old Liberalism’ might be regarded as an indication of how the issue of poverty was a political football used by the likes of Samuel to promote new Liberalism. Some may see Sources C and D as placing emphasis on economic motives for tackling poverty. After all, in Source C the poor are decried as ‘useless and costly inefficients’ who have to be transformed into productive members of society. Source D implies, softly perhaps, the economic imperative in highlighting the importance of ‘useful labour’ and ‘the duty of working hard for his family’. Both Source C and D seem concerned to address poverty in the interests of law and order. The former regards the poor as ‘a peril to the whole community’ and the importance of converting the poor into ‘respectable members of the community’ is stressed whilst the latter is concerned to create ‘conditions of a good social order and civic efficiency’. The clinical language of both sources may be explained as a feature of an official report and an academic appraisal both of which would have been objective in intention. On the other hand candidates may point out that the Majority report favoured retaining the Poor Law (in contrast to the Minority report), with the reference to poverty being ‘possibly from their own failure and faults’ denying, perhaps, any sense of moral obligation. Some may regard Sources A and D as concerned to tackle poverty as a matter of libertarian principle. Source A emphasises that ‘Liberty is of supreme importance’ and that extending ‘the bounds of liberty’ appears to be a major objective. Equality of rights is stressed in Source D too. However, in both cases it could be argued that the promotion of liberty and individual rights is synonymous with a moral obligation to help the poor; if liberty was a right, tackling poverty, which stifled freedom, was a moral imperative. Candidates might dismiss the tenor of Sources A and D as the musing of dry theorists but they were, nonetheless, typical of new liberalism.

Page 133: History A Mark Schemes for the Units January 2010 - The ...

F963/02 Mark Scheme January 2011

17

4 (a) Study Sources D and E. Compare these Sources as evidence for views about Churchill’s economic policy in 1925. [30] No set answer is expected, but candidates need to compare the contents, evaluating such matters as authorship, dating, utility and reliability, so using the Source ‘as evidence for…..’ The Headings and attributions should aid evaluation and reference to both is expected in a good answer. Sources D and E take very different views of the policy of returning the £ to the Gold Standard in 1925. The £ had been taken ‘off Gold’ during the First World War and consequently the exchange rate had fallen. The decision to get the £ ‘back to normal’ and to link it to gold reserves had been urged by the financial sector, eager to give complete confidence to users of sterling world wide and to boost income from ‘invisible exports’ which were of huge importance to the economy. However it did mean that revaluation brought an increase in the exchange rate of the £ against the $ and therefore made exports more expensive. One of Britain’s major exports was coal and the decision is said to have had a major impact on the coal industry and to have been a cause of the General Strike, unemployment and falling purchasing power and therefore internal demand. Content: D argues that it brought about ‘the present troubles’ of British export industries; E in direct comparison argues that it would be to the advantage of British industries. For E it is a heroic move which will lessen the cost of living (by making imports cheaper). For D it is a dangerous and unnecessary decision, rather than being momentous and heroic and will reduce spending power by reducing everybody’s wages by 2s (presumably because export industries will sell less and cut wages), E congratulates Churchill; D sees his policy having a negative effect on exports. Provenance: D is from an economic theorist and E is from the professional world of finance. D has no real vested interest in policy whereas the Bankers’ Association had everything to gain from greater confidence in sterling even at the cost of higher prices for the manufacturing sector. Both are contemporary views but whereas E is a report of the head of an association congratulating the government on a policy they welcomed; D is a controversial study from someone outside manufacturing and finance. Some may know that Keynes was famous for his criticisms of Versailles and for his later advocacy of deficit finance so will be approaching this from a more radical perspective than the ‘establishment’ opinion of E but knowledge of Keynes is not to be expected and marks should not be held back if there is not a focus on this. Judgement. Some may say that Keynes (D) is far more justified – coal exports did fall and there was discontent; unemployment stayed high in the 1920s; Gold had to be abandoned in 1931. However, in 1925 there was a case for E’s view of the Return – Italy too revalued its currency; there was a strong fear of inflation such as occurred in Germany in 1923 without a firm gold basis for the currency. There was a considerable shift in the economy away from the value of manufactured exports to a more modern economy based on investment, insurance, and financial services which needed a secure currency. Churchill was at one with most of the experts in 1925 and Keynes was not the ‘miracle worker’ that he became to post-war British politicians. Do not look for a particular point of view, but reward attempts to assess the relative value of the Sources.

Page 134: History A Mark Schemes for the Units January 2010 - The ...

F963/02 Mark Scheme January 2011

18

4 (b) Study all the Sources. Use your own knowledge to assess how far the Sources support the interpretation that Churchill’s domestic policies and attitudes in the 1920s were disastrous for his reputation. [70] Successful answers will need to make use of all five Sources, testing them against contextual evidence and evaluating their strengths and weaknesses, any limitations as evidence. A range of issues may be addressed in focusing upon the terms of the question but no set conclusion is expected. Sources that suggest his policies were disastrous for his reputation in the 1920s are A , B, C and D. Source E is full of praise and there are elements of A and B which stress that he retained his reputation due to his qualities of confidence, patriotism, guts, brilliance and his effectiveness as an opponent of the left. The debate here is whether Churchill was a discredited politician by the end of the 1920s. One view might be that his rabid anti-communism had made him a target for the left and almost a caricature of a Tory ( as suggested by A and B) Some claim that his tenure of the Exchequer had resulted in unwise economies that had alienated the armed services (C) and many in the Conservative party. His handling of the General Strike has come in for considerable criticism, both in terms of his own financial policy helping to cause it (D) and his misunderstanding of the challenge to the Constitution being the issue (B). For some he came close to Fascism (A), for others he took a responsible and statesmanlike path, accepting sound advice (E). Was he out of office after 1929 because his reputation was already tarnished? The most critical sources are from the left, B, and from the intellectual centre, D. These are hardly impartial. Shinwell was a Labour supporter on the left of his party; Keynes was a radical academic. Both write in quite a partisan way. If Churchill was a hate figure at Labour conferences, then this could be seen as entirely natural in politics. Shinwell shows his admiration, nonetheless for Churchill’s brilliance. However, evidence could be presented to support Labour’s dislike, for instance by Churchill’s over reaction to the General Strike and his branding all labour movements as Communist. Contrarily, Churchill’s attempts at conciliation and his concern for social conditions at home might be presented to counter this. Some cross reference might be made with A which sees Churchill as being close to Fascism. He did admire elements of Fascist Italy and there are certainly elements of his hostility to the left that could be seen as being unbalanced and detrimental to his reputation. Low as a cartoonist was a close observer of the political scene; but a cartoonist is by nature critical. Note that both he and Shinwell offer some balance in their views. D does not - for Keynes Churchill is pursuing a disastrous economic policy which will, by implication, harm his reputation as export industries, purchasing power and exports suffer. However, critics might point out that Keynes was not actually describing effects but predicting them. British export industries had been in long term decline and faced foreign competition. The Return to Gold was not the cause of the problem and Churchill had to balance making exports dearer with the supposed benefit to the more thriving sectors of the economy. This is quite an opinionated view and needs to be considered as an economic opinion. C too is opinionated, written by someone with as much of a special interest as E. Beatty might have expected special treatment given Churchill’s previous work with the admiralty, but Churchill was his own man. Should the cuts actually be admired? Would millions of pounds spent on 1920s battleships have actually been of much use after 1939? (especially given the rise of air power) . Why did Beatty want big naval expenditure? The letter expresses a candid view but it is a

Page 135: History A Mark Schemes for the Units January 2010 - The ...

F963/02 Mark Scheme January 2011

19

‘heat of the moment’ response. Candidates may well agree with him and point to the damaging effects of the 10 Year Rule and military cuts. E is more admiring, but the bankers have got just what they wanted. The origin of the source could be discussed – this is from a particular sector of the economy. Manufacturing industry might have taken a different view. However, Churchill’s financial policies as a whole have not been overly criticised by historians, and though without expertise when appointed, he did hold his position and showed some flair as Chancellor. Keynes seems to have won the argument about Gold so many candidates will take issue with E and see it as selfish and overpaid bankers getting their own way.

Page 136: History A Mark Schemes for the Units January 2010 - The ...

Oxford Cambridge and RSA Examinations is a Company Limited by Guarantee Registered in England Registered Office; 1 Hills Road, Cambridge, CB1 2EU Registered Company Number: 3484466 OCR is an exempt Charity OCR (Oxford Cambridge and RSA Examinations) Head office Telephone: 01223 552552 Facsimile: 01223 552553 © OCR 2011

OCR (Oxford Cambridge and RSA Examinations) 1 Hills Road Cambridge CB1 2EU OCR Customer Contact Centre 14 – 19 Qualifications (General) Telephone: 01223 553998 Facsimile: 01223 552627 Email: [email protected] www.ocr.org.uk For staff training purposes and as part of our quality assurance programme your call may be recorded or monitored

Page 137: History A Mark Schemes for the Units January 2010 - The ...

Oxford Cambridge and RSA Examinations

GCE

History A

Advanced Subsidiary GCE

Unit F963/02: Option B Modern 1815-1945

Mark Scheme for January 2013

Page 138: History A Mark Schemes for the Units January 2010 - The ...

OCR (Oxford Cambridge and RSA) is a leading UK awarding body, providing a wide range of qualifications to meet the needs of candidates of all ages and abilities. OCR qualifications include AS/A Levels, Diplomas, GCSEs, Cambridge Nationals, Cambridge Technicals, Functional Skills, Key Skills, Entry Level qualifications, NVQs and vocational qualifications in areas such as IT, business, languages, teaching/training, administration and secretarial skills. It is also responsible for developing new specifications to meet national requirements and the needs of students and teachers. OCR is a not-for-profit organisation; any surplus made is invested back into the establishment to help towards the development of qualifications and support, which keep pace with the changing needs of today’s society. This mark scheme is published as an aid to teachers and students, to indicate the requirements of the examination. It shows the basis on which marks were awarded by examiners. It does not indicate the details of the discussions which took place at an examiners’ meeting before marking commenced. All examiners are instructed that alternative correct answers and unexpected approaches in candidates’ scripts must be given marks that fairly reflect the relevant knowledge and skills demonstrated. Mark schemes should be read in conjunction with the published question papers and the report on the examination. OCR will not enter into any discussion or correspondence in connection with this mark scheme. © OCR 2013

Page 139: History A Mark Schemes for the Units January 2010 - The ...

F963/02 Mark Scheme January 2013

1

Subject-specific Marking Instructions Question (a) Maximum mark 30

A01a and b AO2a

1 13–14 15–16

2 11–12 13–14

3 9–10 10–12

4 7–8 8–9

5 5–6 6–7

6 3–4 3–5

7 0–2 0–2

Notes related to Part A: (i) Allocate marks to the most appropriate level for each AO (ii) If several marks are available in a box, work from the top mark down until the best fit has been found (iii) Many answers will not be at the same level for each AO

Page 140: History A Mark Schemes for the Units January 2010 - The ...

F963/02 Mark Scheme January 2013

2

Marking Grid for Question (a)

A0s A01a and b A02a Total for each question = 30

Recall, select and deploy historical knowledge appropriately, and communicate knowledge and understanding of history in a clear and effective manner. Demonstrate understanding of the past through explanation, analysis and arriving at substantiated judgements of: - key concepts such as causation, consequence,

continuity, change and significance within an historical context;

- the relationships between key features and characteristics of the periods studied.

As part of an historical enquiry, analyse and evaluate a range of appropriate source material with discrimination.

Level 1 · Consistent and developed comparison of the key issue with a balanced and well-supported judgement. There will be little or no unevenness.

· Focused use of a range of relevant historical concepts and context to address the key issue.

· The answer is clearly structured and organised. Communicates coherently, accurately and effectively.

13–14

· Focused comparative analysis. Controlled and discriminating evaluation of content and provenance, whether integrated or treated separately.

· Evaluates using a range of relevant provenance points in relation to the sources and question. There is a thorough but not necessarily exhaustive exploration of these.

15–16

Level 2 · Largely comparative evaluation of the key issue with a balanced and supported judgement. There may be a little unevenness in parts.

· Focused use of some relevant historical context with a good conceptual understanding to address the key issue.

· The answer is well structured and organised. Communicates clearly.

11–12

· Relevant comparative analysis of content and evaluation of provenance but there may be some unevenness in coverage or control.

· Source evaluation is reasonably full and appropriate but lacks completeness on the issues raised by the sources in the light of the question.

13–14

Page 141: History A Mark Schemes for the Units January 2010 - The ...

F963/02 Mark Scheme January 2013

3

A0s A01a and b A02a Level 3 · Some comparison linked to the key issue. Is aware of

some similarity and/or difference. Judgements may be limited and/or inconsistent with the analysis made.

· Some use of relevant historical concepts and contexts but uneven understanding. Inconsistent focus on the key issue.

· The answer has some structure and organisation but there is also some description. Communication may be clear but may not be consistent.

9–10

· Provides a comparison but there is unevenness, confining the comparison to the second half of the answer or simply to a concluding paragraph. Either the focus is on content or provenance, rarely both.

· Source evaluation is partial and it is likely that the provenance itself is not compared, may be undeveloped or merely commented on discretely.

10–12 Level 4 · Some general comparison but undeveloped with some

assertion, description and/or narrative. Judgement is unlikely, unconvincing or asserted.

· A general sense of historical concepts and context but understanding is partial or limited, with some tangential and/or irrelevant evidence.

· Structure may be rather disorganised with some unclear sections. Communication is satisfactory but with some inaccuracy of expression.

7–8

· Attempts a comparison but most of the comment is sequential. Imparts content or provenance rather than using it.

· Comparative comments are few or only partially developed, often asserted and/or ‘stock’ in approach.

8–9 Level 5 · Limited comparison with few links to the key issue.

Imparts generalised comment and/or a weak understanding of the key points. The answer lacks judgement or makes a basic assertion.

· Basic, often inaccurate or irrelevant historical context and conceptual understanding.

· Structure lacks organisation with weak or basic communication.

5–6

· Identifies some comparative points but is very sequential and perhaps implicit

· Comment on the sources is basic, general, undeveloped or juxtaposed, often through poorly understood quotation.

6–7

Page 142: History A Mark Schemes for the Units January 2010 - The ...

F963/02 Mark Scheme January 2013

4

A0s A01a and b A02a Level 6 · Comparison is minimal and basic with very limited links

to the key issue. Mainly paraphrase and description with very limited understanding. There is no judgement.

· Irrelevant and inaccurate concepts and context. · Has little organisation or structure with very weak

communication. 3–4

· Little attempt to compare. Weak commentary on one or two undeveloped points, with basic paraphrase. Sequencing is characteristic.

· Comments on individual sources are generalised and confused.

3–5

Level 7 · Fragmentary, descriptive, incomplete and with few or no links to the key issue. There is little or no understanding. Much irrelevance.

· Weak or non existent context with no conceptual understanding.

· No structure with extremely weak communication. 0–2

· No attempt to compare either content or provenance with fragmentary, brief or inaccurate comment.

· Makes no attempt to use any aspects of the sources.

0–2 Question (b) Maximum mark 70

A01a and b AO2a and b

1 20–22 42–48

2 17–19 35–41

3 13–16 28–34

4 9–12 21–27

5 6–8 14–20

6 3–5 7–13

7 0–2 0–6

Notes related to Part B: (iv) Allocate marks to the most appropriate level for each AO (v) If several marks are available in a box, work from the top mark down until the best fit has been found (vi) Many answers will not be at the same level for each AO

Page 143: History A Mark Schemes for the Units January 2010 - The ...

F963/02 Mark Scheme January 2013

5

AOs A01a and b Ao2a and b

Total mark for the question = 70

Recall, select and deploy historical knowledge appropriately, and communicate knowledge and understanding of history in a clear and effective manner. Demonstrate understanding of the past through explanation, analysis and arriving at substantiated judgements of: - key concepts such as causation, consequence, continuity, change and significance within an historical context; - the relationships between key features and characteristics of the periods studied.

As part of an historical enquiry, analyse and evaluate a range of appropriate source material with discrimination. Analyse and evaluate, in relation to the historical context, how aspects of the past have been interpreted and represented in different ways.

Level 1 · Convincing analysis and argument with developed explanation leading to careful, supported and persuasive judgement arising from a consideration of both content and provenance. There may be a little unevenness at the bottom of the level.

· Sharply focused use and control of a range of reliable evidence to confirm, qualify, extend or question the sources.

· Coherent organised structure. Accurate and effective communication.

20–22

· A carefully grouped and comparative evaluation of all the sources with effective levels of discrimination sharply focused on the interpretation.

· Analyses and evaluates the strengths, limitations and utility of the sources in relation to the interpretation. Uses and cross references points in individual or grouped sources to support or refute an interpretation.

· Integrates sources with contextual knowledge in analysis and evaluation and is convincing in most respects. Has synthesis within the argument through most of the answer.

42–48 Level 2 · Good attempt at focused analysis, argument and

explanation leading to a supported judgement that is based on the use of most of the content and provenance.

· A focused use of relevant evidence to put the sources into context.

· Mostly coherent structure and organisation if uneven in parts. Good communication.

17–19

· Grouped analysis and use of most of the sources with good levels of discrimination and a reasonable focus on the interpretation.

· Analyses and evaluates some of the strengths and limitations of the sources in relation to the interpretation. May focus more on individual sources within a grouping, so cross referencing may be less frequent.

· Some, perhaps less balanced, integration of sources and contextual knowledge to analyse and evaluate the interpretation. Synthesis of the skills may be less developed. The analysis and evaluation is reasonably convincing.

35–41

Page 144: History A Mark Schemes for the Units January 2010 - The ...

F963/02 Mark Scheme January 2013

6

AOs A01a and b Ao2a and b Level 3 · Mainly sound analysis, argument and explanation, but there

may be some description and unevenness. Judgement may be incomplete or inconsistent with the analysis of content and provenance.

· Some relevant evidence but less effectively used and may not be extensive.

· Reasonably coherent structure and organisation but uneven. Reasonable communication.

13–16

· Some grouping although not sustained or developed. Sources are mainly approached discretely with limited cross reference. Their use is less developed and may, in parts, lose focus on the interpretation. There may be some description of content and provenance.

· Is aware of some of the limitations of the sources, individually or as a group, but mostly uses them for reference and to illustrate an argument rather than analysing and evaluating them as evidence. There is little cross referencing.

· There may be unevenness in using knowledge in relation to the sources. Synthesis may be patchy or bolted on. Analysis and evaluation are only partially convincing.

28–34 Level 4 · Attempts some analysis, argument and explanation but

underdeveloped and not always linked to the question. There will be more assertion, description and narrative. Judgements are less substantiated and much less convincing.

· Some relevant evidence is deployed, but evidence will vary in accuracy, relevance and extent. It may be generalised or tangential.

· Structure is less organised, communication less clear and some inaccuracies of expression.

9–12

· Sources are discussed discretely and largely sequentially, perhaps within very basic groups. Loses focus on the interpretation. The sources are frequently described.

· May mention some limitations of individual sources but largely uses them for reference and illustration. Cross referencing is unlikely.

· An imbalance and lack of integration between sources and knowledge often with discrete sections. There is little synthesis. Analysis and explanation may be muddled and unconvincing in part.

21–27 Level 5 · Little argument or explanation, inaccurate understanding of

the issues and concepts. The answer lacks judgement. · Limited use of relevant evidence or context which is largely

inaccurate or irrelevant. · Structure is disorganised, communication basic and the

sense not always clear.

5–8

· A limited attempt to use the sources or discriminate between them. The approach is very sequential and referential, with much description. Points are undeveloped.

· There is little attempt to analyse, explain or use the sources in relation to the question. Comment may be general.

· There is a marked imbalance with no synthesis. Analysis and explanation are rare and comments are unconvincing.

14–20

Page 145: History A Mark Schemes for the Units January 2010 - The ...

F963/02 Mark Scheme January 2013

7

AOs A01a and b Ao2a and b Level 6 · There is very little explanation or understanding. Largely

assertion, description and narrative with no judgement. Extremely limited relevance to the question.

· Evidence is basic, generalised, patchy, inaccurate or irrelevant.

· Little organisation or structure with poor communication. 3–4

· Very weak and partial use of the sources for the question. No focus on interpretation.

· A very weak, general and paraphrased use of source content.

· No synthesis or balance. Comments are entirely unconvincing.

7–13 Level 7 · No argument or explanation. Fragmentary and descriptive

with no relevance to the question. · No understanding underpins what little use is made of

evidence or context. · Disorganised and partial with weak communication and

expression. 0–2

· Little application of the sources to the question with inaccuracies and irrelevant comment. Fragmentary and heavily descriptive.

· No attempt to use any aspect of the sources appropriately. · No contextual knowledge, synthesis or balance. There is no

attempt to convince. 0–6

Page 146: History A Mark Schemes for the Units January 2010 - The ...

F963/02 Mark Scheme January 2013

8

Question Answer Marks Guidance 1 (a) The context is immediate post war distress, specifically the winter of 1816/17 which saw the

two Spa Fields Meetings, the second of which degenerated into violence and alleged revolutionary plotting. This brought into focus the issue of radical methods. Both sources agree that the focus should be parliamentary reform. They are similar in agreeing on moderate methods – ‘the superiority of mental over physical force’ by Hunt in Source A and the stress on ‘moderate methods’ by Cobbett in Source B. Both stress petitions as a primary method and both put great stress on Open Meetings. At these there could be a ‘calling aloud for timely reform’ in Source A, a trust in individual effort and open meetings in Source B. However there are important differences. Hunt in Source A argues that physical force could be resorted to if moderate methods failed. Cobbett does not. Cobbett, unlike Hunt disapproves of Political Clubs, plots and Corresponding Societies, considering them all ineffective. They also differ on the use of language. Cobbett in Source B stresses that language must be moderately used in both petitions and meetings. However Hunt in Source A indulges in rousing rhetoric that implies the right of rebellion. Thus he hints and warns about an ultimate recourse to violence and physical force. Candidates may note that his reference to the ‘fatal day’ was greeted by a ‘hurrah’ from the crowd. He himself will not be backward in coming forward. As regards provenance the dates and geography of the sources are important. Hunt is addressing a radical part of artisan London and he is aware that he must not go too far whilst using the mass platform to enthuse (‘the fatal day’). The meeting passed off peacefully. The second Spa Fields meeting on the 2nd December was hijacked by radical Spenceans and ended in a drunken riot. This led the government to clamp down in the Gagging Acts of 1817. This provides the different context for Cobbett, who is writing, in contrast, to a rural area advising more moderate and legal methods, such as petitions and open meetings. His purpose is to calm things and avoid giving the government any excuse to further move against the radicals, hence his more moderate tone. In judgement candidates may well see Hunt’s evidence in Source A as better for explaining and demonstrating the rhetoric of the mass platform and his particular form of crowd control. Cobbett in B is better for general tactics following the debacle of the 2nd Spa Fields Meeting, although his dislike of other radicals like Hunt may also colour his evidence.

30 Focus: Comparison of two Sources No set answer is expected, but candidates need to compare the contents, evaluating such matters as authorship, dating, utility and reliability, so using the Source ‘as evidence for…..’ The Headings and attributions should aid evaluation and reference to both is expected in a good answer.

Page 147: History A Mark Schemes for the Units January 2010 - The ...

F963/02 Mark Scheme January 2013

9

Question Answer Marks Guidance (b) Various views on the reasons for radical failure in this period are possible. The suggestion is

that divisions within the radicals, both over personality and policy clashes and disagreement over tactics, rendered them weak and prone to failure, Sources B and D. The main alternative is that they were in fact relatively united and strong, both in purpose and in tactical ingenuity, and it was government strength and repression that ensured failure (Sources A, B, C and E). All the sources are from radicals, including the cartoon, and provide a variety of internal perspectives and on the problems that they faced in a society that restricted and denied them access to the franchise and parliament. The view that the radicals failed through their own divisions can be found in Sources B and D. Cobbett in Source B is contemptuous of the failure of Spa Fields and firmly holds to moderate methods. He dislikes radical Spenceans like Watson, and Thistlewood, whom he blames for Spa Fields and he would also include Hunt in this. However he also condemns Cartwright’s Hampden Clubs, in contrast to Bamford in Source E. These were the major post Spa Fields initiative from the radicals to organise both locally and then move to a central organisation. Yet Cobbett considers them ineffective. However his evidence is tainted by his dislike of Hunt and Cartwright and by his own preference for local meetings, issues and his Political Register. Nonetheless its dislike of other radicals and their methods is corroborated by Hazlitt in Source D. Writing in 1819 in the context of Peterloo and from a detached perspective (he was a literary radical who was also a sought after journalist and writer) he is very explicit in his condemnation of radical divisions. He blames the personalities and leaders rather than their policies. He argues they all hated each other, took individual lines and devoted their respective organisations to undermining each other. Knowledge may bear this out. This was an age of self dependent politicians who found it difficult to create a central organisation, hence Carlyle’s long contest with authority and Hunt’s efforts to hog the limelight (as Source A and Peterloo illustrate). Place saw most other radicals as fools to be manipulated. Bamford became ever more moderate and was prone to self esteem, hence his grand autobiographical descriptions in ‘Passages in the Life of a Radical’, (Source E is an example in point). Carlyle despised organisation and consulting others. As Source D suggests the radicals were prima donnas demanding and rarely getting respect from each other. They liked slogans and political theatre (the platform and grand entrances to towns and cities). Whilst this and their demagoguery could rouse numbers they were not always good leaders. As Hazlitt comments in Source D their parties and committees were often aimed more at each other than the government and authority. In Source A Hunt deals with emotion rather than strategy.

70 Focus: Judgement in context, based on a set of Sources and own knowledge. Successful answers will need to make use of all five Sources, testing them against contextual evidence and evaluating their strengths and weaknesses, any limitations as evidence. A range of issues may be addressed in focusing upon the terms of the question but no set conclusion is expected. The sources can be read/analysed in different ways and as part of their judgement candidates will need to appreciate this.

Page 148: History A Mark Schemes for the Units January 2010 - The ...

F963/02 Mark Scheme January 2013

10

Question Answer Marks Guidance The alternative view is that they were strong but failed because of government strength. This interpretation can be found in Sources A, B, C and E. Hunt in A is clearly a master of crowd manipulation and, like most of the leaders, good at staying just this side of the law in his rhetoric. His management of the 1st Spa Fields meeting is effective. Similarly Cobbett in B is careful not to provoke and provides evidence of considerable activity. However the main evidence of radical purpose and unity is Bamford in Source E. He comments on increased membership of the Hampden Clubs, good funding, the use of chapels for regular meetings and also signs of a national organisation emerging, a national Meeting or Convention in London. The aims (parliamentary reform) are clear – all over the age of 18 paying taxes are to vote annually and each block of population is to receive a representative with no corrupt placemen allowed. There appears to be unity at the base and in the localities, whatever the divisions at the top. The stalwarts were local trade unionists, booksellers and local speakers. This would suggest that the Cartoon in Source C is accurate and that, as Cobbett realised in Source B, the Government is repressive and proactive. The government should be given no excuse to act. The comment in C comes from March 1817, after the government clamp down following Spa Fields. The Gagging Acts had suspended habeas corpus, there were press restrictions and the law was being used to arrest and imprison radicals. A Committee of secrecy had been set up to collate evidence of widespread conspiracy and revolution, Lord Eldon’s large sack. Church and patronage were used to counter the radicals and the use of troops was more frequent (culminating in Peterloo). Spies were used to infiltrate and incriminate the radicals. The radicals were a ‘weeping’ and marginalised group. Although obviously exaggerated Source C points to government strength rather than radical weakness. The sources can be read/analysed in different ways and as part of their judgement candidates will need to appreciate this. As a set they are all radical, revealing elements both of strength and division.

Page 149: History A Mark Schemes for the Units January 2010 - The ...

F963/02 Mark Scheme January 2013

11

Question Answer Marks Guidance 2 (a) The context of both sources are Gladstone’s campaigns in late 1879 and 1880, the first

Midlothian campaign, dating from November/December 1879 when Gladstone accepted the nomination to be Liberal candidate for the normally solid Conservative constituency of Midlothian, the second being the result of the General Election that ensued in 1880. Gladstone’s focus in both was a condemnation of Disraeli’s foreign and imperial policy. Both sources are similar in agreeing on the power of Gladstone’s mass oratory, using very similar language to describe its impact. The Times in B refers to a ‘passionate temper’; Watson in C to the ‘launching upon a sea of passion’. Both have occasion to question Gladstone’s reasoning, albeit in rather different ways. The Times in B considers that such oratory lacks a sense of calm judgement and ‘considered argument’; Watson in C, after the event, comments on a sense of ‘ecstasy beyond reason’. However they differ on the impact of the campaigns. The Times tends to dismiss it as temporary and shallow, commanding only brief popular enthusiasm from the electorate, an electorate which it thinks is still dominated by the cautious and intelligent voter, not yet the mass democracy of post 1884. The emotion it thinks Gladstone wrongly whips up will evaporate quickly, having little (probably deleterious) impact on the thoughtful voting classes it thankfully thinks are still in a majority. The Times is confident that most will not surrender to Gladstone’s public display of ‘passionate temper’. In contrast Watson in Source C comments from the vantage point of a voter who did succumb and who appears both intelligent and perceptive in his comments. The Times in B misses the point that C makes about Gladstone’s ability to empower people. It considers Gladstone’s impact to be temporary –‘for a while command’. Watson in C makes the point that Gladstone gave his audience a sense of power, putting decisions into the hands of people and nation. It was their ‘verdict’. They would make the decisions. Gladstone, according to Watson, enabled people to understand complex issues as matters of simple and great principle, ‘moral commands’. This effect is missed completely by the Times. As regards provenance the key lies in the respective positions of the two sources. The Times in B was the major ‘establishment’ newspaper commenting at the beginning of the 1st Midlothian campaign when the probable result of an ex Liberal leader challenging in a safe Conservative seat may well have appeared foolish and unlikely to succeed. It clearly disapproves of Gladstone’s campaigning tactics. Such campaigns were relatively new and, if Bulgaria was anything to go by, likely to either fizzle out or work against Gladstone when events changed.

30 Focus: Comparison of two Sources. No set answer is expected, but candidates need to compare the contents, evaluating such matters as authorship, dating, utility and reliability, so using the Sources ‘as evidence for …’. The headings and attributions should aid evaluation and reference to both is expected in a good answer.

Page 150: History A Mark Schemes for the Units January 2010 - The ...

F963/02 Mark Scheme January 2013

12

Question Answer Marks Guidance The Times’ audience was a prosperous and educated Upper and Middle Class who followed politics through the word rather than the traditionally radical mass platform. They would, like the Times, disapprove of Gladstone’s policies and tactics. In contrast Watson attended the 1880 meeting as part of a later General Election campaign when Gladstone was clearly at the height of his powers following many such events. His purpose is different – to describe the personal impact of Gladstone on himself as an audience member and to analyse what he described as being ‘Gladstonised’. Although largely supportive of Gladstone’s overall stance, he is careful to assess the precise impact of Gladstone on large crowds. He was writing in a political magazine and assessing his immediate reactions. Candidates are likely to judge that both sources are useful in demonstrating how Gladstone could polarise opinion. Both provide useful evidence for a differing impact. The Times is effective evidence for establishment distaste of Gladstone’s popular campaigns, but Watson may be considered the better evidence as his account is from the perspective of Gladstone’s audience and contains some balance. It is more careful in its analysis of Gladstone’s play upon emotions and upon the popular reaction he was having. Contrary to the Times’ opinion, Gladstone won the election of 1880.

Page 151: History A Mark Schemes for the Units January 2010 - The ...

F963/02 Mark Scheme January 2013

13

Question Answer Marks Guidance (b) The sources support two possible interpretations of Gladstone’s campaigns. The view in

the question is that he was reluctantly drawn out of retirement by a simple sense of moral outrage at Disraeli’s irresponsibility over foreign and imperial issues (Bulgaria and the Eastern Question in 1876; Afghanistan and South Africa in 1879–80). The other view is that Gladstone as politically ambitious, using the campaigns as an excuse to stage a political comeback after the debacle of the 1874 election, reasserting his hold over Liberalism and sweeping aside his successors, the Whig Liberals Granville and Hartington, and potential future rivals, like the coming Joe Chamberlain. The sources represent a variety of views – Gladstone himself commenting in public and private both during and after the event; an intelligent observer of the campaigns themselves; an establishment view; and a modern historian. All the sources, except possibly E which suggests politics and ambition, are capable of supporting either interpretation, depending on how their information and provenance are assessed. The interpretation in the question, Gladstone returning out of a sense of duty and moral outrage to conduct a crusade against ‘Beaconsfieldism’, can be found in all the sources except E. As might be expected Gladstone in sources A and D is at pains, both privately (A) and publicly (D), to deny political ambition and stress instead his moral duty, responding to the pressures of both others and events. To Bright in A he cites five very cogent reasons why he is no longer ambitious. Nor does he think it realistic to challenge for the leadership. In particular he cites loyalty to Granville and Hartington and to the great hostility such ambition would produce (confirmed with reference to the views of the Times in B). Source D most clearly supports the view of a moral crusade. In his Reminiscences he stresses that he acted out of a moral sense of duty to the subject peoples of the Balkans. It was an act of public justice. However candidates could question the extent to which, by 1896, Gladstone was weaving a story about his career, one based on a series of moral missions, that of the 1870s being on Britain’s European and World role. Certainly in D he argues for a reluctantly adopted mission, saying he was slow to see Disraeli’s unconditional support for the Ottomans and was content to allow others (Forster) to take the lead in opposing him. The stress is on compulsion (from Bright?) and he explicitly states that he had no thought as to leadership. The Times in Source B doubts Gladstone’s political ability, commenting unfavourably on his campaigns. It is convinced he will not succeed. Its reference to ‘passionate temper’ would suggest moral outrage on policies abroad rather than political ambition. Knowledge might suggest that the Liberal leadership was reluctant to

70 Focus: Judgement in context, based on the set of Sources and own knowledge. Successful answers will need to make use of all five Sources, testing them against contextual knowledge and evaluating their strengths and weaknesses, any limitations as evidence. A range of issues may be addressed in focusing upon the terms of the question but no set conclusion is expected.

Page 152: History A Mark Schemes for the Units January 2010 - The ...

F963/02 Mark Scheme January 2013

14

Question Answer Marks Guidance support him, not just because he was a threat to them but because his opinions would make it more difficult to come to a favourable resolution of Britain’s foreign and imperial interests. Watson in C comments purely in terms of great principles, an appeal to the people, ‘moral commands’. Shannon in E could be considered unduly cynical in his view, his Gladstone’s ‘game’ quotation applying equally to a moral campaign as a political one. The alternative argument, for ambition is largely to be found in Sources A (reading between the lines), C (gaining votes), in part D and especially E. Although Gladstone in Source A, writing privately to his mentor Bright (who had raised the issue of his resuming the leadership), denies it, the fact that Bright mentions it meant that the Midlothian campaign was having a huge political impact. There was pressure from the radical wing for Gladstone to come out of retirement to challenge the Whig leadership over partial support for Disraeli’s policies abroad. This is supported in Source E where Chamberlain hopes a return will discomfit the Whigs. Gladstone had to write to Bright to deny it. As he later commented, in Source D, of 1876 and Bulgaria, he was less than impressed with the existing leadership’s reactions to Disraeli’s government. Source C supports the passion that Gladstone aroused but, despite reservations, Watson votes for him in 1880. That sort of response made it impossible for the Whig leadership of the Liberals or the Queen, to hold out against his resumption of the premiership. Candidates may well spot the last sentence of Source A, where Gladstone changes his modest tack and comments on the ‘wonderful enthusiasm’ in Scotland. He shares his ‘content’ with Bright on the ‘progress of popular opinion’ and its response to the Midlothian campaign. One reading of this source is that Gladstone is rousing opinion in a way that Bright approves, with the leadership in mind, but is careful to reign in Bright and not appear to be queering Granville and Hartington’s pitch as existing leaders. Commenting after the event, in Source D, Gladstone arguably continues to be ingenuous. On the one hand he comments on his re-emergence from 1876 (over the Eastern Question, Afghanistan and South Africa – the ‘main business of my life’), implying simply an innocent reaction to events. However candidates might comment on the penultimate sentence – ‘nevertheless it made me leader again whether I wanted it or not’. Was Gladstone commenting in a knowing manner, aware that his campaign was meant to deliver him the leadership, or is this simply a statement of innocent fact? Taken together the two Gladstonian sources, A and D, certainly show that leadership and ambition could be seen as the unwritten agenda of the campaigns, although the private/public aspects of both confirm that Gladstone was always careful on issues of ambition. Own knowledge might suggest that, at least on Bulgaria, Gladstone was reluctant to come forward. Public opinion

Page 153: History A Mark Schemes for the Units January 2010 - The ...

F963/02 Mark Scheme January 2013

15

Question Answer Marks Guidance led him rather than the other way around, but by 1879 he had taken charge. Watson in C and Shannon in E both confirms Gladstone’s ability to arouse emotion, set agendas and win elections, (in this case 1880). Watson in C, despite some reservations, votes for him and confesses to be ‘Gladstonised’. He and Gladstone consider the victory of 1880 to be that of the nation, not the Whig and Tory upper classes, as Gladstone may have intended. If it is Gladstone who wins, it will inevitably be he who returns to both leadership and premiership. Candidates could use their knowledge to show how Gladstone overcame even the Queen’s objections to his resumption of the government. Given the balance of Watson’s comments he may be considered effective evidence of Gladstone’s political tenacity. Shannon in E, from a more modern and cynical perspective, quotes Gladstone himself in 1876 as evidence that a ‘game’ was afoot’, a game in which he was backed by formerly alienated Nonconformists and the rising radical star of Chamberlain. For Shannon the ‘abdication’ of 1875 was both a ‘fiction’ and short-lived (he re-emerged in 1876), Gladstone feeling only ‘slight embarrassment’ at pushing the Whigs aside in 1880 (in contrast to his protestations to Bright about the leadership in Source A). However candidates may feel that this ignores the seriousness of his intentions, his real commitment to retirement and only temporary re-emergence (?). He could also be quoting Gladstone out of context. The ‘game’ could merely refer to his campaign rather than a wider political agenda. A consideration of 1876–80 might stress Gladstone’s reluctance, at least in 1876–78 to embark on a crusade and his acceptance of the Midlothian liberal candidacy may have owed more to his desire to escape from Greenwich, scene of his 1874 humiliation, and to the traditional Rosebery interest in Midlothian than to a moral crusade that only developed once underway. In terms of judgement candidates may well consider the evidence for moral issues and ‘reluctance’ to be more compelling, given the framework of reference that all the sources adopt. However Gladstone’s views in A and D are, as always, politically careful and the consequence of both Bulgaria and Midlothian was to move Gladstone centre stage, enable him to take the credit for Liberal victory in 1880 and resume government and leadership without the qualms his comments to Bright in Source A suggest.

Page 154: History A Mark Schemes for the Units January 2010 - The ...

F963/02 Mark Scheme January 2013

16

Question Answer Marks Guidance 3 (a) These sources differ on several points. Source B accuses Labour MPs of failing to support

unemployed textile workers in Manchester whereas Source D demonstrates the solidarity of local Labour politicians and local people in the march to the High Court. In Source B the charge against Labour is that they preferred to talk, ‘empty and meaningless slogans at Liberal meetings’ in contrast to the politicians of Poplar who were prepared to take action in pursuit of changing the rating system. This point could be extended by consideration of the leadership shown by Labour politicians. In Source B the Party is described as without ‘teeth or claws’ and with ‘blind leaders’. However, in Source D the leaders of the council were prepared to go to jail and to remain defiant in prison by singing the Red Flag and Lansbury addressed their supporters from his ‘cell window’. According to Source B Labour MPs were more concerned with temperance reform than ‘the subject of unemployment’. However, the whole thrust of Source D indicates that the priority of the Labour Party in Poplar was to improve the lot of the poor and unemployed. In Source B MPs are said to ‘have displayed greater activity for temperance reform’ with the implication that some MPs regarded moral virtue as a remedy for poverty. This contrasts starkly with Source D which recognises clearly the imperative of financial matters to deal with the problems of the unemployed and they were prepared to flout their obligations to the LCC by withholding money from them.

A very obvious but important point should be made to explain the differences. Source B is commenting on the Parliamentary Labour Party which was in ‘alliance’ with the Liberals who headed the government. As such they were not in a position to determine policy whereas the Labour Party was in control of the county council in Poplar with some scope to decide policy and an ability to rebel against the LCC, albeit at a cost. However, there is convergence of purpose between the authors of Sources B and D: they both want change. When Source B was written the Liberal Party was addressing a range of social problems and expectations of change were high and, although reforms for the working man were introduced later, there had not been any significant progress by 1908 which explains the frustration of the author of Source B. After World War One there was great pressure on government, even at local level, to create ‘Homes for Heroes’. This helps explain the determination of the author of Source D to affect change from a Conservative dominated Coalition Government. Furthermore, the author was writing almost ten years after the rates row and perhaps exaggerates the role he and his colleagues played in reforming the rates. This could be linked to a broader point of motive. Lansbury was an ambitious Labour MP in 1928 and he may have been trying to enhance his reputation by writing his autobiography at this stage. Indeed, he assumed the leadership of the Party in 1931 and held the post till 1935.

30 Focus: Comparison of two Sources. No set answer is expected, but candidates need to compare the contents, evaluating such matters as authorship, dating, utility and reliability, so using the Sources ‘as evidence for …’. The headings and attributions should aid evaluation and reference to both is expected in a good answer.

Page 155: History A Mark Schemes for the Units January 2010 - The ...

F963/02 Mark Scheme January 2013

17

Question Answer Marks Guidance The author of Source B was a frustrated radical hoping to stimulate or embarrass the Labour Party into action and he wanted to be entirely detached from the Liberals. In judgement on the evidence candidates might argue that a comparison of the Party at national and local level has its limitations especially as Lansbury was also an MP. Given the different personal motives of the authors some might argue that Source B was more reliable. Either way both sources show how the problem of unemployment was a major issue as far many in the Party were concerned and that there were differences within the Party in how to deal with it, and over political tactics in the different circumstances of 1908 and 1921.

(b) Sources that support the interpretation strongly are B and E, although Source C is weighted that way too. However, Source C also provides evidence to the contrary: even Source E can be used to challenge the interpretation. Sources A and D provide less ambiguous evidence to challenge the interpretation. Sources B and E both charge Labour with ‘betrayal’ (Source B) and ‘treachery’ (Source E) specifically of the working class but more generally of party principles. Source B sarcastically suggests ‘they have displayed greater activity for temperance reform than for Labour interests’ and Source E accuses the Labour Government of ‘wholesale scrapping of principles’. This is developed in Source B which suggests Labour MPs are more interested in playing at politics and attending Liberal meetings where they mouth ‘empty and meaningless slogans’ implying a willingness to compromise their principles. Source E damns the Labour Party as working in ‘the service of capitalism’ which explicitly suggests they have abandoned their principles. Source C could be integrated into this analysis as it is particularly critical of MacDonald who is thought to be ‘preparing his exit from the ILP’ and of ‘joining the Liberal Party’. Even the founder of the Party, Kier Hardie, is described as disillusioned ‘with no real faith left in the Labour movement as a revolutionary force’. It is implied that ’the rank and file become more restive’ because they too are disillusioned. However, the authors of Sources B and C are of the radical wing of the Party and the author of Source E is from the Communist Party. As such the latter has a vested interest in painting the Labour Party as unprincipled and part of the establishment, ‘anxious to conserve the whole worn out fabric of Parliament’. He also ignores the fact that Mac Donald headed a minority government. The criticism of Tillett and Webb might be explained by virtue of being some of the early members of the Party and intellectuals who were intolerant of compromise, even naive in failing to appreciate the realities of politics.

70 Focus: Judgement in context, based on the set of Sources and own knowledge. Successful answers will need to make use of all five Sources, testing them against contextual knowledge and evaluating their strengths and weaknesses, any limitations as evidence. A range of issues may be addressed in focusing upon the terms of the question but no set conclusion is expected.

Page 156: History A Mark Schemes for the Units January 2010 - The ...

F963/02 Mark Scheme January 2013

18

Question Answer Marks Guidance Perhaps their criticism might be considered extreme given the relatively small size of the parliamentary party. However, both Sources C and E are less critical of the Party than Source B. The latter seems to condemn the parliamentary party as a whole as unprincipled: in reference to ‘Labour MPs could have been in Manchester’ the implication is that none were and that they were all culpable of not ‘supporting textile workers suffering starvation’. However, in Source C an MP of principle argued MPs ‘ought to vote ... ‘‘according to the merits’ of the particular issue’, and, that he ‘carried his resolution – however impractical’. In describing it in this way the author reveals her solidarity with this MP but the fact that he ‘shone out’ indicates that he was an exception, as a man of principle, amongst Labour MPs. In Source E a distinction is made between the Labour Government, against which the charge of being unprincipled and more concerned with power is made, and backbenchers, as the remarks ‘causing disquiet amongst their own followers in Parliament’, suggests that some MPs, at least, were more principled. Most candidates will use Source D to oppose the interpretation arguing that ‘the local Labour Party programme’ was taken seriously and action taken to achieve it. The fact that local politicians were prepared to go to jail in pursuit of the policy of ‘equalisation of the rates’ could be offered as evidence for their commitment to principles. Also, the singing of the Red Flag by the imprisoned Labour Party politicians suggests allegiance to socialist principles. The fact that they appear to have had the support of the unemployed suggests they were regarded as committed to the interests of the working class and true to their principles. It could be argued that the record of one local council is not representative of the Labour Party as a whole. Some might question whether the reform of the rates was merely tinkering with the system and did not amount to radical change of the sort associated with Labour Party rhetoric and constitutional principles of nationalisation and the social equality. The rates were merely one issue and we do not know if the local party was similarly principled on other matters. Source A is fairly unambiguous in highlighting the priority of principle. Socialism is said to have had ‘ethical appeal’ and ‘it was politics inspired by idealism’. In addition, the ways in which the ‘word’ was spread indicates the commitment of the supporters of the Labour Party to its ideals.

Page 157: History A Mark Schemes for the Units January 2010 - The ...

F963/02 Mark Scheme January 2013

19

Question Answer Marks Guidance References to the ‘Vocal Unions’ and ‘cycling corps’, ‘the singing of choirs’ and the enthusiasm with which slogans such as ‘Workers of the World Unite’ were distributed suggests a pride and honesty in those involved. The piece as a whole stresses the importance of the message and the principles behind the movement. It might be argued that this is not surprising given the passage is about the propaganda work of the ILP and does not comment on Labour in power. Further, these recollections are reflecting on the early years of the party when it was trying to gain support when enthusiasm for a new venture created an optimism and belief that the ILP would make a difference because it was different. The fact that the author is writing many years after his career has ended may be something candidates could comment on to assess the utility of the evidence. The evidence can clearly be used to support the interpretation but a strong counter argument can be constructed. This is more likely to be effective if candidates make distinctions between different parts of the Labour Party: Governments, MPs, rank and file, ILP, Fabians etc, or comment on the question in relation to various periods. However, even candidates who are less discerning in this regard should be able to produce a sound analysis.

Page 158: History A Mark Schemes for the Units January 2010 - The ...

F963/02 Mark Scheme January 2013

20

Question Answer Marks Guidance 4 (a) Both Source B and Source D see bombing as destructive – B sees ‘a whirlwind’ and talks

of the scale of bombing – millions of tons. D talks of impressive destruction and the impact on Germany – creating a ‘ruined land’. So both are aware of the power and destructiveness of the bombing. However, the thrust is different in other ways. B sees bombing as an act of revenge – the key sentence is that Germany has sowed the wind and reaped the whirlwind and Hitler’s boasts have rebounded on him. The RAF campaign is seen in terms of a technical success. There is no hint of remorse or self-criticism. In D however, the bombing is seen in a different light – there is an element of deception – ‘terror under other pretexts’ which is not present in B at all. Dresden, as opposed to Düsseldorf raises a serious query about the conduct of the war not at all raised in B. The emphasis in B is on success and achievement; but D has ‘terror and wanton destruction’. The object of bombing is not queried in B but the creation of ‘an utterly ruined land’ casts doubts on the wisdom of the policy in D. When considering provenance, what explains the difference is the very different context and purpose of the two sources. The speech to Londoners in 1943 (B) is to offer the consolations of revenge after the sufferings of the Blitz and in the middle of a very protracted war in which it was difficult to strike directly at the heart of German power. By 1943, with the aid of the US air force it was possible to inflict heavy damage on German cities and it was a matter of raising morale by drawing attention to the power of the allied attacks. Victory was so far away that it would not have occurred to Churchill to consider what the allies might take over when their forces came to control Germany. By 1945, with the end of the war in sight this became a more pressing concern. Dusseldorf had been an industrial area; Dresden far less so. D is not a public document and Churchill would not have criticised any aspect of the war in public, however ferocious such was the intense desire to bring the war to an end; but in private there could be greater reflection, particularly as the matter of post-war political and historical reputation was now a matter to be considered. In other words, moral qualms were able to be indulged in the situation in 1945, but had little place in the midst of the war in 1943. Also, what might be raised in confidence was very different to what could be said in public, particularly in the light of the very heavy losses incurred by RAF bombing crews.

30 No set answer is expected, but candidates need to compare the contents, evaluating such matters as authorship, dating, utility and reliability, so using the Source ‘as evidence for…..’ The Headings and attributions should aid evaluation and reference to both is expected in a good answer.

Page 159: History A Mark Schemes for the Units January 2010 - The ...

F963/02 Mark Scheme January 2013

21

Question Answer Marks Guidance When making a judgement, candidates might consider that In the light of the bitter feelings about the bombing of Britain, it may be that B is more typical of Churchill and other war leaders’ attitude to bombing; However given Churchill’s generosity to defeated enemies and his scepticism about the policies of his military commanders, it might be argued that D is also typical, but not of the war as a whole. B may be seen as a more useful guide to understanding British bombing policy given Harris’s beliefs that German morale would break and the heavy commitment of manpower and resources to bombing. The exceptionally high casualties of Dresden and the fact that it was primarily for the benefit of the USSR may make Churchill’s regrets in D rather less typical.

Page 160: History A Mark Schemes for the Units January 2010 - The ...

F963/02 Mark Scheme January 2013

22

Question Answer Marks Guidance (b) This remains a controversial topic and Bomber Command and its chief, Sir Arthur Harris, felt

ill-used by Churchill after the war. Some German historians have seen the allied bombing as a war crime and the efficacy of bombing both in undermining morale in Germany and in restricting war production have been questioned. Sources E and C offer a case for the policy to have been ill-judged as a whole. Source D suggests some lack of judgement while Sources A and B see a justified and successful policy. The case for poor judgement: Source E sees a change of policy in 1942 leading to retaliation and having an opposite effect on German morale from that intended with the population closing ranks behind Hitler as never before. The civilian bombing at the expense of strategic bombing was ineffective and the general effect was to lengthen rather than to shorten the war. Fest offers historical rather than moral judgements, but his generation suffered directly from the bombing and he may not be an impartial observer. However, he is an expert on German wartime armament through his work on Speer who did remarkably keep production going despite the bombing.

In Source C Bishop Bell in a brave and highly unpopular attack on wartime policy looked to the future and saw the policy as ill-judged in threatening the very values for which the war was being fought and ending any chance of the millions who were not Nazis supporting an end to the war – making a similar point to Speer about increasing rather than decreasing German support for the Hitler regime. There is a moral condemnation that ‘the roots of civilization’ are threatened; candidates might see some evidence for this in the increasing brutalization and carelessness for civilian life. Obviously, as a religious figure, Bell is speaking from a moral and philosophical perspective, but his arguments have some practical basis as well. This is a public speech with the intention to warn, but it does have some echoes in Churchill’s own view of Dresden in Source D. The point regarding poor judgement could also be developed using E’s comments about Harris and D’s regrets – did Churchill show poor judgement in first instructing him to ‘area bomb’ and then not reigning Harris in over widening the strategy. Later relations between Churchill and Harris might suggest this.

70 Focus: Judgement in context, based on a set of Sources and own knowledge. Successful answers will need to make use of all five Sources, testing them against contextual evidence and evaluating their strengths and weaknesses, any limitations as evidence. A range of issues may be addressed in focusing upon the terms of the question but no set conclusion is expected.

Page 161: History A Mark Schemes for the Units January 2010 - The ...

F963/02 Mark Scheme January 2013

23

Question Answer Marks Guidance The counter-view. However, by 1945 the outcome of war was no longer in question and criticisms of bombing may not take into account the problems of engaging with Germany in a decisive way before a Second Front was opened in June 1944. Before that the burden of the war fell on the USSR and as Source A makes clear, part of the rationale for heavy bombing of Germany was to support Stalin (or some may say to offer at relatively low cost allied support which did not involve the huge casualties of a land–based Second Front). There was a rational argument that German air strength was being diverted and depleted and it was certainly important to do everything possible to encourage the Russian victories in Russia in 1943 which were the turning point in the war. Stalin however regarded this as a poor substitute for actually landing large scale forces in France. There is rather more strategic justification in Source A as one would expect from the nature of the source – a joint letter to Stalin at a time when relations were quite difficult within the alliance. Source B is different, being a rousing speech to the people of London who would be less interested in strategic concerns than in seeing revenge for the raids inflicted on the capital and having their confidence boosted in the midst of a long and seemingly endless struggle by an assurance of the power and success of the RAF. Churchill’s judgement here might be a morale and political one for a domestic audience. However, the effectiveness of the raids celebrated in Source B might be questioned in the light of Source E and the ability of Germany to sustain war production well into 1944–5. D might question the wisdom of persisting with terror bombing, but this was at a time when Churchill could reflect on the morality of it all and look forward to occupying Germany – perhaps luxuries not available earlier in the war when, as he said, Britain had to fight as it could rather than how it wished.

Page 162: History A Mark Schemes for the Units January 2010 - The ...

Oxford Cambridge and RSA Examinations is a Company Limited by Guarantee Registered in England Registered Office; 1 Hills Road, Cambridge, CB1 2EU Registered Company Number: 3484466 OCR is an exempt Charity OCR (Oxford Cambridge and RSA Examinations) Head office Telephone: 01223 552552 Facsimile: 01223 552553 © OCR 2013

OCR (Oxford Cambridge and RSA Examinations) 1 Hills Road Cambridge CB1 2EU OCR Customer Contact Centre Education and Learning Telephone: 01223 553998 Facsimile: 01223 552627 Email: [email protected] www.ocr.org.uk For staff training purposes and as part of our quality assurance programme your call may be recorded or monitored

Page 163: History A Mark Schemes for the Units January 2010 - The ...

GCE

Oxford Cambridge and RSA Examinations

Option B Modern 1815-1945

Advanced GCE F963/02

History A

Mark Scheme for June 2010

Page 164: History A Mark Schemes for the Units January 2010 - The ...

OCR (Oxford Cambridge and RSA) is a leading UK awarding body, providing a wide range of qualifications to meet the needs of pupils of all ages and abilities. OCR qualifications include AS/A Levels, Diplomas, GCSEs, OCR Nationals, Functional Skills, Key Skills, Entry Level qualifications, NVQs and vocational qualifications in areas such as IT, business, languages, teaching/training, administration and secretarial skills. It is also responsible for developing new specifications to meet national requirements and the needs of students and teachers. OCR is a not-for-profit organisation; any surplus made is invested back into the establishment to help towards the development of qualifications and support which keep pace with the changing needs of today’s society. This mark scheme is published as an aid to teachers and students, to indicate the requirements of the examination. It shows the basis on which marks were awarded by Examiners. It does not indicate the details of the discussions which took place at an Examiners’ meeting before marking commenced. All Examiners are instructed that alternative correct answers and unexpected approaches in candidates’ scripts must be given marks that fairly reflect the relevant knowledge and skills demonstrated. Mark schemes should be read in conjunction with the published question papers and the Report on the Examination. OCR will not enter into any discussion or correspondence in connection with this mark scheme. © OCR 2010 Any enquiries about publications should be addressed to: OCR Publications PO Box 5050 Annesley NOTTINGHAM NG15 0DL Telephone: 0870 770 6622 Facsimile: 01223 552610 E-mail: [email protected]

Page 165: History A Mark Schemes for the Units January 2010 - The ...

F963/02 Mark Scheme June 2010

Question (a) Maximum mark 30 A01a and b AO2a

1 13-14 15-16

2 11-12 13-14

3 9-10 10-12

4 7-8 8-9

5 5-6 6-7

6 3-4 3-5

7 0-2 0-2

Notes related to Part A:

(i) Allocate marks to the most appropriate level for each AO (ii) If several marks are available in a box, work from the top mark down until the best fit

has been found (iii) Many answers will not be at the same level for each AO

1

Page 166: History A Mark Schemes for the Units January 2010 - The ...

F963/02 Mark Scheme June 2010

Marking Grid for Question (a) A0s A01a and b A02a

Total for each question =30

Recall, select and deploy historical knowledge appropriately, and communicate knowledge and understanding of history in a clear and effective manner. Demonstrate understanding of the past through explanation, analysis and arriving at substantiated judgements of: - key concepts such as causation, consequence, continuity, change and significance within an historical context; - the relationships between key features and characteristics of the periods studied.

As part of an historical enquiry, analyse and evaluate a range of appropriate source material with discrimination.

Level 1   Consistent and developed comparison of the key issue with a balanced and well‐supported judgement. There will be little or no unevenness. 

  Focused use of a range of relevant historical concepts and context to address the key issue. 

  The answer is clearly structured and organised. Communicates coherently, accurately and effectively.                            

13-14

  Focused comparative analysis. Controlled and discriminating evaluation of content and provenance, whether integrated or treated separately. 

  Evaluates using a range of relevant provenance points in relation to the sources and question. There is a thorough but not necessarily exhaustive exploration of these. 

 15-16

Level 2   Largely comparative evaluation of the key issue with a balanced and supported judgement. There may be a little unevenness in parts.  

  Focused use of some relevant historical context with a good conceptual understanding to address the key issue. 

  The answer is well structured and organised. Communicates clearly. 

11-12

  Relevant comparative analysis of content and evaluation of provenance but there may be some unevenness in coverage or control. 

  Source evaluation is reasonably full and appropriate but lacks completeness on the issues raised by the sources in the light of the question. 

13-14 Level 3   Some comparison linked to the key 

issue. Is aware of some similarity and/or difference. Judgements may be limited and/or inconsistent with the analysis made.  

  Some use of relevant historical concepts and contexts but uneven understanding. Inconsistent focus on the key issue. 

  The answer has some structure and organisation but there is also some description. Communication may be clear but may not be consistent. 

9-10

  Provides a comparison but there is unevenness, confining the comparison to the second half of the answer or simply to a concluding paragraph. Either the focus is on content or provenance, rarely both. 

  Source evaluation is partial and it is likely that the provenance itself is not compared, may be undeveloped or merely commented on discretely. 

10-12

2

Page 167: History A Mark Schemes for the Units January 2010 - The ...

F963/02 Mark Scheme June 2010

Level 4 Some general comparison but undeveloped with some assertion, description and/or narrative. Judgement is unlikely, unconvincing or asserted. 

A general sense of historical concepts and context but understanding is partial or limited, with some tangential and/or irrelevant evidence. 

Structure may be rather disorganised with some unclear sections. Communication is satisfactory but with some inaccuracy of expression. 

7-8

Attempts a comparison but most of the comment is sequential. Imparts content or provenance rather than using it. 

Comparative comments are few or only partially developed, often asserted and/or ‘stock’ in approach. 

8-9

Level 5 Very Limited comparison with few links to the key issue. Imparts generalised comment and /or a weak understanding of the key points. The answer lacks judgement or makes a basic assertion. 

Basic, often inaccurate or irrelevant historical context and conceptual understanding. 

Structure lacks organisation with weak or basic communication. 

5-6

Identifies some comparative points but is very sequential and perhaps implicit 

Comment on the sources is basic, general, undeveloped or juxtaposed, often through poorly understood quotation. 

6-7

Level 6 Comparison is minimal and basic with very limited links to the key issue. Mainly paraphrase and description with very limited understanding. There is no judgement. 

Irrelevant and inaccurate concepts and context. 

Has little organisation or structure with very weak communication. 

3-4

Little attempt to compare. Weak commentary on one or two undeveloped points, with basic paraphrase. Sequencing is characteristic.  

Comments on individual sources are generalised and confused. 

3-5 Level 7 Fragmentary, descriptive, incomplete 

and with few or no links to the key issue. There is little or no understanding. Much irrelevance. 

Weak or non existent context with no conceptual understanding. 

No structure with extremely weak communication. 

0-2

No attempt to compare either content or provenance with fragmentary, brief or inaccurate comment. 

Makes no attempt to use any aspects of the sources. 

0-2

3

Page 168: History A Mark Schemes for the Units January 2010 - The ...

F963/02 Mark Scheme June 2010

Question (b) Maximum mark 70 A01a and b AO2a and b

1 20-22 42-48

2 17-19 35-41

3 13-16 28-34

4 9-12 21-27

5 6-8 14-20

6 3-5 7-13

7 0-2 0-6

Notes related to Part B:

(iv) Allocate marks to the most appropriate level for each AO (v) If several marks are available in a box, work from the top mark down until the best fit

has been found (vi) Many answers will not be at the same level for each AO

4

Page 169: History A Mark Schemes for the Units January 2010 - The ...

F963/02 Mark Scheme June 2010

AOs A0Ia and b Ao2a and b

Total mark for the question = 70

Recall, select and deploy historical knowledge appropriately, and communicate knowledge and understanding of history in a clear and effective manner. Demonstrate understanding of the past through explanation, analysis and arriving at substantiated judgements of: - key concepts such as causation, consequence, continuity, change and significance within an historical context; - the relationships between key features and characteristics of the periods studied.

As part of an historical enquiry, analyse and evaluate a range of appropriate source material with discrimination. Analyse and evaluate, in relation to the historical context, how aspects of the past have been interpreted and represented in different ways.

Level 1   Convincing analysis and argument with developed explanation leading to careful, supported and persuasive judgement arising from a consideration of both content and provenance. There may be a little unevenness at the bottom of the level. 

  Sharply focused use and control of a range of reliable evidence to confirm, qualify, extend or question the sources. 

  Coherent organised structure. Accurate and effective communication. 

20-22

  A carefully grouped and comparative evaluation of all the sources with effective levels of discrimination sharply focused on the interpretation. 

  Analyses and evaluates the strengths, limitations and utility of the sources in relation to the interpretation. Uses and cross references points in individual or grouped sources to support or refute an interpretation. 

  Integrates sources with contextual knowledge in analysis and evaluation and is convincing in most respects. Has synthesis within the argument through most of the answer. 

42-48

Level 2   Good attempt at focused analysis, argument and explanation leading to a supported judgement that is based on the use of most of the content and provenance. 

  A focused use of relevant evidence to put the sources into context. 

  Mostly coherent structure and organisation if uneven in parts. Good communication.        

17-19

  Grouped analysis and use of most of the sources with good levels of discrimination and a reasonable focus on the interpretation. 

  Analyses and evaluates some of the strengths and limitations of the sources in relation to the interpretation. May focus more on individual sources within a grouping, so cross referencing may be less frequent. 

  Some, perhaps less balanced, integration of sources and contextual knowledge to analyse and evaluate the interpretation. Synthesis of the skills may be less developed. The analysis and evaluation is reasonably convincing. 

35-41

5

Page 170: History A Mark Schemes for the Units January 2010 - The ...

F963/02 Mark Scheme June 2010

Level 3   Mainly sound analysis, argument and explanation, but there may be some description and unevenness. Judgement may be incomplete or inconsistent with the analysis of content and provenance. 

  Some relevant evidence but less effectively used and may not be extensive. 

Reasonably coherent structure and organisation but uneven. Reasonable communication. 

13-16

  Some grouping although not sustained or developed. Sources are mainly approached discretely with limited cross reference. Their use is less developed and may, in parts, lose focus on the interpretation. There may be some description of content and provenance. 

  Is aware of some of the limitations of the sources, individually or as a group, but mostly uses them for reference and to illustrate an argument rather than analysing and evaluating them as evidence. There is little cross referencing. 

  There may be unevenness in using knowledge in relation to the sources. Synthesis may be patchy or bolted on. Analysis and evaluation are only partially convincing. 

28-34 Level 4   Attempts some analysis, argument 

and explanation but underdeveloped and not always linked to the question. There will be more assertion, description and narrative. Judgements are less substantiated and much less convincing. 

  Some relevant evidence is deployed, but evidence will vary in accuracy, relevance and extent. It may be generalised or tangential. 

  Structure is less organised, communication less clear and some inaccuracies of expression.  

9-12

  Sources are discussed discretely and largely sequentially, perhaps within very basic groups. Loses focus on the interpretation.  The sources are frequently described. 

  May mention some limitations of individual sources but largely uses them for reference and illustration. Cross referencing is unlikely. 

  An imbalance and lack of integration between sources and knowledge often with discrete sections. There is little synthesis. Analysis and explanation may be muddled and unconvincing in part. 

21-27 Level 5   Little argument or explanation, 

inaccurate understanding of the issues and concepts. The answer lacks judgement. 

  Limited use of relevant evidence or context which is largely inaccurate or irrelevant. 

  Structure is disorganised, communication basic and the sense not always clear. 

5-8

  A limited attempt to use the sources or discriminate between them. The approach is very sequential and referential, with much description. Points are undeveloped. 

  There is little attempt to analyse, explain or use the sources in relation to the question. Comment may be general. 

  There is a marked imbalance with no synthesis. Analysis and explanation are rare and comments are unconvincing. 

14-20 

6

Page 171: History A Mark Schemes for the Units January 2010 - The ...

F963/02 Mark Scheme June 2010

Level 6   There is very little explanation or understanding. Largely assertion, description and narrative with no judgement. Extremely limited relevance to the question. 

  Evidence is basic, generalised, patchy, inaccurate or irrelevant. 

  Little organisation or structure with poor communication. 

3-4

  Very weak and partial use of the sources for the question. No focus on interpretation. 

  A very weak, general and paraphrased use of source content.

  No synthesis or balance. Comments are entirely unconvincing. 

7-13 Level 7   No argument or explanation. 

Fragmentary and descriptive with no relevance to the question. 

  No understanding underpins what little use is made of evidence or context. 

  Disorganised and partial with weak communication and expression. 

0-2

  Little application of the sources to the question with inaccuracies and irrelevant comment. Fragmentary and heavily descriptive. 

  No attempt to use any aspect of the sources appropriately. 

  No contextual knowledge, synthesis or balance. There is no attempt to convince. 

0-6

7

Page 172: History A Mark Schemes for the Units January 2010 - The ...

F963/02 Mark Scheme June 2010

1 The Condition of England 1815-1853

(a) Study Sources B and D Compare these sources as evidence for the causes of the Plug Plot disturbances in 1842. [30]

No set answer is expected, but candidates need to compare the contents, evaluating such matters as authorship, dating, utility and reliability, so using the Source ‘as evidence for…’. The headings and attributions should aid evaluation and reference to both is expected in a good answer.

Both Sources agree that the reduction of wages would appear to have been the trigger and that it would seem that that this was done deliberately by employers. They also stress that others encouraged workers to strike and disturb the peace for their own ends. This is especially elaborated on by Cooper in D. They differ on the reasons why wages were reduced. The magazine in B hints at a general ‘intention’ to do this quickly, presumably as part of an employer reaction to the great Victorian Depression (1842 was the worst year of the 19th century). The need would be to reduce labour costs. B also lays the blame on the Chartists, to be expected from a new magazine anxious to appeal to the wealthy and to stress their command of northern topics. It identifies a ‘spreading organisation of a most formidable’ type, but it also adopts a dim view of the Anti Corn Law League. In contrast Source D, Cooper, blames the Anti Corn Law League, who, it claims, deliberately encouraged employers to pick a strike to paralyse the government and assist their case for the repeal of the Corn Laws. Source B also stresses poverty, hence the references to plundering (clothes from workhouses and food from shops). Source D introduces drunkenness and the Charter (“Strike for the Charter”). In part the similarities are explained by the date, 1842, a year of exceptional depression and poverty. However the Illustrated News (B), a contemporary source, takes a more propertied view, blaming radical organisations for stirring things up to satisfy their own ends. Source D is a Chartist source written by a key witness and participant in the event. It places the blame elsewhere, on employers, on drunken workers and the depression. Cooper takes care to stress drunkenness because he was almost immediately held responsible, with others, for the Pottery disturbances. As a radical Chartist speaker he was imprisoned. In his memoirs he may have wanted to clear himself, particularly as he later withdrew from radicalism and repented for a misspent youth. He claims that he was ashamed of the actions of those who destroyed property and broke the law. In practice he may have encouraged disturbance and be one of those members of Source B’s ‘spreading organisation’. He certainly blamed the Anti Corn Law League. Both are useful for the different perspectives. It is difficult to verify the sources of B’s information and it is certainly generalised. Its respectable perspective ensures Chartist and League blame but it does sympathise with the poor. Cooper in contrast was a participant and potentially the more informative but his memoirs portray reticence on his own involvement in physical force.

(b) Study all the Sources

Use your own knowledge to assess how far the sources support the interpretation that force and provocation were tactics more used by the authorities and employers than the Chartists. [70]

Successful answers will need to make use of all five sources, testing them against contextual knowledge and evaluating their strengths and weaknesses, including any limitations as evidence. A range of issues may be addressed in focusing upon the terms of the question but no set response is expected.

The sources may be used in a variety of ways to assess the relative use of force in the Chartist period. Only Source A, Thomas Dunning, unequivocally comments on state

8

Page 173: History A Mark Schemes for the Units January 2010 - The ...

F963/02 Mark Scheme June 2010

violence and force. The other four sources can bear different interpretations. However when discussing Chartist physical force it is worth remembering that whilst four of the sources are from Chartist participants all are memoirs, often written long after Chartist collapse in a less violent and more reforming age. Three of these four represent moral force Chartism, Gammage in C particularly so. Only Cooper was identified with physical force in 1842 and by the time he wrote his memoir he had long changed his mind. All are likely to play down Chartist violence yet play up O’Connor’s provocative stance, although Adams in E admits that this was more self delusional than forceful in practice. Nonetheless the argument that Chartists used force and provocation as a tactic can be found in Sources B, D and E. Riots are mentioned in the magazine (B), with hints of trained ‘discipline’ and the possibility of ‘rebellion’. Gammage in C refers disparagingly to O’Connor’s provocative demagogy and posturing that own knowledge can link to threats of physical force (‘peacefully if we may, forcefully if we must’).However Gammage is also at pains to point out that other chartist organisations, notably the London Working Men’s Association, disclaimed physical force. There is a convincing argument that the Chartists avoided force whenever possible, knowing its potential for harming the cause, as Cooper reminds his audience outside the Crown in Source D. Adams in E corroborates Gammage’s view on O’Connor provocations with his comments on ‘denunciations’, ‘rabble rousing’ and regal ‘delusions’, something which his republican sympathies would find uncongenial. As noted, only Cooper in D fully describes one of the two main physical force moments in Chartism, the Plug Plots of 1842. The other moment, the Newport Rising in 1839 may have had hopes of triggering a wider rising but was localised and the jury remains out on who was responsible for the shooting. Cooper refers to a ‘wild general strike’ which wasn’t necessarily ‘physical’. The evidence in the Potteries is muddied by Cooper’s subsequent arrest and trial, but it would seem from this account that much of the force was random, disorganised and focused on the destruction of property. From hindsight he clearly disapproves. The magazine in B also refers to plundering. Own knowledge might refer to the violent and physical force language used, the reports of military drilling , the intimidating tactics of large scale meetings and the occasional surfacing of weapons. Three of the sources contain strong suggestions that the authorities and employers used force and provocation on every possible occasion (A, B, and D). In B we are told that the employers reduced wages without notice and troops ‘poured into the disturbed districts’ during the Plug disturbances, a fact confirmed by Cooper in D who refers to employers who, for their own ACLL ends, provoked workers whilst cavalry and artillery were deployed by the authorities. Own knowledge may mention the use of the new railways enabling troops from other areas to be deployed quickly. Source A, Dunning, is however the strongest evidence for forceful tactics by the authorities. He refers to persecution and imprisonment, a long standing tactic used by governments to take dangerous radicals out of circulation and starve them of the oxygen of publicity. A series of mock trials were held and, on grounds of sedition, radicals were sentenced to a couple of years in gaol. Dunning cites precise numbers (93) and provides effective commentary given his legal role in challenging the evidence in these trials (and his success in 1834 in defending some trade unionists). He is well aware of the pressure brought to bear in Court (the weapons on constant display to remind juries of a Chartist threat of force). The fate of Cooper in 1842 (D) and of all the Chartist leadership, including moral force leaders like Lovett, supports this. During the Chartist Petitions and especially at Kennington Common in 1848 the government was armed to the teeth, using the tactic of special constables sworn in for an auxiliary role. Although Dunning’s memoirs were written at the very end of a long life he clearly kept records and writes of the cruel conviction of all governments.

9

Page 174: History A Mark Schemes for the Units January 2010 - The ...

F963/02 Mark Scheme June 2010

2 The Age of Gladstone and Disraeli 1865-1886

(a) Study Sources C and E Compare these sources as evidence for the attitudes of those who opposed British imperial expansion. [30]

No set answer is expected, but candidates need to compare the contents, evaluating such matters as authorship, dating, utility and reliability, so using the sources ‘as evidence for…’. The headings and attributions should aid evaluation and reference to both is expected in a good answer.

Both sources C and E agree that there was resistance to expansion although their attitudes are different, as is the type of resistance they are discussing (from Afghans in C and Egyptians in E). Both are from a western, British viewpoint and stress in C a sense of ‘freedom’ (Gladstone) and in E (Churchill) a sense of nationhood (Arabi as ‘the movement of the nation’). Both agree that the opposition is righteous (‘naturally and not wrongfully’ in C; ‘we should have approved their action’ in E) and they agree that Britain has wrongfully oppressed opposition. Their tone is remarkably similar – to Gladstone such oppression is ‘horrible’, to Churchill it is ‘devastation’, ‘struggling wretches’ and the ‘mess of oppression’. They both condemn British military expansion and intervention and sympathise with the attitudes of those who resisted. They differ in that Gladstone in C is discussing the Afghan attitude to British political methods (the Indian model of establishing British Residents as Trojan Horses, hence the Afghan opposition to Disraeli, Carnarvon, and Lytton’s proposal for a Resident at Kabul to counter Russian influence) whereas Churchill in E is discussing fiscal infiltration. He objects to the entrapment of the Egyptians via debt and the crippling interest rates incurred through spendthrift rulers. However Gladstone would not have agreed with Churchill’s point on the legitimacy of Egyptian debt repudiation. The flouting of fiscal rectitude was not, in his eyes, to be equated with the unwarranted destruction of Afghan hill villages. The bombardment of Alexandria was legitimate; burning and subsequent death in the Afghan snow was not. In terms of provenance Gladstone (C)) is speaking in an election campaign to denounce the forward Imperialism of Beaconsfieldism, using emotive language in the wake of disasters in both South Africa and Afghanistan. Its purpose is to see the Afghans in the same light as one would one’s own and to rally popular emotion behind one of the great campaigns of modern political history. Churchill in E is, similarly, a political and public speech, 3 years later, in an area that was part of the Midlothian constituency that Gladstone won from a Conservative landowner. He is concerned to stress the hypocrisy of Gladstone, now Liberal PM., who had justified his decision to bombard Alexandria, having condemned Disraeli for retaliating against the Afghan Amir. Interestingly Churchill was a Conservative but he condemns both Tory and Liberal for pursuing illegitimate ends. His angle is the political outsider keen to make an impression within Conservatism and to pressurise his leader, Lord Salisbury. Identifying with the deceased Disraeli he cannot resist having a go at Gladstone. Both are partisan sources, neither especially balanced, but a case could be made for Churchill as the more useful source, unless one sees Gladstone as more typical in the attitudes of those who opposed imperial expansion. Both sense the attitude of those on the receiving end, as well as opponents at home, and both are useful for gauging public opinion given their attempts to mould it.

10

Page 175: History A Mark Schemes for the Units January 2010 - The ...

F963/02 Mark Scheme June 2010

(b) Study all the Sources Use your own knowledge to assess how far the Sources support the interpretation that there was little difference between the Imperial policies of Gladstone and Disraeli. [70]

Successful answers will need to make use of all five Sources, testing them against contextual knowledge and evaluating their strengths and weaknesses, including any limitations as evidence. A range of issues may be addressed in focusing upon the terms of the question but no set conclusion is expected.

D, E and parts of Sources A and B contain evidence that Imperial policy differed little in practice. Given that D and E come from opposite political ends (Gladstone and Churchill) one might consider them effective evidence on continuity but D is Gladstone in office as PM and this is an official diplomatic offer to Kruger continuing the pursuit of Confederation. It is not necessarily his personal opinion or policy and indeed we are told that the advice came from the Colonial Office. Nonetheless Gladstone did not challenge it. Churchill is making a point about the similarity of intervention under both men, whether it is Afghanistan or Egypt, but his view is that of the outsider and his agenda might be suspect. Perhaps better evidence is parts of A and B, both public statements of Gladstone and Disraeli whilst in opposition (and therefore more reliable, or simply more suspect in terms of point scoring?). Interestingly, in A, Disraeli condemns Gladstone’s Liberal moves whilst PM on Imperial Confederation, although he acknowledges that he has no objection provided the terms were right. The bounds of empire have become too loose. Yet later, whilst PM, Disraeli encourages Frere in Cape Colony to pursue a South African Confederation along Canadian lines, which led to disaster with the Zulus. In D Gladstone, at the instigation of the Colonial Office, is prepared to continue with Confederation and attempts to persuade Kruger to accept British Sovereignty (it cannot be given up). Again, as the introduction indicates, it led to disaster (the First Boer war) and the policy was abandoned in the Convention of Pretoria (which acknowledged the Transvaal’s independence, saving some vague fig leaf on suzerainty). On the issue of Confederation there would seem to be continuity. There is also evidence of a policy of forceful intervention by both when challenged. Source C demonstrates Disraeli’s apparent use of force to woo the Amir (knowledge might suggest he was reluctantly dragged into it by Lytton) whilst Source E shows Gladstone’s use of force in Egypt to preserve financial stability in the Canal zone, perceived as a vital British interest since Disraeli’s Purchase in 1875. However both sources come from the mouths of opponents, although the fact of military and naval intervention cannot be disputed. Both justified it, but on very different grounds. In A and B both agree on a ‘mighty mission’. The alternative view, that there was a considerable difference in imperial policy between them, can be found in Sources A, B and C. Sources A and B establish the key public difference between the two men and both are classic statements of their kind. However both are written whilst in opposition, keen to establish clear blue water between them. Each uses the Empire as a stick to beat the other. Thus Disraeli in A accuses the Gladstone government of a plot to dismember the empire using the means of unfettered Confederation. He accuses Gladstone of fiscal meanness, throwing away our greatest asset to save money and reduce taxes to buy further power. There may be some truth in this as Gladstone notoriously economised with both army and navy. On the other hand Gladstone in B accuses Disraeli of needless and reckless expansion regardless of British interests and of the means to pay for it (no ‘men or money to sustain it’). A clear difference emerges on Imperialism. For Gladstone the Empire is a Pax Britannica with Britain as its core, setting an example by justice, reform and improvement. Welfare and peace are the key, even to the extent of ‘friendly independence’. Knowledge would suggest this to be the case given Gladstone’s reluctance to intervene and his Confederation policy in Canada and South Africa in the first and second ministries, (Source D refers to the Transvaal having ‘the fullest liberty to manage their local affairs’, although this was ‘to be without prejudice to the rest of the population’, a reference to continued antagonism with the

11

Page 176: History A Mark Schemes for the Units January 2010 - The ...

F963/02 Mark Scheme June 2010

Zulus). However for Disraeli the Empire is the ‘Empire of England’, to be based on a keen sense of national interest, (such as imperial tariffs - although he never attempted anything like this whilst in power). It implied he would spend money (although in power he too was reluctant to incur expenditure). Gladstone maintained in B that Disraeli also desired territorial acquisition, which candidates might dispute in practice. Certainly the Empire increased more in size under Gladstone than under Disraeli. As to Disraeli’s supposed propensity to use force Gladstone also resorted to it, albeit reluctantly. Source C also underlines the differences, but it too is produced in opposition, part of one of the great political campaigns to win an election on the basis of an opponent’s immoral and forward imperialism. Its tone is sincere but emotional. The sources certainly support a rhetorical difference but they are less certain in their evidence and provenance as to practice.

12

Page 177: History A Mark Schemes for the Units January 2010 - The ...

F963/02 Mark Scheme June 2010

3 England and a New Century 1900-1924 Post War Problems 1918 to 1924

(a) Study Sources B and C

Compare these Sources as evidence for views on government spending on social problems during the immediate post war period. [30]

No set answer is expected, but candidates need to compare the contents, evaluating such matters as authorship, dating, utility and reliability, so using the Sources ‘as evidence for...’. The headings and attributions should aid evaluation, and reference to both is expected in a good answer.

There are obvious differences in the content of the two sources. Source B supports government spending, particularly on housing and legislation on industry, as a means of reducing the danger of revolution. But, in Source C the focus of the Geddes Report is on shortage of money, although the committee does suggest greater efficiencies as well as cuts in government expenditure. Source B is mainly concerned with housing, urban renewal and working hours. But it also urges improvements in industry, particularly where working conditions are concerned. Source C has wider economic and financial concerns within a particular remit, and seems to regard social conditions as a lower priority. However a close reading reveals that this extract focuses more on savings than cuts per se. Health programmes are considered to have merit. Both sources reflect a background of post war social and economic difficulty. In terms of provenance, the two years between the dates of the sources reflect a radical change in the situation. By 1922, the Coalition Government is no longer able to provide large sums of money for housing, and other social reforms. In Source B, Tom Jones (a close friend, as well as an adviser) would expect the Prime Minister to respond positively. Lloyd George had promised “Homes for heroes” in the run up to the 1918 Election. He has already supported the setting up of the Whitley Councils, bodies which represented both managers and workers in the main industries. Jones sounds optimistic and clearly goes beyond a civil service remit to endorse a progressive Liberal agenda when Lloyd George was at the height of his power. Lloyd George, already known for his energy as a war time leader, is in a strong position at this time. However, by the time of the Geddes Report (Source C), economic problems and trade union militancy have weakened Lloyd George’s position. Right wing opinion (the Committee is composed entirely of business men and financiers) favours retrenchment to avoid tax increases on business. Less sympathy is shown here for working class interests. Lloyd George, having set up the Committee himself, will be forced to agree to massive cuts in social expenditure.

(b) Study all the Sources.

Use your own knowledge to assess how far the Sources support the interpretation that the Labour Party was better placed than its rivals to deal with Britain’s post war problems 1918-1924. [30]

Successful answers will need to make use of all five Sources, testing them against contextual knowledge, and evaluating their strengths and weaknesses and any limitations as evidence. A range of issues may be addressed in focusing upon the terms of the question, but no set conclusion is expected.

Candidates will benefit from knowing (in outline at least) the main political events of 1918 to 1924. All of the parties, including Labour, suffered from having to confront the serious social and economic problems bequeathed by the First World War. The issue is who was best placed to deal with the social and economic consequences of the War. Supporting the assertion. Source A. Having made important war-time contributions to Government, the Labour Party shows a new confidence in laying out ambitious plans for

13

Page 178: History A Mark Schemes for the Units January 2010 - The ...

F963/02 Mark Scheme June 2010

what would effectively be a welfare state, replacing the discredited efforts of the pre-war Poor Law. After the Election of 1918, Labour only had 63 seats. Yet, effectively, with the Liberals split, and Lloyd George the leader of a Conservative dominated coalition, Labour was moving towards becoming the second party. However, candidates may point out that the conference resolutions in A were probably unrealistic. At this stage, Britain’s post-war economic weaknesses were not yet fully appreciated. Labour was informal opposition aware of the new democratic vote to all men and was eager to claim the progressive agenda. Source B mentions the revolutionary pressures of 1919. These would tend to strengthen Labour’s appeal to the newly-enfranchised working classes but it also suggests Lloyd George’s Coalition to be the best placed, indicating significant achievement to date. Source D strengthens the view that the Liberals are on the way out, and that Labour has now (late 1923, and with 191 seats gained in the recent election) replaced them as the chief rival to the Conservatives although Amery calculates politically that both Liberal and Labour would be hampered by coalition and lack achievement – a calculation that was proved correct. The eventual result, mentioned in Source E, is the formation of the First Labour Government under Ramsay MacDonald. One might suppose that (by 1924) Labour was in a much stronger position to introduce its social reforms, although it was a minority government. E condemns it for selling out on more radical solutions to post-War problems. The main success was the Wheatley Housing Act of 1924. Opposing the assertion. Source B suggests that Lloyd George (Prime Minister in a strong Coalition Government, dominated by the Conservatives) by taking bold action, especially on housing, can head off threats from the Left. The Whitley Councils are an example of LG’s success in his early reforms, as is Addison’s Housing Act of 1919 and Fisher’s Education Act. However, by 1922, Lloyd George is being overwhelmed by his problems. The Whitley Councils were scrapped in 1921. Lloyd George was forced to accept the Geddes call for cuts in expenditure (see Source C). This source could be used to measure the extent of the problem fore all parties. It clearly limited the prospect of the then government and especially the Lloyd George Liberals. The economy continued to stagnate. Later in 1922, LG fell from office, and was replaced by Bonar Law. The Conservatives had a tendency to split over tariff reform, the traditional Conservative solution to Britain’s problems, but soon recovered given the prospect of government freed from Lloyd George’s spell. Source C clearly suggests a severe blow to any political party considering social reform, especially the kind of proposals favoured by Labour. The implication in Source D is that the Conservative Party, despite a loss of electoral support in 1923 over protectionist policies, will soon recover its pre-eminent position, although it fails to mention post-War economic/social problems. With hindsight, Amery’s advice to Baldwin (which Baldwin took) is seen to have been well-founded. Indeed, the First Labour Government was to be short lived. And Source E indicates that a Labour Government under Ramsay Macdonald was less likely to support the radical social reform that socialists like the Webbs wanted than the Lloyd George Liberals. There were some radical plans to deal with rising unemployment, but most never came to fruition.

14

Page 179: History A Mark Schemes for the Units January 2010 - The ...

F963/02 Mark Scheme June 2010

4 Churchill 1920-45 The Election of 1945

(a) Study Sources A and B.

Compare these Sources as evidence for the appeals made to the electorate in the campaign leading to the General Election of 1945. [30]

No set answer is expected, but candidates need to compare the contents, evaluating such matters as authorship, dating, utility and reliability, so using the Sources ‘as evidence’ for….The headings and attributions should aid evaluation and reference to both is expected in a good answer. Content: A refers to Labour’s experience in war; B seems to denigrate it by reference to ‘Gestapo’ – something that Labour had been fighting against. Both refer to the ordinary man- but A is more effective, claiming that the victory in the war was won by the people. For B the people must be protected against the power of the state. A says that the state will provide full employment and proper social security – the emphasis is on collective protection against hard times. B stresses individual liberty against the power of the state. Essentially A is offering a view of a beneficent state making up for deficiencies in past Conservative policy. B is seeing an oppressive state and equating Labour with totalitarianism, both Nazi and Communist. A is far more positive and effective, pointing to actual past Conservative failures. B relies on a less realistic view of Labour with the emphasis on what Labour might do – ignoring their wartime achievements. The tone of the passages is different – A looks forward positively, though it is critical of opponents; B offers, here, no positive commitments but a negative – Labour is going to threaten liberty and the Conservatives offer the status quo of traditional liberty, something A dismissed as equating to “mean and shabby treatment”. Provenance of Sources: Both are election appeals; both exaggerate the weaknesses of the opponents because it is election time. However, Source A is a considered and prepared written response. In fact the Conservative written manifesto said much the same about social reform and Churchill’s government endorsed the Beveridge Report. However, Source B, Churchill’s speech is much less considered and more improvised – he seems to have gone back to his early attacks on socialism and is equating Labour with totalitarianism in an extreme manner which runs contrary to his own manifesto. A, a collective document prepared by Labour as a whole must be contrasted with B, a flight of oratory which met with little enthusiasm within the Conservatives and showed Churchill to be out of touch with the mood of the times. The sources are useful for showing the reasons for Labour’s victory and Churchill’s limitation as a peacetime politician. Whereas A is quite typical of Labour’s appeal in 1945, B is not typical of what the Conservatives were actually offering although they are, classically, Churchill. Candidates might consider that A is more valuable for explaining the issues of the election while B showed why the Labour appeal was strengthened, so there is no set answer required for the relative value of the two sources, but candidates have the opportunity to assess this.

(b) Study all the Sources

Use your own knowledge to assess how far the Sources support the interpretation that the main reason for Churchill’s election defeat in 1945 was the policy of social reform offered by Labour. [70]

Successful answers will need to make use of all five Sources, testing them against contextual knowledge and evaluating their strengths and weaknesses, any limitations as evidence. A range of issues may be addressed in focusing upon the terms of the question but no set conclusion is expected. The debate here is whether Churchill’s conduct of the election and the poor Conservative record on social issues before the war were the key reasons, or whether it was Labour’s ability to offer a strong campaign and the public’s expectations for greater social reform

15

Page 180: History A Mark Schemes for the Units January 2010 - The ...

F963/02 Mark Scheme June 2010

that were the main reasons. The war had brought a lot of changes and hopes. Did the electorate think that Labour was more likely to deliver those hopes; or did the Conservatives throw away the trump cards they held by poor electioneering? The actual popular vote was closer than the large parliamentary majority gained by Labour suggests and Churchill thought that he would win until very close to the actual poll. Sources B, C and E support the view that Churchill misjudged the campaign, and by implication this rather than the offers of social change was the most important factor. though Thatcher also stresses some advantages that Labour had. B is a famous opening speech in which Churchill equates Labour with the Gestapo and stands, as he did in 1940, as the champion of liberty. With wartime propaganda stressing the fight for freedom, this might have seemed a good idea. However, as Attlee and his colleagues had stood alongside Churchill since 1940 as bitter enemies of Fascism, this sort of rhetoric seemed absurd. Also, the USSR was popular at this time and there was an increased interest in state planning. Wartime reports such as Barlow and Uthwatt had recommended this. Full employment was thought possible and the Conservatives were pledged to implement Beveridge and to provide a health service. Wartime reconstruction could not be left to private enterprise. Few were convinced by the simplistic rhetoric produced by a leader whose main preoccupations had been foreign policy. The reactions of Thatcher (E) and Sackville-West (C) are similar. However Thatcher is writing with the benefit of hindsight. Knowing that Churchill lost the election might colour her view of remembering that he had gone too far as she listened to the broadcast in her Oxford College. She is balanced enough to provide some wider analysis – this is after all a published work. Vita Sackville West is not attempting analysis but a ‘gut reaction’ in a private letter. Unlike Thatcher, she knew Churchill and there is some personal disappointment that he seems not to be rising to the challenge of domestic politics. Labour on the other hand responded more successfully to the highly emotional mood of the nation which the cartoon (D) so vividly represents. The issue of a lasting peace and a post-war Europe which will avoid the destruction of war and the sufferings of soldiers seems here to be paramount rather than social reforms or Churchill’s misjudgements. This is an appeal to the emotions from a left wing newspaper on the eve of the election. That cartoon represents an injured and battered soldier amid destruction offering a very hard won peace to the nation. The poor Conservative policies of the inter-war years referred to in A must not come again. As A says, this was a war won by the ordinary people represented in D. A is obviously intended as election propaganda but offers ‘proper social security’. This is the source that supports the view of the question most fully. Not the rhetoric of B, but the firm promises of social services and full employment were needed. As E says Labour could outbid the Conservatives on this front. The Labour ministers had been in charge of domestic policy during the war. Conservative commitment to Beveridge was doubted by many. This was Labour’s home ground and they could point to the apparently poor record of the National Governments of the 1930s – Means Test, high unemployment etc. Churchill had not been part of this, but he was shackled to a Conservative party which had been seen, despite its reforming aspects in the 1930s, as uncaring. Thatcher’s analysis can be defended. Also Churchill could not gain all the credit for victory in what A rightly identifies as a people’s war led by a coalition involving Labour and Liberals. Candidates might know about the younger generation’s radicalism, the influence of the Army Current Affairs discussions, the influence of wartime discussions about the post-war world, especially the Beveridge Report of 1942, about which many Conservatives were privately and publicly luke warm. Churchill gave priority to military and diplomatic concerns rather than post-war social policy. There were concerns about the disappointing aftermath of World War I, and an admiration for the Soviet achievement. They might contrast dynamic Labour figures like Bevin, Bevan, and Morrison with some lacklustre performances by the Conservatives who were over-reliant on Churchill’s prestige now squandered in what C considered a “confused and wordy” response. Certainly, Churchill’s

16

Page 181: History A Mark Schemes for the Units January 2010 - The ...

F963/02 Mark Scheme June 2010

17

concerns in 1945 – for the post-war settlement, for Eastern Europe, for Britain’s Empire – were not the concerns of many of his voters. Three of the sources are Conservative, their focus is on Churchill (his speech and the reaction to it) but B and E, implicitly and explicitly, acknowledge the role of social reform. The two Labour sources, A and D, focus largely on this and imply success for this very reason.

Page 182: History A Mark Schemes for the Units January 2010 - The ...

OCR (Oxford Cambridge and RSA Examinations) 1 Hills Road Cambridge CB1 2EU OCR Customer Contact Centre 14 – 19 Qualifications (General) Telephone: 01223 553998 Facsimile: 01223 552627 Email: [email protected] www.ocr.org.uk For staff training purposes and as part of our quality assurance programme your call may be recorded or monitored

Oxford Cambridge and RSA Examinations is a Company Limited by Guarantee Registered in England Registered Office; 1 Hills Road, Cambridge, CB1 2EU Registered Company Number: 3484466 OCR is an exempt Charity OCR (Oxford Cambridge and RSA Examinations) Head office Telephone: 01223 552552 Facsimile: 01223 552553 © OCR 2010

Page 183: History A Mark Schemes for the Units January 2010 - The ...

GCE

Oxford Cambridge and RSA Examinations

Unit F963/02: Option B Modern 1815-1945

Advanced Subsidiary GCE

History A

Mark Scheme for June 2011

Page 184: History A Mark Schemes for the Units January 2010 - The ...

OCR (Oxford Cambridge and RSA) is a leading UK awarding body, providing a wide range of qualifications to meet the needs of pupils of all ages and abilities. OCR qualifications include AS/A Levels, Diplomas, GCSEs, OCR Nationals, Functional Skills, Key Skills, Entry Level qualifications, NVQs and vocational qualifications in areas such as IT, business, languages, teaching/training, administration and secretarial skills. It is also responsible for developing new specifications to meet national requirements and the needs of students and teachers. OCR is a not-for-profit organisation; any surplus made is invested back into the establishment to help towards the development of qualifications and support which keep pace with the changing needs of today’s society. This mark scheme is published as an aid to teachers and students, to indicate the requirements of the examination. It shows the basis on which marks were awarded by Examiners. It does not indicate the details of the discussions which took place at an Examiners’ meeting before marking commenced. All Examiners are instructed that alternative correct answers and unexpected approaches in candidates’ scripts must be given marks that fairly reflect the relevant knowledge and skills demonstrated. Mark schemes should be read in conjunction with the published question papers and the Report on the Examination. OCR will not enter into any discussion or correspondence in connection with this mark scheme. © OCR 2011 Any enquiries about publications should be addressed to: OCR Publications PO Box 5050 Annesley NOTTINGHAM NG15 0DL Telephone: 0870 770 6622 Facsimile: 01223 552610 E-mail: [email protected]

Page 185: History A Mark Schemes for the Units January 2010 - The ...

F963/02 Mark Scheme June 2011

Question (a) Maximum mark 30 A01a and b AO2a

1 13-14 15-16

2 11-12 13-14

3 9-10 10-12

4 7-8 8-9

5 5-6 6-7

6 3-4 3-5

7 0-2 0-2

Notes related to Part A:

(i) Allocate marks to the most appropriate level for each AO (ii) If several marks are available in a box, work from the top mark down until the best fit

has been found (iii) Many answers will not be at the same level for each AO

1

Page 186: History A Mark Schemes for the Units January 2010 - The ...

F963/02 Mark Scheme June 2011

Marking Grid for Question (a) A0s A01a and b A02a

Total for each question =30

Recall, select and deploy historical knowledge appropriately, and communicate knowledge and understanding of history in a clear and effective manner. Demonstrate understanding of the past through explanation, analysis and arriving at substantiated judgements of: - key concepts such as causation, consequence, continuity, change and significance within an historical context; - the relationships between key features and characteristics of the periods studied.

As part of an historical enquiry, analyse and evaluate a range of appropriate source material with discrimination.

Level 1 Consistent and developed comparison of the key issue with a balanced and well-supported judgement. There will be little or no unevenness.

Focused use of a range of relevant historical concepts and context to address the key issue.

The answer is clearly structured and organised. Communicates coherently, accurately and effectively.

13-14

Focused comparative analysis. Controlled and discriminating evaluation of content and provenance, whether integrated or treated separately.

Evaluates using a range of relevant provenance points in relation to the sources and question. There is a thorough but not necessarily exhaustive exploration of these.

15-16 Level 2 Largely comparative evaluation

of the key issue with a balanced and supported judgement. There may be a little unevenness in parts.

Focused use of some relevant historical context with a good conceptual understanding to address the key issue.

The answer is well structured and organised. Communicates clearly.

11-12

Relevant comparative analysis of content and evaluation of provenance but there may be some unevenness in coverage or control.

Source evaluation is reasonably full and appropriate but lacks completeness on the issues raised by the sources in the light of the question.

13-14 Level 3 Some comparison linked to the

key issue. Is aware of some similarity and/or difference. Judgements may be limited and/or inconsistent with the analysis made.

Some use of relevant historical concepts and contexts but uneven understanding. Inconsistent focus on the key

Provides a comparison but there is unevenness, confining the comparison to the second half of the answer or simply to a concluding paragraph. Either the focus is on content or provenance, rarely both.

Source evaluation is partial and it is likely that the provenance itself is not compared, may be

2

Page 187: History A Mark Schemes for the Units January 2010 - The ...

F963/02 Mark Scheme June 2011

issue. The answer has some structure

and organisation but there is also some description. Communication may be clear but may not be consistent.

9-10

undeveloped or merely commented on discretely.

10-12 Level 4 Some general comparison but

undeveloped with some assertion, description and/or narrative. Judgement is unlikely, unconvincing or asserted.

A general sense of historical concepts and context but understanding is partial or limited, with some tangential and/or irrelevant evidence.

Structure may be rather disorganised with some unclear sections. Communication is satisfactory but with some inaccuracy of expression.

7-8

Attempts a comparison but most of the comment is sequential. Imparts content or provenance rather than using it.

Comparative comments are few or only partially developed, often asserted and/or ‘stock’ in approach.

8-9 Level 5 Limited comparison with few links

to the key issue. Imparts generalised comment and /or a weak understanding of the key points. The answer lacks judgement or makes a basic assertion.

Basic, often inaccurate or irrelevant historical context and conceptual understanding.

Structure lacks organisation with weak or basic communication.

5-6

Identifies some comparative points but is very sequential and perhaps implicit

Comment on the sources is basic, general, undeveloped or juxtaposed, often through poorly understood quotation.

6-7 Level 6 Comparison is minimal and basic

with very limited links to the key issue. Mainly paraphrase and description with very limited understanding. There is no judgement.

Irrelevant and inaccurate concepts and context.

Has little organisation or structure with very weak communication.

3-4

Little attempt to compare. Weak commentary on one or two undeveloped points, with basic paraphrase. Sequencing is characteristic.

Comments on individual sources are generalised and confused.

.

3-5

3

Page 188: History A Mark Schemes for the Units January 2010 - The ...

F963/02 Mark Scheme June 2011

Level 7 Fragmentary, descriptive, incomplete and with few or no links to the key issue. There is little or no understanding. Much irrelevance.

Weak or non existent context with no conceptual understanding.

No structure with extremely weak communication.

0-2

No attempt to compare either content or provenance with fragmentary, brief or inaccurate comment.

Makes no attempt to use any aspects of the sources.

0-2

4

Page 189: History A Mark Schemes for the Units January 2010 - The ...

F963/02 Mark Scheme June 2011

Question (b) Maximum mark 70 A01a and b AO2a and b

1 20-22 42-48

2 17-19 35-41

3 13-16 28-34

4 9-12 21-27

5 6-8 14-20

6 3-5 7-13

7 0-2 0-6

Notes related to Part B:

(iv) Allocate marks to the most appropriate level for each AO (v) If several marks are available in a box, work from the top mark down until the best fit

has been found (vi) Many answers will not be at the same level for each AO

5

Page 190: History A Mark Schemes for the Units January 2010 - The ...

F963/02 Mark Scheme June 2011

AOs A0Ia and b AO2a and b

Total mark for the question = 70

Recall, select and deploy historical knowledge appropriately, and communicate knowledge and understanding of history in a clear and effective manner. Demonstrate understanding of the past through explanation, analysis and arriving at substantiated judgements of:- key concepts such as causation, consequence, continuity, change and significance within an historical context; - the relationships between key features and characteristics of the periods studied.

As part of an historical enquiry, analyse and evaluate a range of appropriate source material with discrimination. Analyse and evaluate, in relation to the historical context, how aspects of the past have been interpreted and represented in different ways.

Level 1 Convincing analysis and argument with developed explanation leading to careful, supported and persuasive judgement arising from a consideration of both content and provenance. There may be a little unevenness at the bottom of the level.

Sharply focused use and control of a range of reliable evidence to confirm, qualify, extend or question the sources.

Coherent organised structure. Accurate and effective communication.

20-22

A carefully grouped and comparative evaluation of all the sources with effective levels of discrimination sharply focused on the interpretation.

Analyses and evaluates the strengths, limitations and utility of the sources in relation to the interpretation. Uses and cross references points in individual or grouped sources to support or refute an interpretation.

Integrates sources with contextual knowledge in analysis and evaluation and is convincing in most respects. Has synthesis within the argument through most of the answer.

42-48

Level 2 Good attempt at focused analysis, argument and explanation leading to a supported judgement that is based on the use of most of the content and provenance.

A focused use of relevant evidence to put the sources into context.

Mostly coherent structure and organisation if uneven in parts. Good communication.

Grouped analysis and use of most of the sources with good levels of discrimination and a reasonable focus on the interpretation.

Analyses and evaluates some of the strengths and limitations of the sources in relation to the interpretation. May focus more on individual sources within a grouping, so cross referencing may be less frequent.

Some, perhaps less balanced, integration of sources and contextual knowledge to analyse and evaluate the interpretation. Synthesis of the skills may be less developed.

6

Page 191: History A Mark Schemes for the Units January 2010 - The ...

F963/02 Mark Scheme June 2011

17-19

The analysis and evaluation is reasonably convincing.

35-41

Level 3 Mainly sound analysis, argument and explanation, but there may be some description and unevenness. Judgement may be incomplete or inconsistent with the analysis of content and provenance.

Some relevant evidence but less effectively used and may not be extensive.

Reasonably coherent structure and organisation but uneven. Reasonable communication.

13-16

Some grouping although not sustained or developed. Sources are mainly approached discretely with limited cross reference. Their use is less developed and may, in parts, lose focus on the interpretation. There may be some description of content and provenance.

Is aware of some of the limitations of the sources, individually or as a group, but mostly uses them for reference and to illustrate an argument rather than analysing and evaluating them as evidence. There is little cross referencing.

There may be unevenness in using knowledge in relation to the sources. Synthesis may be patchy or bolted on. Analysis and evaluation are only partially convincing.

28-34

Level 4 Attempts some analysis, argument and explanation but underdeveloped and not always linked to the question. There will be more assertion, description and narrative. Judgements are less substantiated and much less convincing.

Some relevant evidence is deployed, but evidence will vary in accuracy, relevance and extent. It may be generalised or tangential.

Structure is less organised, communication less clear and some inaccuracies of expression.

9-12

Sources are discussed discretely and largely sequentially, perhaps within very basic groups. Loses focus on the interpretation. The sources are frequently described.

May mention some limitations of individual sources but largely uses them for reference and illustration. Cross referencing is unlikely.

An imbalance and lack of integration between sources and knowledge often with discrete sections. There is little synthesis. Analysis and explanation may be muddled and unconvincing in part.

21-27

Level 5 Little argument or explanation, inaccurate understanding of the issues and concepts. The answer lacks judgement.

Limited use of relevant

A limited attempt to use the sources or discriminate between them. The approach is very sequential and referential, with much description. Points are

7

Page 192: History A Mark Schemes for the Units January 2010 - The ...

F963/02 Mark Scheme June 2011

evidence or context which is largely inaccurate or irrelevant.

Structure is disorganised, communication basic and the sense not always clear.

5-8

undeveloped. There is little attempt to analyse,

explain or use the sources in relation to the question. Comment may be general.

There is a marked imbalance with no synthesis. Analysis and explanation are rare and comments are unconvincing. 14-20

Level 6 There is very little explanation or understanding. Largely assertion, description and narrative with no judgement. Extremely limited relevance to the question.

Evidence is basic, generalised, patchy, inaccurate or irrelevant.

Little organisation or structure with poor communication.

3-4

Very weak and partial use of the sources for the question. No focus on interpretation.

A very weak, general and paraphrased use of source content.

No synthesis or balance. Comments are entirely unconvincing.

7-13 Level 7 No argument or explanation.

Fragmentary and descriptive with no relevance to the question.

No understanding underpins what little use is made of evidence or context.

Disorganised and partial with weak communication and expression.

0-2

Little application of the sources to the question with inaccuracies and irrelevant comment. Fragmentary and heavily descriptive.

No attempt to use any aspect of the sources appropriately.

No contextual knowledge, synthesis or balance. There is no attempt to convince.

0-6

8

Page 193: History A Mark Schemes for the Units January 2010 - The ...

F963/02 Mark Scheme June 2011

The Condition of England 1815-1853 1 (a) Study Sources B and C

Compare these Sources as evidence for the causes of the agricultural disturbances in 1830-31 (the Swing riots). [30]

No set answer is expected, but candidates need to compare the contents, evaluating such matters as authorship, dating, utility and reliability, so using the sources ‘as evidence for…’. The headings and attributions should aid evaluation and reference to both is expected in a good answer. The context is the widespread riots (overturning hedges, burning ricks, looting property and destroying machinery) that spread across southern and eastern England in 1830 -31, thoroughly alarming the authorities, the last great labourer’s revolt against agrarian change and conditions. Both sources suggest reliance by the poor on the poor rate and suggest that it has become a feature of their life. For Hudson’s interviewees (C)this is a matter of regret and the Poor Law is seen as one of the several causes of disturbance (seasonal sacking using the Poor Law to keep them over the winter). For the report, (B), it is also a cause for concern. There is some implied support from Hudson (C) for the Report (B) on the reliance of the poor on allowances as the Report (B) argues that it enabled labourers to be supported at very low ratepayer cost as a pool of continuing cheap labour when the need arose (planting and harvesting). For Hudson this is just one example of the poverty and oppression that he sees as at the root of the disturbances. However the differences outweigh the similarities. The Report (B) blames the relative generosity of the poor rates in the disturbed areas, arguing that it produced sturdy and insolent idlers who regarded such payments as a right. It implies the disturbances occurred through to increase such inappropriate rights and payments, - ‘disappointment and hatred if the expectation is not fulfilled’. It argues that the rioters believed that the authorities were withholding their benefits either through an unwillingness to dig into their own pockets (‘greed’) or through corruption. Hudson in C makes no reference to such generosity and mentions the allowance system only as a secondary factor in the riots. There are no sturdy and insolent poor angry at insufficient handouts in his account. Instead his focus is on desperation from acute poverty and oppression, implying that in disturbed areas the authorities were far from generous. He identifies three key factors – very low wages, technological threats to employment (threshing machines) and the persistence of seasonal dependence and underemployment which, in the winter months, forced a subsistence on the Poor Law, something that the young, elderly and weak had to face throughout the year. Thus there are two views of the poor here – a weak and desperate poor, already forced to commit low level crime (poaching), and a sturdy and aggressive poor prepared to take what they arrogantly believed to be theirs. The provenance of the sources is different. Hudson (C) bases his account on the memories of some of the rural poor themselves, albeit remembered long after.. It is an account of a lost way of life, remembered in bitterness and with sympathy by Hudson whose tone betrays these emotions (‘spiritless slaves’). His evidence is from one county, Wiltshire, and from labourers likely to be at the lower end of employment. He blames the oppression of the farmers and landowners but his organisation of points is convincing and is corroborated by other rural evidence, particularly in reference to machinery, enclosures and wages. In contrast the Report (B) is famous for its utilitarian, a priori and partial methodology. Its purpose was not to cast light on a vanished world but to change the way poverty was dealt with. Its commissioners, propertied and educated, often strangers to the areas they descended upon, were looking for evidence on out of control allowances and a consequently deleterious impact on the character of the poor; it was concerned to use the riots as evidence of this and to link outbreaks to high poor rates. It uses selective evidence

9

Page 194: History A Mark Schemes for the Units January 2010 - The ...

F963/02 Mark Scheme June 2011

to conclude that the Old Poor Law caused the riots and would have seized on C’s comments on the young, weak and elderly. In judgement candidates may well see Hudson’s evidence as more nuanced, although there is some evidence of the abuse of the poor rates to support the Report. It is true that the Speenhamland areas, on the whole, saw the main disturbances but these were also the areas of most rural change with the least option of urban factory work. The North had higher wages. This would suggest that the evidence in Hudson, at a time of life when labourers would reflect without fear of the consequences, is more accurate, although we are not told whether he asked leading questions of those he interviewed.

10

Page 195: History A Mark Schemes for the Units January 2010 - The ...

F963/02 Mark Scheme June 2011

1 (b) Study all the Sources. Use your own knowledge to assess how far the Sources support the interpretation that the most important reason for the reform of the Old Poor Law was its demoralising impact upon the poor. [70]

Successful answers will need to make use of all five Sources, testing them against contextual knowledge and evaluating their strengths and weaknesses, including any limitations as evidence. A range of issues may be addressed in focusing upon the terms of the question but no set conclusion is expected. A variety of factors may be considered of importance in deciding the fate of the Old Poor law, including the assertion in the question. In addition there were fears over the rising costs, that the Laws were no longer the guarantor of law and order, new economists argued that they constituted a barrier to a free labour market and the Benthamites who stressed both the negative moral dimensions of the old system and its administrative inefficiencies. All these views are evident in the sources but candidates will need to prioritise their relative importance in the debate. Three sources (A, B and E) come from opponents of the Old Law, (a northern overseer in A, the condemnatory Poor Law Report in B, based on some rather selective evidence and approaches, and an Assistant Commissioner in E, although much of the extract here is a conversation intended to ridicule a local official whose view is that the poor and their problems were the way of the world). Candidates could view these as highly selective but they were influential in moulding propertied and governmental opinion. Two sources support the Old Law (C and D) ,to a greater or lesser degree, although Hudson in C considers that wages are so low that a Poor law can only scratch the surface, whilst the local official in E takes the view that all need a little land and a cow and until such time bread and wage handouts are inevitable. The MP in D affirms the right of the poor to some assistance and tries to counter the view that they are thus dependent and demoralised, although such views were not necessarily very typical amongst the governing classes. The view that the Old Poor Law demoralised can be found in various forms in A, B, C (from different perspectives) and E. Sources A, B and E all come from those under the influence of Chadwick and the Benthamites who, taking their cue from Malthus, were worried that the Old Laws had become a cause of poverty rather than a solution to it. They stressed dependence and large families. Thus the overseer in A attacks the allowance system for de-incentivising the poor. The example he quotes, (a solitary one), is able to obtain well above the going wage rate in the rural South (55p as opposed to 35p) because of his large family and higher bread prices, both considered when his allowance was worked out. The overseer stresses that this is one reason for large families amongst the poor. He collects such evidence and is concerned to use it to question the existing system. The Assistant Commissioner in E corroborates this by quoting, with patronising concern, a local ratepayer who considers it natural and a God given right to procreate. The provenance of this might suggest that such a view on demoralisation was less widely held in local society and care needs to be taken not to overestimate the influence of Benthamite views. In the Report (B) the undermining of good work habits is also picked up, the emphasis is on a change of character from deferential and hard working to sturdy, insolent and belligerent concerned with handouts as of ‘right’. Source C, from a much later period, takes a different view of demoralisation. For Hudson it is the demoralisation of poverty, low wage and underemployment. We find little example of generous allowances in his account of the rural areas. Candiates could stress either the untypicality of A, B and E (the methodology of the Report and those involved with it – leading questions etc) or their relative influence on government thinking and thus the importance of demoralisation in the debate. Another nail in the coffin of the Old Law was cost. The sources, in stressing the generosity of the allowance system as the Overseer in A does, imply rising costs and candidates may well look at the rise of the poor rate in the period to 1834 and the pressures on government

11

Page 196: History A Mark Schemes for the Units January 2010 - The ...

F963/02 Mark Scheme June 2011

to reduce it from the propertied classes. Scrope in D refers to this when commenting on the rate payers, whom he considers excessively penny pinching. However if this was the case, and Hudson in C confirms it with his comments on hard masters, then in the rural areas the squeeze on costs was already underway. Nonetheless governments might well not listen to Vestry opinion. Scrope’s comments on them (D) betray a bias against such attitudes and this may well be shared by larger landowners. The Old Poor Law had always been seen as a guarantor of stability but Sources B and C, would suggest this was no longer the case. The Report in B is especially worried that the poor are demanding their ‘rights’ under the old system and that Speenhamland, devised as a temporary expedient during the revolutionary wars in the 1790s, was now seen as permanent. If the comments in the Report (B) are accepted then the Poor Law had caused the riots not prevented them. Scrope’s language in D and his discussion of the poor’s rights in Parliament would seem to corroborate this, although his is a very different perspective, comparing their rights equally to those of the propertied and titled and fully endorsing them. So too does the local vestry official quoted in E. He goes as far as to talk of ancient rights to a little land and a cow, alongside a natural order that welcomed children as part of God’s plan. Candidates could comment on the impact of such traditional ideas in the 1830s when change and new intellectual views challenged them. Scrope is making Parliament aware that abolition of outdoor relief runs the risk of breaking traditional ‘compacts’ based on natural justice and he warns of the consequences. Nonetheless candidates need to question his typicality within Parliament, and thus the weight they would allow this source. Other issues would be the increasing acceptance of the need to achieve a free labour market in early industrial society and the perception that the Old Poor Law was an obstacle to this. Candidates may mention Ricardo and the new economists who took a dim view of the Poor Law settlement laws which prescribed that relief could only be administered in the parish of birth, thus discouraging labour mobility and the chance of the poor moving to new areas of employment. The Overseer in A makes this point when citing Robert Smith’s reluctance to move outside his parish. In addition the Benthamites stressed the administrative inefficiencies of the Old Law and the sources amply illustrate this, albeit from a slanted angle. They clearly despise the ignorance and parochial nature of the Vestries, responsible for presiding over a haphazard system. The Report (B) refers to corruption and inefficiency, whilst Hudson in C, from a different perspective, condemns a system that sacks men after the harvest and allows the poor rate to keep them through the winter. Candidates may well know this was condemned as the larger landowners were enabled to keep a pool of cheap labour at the expense of the smaller farmers and local shopkeepers. The Assistant Commissioner writing in E to Chadwick, the main antagonist of the Old Law, mentions disapprovingly the parochial attitudes of a ratepayer, Vestry voter and tenant who chose those who administered the Old Poor Law. Candidates may well conclude that the official voices in the sources (Overseers and Commissioners) were the most influential and their critique of a demoralised and rebellious poor the key factor but costs and administrative problems are also evident.

12

Page 197: History A Mark Schemes for the Units January 2010 - The ...

F963/02 Mark Scheme June 2011

The Age of Gladstone and Disraeli 1865-1886 2 (a) Study Sources D and E.

Compare these sources as evidence for attitudes to British achievements at the Congress of Berlin in 1878. [30]

No set answer is expected, but candidates need to compare the contents, evaluating such matters as authorship, dating, utility and reliability, so using the sources ‘as evidence for…’. The headings and attributions should aid evaluation and reference to both is expected in a good answer. The sources are from the two main protagonists, Gladstone in D and Disraeli in E so the differences are more evident than the similarities. The only achievement that they can agree upon is that Britain’s material strength is regarded as impressive by her opponents and even here Gladstone only very begrudgingly acknowledges this; given his dislike of Disraelian sabre rattling. On all else the achievements at Berlin are disputed. Gladstone considers Britain’s moral respect to have been lessened given Disraeli’s lack of interest in the Balkan peoples but Disraeli stresses that he has had the interests of the subject peoples at heart, considering their conciliation vital to the peace of the area – ‘improve the condition of its subjects’. Gladstone thinks that Disraeli and Salisbury have done deals with Europe and fallen too much under their self interested sway, whereas the latter disagrees, stressing the twofold set of principles he held to, one of which was Ottoman integrity the other a peaceful solution. Gladstone dismisses Disraeli’s pursuit of interests as illusory but Disraeli rebuts this by stressing the over- riding interest of maintaining the Ottomans, something he feels he has achieved with the dismemberment of the large Bulgaria of San Stephano. To Gladstone there is no principle but Disraeli stresses precisely these, perhaps as a means of irritating Gladstone. On provenance candidates could point to the context. In 1878 Gladstone had been sidelined on the Eastern Question having initially seized the initiative over the Bulgarian massacres. Once Russia declared war British opinion changed and Disraeli was able to pose as the defender of British interests. In D Gladstone is still seeking to capitalise on the moral issue, perhaps with one eye on the nonconformists and Christian opinion in general, drawing attention to the re- imposition of Ottoman control over Macedonia and Eastern Rumelia, hence Disraeli’s reference to a concern for their interests. However Disraeli was now safe in the Lords (Earl of Beaconsfield) and can seek to gloss his achievements, confident in the knowledge that public and royal opinion had backed him on the issue. He can point to both European respect and to peace, thus taking the wind out of Gladstone’s sails. He certainly touches, no doubt annoyingly, the Gladstonian buttons of principle and peace. As to judgement , Gladstone’s was a minority view on British achievements at Berlin, although some candidates might question exactly what Disraeli achieved for the subject peoples there. His previous record would suggest that he prevented European pressure being effectively applied to the Ottomans to persuade them to implement reform and toleration in the Balkan provinces.

13

Page 198: History A Mark Schemes for the Units January 2010 - The ...

F963/02 Mark Scheme June 2011

2 (b) Study all the Sources Use your own knowledge to assess how far the Sources support the interpretation that Disraeli pursued irresponsible policies on the Eastern Question during the period from 1871 to 1878. [70]

Successful answers will need to make use of all five Sources, testing them against contextual knowledge and evaluating their strengths and weaknesses, including any limitations as evidence. A range of issues may be addressed in focusing upon the terms of the question but no set conclusion is expected. Gladstone and many highly placed Tories like Derby and Salisbury were convinced that Disraeli did pursue irresponsible policies on eastern issues but, from 1877 to 1878, popular and royal opinion saw his policies as both responsible and as representing a return to the prestige of the Palmerston years. The sources represent these opposing views, Gladstone in A and D (at the beginning and end of the 1871 -1878 period) and Derby in C being critical, Disraeli in E maintaining a responsible approach and Barrington’s report in B of a private conversation with Disraeli perhaps providing a more critical yet friendly approach to Eastern policy. Together they provide a broad chronological survey of policy in the 1870s and candidates will need to put them into their appropriate context. The accusation of irresponsibility is best seen in Sources A, C and D, although one reading of Barrington in B lends weight to the charge that he overestimated the ‘treachery’ and ambition of the Russians and might well have done better to work more closely with Russia on imposing a settlement on a disturbed Ottoman Empire in the 1874-6 period if he sought to avoid war. Gladstone in A might be expected to take a dim view of Disraeli’s views yet his analysis of Disraeli’s policies is convincing. He is responding to a speech that Disraeli made attacking the London Conference, which Gladstone had retrospectively called to give international ‘approval’ for unilateral Russian action in abrogating the Black Sea clauses during the Franco-Prussian war of 1870 (forbidding Russian warships access to their own naval backyard). He was in a position to know that they had never been considered permanent and he rightly points out that Disraelian action was impossible given the deals other powers had made with Russia. Britain would have to fight alone without a European ally. As a naval power this was ridiculous. Disraeli’s propensity to threaten a war that was unlikely to be won or prove smooth is corroborated in Source C by Derby. He writes with horror to Salisbury about Disraeli’s sabre rattling. He is aware that Disraeli doesn’t want war with Russia but he mistrusts the policy of military threats that could easily become war. Such views could be supported by the outbreak of jingoism and Disraeli’s gestures of sending the fleet to the Dardanelles or plans to occupy the Gallipoli peninsula, and his playing up to war fever. Source C is telling evidence from his closest colleague, the Foreign Secretary, who was to resign over the issue by 1877. His view in C is that Disraeli was too obsessed with chimera like prestige and Empire to be realistic about cutting a diplomatic deal with Russia. His irresponsible posturing prevented this. However Derby himself took an independent line and eventually joined the liberals in 1880. Temperamentally he was more inclined to a peaceful solution, as was his colleague Lord Salisbury, whose work before and behind the scenes at the Congress of Berlin helped Disraeli appear triumphant. Candidates might refer to these deals to put Disraeli’s policy at Berlin into perspective. His behaviour there was gauche and irresponsible - threatening to leave - and more concerned with prestige. Gladstone in D, with much less ammunition than in A, again tries to suggest that Disraeli ignored real British interests in protecting the weak, a minor point for most by 1878. He was on stronger ground when referring to ‘imaginary interests’ and candidates could consider the wisdom of continuing to support a decaying Turkey through a military alliance, especially the guarantee Britain gave to maintain Asian Turkey, an all but impossible and unrealistic commitment, taken on board as a quid pro quo for Cyprus ( all part of his imperial string of naval bases to protect the route to India, soon

14

Page 199: History A Mark Schemes for the Units January 2010 - The ...

F963/02 Mark Scheme June 2011

redundant once Alexandria was acquired). Candidates could comment on Disraeli’s apparent lack of responsibility over the Bulgarian horrors prior to this. The alternative view, that Disraeli took a responsible view on the Eastern Question is to be found by questioning the reliability of A, C and D and by taking seriously the evidence in B and E. It could be argued that he was right to be sceptical about the atrocity stories in the Balkans and he certainly feared, probably rightly, that it would enflame Russian passions and make war against Turkey more likely. In Source B Disraeli speculates in private with a friend and colleague, in E he glosses and explains his work at Berlin, after the event. In opposition Disraeli would appear to have been irresponsible in demanding action be taken over the Black Sea clauses but Derby in C perhaps underestimates the use of bluff (arguably the fleet was in no fit state to make much of a show) to get to the negotiating table, whilst Gladstone in D is clearly clutching at ancient straws (Bulgarian atrocities). Barrington in B indicates that Disraeli had a clear idea of the need to stop Russian domination of the Ottomans and that a simple carve up along the lines of Russian control at Constantinople and British control of Egypt would ultimately prove counter productive. Whether or not Constantinople was the key to India was and is a moot point. In contrast to Derby’s view of Disraeli’s jingoism, Barrington in B at least in 1876, comments that Disraeli was ‘guarded’ as to peace or war and stresses his ‘grand object is victory in diplomacy’. Nonetheless, by 1877 and San Stephano, Disraeli, as promised to Barrington, has treated Russian entry to Bulgaria very seriously. His policy was responsibly firm. He was determined to uphold traditional British policy in the area. Source E sees Disraeli justify himself in relation to a tradition that he has become the triumphant heir to, but was he wise to do so? He was aided by the horrified reaction of the other European powers to San Stephano which made Salisbury’s deals at Berlin easier. Conclusions will largely be determined by a convincing evaluation of the sources, with much weight being put on Gladstone in A, Barrington in B and to the chronology of Eastern policy in the 1870s.

15

Page 200: History A Mark Schemes for the Units January 2010 - The ...

F963/02 Mark Scheme June 2011

3 (a) Study Sources C and D Compare these Sources as evidence for the relationship between Asquith and Lloyd George in 1916. [30]

No set answer is expected, but candidates need to compare the contents, evaluating such matters as authorship, dating, utility and reliability, so using the Sources ‘as evidence for …’. The headings and attributions should aid evaluation and reference to both is expected in a good answer. Their view on how to direct the war is central. Both sources recognise the importance of Asquith’s reluctance to accept the proposals to change the War Committee as the source of conflict between him and Lloyd George. In Source C this is made explicit in the second sentence and, later, Asquith makes it clear that the proposals risk ‘undermining my own authority’. The entry for December 2nd in Source D explains that Asquith refused the proposals to preserve his influence and control. It is also apparent that neither man respects the other. Whilst acknowledging Lloyd George’s ‘many qualities’ Asquith, in Source C, nonetheless, considers Lloyd George to be untrustworthy. In return, it is claimed in Source D that Lloyd George thought Asquith was ‘absolutely hopeless’. Bonar Law was an important figure without whose support each would be weaker in relation to the other. In Source C Asquith states how he appreciates his ‘loyalty’ absolving him of any treachery. In Source D, it appears that Lloyd George does not feel strong enough to act without the support of Bonar Law. A key point of difference between the sources is the motives ascribed to Lloyd George. Asquith is suspicious of Lloyd George who, in Source C, he accuses of wanting to ‘displace me’, simply, it implies, to satisfy personal ambition. By contrast, in the entry for 30th November in Source D, Lloyd George is motivated by the obligation to serve his country responding to the call of the people. However, some may read the first line of the same entry as similar to Source C in suggesting selfish ambitions by Lloyd George. The reliability of the sources is questionable. Asquith, in Source C, is defending his stance by attacking Lloyd George whom he regarded as responsible for the situation. In contrast, in Source D, Stevenson is naturally protective of her lover. Indeed, the comments in Source D are not explicitly those of Lloyd George and their veracity might be challenged. Candidates may consider the reliability of the sources by another approach. Both sources are confidential: the opening line of Source C makes this point and Source D was clearly intended to be private. As such both may be regarded as the free expression of the writers’ views although Asquith could not be sure his letter would remain secret given Bonar Law’s association with Lloyd George and Stevenson’s diary was clearly published later, an event the authoress may have anticipated. Yet, Asquith’s charge against Lloyd George was an assumption, never proved, and Stevenson’s account about the War Committee (Source D) and Asquith’s response to it (Source C) fits the facts. Something might be made of the dates. Source C was written immediately after the first moves in the crisis whereas the last two entries in Source D were reflections at a time when the crisis had deepened. Judgement will be a matter of weighing up these provenance factors and many will conclude that they are both useful in terms of the perspective taken.

16

Page 201: History A Mark Schemes for the Units January 2010 - The ...

F963/02 Mark Scheme June 2011

3 (b) Study all the Sources Use your own knowledge to assess how far the Sources support the interpretation that Asquith should bear the major responsibility for splitting the Liberal Party in 1916. [70]

Successful answers will need to make use of all five Sources, testing them against contextual knowledge and evaluating their strengths and weaknesses, any limitations as evidence. A range of issues may be addressed in focusing upon the terms of the question but no set conclusion is expected. Elements of support for Asquith’s role can be found in all the sources but mainly in Source D. The alternative view, that Lloyd George and/or the Conservatives bear the major responsibility, can be found in Sources B, C, D and E with evidence that Asquith held the party together in Sources A and B. Most sources provide evidence on Asquith and some candidates might present the evidence thematically. A key charge against Asquith is that he lacked the qualities needed for effective leadership. Most damning in this respect is Source D where he is described as indecisive and to ‘have lost all will-power’. However, candidates should evaluate the source as unreliable to the extent that the authorship of the comment is ambiguous and, whether the views of Stevenson or Lloyd George, they represent the views of opponents of Asquith. In Source A Asquith cuts a sorry figure who ‘looked old and worried’ and, rather than provide direction, he appears dependent on his colleagues in so far as ‘He flung himself on our mercy’. However, the fact that Asquith received ‘an over-powering ovation’ suggests his speech had inspired his colleagues. Knowledge about Asquith’s formidable intellect and ability and also how effective he was as leader before 1914 may be known by candidates. Even Asquith, in Source C, admits to the ‘intolerable daily burden’ a comment candidates may evaluate as reliable given it is self-critical and knowledge of his weaknesses (drinking problems etc) could be added. Asquith might also be criticised for the formation of the Coalition in 1915. Source A emphasises the opposition within the ranks of the Liberal Party to it with some speaking ‘very strongly against a coalition’ although he could be defended as simply taking a position that ‘had become inevitable’. It is clear that Asquith did not want to join forces with the Conservatives who he describes as ‘his bitterest enemies’. Candidates may explain this reluctance as a consequence of the struggles of 1910-11 and more recent criticism by the Conservatives of the conduct of the war. The fact that the split did not occur till late 1916 suggests that the formation of the Coalition was not an immediate cause of the split at least. Nonetheless, Asquith’s failure to foresee the consequence of the Coalition in terms of the Press is picked up in Source E. Candidates might explain this reference: the Press now adopted the role of opposition which had been forfeit by the Tories when they joined the Coalition. Indeed, his naivety is suggested in the fact that he ‘informed Northcliffe of the shell shortage’. Over time, the Press was a factor ‘in the overthrow of Asquith’s administration’. Asquith’s war policy might be assessed. Source B portrays a Cabinet divided on conscription and Gallipoli and problems with the supply of munitions. Knowledge about each of these issues might be provided and some may add to the list details about events in Europe, then and later notably the Battle of the Somme, all of which weakened Asquith. However, candidates might excuse Asquith responsibility for these divisions and instead blame the failure of commanders and ministers: Lloyd George is said to have ‘muddled ... Munitions’. However, demands for the reform of the War Committee, mentioned in Sources C, D and E, hint at disappointment with Asquith’s conduct of the war and his refusal to concede could be viewed as culpability on his part. Knowledge of the discussions and letters between Asquith and Lloyd George could be discussed by those well versed in the subject. In evaluation candidates could argue that Asquith’s defensive position reflects a sensitivity and pride, revealed in Source C in his concern not to ‘undermine his own

17

Page 202: History A Mark Schemes for the Units January 2010 - The ...

F963/02 Mark Scheme June 2011

authority, even if the other two sources are treated with some suspicion given the position of their authors. Clearly, Lloyd George could be blamed for splitting the Liberal Party. In several sources Lloyd George’s actions are described as deliberately targeted on his attaining the top spot. As early as October 1915 his eye for an ‘opening to the leadership’ is identified, in Source B, which may be dismissed as the subjective view of one man but which suggests some observation of Lloyd George over time. In Source C Asquith is convinced that Lloyd George had ‘engineered’ things ‘with the purpose of displacing me’ but such a judgement might be dismissed as a ploy by Asquith to undermine the credibility of Lloyd George’s proposals. The implication of the comments for 30 November in Source D is that Lloyd George intended to ‘smash the government’ but this is the assessment of his mistress. In Source E ‘Churchill claims that the resignation of Lloyd George led to the fall of the government’ which might be used to argue that the latter bears responsibility for the split in the Party. However, it could be argued that Lloyd George had no alternative given the contradictory positions adopted by Asquith in his dialogue with Lloyd George on the reform of the War Committee. In all cases the evidence that Lloyd George conspired against Asquith is hard to verify. Strong candidates will make mention of other politicians. Liberals inside and out of the Cabinet, could be blamed for their reluctance to back Asquith fully or to put principle to one side at a time of national emergency. This could be substantiated with reference to Source A although the source explains that initial concerns about the Coalition were calmed by Asquith. The fragility of support for Asquith from colleagues is evident in Source C and their stubborn adherence to principle is revealed in Source B. Bonar Law might be blamed for siding with Lloyd George as indicated in Source E, the importance of his alliance with Lloyd George recognised in Source D. Although Bonar Law is portrayed as a reliable colleague in Source C this might be evaluated as insincere as a device to divide him from Lloyd George and Source B shows Bonar Law was prepared to resign over Gallipoli. In Source E Northcliffe is said to have ‘exercised a commanding influence’ and candidates may be aware of some of the devastating attacks made on the Coalition by the papers mentioned which confirms the influence they had even if it does not explain the split in the Liberal Party.

18

Page 203: History A Mark Schemes for the Units January 2010 - The ...

F963/02 Mark Scheme June 2011

4 (a) Study Sources B and D. Compare these Sources as evidence for the relationship between Churchill and Roosevelt. [30]

No set answer is expected, but candidates need to compare the contents, evaluating such matters as authorship, dating, utility and reliability, so using the Source ‘as evidence for…..’ The Headings and attributions should aid evaluation and reference to both is expected in a good answer. The issue here is whether the apparently warm relations between Churchill and Roosevelt which D exemplifies were borne out by the realities of US-British relations. B seems to offer a more hostile view. Roosevelt may have been concerned that the US should not be drawn into spending US lives in propping up the British Empire. Churchill worked very hard to charm and impress Roosevelt, but was not always successful in actually influencing US policy. Content: D refers to the help and comfort America brought. B makes it clear that the help was not extended to preserve the Empire with its ‘mediaeval’ ideas. There is no hint of this resentment by FDR in D. B does not refer to the ‘unsordid’ Lease Lend, but there is a direct reference to the Atlantic Charter – giving a very different interpretation. In B the Charter is something that Churchill might want to wriggle out of, by implication because of the Empire. In D the Charter is seen as part of a great cooperative effort by both sides (We drew up together the Atlantic Charter) although Churchill is careful to refer to its impact on ‘other peoples’ which may not include the subjects of the British Empire. There is little personal warmth expressed by FDR in B, and Churchill is not mentioned by name, whereas D is in the nature of an emotional tribute to a personal friendship (personal regard and affection) There is little reflection in D of the obvious concern that the US is not subordinated to Britain that appears in B. Roosevelt in B is referring to British interests in general; Churchill in D to a personal relationship based on US generosity. Nature : B is a reported conversation between father and son after FDR had made a considerable journey to an unfamiliar environment. It was at a difficult time in the war, when there had been a number of differences between the USA and Britain over strategy, Vichy France and the post-war world. D on the other hand was spoken when the war was nearly over. Whatever the differences, a massive Anglo-American expeditionary force had landed in France and was approaching final victory. Churchill’s hopes for US help had materialised and it was important that differences should be forgotten and ongoing US economic help and help against the Soviet threat should be continued. This was a public speech as opposed to a private conversation made in very different circumstances and with a different purpose. Judgment: Neither is entirely trustworthy or typical. FDR did have warmer relations with Churchill than this outburst suggests; Churchill had more problems with FDR than his eulogy suggests. Both are problematic – there is no corroboration that these were FDRs words and we have to accept Elliot’s version (Churchill was angry at the book and thought it misrepresented the facts). Similarly, in the emotional aftermath of FDR’s death, Churchill would not be likely to produce a balanced analysis. He was a highly emotional person and this was a very emotional tribute. Candidates are free to decide which offers the more typical and authentic view of the relationship.

19

Page 204: History A Mark Schemes for the Units January 2010 - The ...

F963/02 Mark Scheme June 2011

20

4 (b) Study all the Sources. Use your own knowledge to assess how far the Sources support the interpretation that relations between Britain and the USA were very successfully managed by Churchill between 1941 and 1945. [70]

Successful answers will need to make use of all five Sources, testing them against contextual evidence and evaluating their strengths and weaknesses, any limitations as evidence. A range of issues may be addressed in focusing upon the terms of the question but no set conclusion is expected. The debate here is whether a successful management of Churchill’s relations with the US is valid. Sources A, B and C suggest a failure to manage relations, or at least that Britain had to take a second seat to the US. Sources D and to some extent E implies more success based on Churchill’s assiduous personal diplomacy and management of Roosevelt. Of the sources here E attempts a balance – a special relationship but not as special as Churchill thought. D suggests the special personal relationship, but has to be viewed with some caution, given the circumstances and intentions of the tribute. B suggests more that Best is right to see the relationship as less special than Churchill supposed. It can be supported by US sympathy for Indian independence and an unwillingness to divert any resources to the recovery of British colonies taken by Japan. It can also be supported by the insistence of US military predominance in the second front and US refusal to press ahead to get to Berlin before Stalin. However, the nature of the Source does need to be considered critically as a hearsay account published some years afterwards. A is the German view which actually confirms the view in B that the US wanted to remain dominant. Obviously the purpose is to show Churchill in a humiliating way and hope for a rift between the allies. By the time of publication in 1943 the campaign in Russia had suffered a fatal blow. A second front was feared and the German people needed to be reassured that the Anglo-US alliance was fragile. This was actually widely held to be the case. Hitler is said to have believed that FDR’s death in April 1945 was a sign that Germany now had a chance of survival if the US split from its ally. However the cartoon also shows US economic power and actually FDR and Churchill are hand in hand. The cartoon is not however entirely fanciful as Lord Moran’s diary reveals Churchill’s pique at being treated as the junior partner. As Moran was close to Churchill and saw the immediate effect of Roosevelt’s snub, this is a useful source; but it does refer to a specific conference. The diaries were not published until after the war. They were seen as a breach of confidentiality, but do offer a unique view. Both sources show a Churchill out of control in his relations with the USA. Roosevelt was concerned that Churchill did not see the Atlantic Charter as having application to Britain; Churchill was concerned that FDR did not see that Stalin would have little interest in the principles of the Atlantic Charter in post-war expansion – hence his desperate deal with Stalin done independently of the US to establish spheres of influence in Eastern Europe in October 1944. Candidates could question the typicality of A and C and the sources as a whole do not have enough evidence of the good humour and warmth of the personal relationship between the leaders, for example in Churchill’s successful visit to Washington in December 1941 or his witty and well-received speech to Congress. His face to face meetings are said to have overcome FDR’s initial hostility to him when they first met and begun a remarkable partnership in which shared democratic ideals led to cooperation and victory. Candidates could point to Churchill’s success in persuading FDR to invade North Africa, in persuading him to delay D Day and then giving massive military aid – these could support the judicious view in E. However, there is enough material to support some truth in the more skeptical views in A, B and C. Churchill had to accept a hard bargain for any US help, no certainty of US intervention against Germany and then facing a lot of problems in relations with the USA which involved virtual appeasement of FDR. By 1943 Churchill could not prevent Roosevelt becoming closer to Stalin and failed to get US understanding of the threat from Stalin. Churchill had to accept US military leadership and the predominance of US Strategy.

Page 205: History A Mark Schemes for the Units January 2010 - The ...

OCR (Oxford Cambridge and RSA Examinations) 1 Hills Road Cambridge CB1 2EU OCR Customer Contact Centre 14 – 19 Qualifications (General) Telephone: 01223 553998 Facsimile: 01223 552627 Email: [email protected] www.ocr.org.uk For staff training purposes and as part of our quality assurance programme your call may be recorded or monitored

Oxford Cambridge and RSA Examinations is a Company Limited by Guarantee Registered in England Registered Office; 1 Hills Road, Cambridge, CB1 2EU Registered Company Number: 3484466 OCR is an exempt Charity OCR (Oxford Cambridge and RSA Examinations) Head office Telephone: 01223 552552 Facsimile: 01223 552553 © OCR 2011

Page 206: History A Mark Schemes for the Units January 2010 - The ...

Oxford Cambridge and RSA Examinations

GCE

Unit F963/02: Option B Modern 1815-1945

Advanced Subsidiary GCE

History A

Mark Scheme for June 2012

Page 207: History A Mark Schemes for the Units January 2010 - The ...

OCR (Oxford Cambridge and RSA) is a leading UK awarding body, providing a wide range of qualifications to meet the needs of candidates of all ages and abilities. OCR qualifications include AS/A Levels, Diplomas, GCSEs, OCR Nationals, Functional Skills, Key Skills, Entry Level qualifications, NVQs and vocational qualifications in areas such as IT, business, languages, teaching/training, administration and secretarial skills. It is also responsible for developing new specifications to meet national requirements and the needs of students and teachers. OCR is a not-for-profit organisation; any surplus made is invested back into the establishment to help towards the development of qualifications and support, which keep pace with the changing needs of today’s society. This mark scheme is published as an aid to teachers and students, to indicate the requirements of the examination. It shows the basis on which marks were awarded by examiners. It does not indicate the details of the discussions which took place at an examiners’ meeting before marking commenced. All examiners are instructed that alternative correct answers and unexpected approaches in candidates’ scripts must be given marks that fairly reflect the relevant knowledge and skills demonstrated. Mark schemes should be read in conjunction with the published question papers and the report on the examination. OCR will not enter into any discussion or correspondence in connection with this mark scheme. © OCR 2012 Any enquiries about publications should be addressed to: OCR Publications PO Box 5050 Annesley NOTTINGHAM NG15 0DL Telephone: 0870 770 6622 Facsimile: 01223 552610 E-mail: [email protected]

Page 208: History A Mark Schemes for the Units January 2010 - The ...

F963/02 Mark Scheme June 2012

Subject-specific Marking Instructions that apply across the whole question paper to be included here. Question (a) Maximum mark 30

A01a and b AO2a

1 13-14 15-16

2 11-12 13-14

3 9-10 10-12

4 7-8 8-9

5 5-6 6-7

6 3-4 3-5

7 0-2 0-2

Notes related to Part A: (i) Allocate marks to the most appropriate level for each AO (ii) If several marks are available in a box, work from the top mark down until the best fit has been found (iii) Many answers will not be at the same level for each AO

1

Page 209: History A Mark Schemes for the Units January 2010 - The ...

F963/02 Mark Scheme June 2012

Marking Grid for Question (a)

AOs AO1a and b AO2a Total for each question =30

Recall, select and deploy historical knowledge appropriately, and communicate knowledge and understanding of history in a clear and effective manner. Demonstrate understanding of the past through explanation, analysis and arriving at substantiated judgements of:

- key concepts such as causation, consequence, continuity, change and significance within an historical context;

- the relationships between key features and characteristics of the periods studied.

As part of an historical enquiry, analyse and evaluate a range of appropriate source material with discrimination.

Level 1 Consistent and developed comparison of the key issue with a balanced and well-supported judgement. There will be little or no unevenness.

Focused use of a range of relevant historical concepts and context to address the key issue.

The answer is clearly structured and organised. Communicates coherently, accurately and effectively.

13-14

Focused comparative analysis. Controlled and discriminating evaluation of content and provenance, whether integrated or treated separately.

Evaluates using a range of relevant provenance points in relation to the sources and question. There is a thorough but not necessarily exhaustive exploration of these.

15-16 Level 2 Largely comparative evaluation of the key issue with a

balanced and supported judgement. There may be a little unevenness in parts.

Focused use of some relevant historical context with a good conceptual understanding to address the key issue.

The answer is well structured and organised. Communicates clearly.

11-12

Relevant comparative analysis of content and evaluation of provenance but there may be some unevenness in coverage or control.

Source evaluation is reasonably full and appropriate but lacks completeness on the issues raised by the sources in the light of the question.

13-14

2

Page 210: History A Mark Schemes for the Units January 2010 - The ...

F963/02 Mark Scheme June 2012

AOs AO1a and b AO2a Level 3 Some comparison linked to the key issue. Is aware of

some similarity and/or difference. Judgements may be limited and/or inconsistent with the analysis made.

Some use of relevant historical concepts and contexts but uneven understanding. Inconsistent focus on the key issue.

The answer has some structure and organisation but there is also some description. Communication may be clear but may not be consistent.

9-10

Provides a comparison but there is unevenness, confining the comparison to the second half of the answer or simply to a concluding paragraph. Either the focus is on content or provenance, rarely both.

Source evaluation is partial and it is likely that the provenance itself is not compared, may be undeveloped or merely commented on discretely.

10-12 Level 4 Some general comparison but undeveloped with some

assertion, description and/or narrative. Judgement is unlikely, unconvincing or asserted.

A general sense of historical concepts and context but understanding is partial or limited, with some tangential and/or irrelevant evidence.

Structure may be rather disorganised with some unclear sections. Communication is satisfactory but with some inaccuracy of expression.

7-8

Attempts a comparison but most of the comment is sequential. Imparts content or provenance rather than using it.

Comparative comments are few or only partially developed, often asserted and/or ‘stock’ in approach.

8-9 Level 5 Limited comparison with few links to the key issue.

Imparts generalised comment and /or a weak understanding of the key points. The answer lacks judgement or makes a basic assertion.

Basic, often inaccurate or irrelevant historical context and conceptual understanding.

Structure lacks organisation with weak or basic communication.

5-6

Identifies some comparative points but is very sequential and perhaps implicit

Comment on the sources is basic, general, undeveloped or juxtaposed, often through poorly understood quotation.

6-7

3

Page 211: History A Mark Schemes for the Units January 2010 - The ...

F963/02 Mark Scheme June 2012

AOs AO1a and b AO2a Level 6 Comparison is minimal and basic with very limited links to

the key issue. Mainly paraphrase and description with very limited understanding. There is no judgement.

Irrelevant and inaccurate concepts and context. Has little organisation or structure with very weak

communication.

3-4

Little attempt to compare. Weak commentary on one or two undeveloped points, with basic paraphrase. Sequencing is characteristic.

Comments on individual sources are generalised and confused.

3-5 Level 7 Fragmentary, descriptive, incomplete and with few or no

links to the key issue. There is little or no understanding. Much irrelevance.

Weak or non existent context with no conceptual understanding.

No structure with extremely weak communication.

0-2

No attempt to compare either content or provenance with fragmentary, brief or inaccurate comment.

Makes no attempt to use any aspects of the sources.

0-2

4

Page 212: History A Mark Schemes for the Units January 2010 - The ...

F963/02 Mark Scheme June 2012

Question (b) Maximum mark 70

A01a and b AO2a and b

1 20-22 42-48

2 17-19 35-41

3 13-16 28-34

4 9-12 21-27

5 6-8 14-20

6 3-5 7-13

7 0-2 0-6

Notes related to Part B: (iv) Allocate marks to the most appropriate level for each AO (v) If several marks are available in a box, work from the top mark down until the best fit has been found (vi) Many answers will not be at the same level for each AO

5

Page 213: History A Mark Schemes for the Units January 2010 - The ...

F963/02 Mark Scheme June 2012

AOs AO1a and b AO2a and b

Total mark for the question = 70

Recall, select and deploy historical knowledge appropriately, and communicate knowledge and understanding of history in a clear and effective manner. Demonstrate understanding of the past through explanation, analysis and arriving at substantiated judgements of:

- key concepts such as causation, consequence, continuity, change and significance within an historical context;

- the relationships between key features and characteristics of the periods studied.

As part of an historical enquiry, analyse and evaluate a range of appropriate source material with discrimination. Analyse and evaluate, in relation to the historical context, how aspects of the past have been interpreted and represented in different ways.

Level 1 Convincing analysis and argument with developed explanation leading to careful, supported and persuasive judgement arising from a consideration of both content and provenance. There may be a little unevenness at the bottom of the level.

Sharply focused use and control of a range of reliable evidence to confirm, qualify, extend or question the sources.

Coherent organised structure. Accurate and effective communication.

20-22

A carefully grouped and comparative evaluation of all the sources with effective levels of discrimination sharply focused on the interpretation.

Analyses and evaluates the strengths, limitations and utility of the sources in relation to the interpretation. Uses and cross references points in individual or grouped sources to support or refute an interpretation.

Integrates sources with contextual knowledge in analysis and evaluation and is convincing in most respects. Has synthesis within the argument through most of the answer.

42-48

Level 2 Good attempt at focused analysis, argument and explanation leading to a supported judgement that is based on the use of most of the content and provenance.

A focused use of relevant evidence to put the sources into context.

Mostly coherent structure and organisation if uneven in parts. Good communication.

17-19

Grouped analysis and use of most of the sources with good levels of discrimination and a reasonable focus on the interpretation.

Analyses and evaluates some of the strengths and limitations of the sources in relation to the interpretation. May focus more on individual sources within a grouping, so cross referencing may be less frequent.

Some, perhaps less balanced, integration of sources and contextual knowledge to analyse and evaluate the interpretation. Synthesis of the skills may be less developed. The analysis and evaluation is reasonably convincing.

35-41

6

Page 214: History A Mark Schemes for the Units January 2010 - The ...

F963/02 Mark Scheme June 2012

AOs AO1a and b AO2a and b Level 3 Mainly sound analysis, argument and explanation, but

there may be some description and unevenness. Judgement may be incomplete or inconsistent with the analysis of content and provenance.

Some relevant evidence but less effectively used and may not be extensive.

Reasonably coherent structure and organisation but uneven. Reasonable communication.

13-16

Some grouping although not sustained or developed. Sources are mainly approached discretely with limited cross reference. Their use is less developed and may, in parts, lose focus on the interpretation. There may be some description of content and provenance.

Is aware of some of the limitations of the sources, individually or as a group, but mostly uses them for reference and to illustrate an argument rather than analysing and evaluating them as evidence. There is little cross referencing.

There may be unevenness in using knowledge in relation to the sources. Synthesis may be patchy or bolted on. Analysis and evaluation are only partially convincing.

28-34

Level 4 Attempts some analysis, argument and explanation but underdeveloped and not always linked to the question. There will be more assertion, description and narrative. Judgements are less substantiated and much less convincing.

Some relevant evidence is deployed, but evidence will vary in accuracy, relevance and extent. It may be generalised or tangential.

Structure is less organised, communication less clear and some inaccuracies of expression.

9-12

Sources are discussed discretely and largely sequentially, perhaps within very basic groups. Loses focus on the interpretation. The sources are frequently described.

May mention some limitations of individual sources but largely uses them for reference and illustration. Cross referencing is unlikely.

An imbalance and lack of integration between sources and knowledge often with discrete sections. There is little synthesis. Analysis and explanation may be muddled and unconvincing in part.

21-27

Level 5 Little argument or explanation, inaccurate understanding of the issues and concepts. The answer lacks judgement.

Limited use of relevant evidence or context which is largely inaccurate or irrelevant.

Structure is disorganised, communication basic and the sense not always clear.

5-8

A limited attempt to use the sources or discriminate between them. The approach is very sequential and referential, with much description. Points are undeveloped.

There is little attempt to analyse, explain or use the sources in relation to the question. Comment may be general.

There is a marked imbalance with no synthesis. Analysis and explanation are rare and comments are unconvincing.

14-20

7

Page 215: History A Mark Schemes for the Units January 2010 - The ...

F963/02 h me

8

AOs

Mark Sc

AO1a and b

e June 2012

AO2a and b Level 6 There is very little explanation or understanding. Largely

assertion, description and narrative with no judgement. Extremely limited relevance to the question.

Evidence is basic, generalised, patchy, inaccurate or irrelevant.

Little organisation or structure with poor communication.

3-4

Very weak and partial use of the sources for the question. No focus on interpretation.

A very weak, general and paraphrased use of source content. No synthesis or balance. Comments are entirely unconvincing.

7-13 Level 7 No argument or explanation. Fragmentary and

descriptive with no relevance to the question. No understanding underpins what little use is made of

evidence or context. Disorganised and partial with weak communication and

expression. 0-2

Little application of the sources to the question with inaccuracies and irrelevant comment. Fragmentary and heavily descriptive.

No attempt to use any aspect of the sources appropriately. No contextual knowledge, synthesis or balance. There is no

attempt to convince.

0-6

Page 216: History A Mark Schemes for the Units January 2010 - The ...

F963/02 Mark Scheme June 2012

Question Answer Marks Guidance 1 (a) The context is the increasing interest in public health issues in the 1840s. ‘1842’

saw the first big move towards raising awareness of public health with Chadwick’s famous Sanitary Report of that year (Source A); whilst by 1847 bills had been presented to parliament and had become government sponsored. Candidates may also be aware of the limited approach and jurisdictional authority of local authorities, even after the Municipal Corporations Act of 1835 opened many to ratepayer suffrage which, in the case of health, could limit what was done by a more rigorous electoral control over what could and was spent. Both reflect the need to do something and comment on the role of local authorities in this light. The similarities here outweigh the differences, partly because both come from sanitary reformers. They are convinced that local authorities (local corporations) are inadequate in their approach and there is a stress on the limited money and resources available to them. Chadwick in A comments on their inefficient operations whilst Guy in B goes further and points to their lack of sufficient scale (in terms of extended drainage and a lack of authority over a sufficiently large area to make a difference). Both stress the importance of drainage and the need for efficient administration through the appointment of salaried and professional officers of health, lacking according to both sources. The differences are ones of emphasis. Chadwick in A focuses on the revenue problem – rates are unequal in their distribution, of little value in practical terms (most would be absorbed in the new poor law rate) and inefficiently collected and administered. He promises economy and saving money, perhaps over optimistically. In contrast the problem for Guy in B is the local electoral system. Ratepayers were potentially removed from public health problems and were unlikely to appreciate the scale of action needed or be bold enough to demand action from their representatives. As a result Guy stresses the need for government appointment to sanitary office, not the local patronage that produced officers for whom local and political loyalties trumped health and administrative expertise. The provenance explains the critical approach to local authorities. Both sources are written by experts, both by committed sanitary reformers, especially Edwin Chadwick, for whom it had become something of a crusade in the 1840s. Both have an interest in stressing local authority inefficiency. Chadwick had become an object of local authority suspicion and hatred and he reciprocated. Both look to central government for the answers and the authority to over-ride local corporations,

30 Focus: Comparison of two Sources No set answer is expected, but candidates need to compare the contents, evaluating such matters as authorship, dating, utility and reliability, so using the Source ‘as evidence for…..’ The Headings and attributions should aid evaluation and reference to both is expected in a good answer.

9

Page 217: History A Mark Schemes for the Units January 2010 - The ...

F963/02 Mark Scheme June 2012

Question Answer Marks Guidance although Guy does so with some reluctance. The purpose of both is to persuade, although their audience differs – Chadwick in A is seeking to mould intelligent propertied opinion (and stresses economy and saving money). He is seeking official sanction (a Report that went beyond its remit and was only published to a wider audience at his own expense). Guy in B is addressing a more specialist audience via a journal, although it reflects a widening interest in health issues by intelligent public opinion. The author is predisposed to oppose centralisation and is at pains to state a problem – local authorities simply lack the boundaries to deal with a problem that is beyond them. This explains the emphasis on election. In terms of judgement both are equally valid in the points they make on local authority limitation but both generalise. Neither acknowledges the admittedly few authorities that were seizing the initiative (Liverpool for example). Some may consider Guy the better source as he focuses on the institutional limitations whereas Chadwick rather speciously claims that savings could be made in the Poor Law bill to compensate for the health expenditure he claims is so necessary.

(b) The Sources provide a variety of views on the issue of what provided the major impetus to tackle health in the 1840s. The question asserts the centrality of Chadwick, his work and efforts. However, there were other factors at work – the role of cholera in panicking the political classes; public awareness and the state of knowledge on the issue; the issue of local government reform and the impact of the industrial revolution on the health of towns and cities. All of the sources are, to a greater or lesser extent, predisposed towards public health reform. Two sources, A and B, are self appointed experts on the issue whilst C, D and E are laymen, looking critically at the unfolding debate and those involved in it. The view that Chadwick’s work was the key to what was achieved can be largely found in A, with some support from B and C. Source A is Chadwick himself and the provenance point here is obvious. This is an extract from his famous report of 1842, published under his own name and the product of his work as a poor law commissioner. By 1840, from poor law reports, he had concluded that the occupants of workhouses were often there because of poor health. Increasingly much of the continued expense was incurred in their role as unofficial local hospitals. Health was the key to the operation of a smooth and free labour market. However, he needed to convince both a suspicious public and government that the health of the poor

70 Focus: Judgement in context, based on a set of Sources and own knowledge. Successful answers will need to make use of all five Sources, testing them against contextual evidence and evaluating their strengths and weaknesses, any limitations as evidence. A range of issues may be addressed in focusing upon the terms of the question but no set conclusion is expected. The sources can be read/analysed in different ways and as part of their judgement candidates will need to appreciate this.

10

Page 218: History A Mark Schemes for the Units January 2010 - The ...

F963/02 Mark Scheme June 2012

Question Answer Marks Guidance affected all, both in terms of infectious disease and in its indirect impact on the pockets of the employing classes. His 1842 report arguably did precisely that. He asserted the causes of disease (wrongly as it turned out – miasma and atmospheric causes were to be disproved in the 1850s) and succinctly summarised them in the opening sentences of the source. His analogies are striking – annual loss of life was greater than in Britain’s wars. Local authorities were not up to the job. Savings were promised (again rather speciously – the claim is a future reduction of one thirtieth in expenditure provided an initial large outlay was made on new, water flushed, drainage). New civil servants were in fact an economy! Thus he provided solutions and savings, all in one go. His theories fitted the facts that he deployed and he could claim that he had been asked to investigate by the Whig government. This had a considerable impact at the time (a Health of Towns Association with branches all over Britain was set up) and Peel’s government was forced further to consider urban health. 100,000 copies of his report were sold. Candidates however could point to the assertions in the source on the causes of disease and to the existing interests that were implicitly condemned to oblivion – improvement commissioners and often powerful water companies would have to be wound up. The result of his report was a polarisation into ‘clean’ and ‘dirty’ parties that arguably hindered health reform. Guy in B lends some support to the view of Chadwick’s pivotal role by echoing his 1842 findings – on drainage, local authority inadequacy and the need for proper sanitary officials, but his is the view of a minority, albeit an increasing one, and he is careful to voice appropriate caveats as to centralisation and officialdom This was a recognition of the opposition’s case against Chadwick. The author in C (Fraser’s magazine), also gives cautious recognition of Chadwick’s centrality in the debate. It is in the form of a warning to Chadwick based on an awareness of his controversial poor law role – his tendency to doctrinaire approaches (a perceived inhumane Benthamite utilitarianism) and compulsion. The view that minimises Chadwick’s importance is to be found in a different reading of Chadwick in source A (see above), in combination with C, which points to negative public perceptions of him. A focus on increasing criticism of local authorities is to be found in B but Sources D and E stress that the main factor was fear of cholera. Sources D and E both provide telling evidence of the powerful fear of cholera, a fear far more concrete than from other epidemics (typhus) –arriving from Asia in 1830, it had not been seen before and government immediately took

11

Page 219: History A Mark Schemes for the Units January 2010 - The ...

F963/02 Mark Scheme June 2012

Question Answer Marks Guidance action to hold ships in quarantine and set up a temporary Board of Health to deal with the first outbreak in 1832. Opinion was divided on causes –victim contact or miasma (as Chadwick believed in Source A) – and on treatments. Clearly it was behind early public health reform in the early 1830s when there had been little by way of Chadwick’s statistics, solutions and advocacy (the product of the 1840s) and Dickens in Source E claims that it was again the key in the 1840s. With another epidemic in 1848/9 he argued cholera lay behind the Public Health Act of 1848 (this time a permanent Central Board with power to act if death rates climbed and to appoint an inspector). The delays after 1842 and the timing of the Act would confirm Dickens’ view. Similarly his point about bulletins ceasing when cholera abated and the ‘buttoning of pockets’ to pay for health measures is corroborated by events – the Board was to be for a trial period of 6 years only and was to be permissive. It was closed in 1854 and Chadwick dismissed. Shopkeepers, labourers and ratepayers resented interference and Chadwick made enemies, as Source C warned. He seemed incapable of realising that it would take time and cooperation for large cities to build arterial systems. Dickens was a persuasive and engaged campaigner. His evidence is compelling and is indirectly supported by Punch in Source D. This, too, highlights action as a result of cholera and interestingly makes what turned out to be the correct link between it and the water supply. In this sense Chadwick’s new Central Board was guilty of making it worse – flushing the London sewers into the Thames and encouraging the continuance of rivers, sources of drinking water, as dumping grounds for waste. However, Punch in Source D had no scientific backing for this claim in 1849. Another view is the general raising of health consciousness in the period, a point stressed in Source C in relation to local government inadequacy but also in the form of the sort of publicity provided by Punch in Source D and Dickens in Source E, both more popular and arguably influential than Chadwick. The latter was regarded with suspicion by populace and government alike. Sources like A and B, once the implications were realised, may well have been counter-productive. The satire of Punch (a well known verse with obvious and popular targets in the form of corrupt vested interests) and the adulation commanded by Dickens may lead to a conclusion that marginalises the role of Chadwick in favour of cholera and corruption. Alternatively the elegance and persuasiveness of Chadwick, not least the savings argument, could well form the basis for an acceptance of the view in the question.

12

Page 220: History A Mark Schemes for the Units January 2010 - The ...

F963/02 Mark Scheme June 2012

Question Answer Marks Guidance 2 (a) The context for both sources is the 1874 election. Both are liberal sources and

comment from this perspective on Liberal achievements. Both talk of ‘enabling’ the British people and both refer to the achievement of moral policy and reform, on issues such as the Alabama arbitration. However there are considerable differences. For Gladstone in A the whole point of a Liberal government is fiscal rectitude – economy and debt reduction, a continued lowering of tariffs (Free Trade) and low taxation. His tone reveals considerable pride in the achievement of this and he proffers the hope that the Income Tax would finally be abolished should the Liberals be re-elected. He is also proud of liberal diplomacy abroad which brought peace and an end to factious disputes like the Alabama case. Halifax in B fails to even mention financial achievements, although he hints at unease at ‘what was going on abroad’. He doesn’t share Gladstone’s optimism as to what the Liberals had sought and achieved abroad. For him the key Liberal achievement had been the removal of abuse, privilege and corruption. Instead of focusing on tariffs and taxation he mentions the removal of the Irish Church (Disestablishment and Dis-Endowment in 1869) of army purchase (Cardwell’s Army reforms) and the Secret Ballot Act in 1872. This was political rather than financial and economic achievement. As regards provenance the key lies in the respective political positions of the two sources, and their dates. Gladstone in Source A is the Liberal leader and PM addressing his constituents immediately prior to the general election and aware that anything he said would be printed (the ‘Times’) and seen as a national Liberal election manifesto. For Gladstone the fiscal imperative was absolute. Always his own Chancellor he was determined to find a great mission to reunite the Liberal party and typically found it in a return to the basics of Free trade and low taxation. However this is a vote winning speech and the achievement of low taxation and a ‘sound economy’ and the proposed abolition of the Income Tax should be seen in this light. He hoped to counter the message of his Tory rival, a brewer who sought to exploit the Licensing issue that had recently bedevilled the Liberals. In contrast Lord Halifax in Source B, albeit from a liberal perspective, is more wide ranging in his assessment. As a member of the Lords, holding an honorary position as elder statesman in Gladstone’s government, he can afford to be more sanguine. His purpose is to console and commiserate immediately after the election defeat. He writes a personal letter to his leader. He studiously avoids the failure of Gladstone’s fiscal mission talking instead of a rebound in the thinking of the electorate, natural

30 Focus: Comparison of two Sources. No set answer is expected, but candidates need to compare the contents, evaluating such matters as authorship, dating, utility and reliability, so using the Sources ‘as evidence for …’. The headings and attributions should aid evaluation and reference to both is expected in a good answer.

13

Page 221: History A Mark Schemes for the Units January 2010 - The ...

F963/02 Mark Scheme June 2012

Question Answer Marks Guidance swings to and from a party or government that had genuinely, and rightly, sought the removal of long standing abuses. He stresses that Liberal achievements may not, in the long run, have resulted in so negative a popular response as the electoral result would suggest. In terms of judgement candidates may consider Halifax to be the better guide on what the Liberals, as a whole, achieved, given his balance of positive and negative and the wider take on the reforms, although Gladstone, as liberal and PM, may well provide better evidence in terms of his own view of what Liberals did, and should, stand for.

(b) The sources support three possible interpretations – that the result was down to Conservative leadership and organisation; that the Liberals had, by their reforms and actions, lost it; that the result was the product of general social and economic trends in the country that advantaged the Conservatives, although candidates may see this as part of the reaction to Liberal reform. Three of the sources are from a liberal perspective, albeit different strands (Gladstone in A, a senior Whig in B and a radical liberal journalist in D). They are divided in their verdict, although no credit is given to Disraeli and the Conservatives. They either stress their own mistakes and policies, well intentioned or otherwise (A and B) or point to long term trends that favour Conservatism (D – suitably vacuous in its reference to Conservatism, as befits a radical assessment). Two of the sources are from a Conservative angle (the Queen in Source C reporting Disraeli’s views and Gorst in Source E commenting on strategies and organisation). The argument for conservative leadership and organisation is to be found in sources C and E, the Queen and Gorst, son of Disraeli’s electoral organiser, and indirectly in D’s implicit reference to Disraeli’s strategy of promising respite from harassing legislation – the preference of the electorate for ‘leaving well alone’. However Sources C and E have different conservative emphases. Queen Victoria’s record of Disraeli’s conversation with her on the results stresses, rather predictably, Disraeli’s own leadership of the party. He takes the credit for the strategy of 1873/4, refusing office when Gladstone sought to resign following the debacle of the Irish Universities bill and allowing him to struggle on with a divided party and no particular policy until January 1874. Candidates might refer to his speeches from 1872 carving out a philosophy for mid Victorian Conservatism and to his witty takes

70 Focus: Judgement in context, based on the set of Sources and own knowledge. Successful answers will need to make use of all five Sources, testing them against contextual knowledge and evaluating their strengths and weaknesses, any limitations as evidence. A range of issues may be addressed in focusing upon the terms of the question but no set conclusion is expected.

14

Page 222: History A Mark Schemes for the Units January 2010 - The ...

F963/02 Mark Scheme June 2012

Question Answer Marks Guidance on Gladstone’s apparent interventionism (‘volcanoes’ etc.). Victoria is partisan but this is a factual record of what was said. It is given more validity by the initial comment on the result as a ‘great surprise’, Disraeli thinking the likelihood to be a small liberal majority. This is hardly an endorsement of a convincing and proactive Conservatism on Disraeli’s part. In contrast Gorst in E almost entirely stresses conservative organisation, countering Disraeli’s own view, in C, that organisation had nothing to do with it. The pro conservative sources diverge here. In 1874 (Source C) Disraeli is dismissive on organisational issues. From hindsight (Gorst wrote his pro conservative biography, Source E, in1900) there is a melding of leadership and organisation, as befits the son of the man who later claimed the credit for victory. He suggests that the idea of bringing together local constituency and a central organisation was Disraeli’s, despite no mention of its beneficial role in Disraeli’s conversation with Victoria. From that alleged suggestion there followed the work of his father – ensuring the party was not caught on the hop by Gladstone’s sudden and unexpected dissolution, targeting winnable seats and moving quickly to back ‘agreed’ candidates. There is no mention here of any other factor. Credit is given to Disraeli and Gorst whom, it is argued in E, would obviously realise the implications of the 1867 Reform Act with its need to mobilise, in new ways, an expanded urban electorate. Candidates may find this convincing given Gorst’s work and calculations. They may be aware of Liberal deficiencies before Chamberlain got to work but Disraeli’s tone and insouciance in C may lead them to consider the Conservative thesis to be unconvincing. They could also cite Harrison’s comments in D on the limits of Conservative organisation – that the party could not control Manchester. Another interpretation, to be found in sources A and B, is that the Liberals lost the election. This is given some weight by their liberal slant. Gladstone in A pins his hopes on fiscal rectitude and the promise of a bribe for the middle classes – the abolition of the income tax. His mention of the Alabama arbitration was, perhaps, unwise, especially its linkage to paying off debt. He studiously ignores the controversial legislation of his government and the precise context of the election in Greenwich (the Licensing Act). Candidates might consider his comments to reflect either desperation or an ill-conceived attempt to pin Liberalism back to its mid century hey-day. They certainly hid more than they reveal about the state of play in Liberal politics. However, Harrison in D challenges the importance of the Licensing Act and Forster’s Education Act (beer having little influence on the London middle

15

Page 223: History A Mark Schemes for the Units January 2010 - The ...

F963/02 Mark Scheme June 2012

Question Answer Marks Guidance class; the nonconformist ‘25th section men...not strong in London’). Halifax in B is similarly more measured. He is concerned to discern wider factors affecting the Liberal vote – an unpopular foreign policy (that could be pinned more squarely on Gladstone as the reference in A suggests) and a John Bright inspired Secret Ballot that rebounded on the Liberals by enabling the expression of private dissatisfaction. Again, however, Harrison is concerned to refute this in D (Halifax’s comments on master/men tension is criticised by the observation that there were few workers in the Home Counties). Halifax may well, as a Whig Liberal, reflect a more staid liberal approach to the perceived radicalism of Gladstone. Whilst acknowledging that the Liberals alienated vested interests, who protested loudly, he prefers to put it down to a swing of the pendulum. A third view can be constructed, using Sources B and D (Halifax and Harrison), although this could equally be made part of the previous argument – that the Liberals lost by losing their grip on the middle ground for whom the tax promises in A would appear slight after reductions in recent years. This view stresses general trends and the swing of the pendulum. Halifax’s thesis in B stresses a general swing in the electorate, still propertied, based on fear. This was partly economic and partly foreign (a balance of power less favourable to Britain). The result, according to Halifax, was the ‘taking refuge in Conservatism’. As a liberal he considers this irrational but, alas, inevitable. Harrison in D presents a more sophisticated analysis but comes to much the same conclusion – that trends were underway which saw the middle class move away from Liberalism. He is concerned to refute short term analyses based on either particular liberal failings (licensing; trade unions; education and the nonconformists; the new electorate of 1867 – residuum; the Gladstone bribe of income tax abolition in A) or on Conservative organisation (controlling Manchester). He argues that the turnaround didn’t occur in areas where liberal issues might rebound against them but amongst the large middle class centres – London, its suburbs, the Home Counties and the big northern cities. Candidates could refute this, at least in the north, where the issues he dismisses, it could be argued, did have an effect. In terms of judgement candidates may find any one of these convincing, depending on their evaluation of the sources and the contextual use of their own knowledge.

16

Page 224: History A Mark Schemes for the Units January 2010 - The ...

F963/02 Mark Scheme June 2012

Question Answer Marks Guidance 3 (a) The sources agree on certain specific points about why votes for women was

opposed. Firstly, opponents argued the woman’s priority was the home, the assumption being that politics was not for them. However, each adopts a different slant on this. Source C is emphatic that ‘Politics will go on without the help of women, but the home will not’ implying they are indispensable in the latter but not the former, whereas Source D suggests the incompetence of women in the home – unable to ‘sew, cook’ etc is such that it would be foolish to ‘introduce into politics these unsatisfactory creatures’. Secondly, opponents thought women lacked knowledge of politics: Source C accepting ‘the special knowledge of men’ in contrast to ‘inexperienced women’ and Source D concedes ‘wives are without the smattering of newspaper information which their husbands exchange’. Thirdly, and related, is the view that women were mentally not equipped for politics. Source C asserts that ‘women are not equal to men ... in intellect’ and the third sentence in Source D makes it clear by inversion that the ‘popular’ view was that women were mentally inferior. Fourthly, opponents argued that women lacked stamina. As with the previous point, a careful reading of the fourth sentence of Source D makes it clear that men believed women lacked ‘perseverance and resolve’ which is stated in Source C which criticised women’s lack of ‘endurance or nervous energy’. In evaluating these sources candidates might highlight the different perspectives of the authors. In Source C the views expressed clearly represent those of the authoress which she claims are supported more broadly by women as a whole drawing on the results of the survey of women in East Grinstead to prove her point. By contrast, in presenting the views of opponents Source D is clearly interpreting the views of ‘politicians’, the press’ and the public but the implication is clear that they are the views of men with which she disagrees. Candidates will likely assert that this is unsurprising given that Mrs Colquhoun was a member of the Anti-Suffrage League and Mrs Martin was a Suffragist. Mention of the integrity of the Empire in Source C might be used as more than simply an argument of opponents against the vote for women but an indication of the conservatism of Mrs Colquhoun which may help explain her resistance to political change. The typicality of these views might be verified as consistent with the organisations represented by the authoresses and a way of judging the reliability of the sources and the equality of their value as evidence.

30 Focus: Comparison of two Sources. No set answer is expected, but candidates need to compare the contents, evaluating such matters as authorship, dating, utility and reliability, so using the Sources ‘as evidence for …’. The headings and attributions should aid evaluation and reference to both is expected in a good answer.

17

Page 225: History A Mark Schemes for the Units January 2010 - The ...

F963/02 Mark Scheme June 2012

Question Answer Marks Guidance (b) This question is about the arguments for giving women the vote, not the

reasons for doing it. In support of the interpretation Source A implies that the present system amounted to ‘tyranny’, the implication being that votes for women would check this. In addition, the record of municipal government is claimed to be ‘in advance of Parliamentary attitudes’, in part it is implied, because women play a part there. Candidates might elaborate on women in local government. It could also be claimed that the point about ‘women have to obey the laws’ suggests that the framing of laws would be better. Source D supports the view that women’s involvement would be a positive political development in stressing the attributes of women including ‘perseverance and resolve’, ‘mental superiority’, ‘commonsense’ and ‘knowledge of the workings of male human nature’. Knowledge of women who had proved these talents and of the hardships endured by many women would usefully re-enforce these claims. The logic of the arguments presented may commend Sources A and D as strong evidence in favour of the interpretation. However, candidates may question their reliability given the commitment of both women to the cause of women’s suffrage. Source C is helpful in developing the counter-argument. The results of the survey suggest that only a minority of women wanted the vote implying a denial of the interpretation. Furthermore, the authoress argues that the political system would not be improved if women had the vote. Indeed, Source C is explicit in stating that ‘the problems of government can only be solved by ... men’. Although the survey quoted may represent the views of women in one locality candidates should note that the Anti-Suffrage League was national, stated in the introduction, and in 1911 the support for the women’s cause was limited: Parliament had dropped a Conciliation Bill and an upsurge of violence from the Suffragettes alienated many. Yet, her reference to women abroad having the vote might be used as support for the argument for giving the vote to women in England. Closely linked to the point that votes for women would improve the political system is the reason that women should be given the vote as affirmation of their equality. This point is made most forcefully, perhaps, in Source B which regards the vote as essential to validate women’s equality in every other sphere of life. This is made clear in ‘disenfranchisement brands women with a permanent mark of inferiority’

70 Focus: Judgement in context, based on the set of Sources and own knowledge. Successful answers will need to make use of all five Sources, testing them against contextual knowledge and evaluating their strengths and weaknesses, any limitations as evidence. A range of issues may be addressed in focusing upon the terms of the question but no set conclusion is expected.

18

Page 226: History A Mark Schemes for the Units January 2010 - The ...

F963/02 Mark Scheme June 2012

Question Answer Marks Guidance which will colour their view of themselves in ‘education, work and social relations’ and also perpetuate the view of men that they are ‘superior’. The tone of Source B is critical of men’s attitudes as a whole which might lead candidates to argue that the author represents the minority view of men. The theme of equality is made explicit in Source A in the first and fourth items listed which suggest women will not enjoy equality of security of interest or justice until they have the vote. The final point listed could be cross-referenced to Source B and its comments on inequality of ‘social relations’. Candidates may dismiss the points made in Source A as the poster was clearly propaganda for the WSPU but given the proximity of Source B to the views expressed in Source A it could be argued otherwise. Indeed, Source D might be used to support the view that equality was the main reason why women should be given the vote. The specific qualities of women considered in the source suggest that women were the equal of men in most respects. Indeed, the final clause claiming women were ‘invaluable as voters’ stresses the point. Pre 1917 Women’s contribution to the war effort and the ending of militancy is regarded as the main argument that they deserved the vote according to Source E, on two counts. In asking ‘how could we have carried on the war without them’ the author acknowledges that women were indispensable to the war effort and as a result he ‘would find it impossible to withhold from women their right’. In addition, the author was impressed by the decision of the Suffragettes to suspend their ‘detested campaign’ of violence and the opportunity presented to politicians to concede to women’s demands without appearing to do so as a result of pressure. Candidates will be able to add detail about Asquith’s hostility to the suffrage movement, the violence of the WSPU and the specific contribution of women to the war effort. Speaking as a politician rather than as Prime Minister also allowed him to express his opinions more frankly than before and by 1917 there was little debate about the contribution of women in WW1.

19

Page 227: History A Mark Schemes for the Units January 2010 - The ...

F963/02 Mark Scheme June 2012

Question Answer Marks Guidance 4 (a) Similarities: Neither is very complimentary about his style. E (Clementine

Churchill) sees an overbearing and sarcastic attitude to subordinates and B sees a lack of consideration, with admirals being brought into early morning conferences. B (Kennedy) sees him overworking and E suggests that rudeness and irascibility may be the result of pressure, though does not say it directly. Neither sees in Churchill a calm or professional approach, but rather both suggest vigour and energy which are not always well-judged. Differences: There is no mention in E of the excessive drinking and dining that figures in B, and there is no mention of the popularity that Churchill enjoys in E. Though B sees a lack of consideration, it is not as direct about Churchill’s rude and abrasive manner as E. This may well be because of the extreme pressure that Churchill was under by the time his wife warned him – he had certainly not had the rest that Kennedy thought was necessary Provenance: E is of course after Churchill has the huge responsibility of guiding Britain in a time of extreme crisis, while B is concerned with speculation, detrimental to Churchill’s claims on the leadership, in the period prior to Chamberlain’s resignation. Neither source had direct evidence of Churchill’s leadership and must have relied on reports from subordinates. E is much closer to Churchill as it had the purpose of improving his relations with others. B had no such purpose and was recording his private thoughts. Both have limitations as evidence – and both omit the charm that Churchill could employ and the devotion he often engendered among those who worked for him. Kennedy as a leading newspaper man would have had inside information and Clementine knew her husband and had been told by those close to him that he was going too far. However neither makes allowances for the exigencies of war and neither is from a current military background – though Brooke’s diaries give a similar picture after 1941. In terms of judgement, it may well be that a close family member would be more reliable and truthful, but that someone like Kennedy who had contacts with the workings of government would have a clearer idea of leadership qualities. No set answer is required for a judgement about the relative usefulness of these sources – E is from the time of Churchill’s premiership while B is not, but may reflect a particularly stressful period. On the other hand there is plenty of other evidence to support these defects of leadership as well as many examples of much greater strengths than this source suggests.

No set answer is expected, but candidates need to compare the contents, evaluating such matters as authorship, dating, utility and reliability, so using the Source ‘as evidence for…..’ The Headings and attributions should aid evaluation and reference to both is expected in a good answer.

20

Page 228: History A Mark Schemes for the Units January 2010 - The ...

F963/02 Mark Scheme June 2012

Question Answer Marks Guidance (b) The issue is whether the view Churchill did not have many appropriate leadership

qualities in terms of being trusted and offering good judgement and only came to office because Halifax did not feel he could accept the responsibility is sustainable. The alternative is that Churchill had those uncompromising, energetic war-like qualities that were needed, even if he did drink, bully and harass subordinates and lack the trust of some, if not all, establishment figures because he was a popular hero. A (Colville) establishes the dilemma of 1940 – Churchill’s reputation of untrustworthiness and instability – borne out by poor judgement over the threat of Communism, India, his choice of associates like Bracken and Boothby and his unwise stance during the Abdication. However against this was the confidence he gave. He had wide experience of office, had direct experience of war and unbounded optimism (at least in public) about Britain’s will and ability to win and a record of urging governments to stand up to Germany. The view that Churchill was the best suited for the premiership in 1940 is to be found, with qualifications in A and B but particularly in C (Boothby). Boothby, though hardly an unbiased observer, being close to Churchill and having worked to have him as leader, was justified in commenting on Labour’s lack of confidence in Chamberlain who was blamed for failures in Norway which Churchill, oddly, was not (though much of the disaster could be laid at his door). The sources do not, rightly, claim that Churchill had the confidence of most of the Conservatives but nevertheless refers to Churchill’s appointment as Prime Minister as ‘inevitable’. This must be seen as more of an encouragement to Churchill than necessarily accurate. The popularity of Churchill might support the evidence in C and some aspects of A. His pre-war stance on Appeasement, his journalistic writing and his speeches, together with the impression that he was leading the war effort (albeit, in reality with some lack of judgement, as in Norway and with some unrealistic ideas of a campaign to help Finland), suggest why he was best suited in the popular if not the official mind to be PM. Better candidates might make something of this distinction in their responses to the question. The alternative view is in B, D and E. George VI in D provides the main corrective – Churchill’s qualities had not impressed the King and he still favoured Halifax who, by that time, had distanced himself from earlier appeasement and stood as a dignified and experienced politician. This useful source makes it clear that Chamberlain was a key figure in having Churchill accepted, once Halifax had made it

70 Focus: Judgement in context, based on a set of Sources and own knowledge. Successful answers will need to make use of all five Sources, testing them against contextual evidence and evaluating their strengths and weaknesses, any limitations as evidence. A range of issues may be addressed in focusing upon the terms of the question but no set conclusion is expected.

21

Page 229: History A Mark Schemes for the Units January 2010 - The ...

F963/02 Mark Scheme June 2012

Question Answer Marks Guidance clear that he did not want to lead. The source is a personal record and consonant with the royal family’s earlier distrust of Churchill. Some doubt is also cast on C by the opinion of a leading journalist, Kennedy of the Times, in B. The erratic and untrustworthy side referred to in A once again is apparent – with the heavy drinking and the unreasonable demands on subordinates and the military leaders. This continued during Churchill’s premiership and is well-documented by sources such as Brooke’s diaries. The heavy drinking is not just gossip but is referred to in many other memoirs. E (Lady Churchill), in a personal and quite wounding letter, raises some of the issues of B and helps to explain the King’s view in D. It does not mention the great charm that Churchill could deploy or the loyalty he could engender and it is written at a particularly difficult time for Churchill which might exacerbate some unfortunate traits of his leadership. However other evidence corroborates an overbearing manner and disregard for subordinates. Some might consider that the fact that such a letter could be written indicates that Churchill was a big enough man to take criticism – and this is justified. Those who stood up to him and questioned him often gained respect and he did listen to different views. Provenance: C is the strongest source for Churchill being best suited to lead. Both A and B see his abilities and B refers to his popularity but neither thought him the natural choice at the time. C is by a strong supporter, part of a group of dissident Conservatives out of favour with mainstream Chamberlain supporters, Though written at the time, it is for the purpose of reassuring Churchill and does not present an unbiased view of the situation – there were plenty of people in Parliament and the country who had more doubts than are expressed here about Churchill’s past record. He himself thought that the disasters of Norway might well have ruined his reputation. Labour favoured Halifax as the PM until Halifax’s doubts and so did the King who was personally upset at Halifax’s refusal. However even someone close to Chamberlain in Source A thought that Churchill was destined to be Prime Minister – but interestingly not in 1940, stressing both his instability and his untrustworthiness. A could well be influenced by his closeness to Chamberlain who was still very well-respected in the party in 1940 and B, though a leading journalist was not as close to the centre of power as A and possibly C. E was certainly personally close to Churchill, but may have been worried by the signs of strain which only she could see and by the reports she was hearing – though regrettable, some bullying in 1940 might have been necessary.

22

Page 230: History A Mark Schemes for the Units January 2010 - The ...

F963/02 Mark Scheme June 2012

23

Question Answer Marks Guidance Context: Candidates might pick up on Churchill’s previously poor reputation and why he might have been considered untrustworthy and unstable. This might be contrasted with knowledge of his own reputation as an opponent of Appeasement. The dangerous situation of May 1940 might be used as evidence to support the views in E and B for the need of someone of Churchill’s energy and uncompromising demands. Alternatively knowledge of his unreasonable behaviour might be used to confirm the views in A and B that he was indeed too dangerous a choice and in E that he was difficult to work for. No set answer is expected here.

Page 231: History A Mark Schemes for the Units January 2010 - The ...

OCR (Oxford Cambridge and RSA Examinations) 1 Hills Road Cambridge CB1 2EU OCR Customer Contact Centre Education and Learning Telephone: 01223 553998 Facsimile: 01223 552627 Email: [email protected] www.ocr.org.uk For staff training purposes and as part of our quality assurance programme your call may be recorded or monitored

Oxford Cambridge and RSA Examinations is a Company Limited by Guarantee Registered in England Registered Office; 1 Hills Road, Cambridge, CB1 2EU Registered Company Number: 3484466 OCR is an exempt Charity OCR (Oxford Cambridge and RSA Examinations) Head office Telephone: 01223 552552 Facsimile: 01223 552553 © OCR 2012

Page 232: History A Mark Schemes for the Units January 2010 - The ...

Oxford Cambridge and RSA Examinations

GCE

History A Advanced Subsidiary GCE

Unit F963/02: Option B Modern 1815-1945

Mark Scheme for June 2013

Page 233: History A Mark Schemes for the Units January 2010 - The ...

F963/02 Mark Scheme

OCR (Oxford Cambridge and RSA) is a leading UK awarding body, providing a wide range of qualifications to meet the needs of candidates of all ages and abilities. OCR qualifications include AS/A Levels, Diplomas, GCSEs, Cambridge Nationals, Cambridge Technicals, Functional Skills, Key Skills, Entry Level qualifications, NVQs and vocational qualifications in areas such as IT, business, languages, teaching/training, administration and secretarial skills. It is also responsible for developing new specifications to meet national requirements and the needs of students and teachers. OCR is a not-for-profit organisation; any surplus made is invested back into the establishment to help towards the development of qualifications and support, which keep pace with the changing needs of today’s society. This mark scheme is published as an aid to teachers and students, to indicate the requirements of the examination. It shows the basis on which marks were awarded by examiners. It does not indicate the details of the discussions which took place at an examiners’ meeting before marking commenced. All examiners are instructed that alternative correct answers and unexpected approaches in candidates’ scripts must be given marks that fairly reflect the relevant knowledge and skills demonstrated. Mark schemes should be read in conjunction with the published question papers and the report on the examination. OCR will not enter into any discussion or correspondence in connection with this mark scheme. © OCR 2013

Page 234: History A Mark Schemes for the Units January 2010 - The ...

F963/02 Mark Scheme June 2013

1

Annotations

Annotation Meaning Subject-specific Marking Instructions

Question (a) Maximum mark 30 A01a and b AO2a

1 13–14 15–16

2 11–12 13–14

3 9–10 10–12

4 7–8 8–9

5 5–6 6–7

6 3–4 3–5

7 0–2 0–2

Notes related to Part A: (i) Allocate marks to the most appropriate level for each AO (ii) If several marks are available in a box, work from the top mark down until the best fit has been found (iii) Many answers will not be at the same level for each AO

Page 235: History A Mark Schemes for the Units January 2010 - The ...

F963/02 Mark Scheme June 2013

2

Marking Grid for Question (a)

A0s A01a and b A02a Total for each question = 30

Recall, select and deploy historical knowledge appropriately, and communicate knowledge and understanding of history in a clear and effective manner. Demonstrate understanding of the past through explanation, analysis and arriving at substantiated judgements of: - key concepts such as causation, consequence,

continuity, change and significance within an historical context;

- the relationships between key features and characteristics of the periods studied.

As part of an historical enquiry, analyse and evaluate a range of appropriate source material with discrimination.

Level 1 Consistent and developed comparison of the key issue with a balanced and well-supported judgement. There will be little or no unevenness.

Focused use of a range of relevant historical concepts and context to address the key issue.

The answer is clearly structured and organised. Communicates coherently, accurately and effectively.

13–14

Focused comparative analysis. Controlled and discriminating evaluation of content and provenance, whether integrated or treated separately.

Evaluates using a range of relevant provenance points in relation to the sources and question. There is a thorough but not necessarily exhaustive exploration of these.

15–16

Level 2 Largely comparative evaluation of the key issue with a balanced and supported judgement. There may be a little unevenness in parts.

Focused use of some relevant historical context with a good conceptual understanding to address the key issue.

The answer is well structured and organised. Communicates clearly.

11–12

Relevant comparative analysis of content and evaluation of provenance but there may be some unevenness in coverage or control.

Source evaluation is reasonably full and appropriate but lacks completeness on the issues raised by the sources in the light of the question.

13–14

Page 236: History A Mark Schemes for the Units January 2010 - The ...

F963/02 Mark Scheme June 2013

3

A0s A01a and b A02a Level 3 Some comparison linked to the key issue. Is aware of

some similarity and/or difference. Judgements may be limited and/or inconsistent with the analysis made.

Some use of relevant historical concepts and contexts but uneven understanding. Inconsistent focus on the key issue.

The answer has some structure and organisation but there is also some description. Communication may be clear but may not be consistent.

9–10

Provides a comparison but there is unevenness, confining the comparison to the second half of the answer or simply to a concluding paragraph. Either the focus is on content or provenance, rarely both.

Source evaluation is partial and it is likely that the provenance itself is not compared, may be undeveloped or merely commented on discretely.

10–12 Level 4 Some general comparison but undeveloped with some

assertion, description and / or narrative. Judgement is unlikely, unconvincing or asserted.

A general sense of historical concepts and context but understanding is partial or limited, with some tangential and/or irrelevant evidence.

Structure may be rather disorganised with some unclear sections. Communication is satisfactory but with some inaccuracy of expression.

7–8

Attempts a comparison but most of the comment is sequential. Imparts content or provenance rather than using it.

Comparative comments are few or only partially developed, often asserted and/or ‘stock’ in approach.

8–9 Level 5 Limited comparison with few links to the key issue.

Imparts generalised comment and / or a weak understanding of the key points. The answer lacks judgement or makes a basic assertion.

Basic, often inaccurate or irrelevant historical context and conceptual understanding.

Structure lacks organisation with weak or basic communication.

5–6

Identifies some comparative points but is very sequential and perhaps implicit

Comment on the sources is basic, general, undeveloped or juxtaposed, often through poorly understood quotation.

6–7

Page 237: History A Mark Schemes for the Units January 2010 - The ...

F963/02 Mark Scheme June 2013

4

A0s A01a and b A02a Level 6 Comparison is minimal and basic with very limited links

to the key issue. Mainly paraphrase and description with very limited understanding. There is no judgement.

Irrelevant and inaccurate concepts and context. Has little organisation or structure with very weak

communication. 3–4

Little attempt to compare. Weak commentary on one or two undeveloped points, with basic paraphrase. Sequencing is characteristic.

Comments on individual sources are generalised and confused.

. 3–5

Level 7 Fragmentary, descriptive, incomplete and with few or no links to the key issue. There is little or no understanding. Much irrelevance.

Weak or non existent context with no conceptual understanding.

No structure with extremely weak communication. 0–2

No attempt to compare either content or provenance with fragmentary, brief or inaccurate comment.

Makes no attempt to use any aspects of the sources.

0–2 Question (b) Maximum mark 70 A01a and b AO2a and b

1 20–22 42–48

2 17–19 35–41

3 13–16 28–34

4 9–12 21–27

5 6–8 14–20

6 3–5 7–13

7 0–2 0–6

Notes related to Part B: (iv) Allocate marks to the most appropriate level for each AO (v) If several marks are available in a box, work from the top mark down until the best fit has been found (vi) Many answers will not be at the same level for each AO

Page 238: History A Mark Schemes for the Units January 2010 - The ...

F963/02 Mark Scheme June 2013

5

AOs A0Ia and b Ao2a and b

Total mark for the question = 70

Recall, select and deploy historical knowledge appropriately, and communicate knowledge and understanding of history in a clear and effective manner. Demonstrate understanding of the past through explanation, analysis and arriving at substantiated judgements of: - key concepts such as causation, consequence, continuity, change and significance within an historical context; - the relationships between key features and characteristics of the periods studied.

As part of an historical enquiry, analyse and evaluate a range of appropriate source material with discrimination. Analyse and evaluate, in relation to the historical context, how aspects of the past have been interpreted and represented in different ways.

Level 1 Convincing analysis and argument with developed explanation leading to careful, supported and persuasive judgement arising from a consideration of both content and provenance. There may be a little unevenness at the bottom of the level.

Sharply focused use and control of a range of reliable evidence to confirm, qualify, extend or question the sources.

Coherent organised structure. Accurate and effective communication.

20–22

A carefully grouped and comparative evaluation of all the sources with effective levels of discrimination sharply focused on the interpretation.

Analyses and evaluates the strengths, limitations and utility of the sources in relation to the interpretation. Uses and cross references points in individual or grouped sources to support or refute an interpretation.

Integrates sources with contextual knowledge in analysis and evaluation and is convincing in most respects. Has synthesis within the argument through most of the answer.

42–48 Level 2 Good attempt at focused analysis, argument and

explanation leading to a supported judgement that is based on the use of most of the content and provenance.

A focused use of relevant evidence to put the sources into context.

Mostly coherent structure and organisation if uneven in parts. Good communication.

17–19

Grouped analysis and use of most of the sources with good levels of discrimination and a reasonable focus on the interpretation.

Analyses and evaluates some of the strengths and limitations of the sources in relation to the interpretation. May focus more on individual sources within a grouping, so cross referencing may be less frequent.

Some, perhaps less balanced, integration of sources and contextual knowledge to analyse and evaluate the interpretation. Synthesis of the skills may be less developed. The analysis and evaluation is reasonably convincing.

35–41

Page 239: History A Mark Schemes for the Units January 2010 - The ...

F963/02 Mark Scheme June 2013

6

AOs A0Ia and b Ao2a and b

Level 3 Mainly sound analysis, argument and explanation, but there may be some description and unevenness. Judgement may be incomplete or inconsistent with the analysis of content and provenance.

Some relevant evidence but less effectively used and may not be extensive.

Reasonably coherent structure and organisation but uneven. Reasonable communication.

13–16

Some grouping although not sustained or developed. Sources are mainly approached discretely with limited cross reference. Their use is less developed and may, in parts, lose focus on the interpretation. There may be some description of content and provenance.

Is aware of some of the limitations of the sources, individually or as a group, but mostly uses them for reference and to illustrate an argument rather than analysing and evaluating them as evidence. There is little cross referencing.

There may be unevenness in using knowledge in relation to the sources. Synthesis may be patchy or bolted on. Analysis and evaluation are only partially convincing.

28–34 Level 4 Attempts some analysis, argument and explanation but

underdeveloped and not always linked to the question. There will be more assertion, description and narrative. Judgements are less substantiated and much less convincing.

Some relevant evidence is deployed, but evidence will vary in accuracy, relevance and extent. It may be generalised or tangential.

Structure is less organised, communication less clear and some inaccuracies of expression.

9–12

Sources are discussed discretely and largely sequentially, perhaps within very basic groups. Loses focus on the interpretation. The sources are frequently described.

May mention some limitations of individual sources but largely uses them for reference and illustration. Cross referencing is unlikely.

An imbalance and lack of integration between sources and knowledge often with discrete sections. There is little synthesis. Analysis and explanation may be muddled and unconvincing in part.

21–27 Level 5 Little argument or explanation, inaccurate understanding of

the issues and concepts. The answer lacks judgement. Limited use of relevant evidence or context which is largely

inaccurate or irrelevant. Structure is disorganised, communication basic and the

sense not always clear.

5–8

A limited attempt to use the sources or discriminate between them. The approach is very sequential and referential, with much description. Points are undeveloped.

There is little attempt to analyse, explain or use the sources in relation to the question. Comment may be general.

There is a marked imbalance with no synthesis. Analysis and explanation are rare and comments are unconvincing.

14–20

Page 240: History A Mark Schemes for the Units January 2010 - The ...

F963/02 Mark Scheme June 2013

7

AOs A0Ia and b Ao2a and b Level 6 There is very little explanation or understanding. Largely

assertion, description and narrative with no judgement. Extremely limited relevance to the question.

Evidence is basic, generalised, patchy, inaccurate or irrelevant.

Little organisation or structure with poor communication. 3–4

Very weak and partial use of the sources for the question. No focus on interpretation.

A very weak, general and paraphrased use of source content.

No synthesis or balance. Comments are entirely unconvincing.

7–13 Level 7 No argument or explanation. Fragmentary and descriptive

with no relevance to the question. No understanding underpins what little use is made of

evidence or context. Disorganised and partial with weak communication and

expression. 0–2

Little application of the sources to the question with inaccuracies and irrelevant comment. Fragmentary and heavily descriptive.

No attempt to use any aspect of the sources appropriately. No contextual knowledge, synthesis or balance. There is no

attempt to convince. 0–6

Page 241: History A Mark Schemes for the Units January 2010 - The ...

F963/02 Mark Scheme June 2013

8

Question Answer Marks Guidance

1 (a) The context is that despite the initial stress on child labour by the 1840s female factory labour had become an issue. How far should it be restricted (the Mines Act of 1842 and the Factory Act of 1844 had imposed restrictions on female hours, ten and a half, and, in the case of the Mines, the type of work allowed women – not underground)? How far was female factory labour desirable? Both agree on the general need to improve the hours and conditions for women and girls in the factories. Both want to protect family life and agree the impact of factory labour has been the neglect of wifely duties– Ashley’s ‘details of domestic life’ and Jameson’s ‘needlework, cleanliness, and the management of wages to provide homely comforts’. There is some agreement on the dangers for women. Jameson refers to those who survive to adulthood, implying many do not, whilst Ashley, although more vague, does imply danger in the competition with more vigorous adult men. The differences are that Jameson argues that factory labour is preferable to other forms – agriculture and domestic service – because of comparative freedom, independence and the stated and regulated hours (after 1844), something missing from the other occupations. Ashley disagrees on the grounds that factory work is cruel for women, although he is careful to say he is talking of 13-18 year olds, not those regulated by the 1833 Act. Ashley doesn’t mention female education in the factories but does refer, in general, to nothing having been done to address 13-18 year old females. Jameson does, citing the Factory Commissioners who comment on the particular neglect of girls’ education in the factory schools. Jameson also stresses the independence that factory work can provide, something significantly not mentioned by Ashley. The provenance is similar. Both are writing/speaking in the context of the 1846 10 Hour Bill. Both appear to want to protect family life. Ashley was known to dislike independent women who demanded the same as men and neglected traditional female roles. Yet neither seems to want equality. There are differences in their backgrounds. Ashley was an aristocrat, a social conservative and one of the key parliamentary factory reform leaders. He was mainly concerned with men and the issue of 10 hours, especially whilst introducing the 10 Hour Bill in this parliamentary speech. His audience was a male one, although the speech would be widely reported. Jameson in Source D was a middle class female and early feminist writing in a journal of memoirs and essays. Her audience was probably more intellectual, middle class and perhaps female. As such she is more accepting of the independence available for some. Nonetheless she too is concerned with morality, family and traditional roles. In terms of judgement both are equally valid in the points they make. Source D provides a middle class female and early feminist slant (the attack on the lack of education for factory women and the awareness of independence), Source E gives a paternal, male, Tory and aristocrat reformer’s view. Jameson may be the more informative and perceptive of the two but she shares a conventional and middle/upper class view of working women, seemingly unaware that work was, usually, not a matter of choice.

30 Focus: Comparison of twoSources No set answer is expected, but candidates need to compare the contents, evaluating such matters as authorship, dating, utility and reliability, so using the Source ‘as evidence for…..’ The Headings and attributions should aid evaluation and reference to both is expected in a good answer.

Page 242: History A Mark Schemes for the Units January 2010 - The ...

F963/02 Mark Scheme June 2013

9

Question Answer Marks Guidance (b) The Sources provide a variety of views on whether the Factory Acts (1833 and 1844) did more

harm than good. Most of the sources are capable of being interpreted either way as they contain a variety of perspectives. Sources A and C, McCulloch and Hyde, are pro factory, an economist and mill owner, but both are worried about the harm that will be done. Sources D and E are factory reformers but can also see the negative aspects, especially on women, and they continue to push for reform. Source B is from a respected and formidable factory inspector, Leonard Horner, who is convinced that the 1833 Act is sensible and doing good. Generally sources B, D and E are more convinced of some good, the ones from the employer perspective still uncertain and resentful at the unprecedented interference. However even the Inspector and the reformers are aware of loopholes and problems, the latter (Ashley and Jameson), especially aware of the negative impact on women. The view that the Acts did more harm for children can be found in Sources A and C, whilst Sources D and E argue that some harm has been done to women. McCulloch in Source A considers that the restrictions on children under 9 will have resulted in them being thrown onto the streets and, by suggesting that factories are places of misery, confinement and ill treatment will have harmed the competitiveness and employment opportunities offered by factories. He angrily refutes the image painted by Sadler’s Report. However as an economist his was a more abstract view, although there is much evidence to suggest that the report and its evidence was deliberately slanted against the factory. Knowledge might suggest that parents, children, campaigners and employers alike would conspire to render it ineffective unless it was properly enforced. Such points are borne out by the reports of the factory inspectors, like that of Horner in Source B, who refers to prosecutions of those who broke the new rules (usually for failing to register child workers and their hours or a failure to provide education) and to a stubborn minority of employers. Here other developments such as registration of births did gradually ensure more ‘good’ came of the act and it became more difficult to evade its educational, safety and working hour clause. Nonetheless it could be pointed out that the 1844 Act actually adversely affected the very young, bringing 8 year olds back into employment. Greg in Source C is convincing evidence. It comes from a noted employer with a firm but sound reputation at Styal Mill in Cheshire. He opposed 10 hours and in this pamphlet, written to refute factory reform, he argues, as McCulloch did, that the consequence of 1833 has been to throw children into a worse situation, either onto the streets, or into the mines where nothing was done until 1842. He also alludes to the educational effects of this. Such children would receive none and he asks the rhetorical question - are they better physically and mentally than before? He assumes not, although it is worth pointing out that he can only be talking of those below 9, or those whose

70 Focus: Judgement in context, based on a set of Sources and own knowledge. Successful answers will need to make use of all five Sources, testing them against contextual evidence and evaluating their strengths and weaknesses, any limitations as evidence. A range of issues may be addressed in focusing upon the terms of the question but no set conclusion is expected. The sources can be read / analysed in different ways and as part of their judgement candidates will need to appreciate this.

Page 243: History A Mark Schemes for the Units January 2010 - The ...

F963/02 Mark Scheme June 2013

10

Question Answer Marks Guidance restricted hours above 9 had led to dismissal. He would be in a position to know, at least in the North. Sources D and E both consider that the factories and acts have harmed young poor women. Jameson in D argues that the attraction of regular factory work encourages girls and women to go into it at the expense of morality and the nation’s domestic life. They become too independent and have no idea how to manage a home. Ashley makes a similar point but the thrust of his point on the existing factory acts is that too many loopholes exist and little has been done for the 13-18 age groups. Both sources are making a point with a view to change. The variety of viewpoints here might suggest that the Factory Acts did little good and, in the case of those sacked and women, much harm. The view that stresses the good can be seen partially in Source A, but particularly in Sources B, C and D. The key source for ‘good’ is Horner in B. As the most influential and hard working of the 4 inspectors set up by the 1833 Act, active in its enforcement, his evidence is especially telling. He is convinced that it has changed attitudes for the better. He argues that most employers accepted the moral and practical points of the act in relation to children, their health and education. He also comments that workers also did – important given that deception about age, given the reliance on children’s wages, was a key element in undermining the Act. The fact that he was actively prosecuting demonstrates that the Act had teeth when it came to age and hours worked. However candidates may point to his lack of typicality and to his vested interest in pushing for more reform. McCulloch in Source A usefully points out, in an otherwise negative source, that no one under 9 is now employed, although it might be argued that most of those employed before were already 9-14. He is supported in this by Greg in Source C who points to the general satisfaction that any child aged 9-13 would now only be working an 8 hour day. Both McCulloch and Greg point out that the bad had been exaggerated by Sadler and they are convinced that factories ‘are our best schools’ in the sense of training children that would otherwise be on the streets. Knowledge would suggest that the educational provisions of the 1833 Act were, at least, something (2 hours a day, after work, for 9-13 year olds). Both McCulloch and Greg seem to accept the point about very young children and possibly the educational aspects of the Act. Their worry is the extension to older children and particularly adults. Jameson in Source D points out factories are now regulated and preferred places of work. Both 1833 and 1844 stated hours for children and the latter extended these to women, so much so that they were attractive to young girls. Source D is also a useful corrective to the grim view similarly refuted by A and C, as it compares factories to other sources of employment and finds their regulated nature, by comparison with domestic service, potteries, sweatshops et al, attractive to workers. Knowledge of the two Acts and the inspectorate could also be used to argue either way.

Page 244: History A Mark Schemes for the Units January 2010 - The ...

F963/02 Mark Scheme June 2013

11

Question Answer Marks Guidance 2 (a) The context for both sources is Bruce’s proposed 1872 Licensing Bill which sought to achieve a

balance between two powerful but opposed liberal interests, the Nonconformists and the Brewing industry. The Nonconformists wanted a variety of restrictions put on alcohol but particularly wanted the local option that would allow local voters and thus local councils to close down pubs and off licences in a locality (local prohibition). The Brewers opposed restrictions, preferring instead a few closures where there were too many. Both sources dislike this particular licensing reform proposal – in B a ‘wretched bill’; in C ‘the strongest dislike’. Both agree that sobriety is desirable and both come from a religious background, one nonconformist, one Anglican. They are similar in that both see the importance of Licensing reform as an issue, agreeing that it raised issues of freedom and social reform. The Advocate in B puts it at the top of Nonconformities reform priorities. However there are considerable differences in attitude but their dissatisfaction arises from different grounds. The Nonconformist Advocate in B wants society protected from the evils of alcohol and sees licensing reform as the key to tackling society’s ills – health, poverty and crime. Society needed to be protected from itself. This is not the issue for Bishop Magee in C. The issue for him is freedom, the principle of minimal intervention in adult choice. Individuals should not be compelled by the State. He ventures the opinion that, through freedom and persuasion, individuals might be led to sobriety but, if compelled, they would lose freedom and would be more likely to find consolation in now illegal drink. The Bishop doesn’t mention the Brewers or indeed any vested interest. The basis of his attitude is the principle of choice. The Advocate does refer to them, scathingly, and would not accept that the poor are free agents given that the Brewers devote themselves to the ‘demoralisation of the people’. For the Advocate society is not a level playing field. It believes that Bruce has been ‘got at’ by the Brewers As regards provenance the key lies in their respective political positions. Source B is from a liberal and nonconformist perspective, specifically a temperance newspaper pushing a particular line (the local option). Its tone makes it plain that the bill is an unwelcome compromise that clearly satisfies the Brewers more than the ‘religious and virtuous’ part of the community. It ignores any of the other arguments against – class, local unfairness if one area was to proceed and another not, and the issue of freedom. It seeks to persuade, particularly amongst its own community and wishes to apply pressure to a Liberal government where it had more influence than usual. Source C is a very different context, a Conservative and active political Bishop speaking in the House of Lords where nonconformist arguments would rarely be heard. He considers any form of compulsion to be wrong. Potentially he is speaking to the converted but he could expect his speech to be reported and taken up by opponents of the Bill, as indeed it was.

30 Focus: Comparison of two Sources. No set answer is expected, but candidates need to compare the contents, evaluating such matters as authorship, dating, utility and reliability, so using the Sources ‘as evidence for …’. The headings and attributions should aid evaluation and reference to both is expected in a good answer.

Page 245: History A Mark Schemes for the Units January 2010 - The ...

F963/02 Mark Scheme June 2013

12

Question Answer Marks Guidance Although a friend of sobriety and morality (his final line points to the potential incongruity of his position) for him the principle of freedom is absolute and he puts it eloquently and with some wit. In terms of judgement candidates may consider both equally useful on the issues and attitudes to licensing reform. The Advocate in B may be better for the wider social issues and attitudes towards vested interests like the Brewers, but Bishop Magee in C is telling on the political and freedom issues raised by licensing reform. Both represent particular constituencies of opinion.

(b) The sources support three possible interpretations – that Disraeli’s 1874-80 2nd Government did more for living and working conditions , either in terms of legislation or rhetoric and in the extent and focus of their reforms; that Gladstone’s 1st Government did more or that both were very similar, differing only in emphasis. Candidates may focus on arguing for either government, although some may consider the third option. The argument for Disraeli’s Government doing more is to be found in Sources B, C D and E. Sources B and C both focus on criticising liberal licensing legislation. That they come from different political and religious agendas is telling. Source B opposes, on the grounds that, merely to limit quite extensive opening hours and close down some pubs where there were too many, was a compromise too far. It would continue to expose the working class to the ‘evils of alcohol’. As far as it was concerned the working classes had been betrayed by the Liberal government. Source C opposes compulsion and knowledge would suggest that the measure was indeed disliked by the working class, who considered it to be unfair in every sense (upper class clubs were immune; middle class magistrates would close down the working man’s pub and control his only pleasure ). They would agree with Bishop Magee. The Conservatives would reverse the measure in the Intoxicating Liquors Act of 1874, which increased pub opening hours with compensation for landlords who had lost their licences, a popular act amongst the working classes. Cross in Source D focuses on insanitary housing, arguing that the State had a right to interfere and compel in this area, one where there ‘was much to be done to take the working class out of miserable conditions’. His Artisans Dwelling Act of 1875 allowed local councils to devise improvement schemes and compel slum owners to sell to them with cheap loans from government to demolish and allow commercial rebuilding. Source E appears to confirm the effectiveness of this, the slum landlord in the cartoon (Bumble) drowning his sorrows at the ‘regular cross’ (burden) he now has to bear (a pun on Disraeli’s Home Secretary Richard Cross) and at the prospect of more ‘harassing legislation’. Mention could also be made to Conservative legislation on Factories (1874 and 1878), Public Health (1875), Food and Drugs (1875), Pollution,

70 Focus: Judgement in context, based on the set of Sources and own knowledge. Successful answers will need to make use of all five Sources, testing them against contextual knowledge and evaluating their strengths and weaknesses, any limitations as evidence. A range of issues may be addressed in focusing upon the terms of the question but no set conclusion is expected.

Page 246: History A Mark Schemes for the Units January 2010 - The ...

F963/02 Mark Scheme June 2013

13

Question Answer Marks Guidance Agricultural Holdings, Trade Unions and education, although candidates should remain focused on the sources. The view that Gladstone’s 1st Government did more can be found in Sources A, D and E. Bright in Source A publicly contrasted the different approach to reform of working and living conditions between the Liberal and Conservative governments. As an old middle class radical liberal and mentor of Gladstone it is unsurprising that he considers Gladstone especially to be concerned with ‘measures for the good of the people’. In contrast Disraeli and the Conservatives are said to see social reform purely as a matter of token gesture rather than genuine reform. To Bright this is patronising, treating the workers not as responsible and equal adults but as ‘indulged children’, voting fodder for a condescending conservative aristocracy. Bright’s liberal stress is upon helping the working class to achieve justice and civil equality in a society based on mutual respect. He argues that what they really want is – protection of Trade Union property, the right to strike and equal liability for employers. Bright’s Liberal emphasis is on institutional and legal reforms rather than health and housing, and mention could be made of the Trade Union Reforms (the Act of 1871 and the Criminal Law Amendment Act) to support this. However candidates may point out that it was the Conservatives that allowed peaceful picketing and, in 1875, achieved employer liability in the Employers and Workmen’s Act. Cross in Source D could also be said to be more Liberal than Conservative in approach. The emphasis of his introductory speech on the Artisans Dwellings Act is on the limits of State responsibility and the importance of not impinging on private building or allowing charities or local government to undercut market rents. He does not see the job of government as the provision of necessaries like housing and education for the working class. As a statement of conservative principle on working class social reform this is effective evidence of limitations and the importance of liberal type moves on legal equality for the working man. His legislation was permissive, the burden falling on local authorities. The Punch Cartoon in Source E suggests that the Liberals had struck at drink and landlords – the ‘harassing legislation’ of the last session, although by implication they had been less inclined to pursue slum landlords. Candidates could point out that this may have been because of nonconformist pressure. They might also point out that this particular landlord seems none the worse for wear. A third view can be constructed, that both governments were similar in their approach to working and living conditions. Sources D and E strongly suggest this. In Source D Cross merely carries on from where the Liberals had left off. Torrens Act of 1868, allowing local councils to demolish slum houses, had been weakened and Cross’ Act strengthened it. Both were permissive and neither saw much of a take-up of powers. Gladstone’s 1872 Public Health Act

Page 247: History A Mark Schemes for the Units January 2010 - The ...

F963/02 Mark Scheme June 2013

14

Question Answer Marks Guidance created sanitary authorities, recognised as a legitimate area by Cross in D. His 1875 Act merely gave clearer statements of what local authorities were expected to do. Source E’s overall thrust is that vested interests were under attack from both – ‘it was our turn last session and now it’s yours’. However, which of these interests harmed the working class more is a moot point. In terms of judgement candidates may find any one of these convincing, depending on their evaluation of the sources and the contextual use of their own knowledge. All, except possibly Cross in D, are partial in their approach. Thus Bright in A takes a particular liberal slant before any legislation is underway, although his emphasis is borne out by what ensued. Source B is also liberal but outraged at betrayal, seeing licensing from a middle class viewpoint. Sources C and D are conservative, arguably C touching a working class chord, albeit of the sort that Bright condemns (token gestures, in this case the freedom not to be sober). Cross in D is more matter of fact and could be said to provide convincing evidence of limitations and similarity. Source E exaggerates but clearly vested interest was under attack in the social reforms of both governments in the 1870s.

Page 248: History A Mark Schemes for the Units January 2010 - The ...

F963/02 Mark Scheme June 2013

15

Question Answer Marks Guidance 3 (a) The sources share some common ground but also differ. In general terms they agree the reforms

did bring benefits. With the words ‘every improvement’ Source C accepts the reforms as positive but emphasises the limitations of them. Source D is more effusive about the impact of the reforms and makes great claims for them without any reservations. More specifically, both regard pensions as a benefit albeit Source C does so grudgingly with the words ‘at least’ and the complaint that workers are only eligible at 70 years whereas Source D considers them to be ‘on higher ground’ which ‘will help millions’. Similarly, Source C seems, in theory, to support insurance. Source D is convinced that insurance will address the problem of an unemployed father being unable to feed his children and will ‘abolish that state of things for ever’. However, Source C is critical of the costs attached, complaining of ‘the irritating conditions and a burdensome payment’ of which Source D makes no direct mention although the reference to the opposition of the Tory Press who regarded ‘the Insurance Bill as an act of tyranny’ obliquely acknowledges the compulsion involved and, by implication the contributions that workers will have to make. Source C refers to the introduction of the minimum wage which he seems to imply had a negative impact as the prices charged to consumers were increased to cover the extra expense incurred by employers: indeed, that it was the capitalist class’ that gained not the workers. Source D makes no mention of the minimum wage. The critical and rather disappointed view of Source C might be considered to be typical of Keir Hardie, a radical socialist, who favoured the redistribution of wealth illustrated by his preference for the cost of insurance to be met by ‘a small addition to income or land tax’. Equally, unsurprising is the optimistic and positive stance of Source D as the author was the architect of the reforms and who had battled hard to implement them. Reference might be made to the struggle of Lloyd George with the House of Lords and the Budget of 1909 to pay for pensions. The industrial context against which these speeches were made could be evaluated. The remarks of Source C are presented as a contribution to a debate on the causes of industrial unrest and in doing so suggests that the strikes and disruption of the time indicate the failure of the social reforms. In contrast, Source D is convinced that the reforms are a means of protecting workers at a time of hardship which given the context implies that the Insurance Bill offering help for those out of work at a time when the level of unemployment was high was less a cause of unrest than a remedy for it. The audiences addressed by the two speakers could be assessed. Hardie was talking in the House of Commons and although his remarks were meant to persuade and were recorded in Hansard his purpose was limited to making a contribution to a motion asking for an investigation into unrest. By contrast, Lloyd George was addressing the public and it would be odd for him to adopt anything other than a positive note on the impact of reform. He is

30 Focus: Comparison of two Sources. No set answer is expected, but candidates need to compare the contents, evaluating such matters as authorship, dating, utility and reliability, so using the Sources ‘as evidence for …’. The headings and attributions should aid evaluation and reference to both is expected in a good answer.

Page 249: History A Mark Schemes for the Units January 2010 - The ...

F963/02 Mark Scheme June 2013

16

Question Answer Marks Guidance clearly intent on persuading people to support the reforms because of their intrinsic value but also their political benefit to the Liberals. The final line illustrates his lofty, perhaps idealistic, vision of the way ahead. In judgement candidates may emphasise the contrast in the political positions of Hardie and Lloyd George (in terms of policies and power) to explain the difference between them yet despite that highlight the general similarity of their ambition for social reform.

(b) The sources provide evidence that show support for the Liberal reforms but also that they aroused opposition. Arguably, each source presents evidence of opposition and only three sources show the support they enjoyed. However, the evaluation of the quality of the evidence will be important in reaching a judgement. Reforms concerning child welfare is confined to Source A which is critical of the Children Act. Opposition is based on the denial of shelter (at times of bad weather) and a place of rest (the name of the refuge) as well as the termination of one strand of the way of life that had been long accepted. There is little doubt that this restriction was unpopular and created problems, not least the abandonment of children at home when their parents went to the local public house. Many parents objected to their liability to prosecution for dereliction of care for their children. Many did not consider it the business of the State to interfere in the affairs of families. However, this was only one of several parts of the Act many of which were applauded, for example, in treating child offenders differently to adults, the registration of children’s homes and so on. The poster ignores this. Further, as a Conservative Party poster it is clearly designed to inflame opposition to the Liberals as the dialogue in the corner makes clear. The poster appears to be aimed at the middle class woman (given the clothes worn by the women), perhaps on holiday at the seaside, who had time and money to spend on leisure and in contrast to working class women who were either at work or too poor to spend money on themselves. Candidates might broaden the discussion by consideration of other reforms affecting children: school meals, medicals. Reforms to improve the lives of the elderly are assessed in Sources B, C and D. Opposition to pensions based on principle is registered in Source B which explains that some argued that pensions would ‘dishearten the thrifty ... and encourage the idle’. Such attitudes were consistent with the ‘self-help’ and ‘laissez-faire’ philosophies of the 19thC and were widely held, not least amongst many of the poor who were too proud to accept support and the tax payers who resented paying for pensions. This suggests the view that there was opposition ‘from both middle

70 Focus: Judgement in context, based on the set of Sources and own knowledge. Successful answers will need to make use of all five Sources, testing them against contextual knowledge and evaluating their strengths and weaknesses, any limitations as evidence. A range of issues may be addressed in focusing upon the terms of the question but no set conclusion is expected.

Page 250: History A Mark Schemes for the Units January 2010 - The ...

F963/02 Mark Scheme June 2013

17

Question Answer Marks Guidance and working classes’ could be reliable. Further, some believed the consequences could be even greater and ‘might even deal a blow at the Empire which could be almost mortal’. This might be dismissed as the view of one man, Lord Rosebery, and a very rich one too, albeit a Liberal, but at a time of increased economic competition and international rivalry this view was widely shared. In Source C the opposition to pensions is not based on principle but the mean scale of them. Hardie considers them to be a distraction from the real issue of low wages and the age of eligibility to be too late. However, Source C backs pensions as a matter of policy as do Sources B and D and also that pensions enjoyed considerable support. Source B confirms that ‘the small sums involved meant life itself for many elderly poor’. Indeed, so welcome were they, many recipients regarded Lloyd George ‘as if he were a saint’. As the author was a small boy at the time pensions were introduced and able to recollect his mother’s assessment of the response to them the evidence of Source B might be considered reliable especially as it conceded that others thought differently. Source D might be considered less reliable as it outlines the views of Lloyd George who introduced the pensions but cross reference to Source B and the reverence with which Lloyd George was held could be made to support his claims and his dismissive views of the Tory Press are perhaps not unreasonable given the nature of the poster in Source A. Reforms designed to help workers are also assessed. On insurance the evidence would appear to be divided. Source C believes the contributions workers have to pay are ‘a burdensome payment’. Candidates could expand on the details of the insurance schemes and the efforts the government made to convince workers their contributions were small in return for the benefits. However, the author fails to acknowledge the contributions that employers and the State were also to make. The political views of Hardie might be assessed and the context in which the scheme was being introduced to explain his critical position. Source E opposes Part II of the Insurance Act as ‘workers who come within its scope are obliged to register with the exchange’. The author objects to this on two counts. He seems to think individuals should have the freedom to decide if they want to register with the labour exchange and by tying a worker to the exchange he was condemned to a system of exploitation. This is explained by the tendency of the exchanges to place non-union labour in jobs rather than unionised workers. In assessing these charges candidates might judge the language of the piece as indicative of a rather prejudiced attitude: phrases like ‘shackles of slavery’, ‘exploited wage-slaves’ and the ‘venomous’ nature of the system, for example. The author was a spokesman for a left-wing group. On the other hand, the exchanges had been in place for four years and his views could reflect practice. Source D is

Page 251: History A Mark Schemes for the Units January 2010 - The ...

F963/02 Mark Scheme June 2013

18

Question Answer Marks Guidance clearly supportive of the Insurance Acts. Again, the claims made might be tested against knowledge. Lloyd George is right to claim that the OAPs and Insurance legislation ‘will help millions’ as at least 15 million were covered by them. However, given the limits of the benefits and the time within which it was possible to claim them it is debatable as to whether they were enough to prevent them ‘from stumbling into wretchedness’. The fact that people in the audience cheered Lloyd George specifically by saying the reforms ‘will win many more (elections)’ suggests popular support: election wins in 1906 and twice in 1910, albeit with lower margins, might be considered a fair reflection of the support in the country for the Liberal reforms. Comments on the problem of low wages might be addressed using Source C. The minimum wage might be considered to be counter-productive but was he right to ascribe the rise in the price of coal to that alone? At face value the sources might lean more to the view that the reforms attracted more opposition than support. However, certain sources are blatant ‘propaganda’ such as Source A or they represent the views of the more radical left like Sources C and E. Although in support of the reforms Sources B and D do allow that there was opposition. Candidates’ judgement will depend on how they view the quality of the evidence and how representative it is of the nation as a whole.

Page 252: History A Mark Schemes for the Units January 2010 - The ...

F963/02 Mark Scheme June 2013

19

Question Answer Marks Guidance 4 (a) The main difference is in the optimism of Source A and the pessimism in Source B about

Britain’s military ability to meet a threat from Germany. Churchill in 1948 thought that Britain could have created a strong enough air force in 1933 or 1934 to restrain Hitler. The Chiefs of Staff in B thought that there was not a time which could be foreseen in which Britain’s defences, including her air force would have been strong enough to even defend British interests against Germany. Churchill in A sees an enhanced military capacity as allowing effective international action against Germany; the Chiefs of Staff in B see the state of Britain’s armed forces as not empowering active diplomacy to restrain Hitler but rather forcing Britain into making concessions. Churchill does not consider the whole context of Britain’s imperial defence, whereas the Chiefs of Staff do. Churchill is concerned with air power, but the Chiefs of staff are considering British Imperial defence as a whole, including the army and navy. There is some similarity in that both Churchill and the Chiefs of Staff value France as a source of military support for the armed forces (A: based on superior air power Britain and France could have invoked the aid of the League; B without overlooking the assistance we might obtain from France) The difference can be explained by the nature of the Sources. Churchill in A had raised the issue of air power repeatedly in the 1930s and in 1948 he was looking back and was anxious to show how right he had been and if his advice had been followed, war could have been prevented. As the war had been won by a grand alliance based on superior air power then it was easy to project this back to the 1930s. B on the other had had no benefit of hindsight and was concerned with the wider responsibilities of Empire – something that Churchill does not refere to here. The cuts that Churchill had actually begun in the 1920s left British armed forces in a weak position to meet the triple threat from Germany, Italy and Japan – Churchill in A makes no reference to Britain’s Far East responsibilities. In terms of judgement, the military chiefs in B were doing their job – to warn government about the actual situation; they were in a position to know the state of the armed forces, but may have been cautious. Churchill in A was not in this position and could therefore speculate, and crucially at a much earlier date than B where Britain can act if she were to have undertaken large scale air rearmament. It is not likely that in the Depression climate of the 1933-4 period there would have been much chance of a massive increase in air power to meet a threat from a Germany which had not yet massively re-armed, so this is really being wise after the event. However, the Chiefs of Staff may be naive to suggest ‘reducing the number of our potential enemies’ given the militaristic and ambitious nature of the leadership Italy, Japan and Germany in place of a rapid expansion of armed forces. Nonetheless, this is what Chamberlain tried to do in the case of Germany through appeasement in 1938.

30 No set answer is expected, but candidates need to compare the contents, evaluating such matters as authorship, dating, utility and reliability, so using the Source ‘as evidence for…..’ The Headings and attributions should aid evaluation and reference to both is expected in a good answer.

Page 253: History A Mark Schemes for the Units January 2010 - The ...

F963/02 Mark Scheme June 2013

20

Question Answer Marks Guidance (b) The support for the realism of gaining international support comes from the two Sources from

Churchill, A and C, one looking back after the war and the other in 1938 and in part from Churchill’s US appeal in E. The case against the realism of international support is put by Chamberlain in Source D, though it was a policy that he had considered and had confirmed by Source B. The evidence in Source E is not conclusive, and candidates could use it either to suggest that there was some public support in the US for support for collective action and thus realistic, or that there was little unanimity in the US. A like C assumes that France was a firm ally and that war was preventable by an alliance of nations with the moral authority of the League of Nations. Both link this to the build up of forces – A postulates the effects of a British air build up and C refers to forces being marshalled, but essentially the policy is to deter Germany by international agreement. A might be influenced by what actually did happen, when Germany was defeated by a sustained alliance – the ‘united nations’ of the second world war. However, this is a post-war justification for the type of policy Churchill is seen to be advocating in C. It is easy to see C as relying on a great many ‘ifs’ – if states assembled; if France were a firm ally; if there were sufficient forces to be marshalled; if there were a ‘moral sense’ in the world. By 1938 it was clear that Germany was a threat and clear that Britain was not likely to resist, so Churchill was proposing the best policy in difficult circumstances which did not involve appeasing Hitler rather than a very realistic alternative. Candidates may use contextual knowledge of the Anschluss with Austria. Troops had entered Austria on 12 March. In context, with British rearmament in its relatively early stages, with a prime minister committed to appeasement, with France weakened by internal disputes, with Russia undergoing domestic turmoil, the USA committed officially to isolation and a League weakened by the Ethiopian crisis, all this did not seem very realistic. The key point in the speech was the warning about approaching war and the need to take some sort of proactive policy, but the reality was that no such policy would emerge. The counterview Churchill’s unrealism, is neatly expressed by Chamberlain in D. In a personal letter he had no need to offer any false optimism, as was the case in the case of the public utterance by Churchill in C. The Chiefs of Staff had rejected the idea of a Grand alliance, probably considering the limitations of France as an ally and the problems faced by Russia whose leading generals were being purged by Stalin and whose military equipment was seen to be weak, The foreign office was sceptical of any agreement with the USA or Russia. The divisions in Eastern Europe with the Poles resentful of the territorial boundary of Czechoslovakia, for instance, would have made effective allies in the east difficult and geographical reality, now that Germany had taken Austria, would indeed have made a campaign to save the Czechs problematic. However, what is not considered here is the alternative – that proposed by the

70 Focus: Judgement in context, based on a set of Sources and own knowledge. Successful answers will need to make use of all five Sources, testing them against contextual evidence and evaluating their strengths and weaknesses, any limitations as evidence. A range of issues may be addressed in focusing upon the terms of the question but no set conclusion is expected.

Page 254: History A Mark Schemes for the Units January 2010 - The ...

F963/02 Mark Scheme June 2013

21

Question Answer Marks Guidance Chiefs of Staff in B of international agreements with potential enemies. Was this any more realistic, given the volatility of the Hitler regime, the alienation of Mussolini over Ethiopia and the pressures to expand to gain raw materials and markets that were affecting Japan, already at war with China? C’s plea for alliances to contain Germany seems to be supported by B’s expressed hope in the final line that action needs to be taken to reduce potential enemies, presumably by alliances or by appeasing Germany and Italy. C seems to be supported by B. If British imperial commitments were too great for Britain’s armed forces, then what could it bring to any proposed Grand Alliance and why should other nations take a role in defending Britain’s Empire? A grand alliance would have had to have had some military ‘teeth’ and it was clear from B that this was not the case. It may be that the Chiefs of Staff, anxious to get more resources and fearful of being drawn into a conflict prematurely were exaggerating the situation and overestimating Germany’s strength and any possibility of coordinated action by the Axis powers. However in the context of economic difficulties and constraints in spending dating from the 1920s, their view could be seen as realistic. Much turned on the possible attitude of the USA and E shows Churchill after Munich appealing directly to its people in the hope of engendering support for cooperation in defence of democracy. Chamberlain, however, had little faith in the USA and given the Neutrality Acts and the strength of feeling against another European conflict, this may have been wishful thinking on Churchill’s part. However the letters do show some support; but they were mixed. There is a telling point about Britain having betrayed her friends by the appeasement policy, something about which Churchill agreed but could not really refute. The letters do not offer sufficient evidence to draw conclusions about the chances of US intervention; but Roosevelt had to bear in mind the USA’s economic problems, the need to maintain the New Deal and the dangers from the Pacific as well as a mass of isolationist opinion. The limited aid given in 1939-41 may be seen as evidence for Churchill’s lack of realism here.

Page 255: History A Mark Schemes for the Units January 2010 - The ...

Oxford Cambridge and RSA Examinations is a Company Limited by Guarantee Registered in England Registered Office; 1 Hills Road, Cambridge, CB1 2EU Registered Company Number: 3484466 OCR is an exempt Charity OCR (Oxford Cambridge and RSA Examinations) Head office Telephone: 01223 552552 Facsimile: 01223 552553 © OCR 2013

OCR (Oxford Cambridge and RSA Examinations) 1 Hills Road Cambridge CB1 2EU OCR Customer Contact Centre Education and Learning Telephone: 01223 553998 Facsimile: 01223 552627 Email: [email protected] www.ocr.org.uk For staff training purposes and as part of our quality assurance programme your call may be recorded or monitored